Anda di halaman 1dari 382

MRCP Paces

180 Clinical Cases


MRCP Paces
180 Clinical Cases

Kevin OGallagher MBBS BA (Hons) MRCP (UK)


Specialty Registrar in Cardiology, London Deanery, London, UK
Daniel Knight MBBS BSc (Hons) MRCP (UK)
Specialty Registrar in Cardiology, London Deanery, London, UK
Michael OGallagher MBBCh BAO
Specialty Registrar in Ophthalmology, Belfast, UK
Eirini Merika MBBS BSc (Hons) MRCP (UK)
Specialty Registrar in Dermatology, London Deanery, London, UK
Omar Malik MBBS BSc PhD FRCP
Consultant Neurologist, Charing Cross Hospital, London, UK
Editors
Peter TK Milton MBBS BSc (Hons) MRCP (UK)
Specialty Registrar in General Practice, London Deanery, London, UK
Nick Oliver MBBS BSc (Hons) FRCP
Consultant Physician and Endocrinologist, Charing Cross Hospital,
London, UK

London Philadelphia Panama City New Delhi


2015 JP Medical Ltd.
Published by JP Medical Ltd
83 Victoria Street, London, SW1H 0HW, UK
Tel: +44 (0)20 3170 8910 Fax: +44 (0)20 3008 6180
Email: info@jpmedpub.com Web: www.jpmedpub.com

The rights of Kevin OGallagher, Daniel Knight, Michael OGallagher, Eirini Merika Omar Malik,
Peter Milton and Nick Oliver to be identified as authors of this work have been asserted by them in
accordance with the Copyright, Designs and Patents Act 1988.

All rights reserved. No part of this publication may be reproduced, stored or transmitted in any
form or by any means, electronic, mechanical, photocopying, recording or otherwise, except as
permitted by the UK Copyright, Designs and Patents Act 1988, without the prior permission in
writing of the publishers. Permissions may be sought directly from JP Medical Ltd at the address
printed above.

All brand names and product names used in this book are trade names, service marks, trademarks
or registered trademarks of their respective owners. The publisher is not associated with any
product or vendor mentioned in this book.

Medical knowledge and practice change constantly. This book is designed to provide accurate,
authoritative information about the subject matter in question. However readers are advised
to check the most current information available on procedures included and check information
from the manufacturer of each product to be administered, to verify the recommended dose,
formula, method and duration of administration, adverse effects and contraindications. It is the
responsibility of the practitioner to take all appropriate safety precautions. Neither the publisher
nor the authors assume any liability for any injury and/or damage to persons or property arising
from or related to use of material in this book.

This book is sold on the understanding that the publisher is not engaged in providing professional
medical services. If such advice or services are required, the services of a competent medical
professional should be sought.

Every effort has been made where necessary to contact holders of copyright to obtain permission
to reproduce copyright material. If any have been inadvertently overlooked, the publisher will be
pleased to make the necessary arrangements at the first opportunity.

ISBN: 978-1-907816-52-9

British Library Cataloguing in Publication Data


A catalogue record for this book is available from the British Library

Library of Congress Cataloging in Publication Data


A catalog record for this book is available from the Library of Congress

Commissioning Editor: Steffan Clements


Editorial Assistant: Sophie Woolven
Design: Designers Collective Ltd

Typeset, printed and bound in India.


Preface

Representing the culmination of the membership examination process, PACES is widely


recognised as an exceptionally difficult challenge. Demanding the incorporation of both clinical
acumen and applied knowledge in a time-pressured environment, it is considered a rite of passage
for the aspiring physician. Candidates can therefore be forgiven for approaching their preparation
with trepidation.
This book aims to encourage an early, targeted approach for each component of the exam. It
serves to guide readers through their preparation by providing a summary of clinical cases likely
to be tested, and offers insight from candidates with recent experience of the current format,
including the new station 5. It helps to remove the mystery from PACES, improving performance
and maximising the chances of success through the development of a structured framework for
the presentation of clinical findings.
The framework for successful presentation need not be complicated, as demonstrated in
this book. Each clinical case is broken down to transform simple concepts and fundamental
knowledge into an impressive display of clinical acumen. The successful candidate should adopt
this framework, in order to display the necessary skills to be a safe, knowledgeable and insightful
doctor suitable for admission to the Royal College of Physicians.
The book guides the reader through the carousel of the exam chapter by chapter. The
introduction highlights the differences between the stations, the approach required for each, and
defines the overall framework. It is therefore paramount for readers to familiarise themselves with
the concepts defined in the introduction before moving on to the remainder of the text.
The format lends itself to both individual study and to group role-play, which will stimulate
dialogue around the themes introduced. Exploring the themes and topics presented in each of the
cases, structured in accordance with the framework, will equip candidates with the necessary skills
to negotiate the PACES process.

Kevin OGallagher
Daniel Knight
Michael OGallagher
Eirini Merika
Omar Malik
Peter Milton
Nick Oliver
October 2014

v
Contents

Preface v
Glossary xv
Introduction: preparation for the exam xix

Chapter 1 Cardiovascular system (station 3) 1


Systolic murmur
Case 1: Aortic stenosis 2
Case 2: Mitral regurgitation 5
Case 3: Ventricular septal defect 8
Case 4: Hypertrophic obstructive cardiomyopathy 10
Case 5: Tricuspid regurgitation 12
Case 6: Pulmonary stenosis 15
Diastolic murmur
Case 7: Mitral stenosis 16
Case 8: Aortic regurgitation 19
Case 9: Pulmonary regurgitation 21
Mixed valve disease: distinguishing the dominant murmur
Case 10: Mixed aortic valve disease 24
Case 11: Mixed mitral valve disease 25
Scars and clicks - the cardiothoracic patient
Case 12: Prosthetic valves and cardiac surgery 26
Oedema, crepitations, and a raised jugular
venous pressure: the failing heart
Case 13: Cardiac failure 30
Case 14: The infraclavicular mass pacemakers 33
implantable devices
Abnormal pulses
Case 15: The irregular pulse: atrial fibrillation 36
Case 16: Coarctation of the aorta 38
Splinter haemorrhages and a murmur: the infected valve
Case 17: Infective endocarditis 40
Congenital heart disease and complex cases
Case 18: Pulmonary hypertension 43
Case 19: Atrial septal defect 45
Case 20: Patent ductus arteriosus 46
Case 21: Ebsteins anomaly: the cyanosed 48
patient with a murmur
Case 22: Tetralogy of Fallot 50
Case 23: Eisenmengers syndrome 51
Case 24: Dextrocardia 54

Chapter 2 Nervous system (station 3) 55


Examination of cranial nerves: visual field defects
Case 25: Monocular blindness 56
Case 26: Homonymous hemianopia 57
Case 27: Bitemporal hemianopia 58
Case 28: Homonymous quadrantanopia 59

vii
Contents

Examination of cranial nerves: ophthalmoplegia


Case 29: Sixth nerve palsy 60
Case 30: Fourth nerve palsy 62
Case 31: Internuclear ophthalmoplegia 63
multiple sclerosis
Examination of cranial nerves: pupillary problems with ptosis
Case 32: Ptosis, large pupil and 65
ophthalmoplegia: third nerve palsy
Case 33: Ptosis and small pupil: Horner syndrome 67
Case 34: Bilateral ptosis with fatigability: 68
myasthenia gravis
Examination of cranial nerves: pupillary problems without ptosis
Case 35: Relative afferent pupillary defect (RAPD) 70
Case 36: Unilateral large pupil in a young 72
female HolmesAdie pupil
Case 37: Bilateral small pupils: 73
Argyll Robertson pupils
Examination of cranial nerves: cranial nerves V, VII and VIII
Case 38: Facial weakness 74
Case 39: Conductive hearing loss 76
Case 40: Sensorineural hearing loss 78
Case 41: V, VII, VIII cranial nerves 79
cerebellopontine angle
Examination of cranial nerves: assessment of speech
Case 42: Dysphasia 81
Case 43: Dysarthria bulbar palsy 82
Examination of cranial nerves: complex presentations
Case 44: Lateral medullary syndrome 84
Case 45: Jugular foramen syndrome 85
Examination of upper limbs: weakness
Case 46: Myotonic dystrophy 86
Case 47: Cervical myelopathy 88
Case 48: Unilateral arm weakness 89
neuralgic amyotrophy
Examination of upper limbs: tremor and assessment of coordination
Case 49: Resting tremor Parkinsonism 90
Case 50: Intention tremor and problems with 92
co-ordination cerebellar syndrome
Examination of upper limbs: spinal cord syndromes
Case 51: Central cord syndrome syringomyelia 93
Case 52: BrownSquard (hemi-cord) syndrome 96
Case 53: Anterior cord syndrome 98
Examination of lower limbs: weakness
Case 54: Spastic paraparesis 100
Case 55: Proximal myopathy 101
Examination of lower limbs: sensory disturbance
Case 56: Glove and stocking distributions 102
sensory polyneuropathy
Examination of lower limbs: the deformed foot pes cavus
Case 57: Friedreichs ataxia 104
Case 58: CharcotMarieTooth disease 105
(hereditary sensorimotor neuropathy type 1)

viii
Contents

Examination of lower limbs: assessment of gait


Case 59: Shuffling gait Parkinsons disease 106
Case 60: Ataxic gait 107
Case 61: Unilateral high-stepping gait 107
Case 62: Spastic hemi-paretic gait 108
Case 63: Waddling gait 110
Examination of lower limbs: cases of note
Case 64: Cauda equina syndrome 110
Case 65: Absent ankle jerks with extensor 112
plantar reflexes sub-acute
degeneration of the cord

Chapter 3 Respiratory system (station 1) 115


Crepitations
Case 66: Pulmonary fibrosis 116
Case 67: Consolidation 118
Case 68: Bronchiectasis 120
Dullness to percussion
Case 69: Pleural effusion 122
Case 70: Collapse 124
Wheeze
Case 71: Chronic obstructive pulmonary 125
disease
Chest wall scars: the thoracic surgery patient
Case 72: Lobectomy 128
Case 73: Pneumonectomy 129
Case 74: Lung transplantation 130
Diagnostic clues in the respiratory patient: facial signs
Case 75: Plethoric face and dilated chest veins 131
superior vena cava obstruction
Case 76: Horners syndrome: Pancoasts tumour 133
Case 77: Cushingoid appearance 135
Diagnostic clues in the respiratory patient: peripheral signs
Case 78: Abnormal nails with pleural effusions 136
and lymphadenopathy yellow nail syndrome
Case 79: Signs of right-sided heart failure 138
cor pulmonale

Chapter 4 Abdominal system (station 1) 141


The liver patient
Case 80: The syndrome of chronic liver disease 142
Case 81: Organomegaly isolated 146
hepatomegaly
Case 82: The distended abdomen ascites 147
Case 83: Portal hypertension 149
Case 84: Abnormal skin pigmentation jaundice 151
Case 85: Abnormal skin pigmentation haemachromatosis 154
Case 86: Periorbital xanthelasma primary 156
biliary cirrhosis
The renal patient
Case 87: The patient requiring renal replacement therapy 157
Case 88: Bilateral ballotable kidneys 159
ix
Contents

Case 89: Iliac fossa scar with a palpable mass renal transplant 160
Case 90: Flank scar nephrectomy 162
Spleen and other abdominal cases
Case 91: Splenomegaly 163
Case 92: Feltys syndrome 165
Case 93: Multiple abdominal scars 166
Case 94: Rooftop incision 167
Case 95: Obesity with evidence of laparoscopic procedure 169

Chapter 5 Station 5 171


Hands: symmetrical deforming polyarthropathy
Case 96: Rheumatoid arthritis 172
Case 97: Psoriatic arthritis 175
Case 98: Jaccouds arthropathy 177
Hands: disease distal interphalangeal joints
Case 99: Osteoarthritis 179
Hands: abnormal hands with skin changes
Case 100: Sclerodactyly/systemic sclerosis 181
Hands: single swollen metocarpophalangeal joint
Case 101: Gout 182
Hands: neurological signs in the hands
Case 102: Carpal tunnel syndrome 184
Case 103: Wrist drop radial nerve lesion 185
Case 104: Ulnar nerve lesion 187
Fundoscopy: diseased lens
Case 105: Cataracts 188
Fundoscopy: diseased retina
Case 106: Diabetic retinopathy 190
Case 107: Hypertensive retinopathy 194
Case 108: Retinitis pigmentosa 195
Case 109: CMV retinitis 197
Fundoscopy: abnormal optic discs
Case 110: Optic atrophy 198
Case 111: Papilloedema 200
Case 112: Foster Kennedy syndrome 201
Endocrinology
Case 113: Thyrotoxicosis 202
Case 114: Hypothyroidism 205
Case 115: Acromegaly 206
Case 116: Weight gain and easy bruising 208
Cushingoid appearance
Case 117: Gynaecomastia 209
Case 118: Pseudohypoparathyroidism 211
Albrights hereditary osteodystrophy
Dermatology: chronic skin conditions
Case 119: Chronic plaque psoriasis 212
Case 120: Atopic dermatitis or atopic eczema 216
Dermatology: manifestations of systemic disease
Case 121: Pyoderma gangrenosum 218
Case 122: Vasculitis (HenochSchnlein purpura) 221
Case 123: Chronic venous insufficiency 223

x
Contents

Case 124: Cutaneous lupus erythematosus 226


Case 125: Discoid lupus erythematosus 230
Case 126: Dermatomyositis 231
Case 127: Alopecia areata 233
Case 128: Sarcoidosis 237
Genetic disease
Case 129: Tuberous sclerosis 239
Case 130: Neurofibromatosis 241
Dermatology spot diagnoses
Case 131 & 132: Dermatology spot diagnoses 245
Commonly encountered spot diagnoses
Case 133: Ankylosing spondylitis 247
Case 134: Marfan syndrome 249
Case 135: Turner syndrome 252
Symptoms as a presenting complaint
Case 136: Tremor 254
Case 137: Seizures 255

Chapter 6 History taking (station 2) 257


Cardiovascular
Case 138: Chest pain 258
Case 139: Irregular pulse 261
Case 140: Palpitations 265
Case 141: Ankle swelling 268
Case 142: Raised lipids 271
Case 143: Hypertension 273
Respiratory
Case 144: Breathlessness and wheeze 277
new diagnosis of asthma
Case 145: Shortness of breath and wheeze 280
established diagnosis of asthma
Case 146: Wheeze established chronic 282
obstructive pulmonary disease
Case 147: Haemoptysis in the returning traveller 285
Case 148: Persistent/change in cough 287
investigation of cancer
Abdominal
Case 149: Change in bowel habit malignancy 290
Case 150: Change in bowel habit established 293
inflammatory bowel disease
Case 151: Difficulty in swallowing 295
Case 152: Cirrhotic liver disease 297
Case 153: Previous renal transplant 299
Neurology
Case 154: Headache 303
Case 155: Visual disturbance 306
Case 156: Tremor 308
Case 157: Established multiple sclerosis 310
Infectious disease
Case 158: Fever in the returning traveller 313
Rheumatology
Case 159: Sore, stiff hands 315
Case 160: Back pain 316
xi
Contents

Endocrinology
Case 161: Established diabetes 318
Case 162: Heat intolerance and weight loss 319
Case 163: Diabetes insipidus 321
Case 164: Weight gain 323

Chapter 7 Communication skills and ethical scenarios 327


(station 4)
Consent and capacity
Case 165: Informed consent for an invasive 328
investigation or procedure chest drain insertion
Case 166: The patient refusing to consent 329
Case 167: Consenting for a clinical trial 330
Case 168: Assessing capacity 332
Case 169: The patient who lacks capacity 333
Breaking bad news and counseling patients
Case 170: Breaking bad news to a relative 335
Case 171: The decision to resuscitate 336
Case 172: Withdrawing (life-dependent) care 337
Case 173: Brainstem death testing 338
Case 174: Organ donation and harvesting 340
Case 175: Requesting a hospital postmortem 341
Case 176: Driving with disease 342
Case 177: Genetic counselling 345
Case 178: Percutaneous endoscopic 347
gastrostomy feeding
Case 179: HIV testing recent unprotected sex 348
Case 180: HIV testing needlestick injury 351
When care goes wrong
Case 181: Dealing with complaints medical errors 353
Dealing with colleagues
Case 182: The trainee in difficulty 355

xii
Contributors
Jamal Hayat BSc MBBS MRCP
Chapter 4
Specialty Registrar in Gastroenterology and General Internal Medicine
St Georges Hospital, University of London, London, UK

Shruthi Konda BSc MBBS MRCP AICSM


Chapter 3
Specialty Registrar in Respiratory and General Internal Medicine
London Deanery, London, UK

Peter TK Milton MBBS BSc (Hons) MRCP (UK)


Chapters 2, 4, 6
Specialty Registrar in General Practice
London Deanery, London, UK

Bejal Pandya MBBS MRCP


Chapter 1
Consultant in Congenital Heart Disease
The Heart Hospital, University College London NHS Foundation Trust, London, UK

xiii
Glossary

ABPA Allergic bronchopulmonary BRCA Breast cancer gene


aspergillosis BTS British Thoracic Society
ABPI Ankle-brachial pressure index CABG Coronary artery bypass graft
ACEi ACE inhibitors CAG Coronary angiography
ACHD Adult congenital heart disease CEA Carcinoembryonic antigen
AChR Acetylcholine receptor CHADS-VASC Congestive heart failure
ACTH Adrenocorticotropic hormone Hypertension Age 75 years
ADH Antidiuretic hormone Diabetes Prior stroke Vascular
AF Atrial fibrillation disease Age 6574 years Sex
AHA American Heart Association category
AIDP Acute inflammatory demyelinating CIDP Chronic inflammatory
polyneuropathy demyelinating polyneuropathy
AKI Acute kidney injury CNS Central nervous system
ALS Advanced life support/amyotrophic CML Chronic myelocytic leukaemia
lateral sclerosis CMV Cytomeglovirus
ALT Alanine aminotransferase COMT Catechol-O-methyl transferase
ALS Amyotrophic lateral sclerosis COPD Chronic obstructive pulmonary
AMTS Abbreviated mental test score disease
ANA Antinuclear antibody CRC Colorectal cancer
ANCA Antineutrophil cytoplasmic CRP C-reactive protein
antibodies CRT-D Cardiac resynchronisation therapy
APKD Autosomal dominant polycystic with biventricular pacemaker
kidney disease and defibrillator
ARB Angiotensin receptor blocker CRT-P Cardiac resynchronisation therapy
ARVD Arrhythmogenic right ventricular with biventricular pacing only
cardiomyopathy CSF Cerebrospinal fluid
ASAS American Society of Abdominal CTPA Computed tomographic pulmonary
Surgeons angiography
ASD Atrial septal defect CVA Cardiovascular accident
ASO Antistreptolysin O CXR Chest X-ray
AST Aspartate aminotransferase DC Direct current
ATN Acute tubular necrosis DIP Distal interpharengeal joints
AVNRT Atrio-ventricular node re-entry DLE Discoid lupus erythematosis
tachycardia DMARD Disease-modifying anti-rheumatic
AZA Azathioprine drugs
BASAI Bath ankylosing spondylitis activity DMPK Myotonic dystrophy protein kinase
index DPLD Diffuse parenchymal lung disease
BCG Bacille Calmette-Gurin dsDNA Double-stranded DNA
BMI Body mass index DVLA Driver and Vehicle Licensing Agency
BNP B-type natriuretic peptide EBV Epstein Barr virus
BOS Bronchiolitis obliterans syndrome ECG Echocardiogram
xv
Glossary

EHRA Electronic health record architecture HOCM Hypertrophic obstructive


EMG Electromyogram cardiomyopathy
ESC European Society of Cardiology HRCT High resolution computed
ESCKD End-stage chronic kidney disease tomography
ESR Erythrocyte sedimentation rate HSMN Hereditary sensory motor
FBC Full blood count neuropathy
FFA Fundus fluorescein angiography HSP Henoch-Schnlein pupura
FNA Fine needle aspiration HTLV-1 Human T-lymphotropic virus type 1
FSH Follicle stimulating hormone HTT Hungtingtin gene
FVC Forced vital capacity IBD Inflammatory bowel disease
GBS Guillain-Barr syndrome ICD Implantable cardiac defibrillator
GGT Gamma-GT ICP Intracranial pressure
GI Gastrointestinal ILD Interstitial lung disease
GMC General medical council INO Internuclear ophthalmoplegia
GORD Gastro-oesophageal reflux disease INR internal normalised ratio
GP General practitioner IPF Idiopathic pulmonary fibrosis
GTN Glyceryl trinitrate idiopathic pulmonary fibrosis
GUCH Grown up congenital heart (disease) ISMN Isosorbide mononitrate
GUM Genito-urinary medicine ITU Intensive therapy unit
HACEK Haemophilus actinobacillus IVIG Intravenous immunoglobulin
cardiobacterium einkella and JVP Jugular venous pressure
kingella LABA Long acting -agonist
HAS-BLED Hypertension (systolic BP > 160 LAMA Long acting muscarinic
mmHg) Abnormal renal function anticholinergic
Abnormal liver function Previous LBT Blastic transformation of
stroke Prior major Bleeding or lymphocytes
predisposition Labile INR (<60% LDH Lactate dehydrogenase
of time in therapeutic range) Age LFTs Liver function tests
>65 (elderly) Drugs predisposing LH Luteinising hormone
to bleeding (antiplatelet agents, LTOT Long term oxygen therapy
NSAIDs) Alcohol use (>8 drinks/ LV Left ventricle
week) LVOT Left ventricular outflow tract
HbA1c Glycosylated haemoglobin MAO Monoamine oxidase
HCC Hepatocellular carcinoma MAZE The Maze procedure open
HDL High density lipoprotein heart procedure requiring
HDU High dependency unit cardiopulmonary bypass
HepA/B/C Hepatitis A/B/C MDT Multi-disciplinary team
HHS Hyperglycaemic hyperosmolar MELD Model end-stage liver disease
syndrome MEN2 Multiple, endocrine neoplasia, type 2
HIV Human immunodeficiency virus MI Myocardial infarction
HLA Human leukocyte antigen MND Motor neurone disease
HNPCC Hereditary non polyposis colorectal MR Mitral regurgitation
cancer MRA Magnetic resonance angiography

xvi
Glossary

MRC Medical Research Council PUVA Photochemotherapy


MRCP Membership of the Royal College of RA Rheumatoid arthritis
Physicians RAPD Relative afferent pupillary defect
MRI Magnetic resonance imaging REM Rapid eye movement
MS Mitral stenosis RNIB Royal National Institute for the Blind
MTP Metatarsophalangeal RNS Repetitive nerve stimulation
MusK Muscle specific kinase RSR Regular sinus rhythm
NASH Non-alcoholic-induced RUQ Right upper quadrant
steatohepatitis RV Right ventricle
NCS Nerve conduction studies SAAG Serum ascetic albumin gradient
NHS National Health Service SBP Spontaneous bacterial peritonitis
NICE National Institute for Health and SCC Squamous cell carcinoma
Care Excellence SCLC Small cell lung cancer
NIV Non-invasive ventilation SCUBA Sub-millimetre common user
NMJ Neuromuscular junction bolometer array
NO Nitric oxide SHBG Sex hormone binding globulin
NOAC Novel oral anticoagulant SHO Senior house officer
NSAIDs Non-steroidal anti-inflammatory SIADH Syndrome of inappropriate
drugs antidiuretic hormone
NYHA New York Heart Association SLE Systemic lupus erythematosus
OA Oesophageal atresia SPECT Single-photon emission
OCP Octacalcium phosphate computerized tomography
OCT Optical coherence tomography SSRI Selective serotonin reuptake
OGD Oesophago-gastro-duodenoscopy inhibitor
OGTT Glucose tolerance test SVC Superior vena cava
PALS Patient Advice and Liaison Service TAVI Transcatheter aortic valve
PAN Polyarteritis nodosa implantation
PANTHER Prednisolone, azathioprine and TB ELISPOT Tuberculosis enzyme-linked
N-acetylcysteine: a study that immunosorbent spot test
evaluates response in idiopathic TBNA Transbronchial needle aspiration
pulmonary fibrosis TEDS Thrombo-embolic deterrent stocking
PAS Periodic acid-Schiff TIA Transient ischaemic attack
PBC Primary biliary cirrhosis TIPS Transjugular intrahepatic
PCP Pneumocystis carinii pneumonia portosystemic shunt placement
PCR Polymerase chain reaction TLCO Carbon monoxide transfer factor
PDA Patent ductus arteriosus TNF Tumour necrosis factor
PEG Percutaneous endoscopic TPMT Thiopurine methyltransferase
gastrostomy TR Tricuspid regurgitation
PEP Post-exposure prophylaxis TSH Thyroid stimulating hormone
PMHx Past medical history TTA Transglutaminase antibodies
PND Paroxysmal nocturnal dyspnea TTE Transthoracic echocardiogram
PSC Pancreatic stellate cells UC Ulcerative colitis
PTH Parathyroid hormone UDCA Ursodeoxycholic acid
PUO Pyrexia of unknown origin US Ultrasound
xvii
Glossary

UTI Urinary tract infection VT Ventricular tachycardia


VDRL Venereal disease research laboratory VZV Varicella zoster virus
VF Ventricular fibrillation WCC White cell count
V/Q Ventilation/perfusion
VSDs Ventricular septal defects

xviii
Introduction: preparation for the exam

The MRCP PACES carousel 1. Physical examination requires a practised


and polished approach. Importantly, the
The PACES carousel consists of five stations,
instruction to the candidate at each station
each lasting 20 minutes, through which
will clearly set out what is required and
candidates progress to encounter a variety of
this should be adhered to as closely as
clinical scenarios. In the first instance, the need
possible. Demonstrating this skill relies upon
to distinguish between the types of stations, and
executing the appropriate examination in a
the nature of their challenges, is paramount to
way that identifies clinical signs while putting
success. Each station requires different skills and
the patient at ease and maintaining patient
demands a tailored approach to demonstrate
welfare (see below)
competency.
2. Identifying clinical signs demands accurate
interpretation of examination findings based
Stations 15 on the foundation of clinical experience and
Candidates can expect to spend 20 minutes at targeted exam preparation
each station. 3. Clinical communication requires articulate,
succinct and skilled use of consultation skills
Station 1 (Chapters 3 & 4)
to engage in discussion with patients and
Respiratory system examination: 10 minutes
examiners alike. The importance of good
Abdominal system examination: 10 minutes
communication skills cannot be emphasised
Station 2 (Chapter 6) enough and forms the foundation of the
History taking skills: 20 minutes framework upon which this book is based
4. Constructing a differential diagnosis
Station 3 (Chapters 1&2) is paramount to demonstrating core
Cardiovascular system examination: 10 minutes knowledge. Appreciating the need to
Nervous system examination: 10 minutes highlight the logic behind the progression
Station 4 (Chapter 7) from physical examination to key clinical
Communication skills and ethics: 20 minutes signs, upon which a list a differential
diagnoses can be based, is crucial
Station 5 (Chapter 5) 5. Clinical judgement places an emphasis
Integrated clinical assessment on clinical prioritisation and the ability to
Two brief clinical consultations: 10 minutes formulate management plans, negotiating
each with the patient where necessary,
rationalising the need for investigation and/
In broad terms the carousel can be broken or treatment
down into three categories of station: 6. Managing patient concerns looking to
The physical examination stations 1 and 3 identify key issues, maintaining an awareness
Communications skills, ethical situations and as to the possibility of sub-plots within
history taking stations 2 and 4 a patient narrative, picking up on verbal
The integrated clinical assessment station 5 and non-verbal cues alluding to fears and
apprehensions
7. Maintaining patient welfare ensuring a
Skills examined patient centred approach where possible,
Having successfully negotiated the initial stages maintaining respect and dignity throughout
of membership, combined with experience
in clinical practice, proficiency in the key In considering each of the three categories of
areas under scrutiny is assumed in candidates station it is possible to appreciate the emphasis
embarking upon PACES. The key to success placed on each of the skills being examined at
lies predominantly in mastering the ability varying points in the exam.
to demonstrate the necessary skills under
exam conditions. It is essential, therefore, to Check the most up-to-date guidance from the
familiarise yourself with the seven areas being Royal College prior to sitting the examination.
examined.
xix
xx Introduction: preparation for the exam

Stations 1 and 3: presenting to be encountered within each station and by


studying relevant chapters in this book, it will
clinical cases in the physical become readily apparent as to how to proceed.
examination stations
Clinical examination demands a structured 2. Follow with relevant negatives
approach, developed throughout undergraduate In deciding upon a predominant positive
medical training and subsequently adopted in finding, it is important to be aware that this
clinical practice, systematically working from will likely lend itself towards a predominant
general inspection in a stepwise progression diagnosis. Indeed, the task thereafter is two
through the hands and peripheral signs fold. First establish how all other positive
towards the target system of interest. Such findings fit with the likely diagnosis or caveat
standardisation of the examination seems towards the possible differential diagnoses. And
intuitive, not least as it lends itself to eliciting secondly, and with equal importance, establish
clinical signs in an organised fashion, applicable the relevant negative findings, which both
to any patient regardless of pathology. Similarly, support the diagnosis and rule out concomitant,
in considering viva questioning, it has long been often related, pathology. Negative findings are
accepted that the structured and organised clinical signs that you would expect to find to
answer, categorising where appropriate, support any given diagnosis, or signs that would
best demonstrates a candidates knowledge be expected if a common complication of a
and encourages the examiner to look more diagnosis were present.
favourably upon them. Thus, in considering
the presentation of clinical findings, we would 3. State the most likely diagnosis
argue that the strong candidate would do well
on the basis of both positive and
to develop a standardised, logical approach,
developing a framework that best demonstrates negative findings
their knowledge. Such a framework Decide upon, and state clearly, your likely
complements the structured approach of clinical diagnosis. To reiterate in adopting this
examination and aids a confident and natural approach and commenting first on both the
progression towards viva questioning, leading positive findings and relevant negatives, the
the examiner to develop a discussion rather than likely diagnosis is one which should already
an interrogation. have been described in all but name. It is
The framework for successful presentation important to understand that patterns of
need not be complicated. In adopting the positive and negative findings are highly
following structure you can transform simple indicative of specific pathologies and so
concepts and fundamental knowledge into an you should be sure that, in establishing a
impressive display of clinical acumen. diagnosis on examination, your presentation
On completion of the physical examination reflects aspects that support and refute such
in stations 1 and 3, take time to conclude the a diagnosis and lead the examiner to a shared
encounter with the patient and make the understanding in a logical manner. A useful
examiner aware of any further bedsides tests you phrase at this point is In summary, this patient
would ordinarily perform at this stage in clinical has signs consistent with a diagnosis of X.
practice. Thereafter:
4. Always proceed to offer relevant
1. Begin with a summary of the differential diagnoses
pertinent positive findings The differential diagnosis is one of the most
The examiner expects a narrative on completion valuable resources in PACES. While it is
of your examination of the patient. That is imperative that you should be sufficiently
not to say that what is required is a running confident in the clinical signs elicited so as to
commentary of each percussion, palpation commit to a diagnosis, thus demonstrating
and auscultation. Prioritise the clinical signs an aptitude for clinical medicine, it is equally
elicited and lead with the predominant finding. important to recognise the potential for
Naturally it takes practice to develop the ability alternative answers to the problem posed.
both to elicit the clinical signs and to decide In everyday practice clinicians proceed
upon their relative importance. However, with with a working diagnosis and sensible list of
some consideration of the likely pathologies differentials when encountering a new clinical
Introduction: preparation for the exam xxi

scenario, relying thereafter on the results of name, alluding thereafter to the expected
investigations in the context of a full history to finding or result. Thus, where aortic stenosis
decide upon a firm diagnosis and subsequent forms the working diagnosis, I would request
management. To assume differently within the an ECG becomes I would request a 12-lead
examination scenario would somewhat miss the electrocardiogram which I would expect to show
point. The differential diagnosis serves both to sinus rhythm, demonstrating left ventricular
demonstrate an understanding of the potential hypertrophy as evidenced by enlarged QRS
diagnoses of any given symptom, and in the complexes within the anterolateral leads, and a
context of the exam, to ensure that marks are strain pattern.
gained for considering the pathology for which At this stage it is important to appreciate
the patient has been selected for examination. the benefits of the framework upon which
Moreover, regardless of whether the actual you are building. Where there is doubt as to
diagnosis forms your working or your differential the underlying pathology suggested by the
option, the importance is placed upon examination findings, you should aim to commit
considering and investigating the possibility, to a working diagnosis, whilst incorporating
thereby ensuring safe practice as a clinician. all other reasonable possibilities as a list of
Decide upon a short, sensible list of differentials. In considering investigations, it
alternative diagnoses, starting with the most is then possible to establish how you would
and ending in the least likely, from which the arrive at a confirmed diagnosis and allows you
subsequent history and investigations will to caveat your working diagnosis. For example,
differentiate between. It will help to place the list where the predominant finding of a systolic
of differential diagnoses in context, for example murmur has resulted in a working diagnosis of
The differential diagnoses of a systolic murmur aortic stenosis, An echocardiogram would prove
would include or The differential diagnosis diagnostic in demonstrating a narrowed valve
for upper motor neuron weakness is. area with increased pressure and a stenotic jet
across the aortic valve, while assessing the mitral
5. Make reference to the importance valve to rule out evidence of regurgitation.
of clinical context a brief history
7. Always proceed to offer a
Briefly list the questions on history that would
most benefit you in determining your diagnosis, management plan
establishing an underlying cause, gauging the Having successfully navigated from
severity of disease, or determining the need examination findings, through diagnoses and
for medical management or intervention. the rationale of investigations the process
While history should not form a large part of reaches a crux. At this point, it is common for
any given presentation, it offers the potential candidates to falter and not proceed to expand
to demonstrate understanding and develop upon the diagnosis, and management thereof,
context, gaining valuable marks. due to concern as to whether they have arrived
Moreover, the accomplished candidate will aim at the correct answer. The strength of this
to demonstrate a holistic approach and include: framework lies in the fact that by discussing
differential diagnoses and investigation first,
Depression screening in chronic conditions
the examiner should be content that in clinical
Seeking to determine the impact of any given
practice such behavior would confirm or refute
condition on the patients quality of life
the working diagnosis and provide a platform
Smoking cessation advice in conditions
on which to base your management. While
where smoking is a prominent modifiable
you are not afforded the results of your chosen
risk factor
investigations, it is safe to assume that you
While on first glance such a list seems can go on to discuss management of a single
cumbersome, rest assured that with practice all condition, regardless of whether the actual
can be covered swiftly and succinctly. diagnosis was correct or lay within one of your
differentials.
6. Demonstrate an understanding In discussing management, it is useful to
consider the distinctions between conservative,
of the value and role of appropriate medical and surgical intervention. While such
further investigation categorisations complement the approach to
As a general rule, an investigation should be management in surgical exams, they can be
referred to using the full and unabbreviated somewhat cumbersome in the medical context.
xxii Introduction: preparation for the exam

That said, they are a useful starting point in Station 2: history taking
all but name. Conservative management
Chapter 6 lays out commonly encountered
should be considered as modifiable risk factors
themes in station 2. The detail included is not
that require lifestyle change such as dietary
intended to serve as a reference text, rather to
advice and smoking cessation. Thereafter, risk
highlight the important principles of each case
factors that require medical treatment such as
and offer insight as to the type of discussion and
blood pressure can be discussed leading the
narrative required to communicate the salient
discussion logically on to pharmacological
points.
management. It is important to note that
conservative management is also often seen as
a watch and wait option, which has a certain Prior to entering the room
implication of doing nothing when really close The candidate information provides details of
monitoring better reflects the rationale. the scenario upon which the station is based,
Most medical specialties have interventional often setting the scene with simple patient
management options such as percutaneous details, the problem or presenting complaint,
coronary intervention (PCI) in cardiology, your role and the environment in which the
transjugular intrahepatic portosystemic encounter takes place. Comparatively, station
shunting (TIPS) in gastroenterology, coiling 2 offers the most detail in the candidate
of subarachnoid haemorrhages in neurology. information with which to prepare prior to
These procedures are considered minimally entering the room. Thus, in each example we
invasive procedures and are held in the offer various approaches to utilise the time
realm of medical management. Confusion afforded to prepare for commonly encountered
as to whether to consider them medical or scenarios. Formulating a plan will prove
surgical is unnecessary; the importance lies invaluable in steering a logical and organised
in understanding their indications for use discussion from the outset and it can be useful
and where applicable their role as either to attempt working through potential lines of
first line treatment or as options only where enquiry in your mind to preempt potentially
pharmacological therapy as failed. difficult areas. Doubtless the patient will
introduce the occasional theme or topic that will
In summary not fit with the best-laid plans but this stresses
the importance of avoiding assumptions and
You should aim to present the salient positive
remaining open to the patients narrative as the
examination findings, avoiding a long list of
station progresses.
normal findings, but including important
It is worth taking the time to consider
negatives. With sufficient preparation and
your individual approach to station 2. Some
practice it will become easier to establish a
candidates choose to make use of paper and
working diagnosis that will form the basis
pen, which is provided, to jot down important
of a differential list. Thereafter, appropriate
phrases from the scenario, possible differential
investigation should reinforce the likely
diagnoses, or pertinent questions to use as a
diagnosis upon which to form a management
prompt when faced with the patient. This can
plan. You should then proceed to give a
be useful but if you do not normally write your
brief and fact-laden prcis of the most likely
history as you talk with patients then dont
diagnosis, and suggest initial management.
suddenly start on the day of the exam! Be aware
This framework encourages a logical
of the risk of failing to establish a rapport with
progression for discussion of any patient
the patient at the cost of focusing on your
within the examination setting. The proposed
penmanship. Indeed, where you intend to use
structure, expanded upon in this book,
notes ask the patient if it is acceptable to write as
offers a framework that makes it possible to
you talk and thereafter be sure not to use it as a
talk for several minutes in an authoritative
crutch. Focus on maintaining eye contact, keep
and insightful manner. It is likely that any
notes short and try to avoid reading off them
subsequent viva questioning will either be
verbatim when dealing with the examiner.
directed towards clarification of points of
interest from your discussion or the examiner
may choose to push the boundaries of your On entering the room, begin with
knowledge on any given topic. Regardless, this open questioning
approach allows you to determine, in large part, The encounter with the patient is allotted
the content of the discussion. 14 minutes, with a 2-minute warning from
Introduction: preparation for the exam xxiii

the examiner at 12 minutes. Thereafter, you Explore ideas, concerns and


may choose to take time to reflect upon the
information gathered and organise their
expectations
thoughts before engaging in a discussion with Actively determining and appropriately
the examiner to conclude the station. The exploring ideas, concerns, and expectations is
importance of stressing the timings of the station central to reaffirming the patients agenda and
at this point is to highlight the considerable uncovering other themes requiring discussion.
amount of time made available to interact with Often patients will have preconceptions as to
the patient. the cause of their symptoms, or have a diagnosis
On entering the room, introduce yourself at the forefront of their mind. Understanding
to the patient and reaffirm your role to them the ideas or beliefs upon which these
as outlined in the scenario. Thereafter, begin preconceptions are based is vital to achieving
with an open question and allow the patient to a shared understanding of the problems
respond, uninterrupted, without directing the encountered. Often, exploring ideas will involve
patients response. Avoid interrogating with unpicking a web of information from previous
closed questioning from the outset, instead consultations, family members, or research from
focus on establishing a rapport and employ search engines and internet sites.
attentive listening. Your aim is to make the Patient concerns or worries often relate to the
patient feel at ease, and facilitate the passage of impact of symptoms or a particular diagnosis
information with reassuring gestures, echoing on quality of life, ability to work, or relationships
important statements, gentle steering and with loved ones or family members. It is
repeated open questioning, underpinned with important in this respect to attempt to gain an
displays of empathy and sensitivity in response understanding of the impact of each problem
to appropriate cues. The initial stages should from the patients perspective. The stigma
be used to allow the patient to unburden him surrounding any given diagnosis is also a
or herself of information and get a feel for the commonly encountered cause for concern.
agenda and themes of the station. Finally, having spent time gathering the
Choosing when to draw a line under open relevant history, establishing a patients
questioning will depend upon the nature of the expectations can be useful in transitioning
information and readiness to proceed along a to a management plan. Incorporating their
particular line of enquiry, the willingness of the expectations from the consultation, goals in
patient to continue to talk openly without the relation to treatment, or envisaged outcomes,
need for continual encouragement, or where forms the basis of shared forward planning.
the information becomes seemingly tangential.
The transition to closed questioning is often What to tell the patient
best achieved by interjecting with a summary Summarise the case as you understand it and
of what the patient has said. Reiterating the formalise a plan considering the need for:
key points, as you understand them, allows the
patient to clarify or add further information Immediate treatment for symptom relief
as necessary. It also serves as a signpost to the The need for further investigation
examiner that you are shifting the emphasis of The need for a period of trial intervention/
the consultation. treatment before review
The need for future consultation to explore
Proceed to closed questioning the problem in more depth, with more time,
after research
Closed questioning should be used to confirm key The need for specialty referral
aspects in the history relating to such areas as: It is not uncommon to be confronted by areas
Disease severity and/or classification of medicine in which you are not well versed,
Risk factors or lack knowledge. In such a situation it is not
Recognised complications unreasonable to admit to the fact and suggest
Relevant past medical history and family a plan in which the information is obtained or
history confirmed with a senior colleague and relayed
Review of medication to the patient at a later date. The premise is that
Consideration of quality of life and social in clinical practice this would be a reasonable
history course of action and places the importance
on patient safety and the provision of accurate
xxiv Introduction: preparation for the exam

information. It should be stressed that this is not admissions. The brief clinical consultation
advocated as an acceptable approach to disguise format challenges candidates to demonstrate
failings in core medical knowledge! a combination of both problem solving
The overall content and detail of the and communication skills, underpinned by
discussion with the patient should focus sound clinical judgment. Indeed, it is the
upon ensuring core understanding of the only station in which all seven areas of the
most important take home messages, using marking scheme are assessed at one time.
layman terminology and avoiding medical As such a significant proportion of the total
jargon. This is a key skill in of itself, vital to a marks available in the exam are attributed
patient-centred approach, which is increasingly to station 5. It is important, however, not to
favoured in these scenarios. Indeed, negotiating over-prepare for this station simply because
a management plan, with the patient at the of the allocation of marks. The foundation of
centre of the decision making process, should a strong performance in station 5 stems from
be prioritised over an in depth discussion of thorough preparation for all other stations
the battery of available investigations and and relies upon successfully adapting the
treatments. approach to demonstrate an amalgamation of
the skills inherent to each.
What to tell the examiner In covering station 5, Chapter 5 focuses
upon scenarios drawn from rheumatology and
Impressing upon the examiner an appreciation
musculoskeletal medicine, endocrinology,
of the pertinent points from the encounter
ophthalmology and dermatology reflecting the
upon which the investigation and management
areas of medicine previously examined prior
plans have been built, will lay the foundations
to the introduction of the new format of station
for discussion of relevant themes. Be prepared
5. However, the nature of the station is such
to expand upon the details of specific issues
that conceivably any area of medicine could
touched upon in the history, including wider
be incorporated. Thus we encourage you to be
aspects such as prognosis and relevant follow up
mindful not only of the content presented but
or specialist input.
also the approach adopted in each scenario.
Developing a systematic framework upon
Station 4: applied communication which to approach this station will equip you to
skills and ethical scenarios face themes from any area of medicine.
To successfully navigate this station requires a
fundamental appreciation of the core principles
Performing a focused history and
involved for any given scenario. Chapter 7 aims examination
to provide you with a prcis of those principles The history and examination should not be
for commonly encountered situations. The considered as separate entities and a stepwise
exact nature of the station will depend upon the approach, taking the history followed by
context of the encounter and the interaction an examination, should be avoided if at all
with the patient. Preparing for a range of patient possible. Instead, begin the dialogue with
types by role-playing with peers, is a useful an open question that establishes the main
exercise in the lead up to the exam. Practising area of concern allowing the early initiation
applied communication skills through role-play of a relevant examination. Thereafter, the
should aim to explore ways of explaining the consultation can continue with an integrative
same information in different ways. Attempting approach, taking the history and examining
to predict the reaction of the angry versus the patient in parallel. With the identification
the withdrawn, depressed, patient or similar of physical signs, targeted questioning should
dichotomies in personalities or behaviours. be employed to develop context and confirm
or refute working diagnoses. Alternatively,
the history may suggest additional areas
Station 5: integrated clinical or systems worth examining. It is through
assessment appropriate questioning and choice of relevant
Station 5 represents a global assessment examination(s), with the potential need to
of applied clinical skills, centred upon prioritise breadth of interrogation at the
commonly encountered problems in daily expense of depth that forms the basis of an
practice on the wards or acute medical assessment of clinical judgment.
Introduction: preparation for the exam xxv

What to tell the patient Exploring the history and examination


findings and how they correlate to diagnosis/
Summarising the case, clearly explaining any
differential diagnosis offered
examination findings and relating them to a
patients symptoms where possible, is important
Evaluating patient understanding and
efficacy of your interaction particularly in
in the first instance. Thereafter be sure to check
answering questions, allaying fears or dealing
the patients understanding and answer any
with areas of concern
questions that arise. The discussion with the
patient should resemble the conclusion of
Justifying the need for further investigation
and appropriate use of follow up
the history station and focus upon agreeing a
arrangements
management plan.
Exploring management strategies
What to tell the examiner As such, succinctly presenting the
The discussion with the examiner is likely to examination findings in the context of the
revolve around: history is fundamental and requires practice to
whittle the case down to the pertinent points.
Chapter 1

Cardiovascular system
(station 3)
Case 1: Aortic stenosis

Instruction to the candidate Soft or absent S2


This 70-year-old man has been complaining
Narrow or reverse split S2 (as A2 is
increasingly delayed)
of worsening shortness of breath on exertion.
Please examine his cardiovascular system and
Timing of the ejection systolic murmur:
peak of the murmur is delayed with
present your findings to the examiner including
increasing severity. Importantly, the
a discussion of your proposed management.
intensity of the murmur does not correlate
with severity
Begin with a summary of Presence of S4
positive findings Peripheral signs may include:
The auscultatory features leading to a working Slow-rising pulse (parvus et tardus: a
diagnosis of aortic stenosis include: diminished volume, delayed carotid upstroke)
A crescendo-decrescendo ejection systolic Narrow pulse pressure: not detectable by
murmur, beginning after S1, peaking in mid- palpation, but should be requested from
late systole, and ending before S2 (grade the the examiner when commenting on blood
loudness, Table 1.1) pressure measurement. A narrow pulse
Most prominent in the right second pressure is a sign of severe aortic stenosis
intercostal space loudest in end-expiration Aortic thrill
with the patient sitting forward Sustained, heaving undisplaced apex beat. In
Equal radiation to the carotids severe disease the left ventricle may become
The murmur can radiate to the apex dilated and thus the apex beat will become
(termed the Gallavardin phenomenon: displaced and harder to palpate with a
high frequency components of the ejection weaker dispersed impulse
systolic murmur radiating to the apex,
falsely suggesting mitral regurgitation). This Follow with a summary of
demonstrates that while the character and
timing of the murmur with respect to the relevant negative findings
cardiac cycle are consistent, anatomical A common statement used in the cardiology
characteristics, such as the radiation, can station is the patient did not display signs of
vary and hence are not always reliable cardiac failure. Such a statement relies upon
An ejection click after S1 indicates a pliable an assessment of fluid status as evidenced by
bicuspid valve auscultation of the lung fields, height of the
jugular venous pressure (JVP), and presence/
The following auscultatory features are
absence of peripheral oedema. If a patient with
consistent with severe aortic stenosis and should
valvular pathology has signs of cardiac failure
be commented upon where detected:
this potentially relates to the inability of the

Table 1.1 The grading of murmurs


Grade Description

1 Faintest murmur, only heard after special effort

2 Soft but readily detected murmur

3 Moderately loud murmur with no thrill

4 Loud murmur with an accompanying thrill

5 Very loud murmur heard with just the rim of the stethoscope held against the patients chest

6 Loudest murmur heard with the stethoscope held just away from the chest wall
Case 1 Aortic stenosis 3

heart to compensate for the valvular pathology for associated bicuspid aortopathy,
thus referred to as decompensation. namely dilatation, dissection, aneurysm
There was no suggestion of associated formation and coarctation. The murmur
valvular pathology. In this case remember to of coarctation is best heard on the back in
comment on mitral regurgitation or, where there the interscapular area. 30% of patients with
is the suspicion of a bicuspid valve (see below), Turners syndrome will have a bicuspid
aortic regurgitation. aortic valve
Whenever valvular pathology is suspected Subvalvular: Subaortic membrane
the need to comment on any peripheral stigmata Supravalvular: Williams syndrome: both
consistent with infective endocarditis is necessary, aortic and pulmonary stenoses are associated
commenting on common sources including with this syndrome. Patients also have
dentition and indwelling intravenous access. transient hypercalcaemia, elfin-like facies
and learning difficulties with overly sociable
persona
State the most likely diagnosis
on the basis of these findings Acquired:
This patient has signs consistent with severe aortic Degenerative (the commonest cause in
stenosis, likely secondary to calcific degeneration. Western countries)
He is clinically euvolaemic, displaying no features Calcific, accelerated by:
of decompensated disease. He does not have any Chronic kidney disease
features of infective endocarditis. Severe Pagets disease (in patients with
more than 15% skeletal involvement, high
output state from bony arteriovenous
Offer relevant differential connections causes increased
diagnoses transvalvular turbulence and accelerated
The differential diagnosis can be considered in valvular calcification)
relation to the clinical sign, that of an ejection Hyperparathyroidism (aortic valve
systolic murmur, and relating to the aetiology of inflammation and calcification)
aortic stenosis itself. Rheumatic fever (often with associated mitral
involvement)
Differential of an ejection systolic murmur: Homozygous familial hypercholesterolaemia

Aortic stenosis
Pulmonary stenosis Demonstrate the importance
Aortic sclerosis (calcified valve, with no of clinical context suggest
stenosis or impediment to flow hence no
radiation to the carotids nor impact upon
relevant questions that would be
pulse pressure) taken in a patient history
Subvalvular (hypertrophic cardiomyopathy Seek to elicit the cardinal symptoms that relate
with left ventricular outflow tract obstruction, to survival and suggest severe disease:
subaortic membrane) or supravalvular
(Williams syndrome) lesions Asymptomatic: sudden death < 1%/year
Flow murmur Dyspnoea secondary to heart failure: up to 2
Aortic stenosis is the most common valve years survival
lesion in Europe and North America with calcific Syncope: average 3 years survival
degenerative disease that occurs with advancing Angina: average 5 years survival
age (prevalence of 27% of individuals over Establish exercise tolerance. It can be
65 years of age) the leading cause. Congenital useful to make reference to the vignette, which
valvulopathy (bicuspid or extremely rarely may describe symptoms as in this case with
unicuspid) which presents in younger individuals shortness of breath on exertion. Beware of
is the second most common aetiology. Thus, asymptomatic patients who may be limiting
the age of the patient may provide a clue to the their activities to avoid symptoms.
underlying pathology. Alternatively, consider the
aetiology as congenital or acquired:
Demonstrate an understanding of
Congenital:
the value of further investigation
Valvular: Bicuspid aortic valve (found in The three investigations to prioritise and discuss
2% of the population). Mention looking first are the electrocardiogram (ECG), plain
4 Chapter 1 Cardiovascular system (station 3)

chest radiograph (CXR) and transthoracic and suitability for peripheral arterial cannulation
echocardiogram (TTE). in patients in whom TAVI is being considered.
ECG: Left ventricular hypertrophy with or
without accompanying strain pattern, Transcatheter aortic valve
i.e. ST depression and T wave inversion implantation (TAVI)
in the lateral leads.
CXR: Look for aortic calcification, which TAVI is becoming a widely utilised interventional
in severe diffuse cases is termed procedure in patients with severe aortic
a porcelain aorta. This precludes stenosis who are either declined conventional
cannulation or clamping of the surgical aortic valve replacement during patient
ascending aorta during cardiothoracic selection or are considered high risk. It has been
surgery. demonstrated to provide both survival and quality
TTE: The gold-standard diagnostic of life benefits versus conservative management
investigation demonstrating: in inoperable patients with severe AS. Patient
selection is key, and a multidisciplinary approach
Transvalvular gradients and aortic valve area amongst non-invasive and invasive cardiologists
(Table 1.2) and cardiothoracic surgeons is essential. It should
Degree of calcification, and tricuspid versus also be remembered that TAVI is still an invasive
bicuspid valves procedure requiring a general anaesthetic, large
Left ventricular wall thickness, systolic and bore vascular access and temporary cardiac
diastolic function pacing. Stroke is a major complication. Other
Aortic root dimensions and associated complications include vascular injury, coronary
aortic pathology (especially important in the embolisation, device embolisation, paravalvular
context of bicuspid valves) leak and death.
Stress testing: Asymptomatic severe
aortic stenosis can be further evaluated Always offer a management plan
with a medically supervised exercise test,
Symptomatic, severe AS requires aortic valve
watching closely for symptoms, and abnormal
replacement. Aortic valve replacement is also
haemodynamic response to exercise or
indicated in moderate AS in the context of
arrhythmia. Discrepancies between transvalvular
surgical coronary revascularisation or other
gradients and aortic valve area do occur
concomitant cardiovascular surgery.
and require further specialist investigation.
Asymptomatic mild to moderate aortic
For example, low dose dobutamine stress
stenosis should receive outpatient follow-up to
echocardiography is used in cases of severely
monitor the natural history of the condition.
stenotic aortic valve area with low transvalvular
In patients with severe aortic stenosis avoid
gradients in the presence of left ventricular
medications that reduce preload (such as ACE
dysfunction. Dobutamine increases myocardial
inhibitors or nitrates), thereby reducing venous
contractility, revealing either a falsely low
return and exacerbating transvalvular gradients.
resting aortic valve area due to left ventricular
impairment (with increasing valve area on stress)
Asymptomatic severe aortic stenosis:
or increasing gradients (but static valve area) on
stress due to true aortic stenosis. Truly asymptomatic patients should be
Cardiac catheterisation: for investigation closely followed-up, with the caveat that
of concomitant coronary artery disease in symptom onset should lead to surgical
pre-operative patients. intervention being expedited
CT aorta and CT coronary angiography: A Asymptomatic patients with an abnormal
non-invasive assessment of coronary anatomy supervised exercise test (e.g. positive for

Table 1.2 Echocardiographic findings in aortic stenosis


Mild Moderate Severe
2)
Valve area (cm 1.52.0 1.01.4 <1.0

Peak gradient (mmHg) <36 3664 >64


Mean gradient (mmHg) <25 2540 >40
Case 2 Mitral regurgitation 5

symptoms or a fall in blood pressure) require Further reading


aortic valve replacement
Vahanian A, Baumgartner H, Bax J, et al. Guidelines
Aortic valve replacement is also indicated
on the management of valvular heart disease:
in asymptomatic patients with evidence of
the task force on the management of valvular
impaired left ventricular systolic function
heart disease of the European Society of
(due to the severe AS) or a rapid rate of
Cardiology. Eur Heart J 2007; 28:230268.
progression of severity of AS (defined by
increasing peak velocities 0.3 m/s per year)

Case 2: Mitral regurgitation

Instruction to the candidate A mid-systolic ejection click followed by a


This 35-year-old man presents with dyspnoea. mid- to late-systolic murmur
Please examine his cardiovascular system. The click results from sudden tensing of the
mitral valve apparatus bowing back into the
left atrium
Begin with a summary of Certain manoeuvres can alter the auscultatory
positive findings characteristics of mitral valve prolapse:
Peripheral signs in mitral regurgitation includes: The murmur is prolonged by reducing
venous return (e.g. standing or Valsalva),
The pulse may be irregular, suggestive of thereby reducing left ventricular filling
atrial fibrillation and thus increasing chordal laxity causing
Palpate at the apex for a displaced (clearly earlier prolapse and an earlier click (with a
refer to anatomical landmarks), thrusting longer subsequent murmur)
(volume-loaded) apex beat, which may be Squatting increases venous return,
associated with a systolic thrill ultimately delaying the click and reducing
Auscultatory features include: the duration of the murmur
A pan-systolic murmur heard loudest at
the apex in expiration in the left lateral
position, radiating to the axilla Follow with a summary of
S3 gallop relevant negative findings
Signs of severity include: The presence or absence of the following
findings should be mentioned to the examiner:
Left ventricular dilatation indicated by a
displaced apex beat An irregular pulse suggestive of atrial
Signs of left ventricular failure fibrillation. Comment on the rate, embolic
A wide split S2 (an early A2 indicating complications and any peripheral stigmata of
premature aortic valve closure due to a large anticoagulation that may be present
regurgitant volume across the mitral valve) Peripheral stigmata of infective endocarditis.
Ask the examiner for the temperature
and a urine dip (looking for microscopic
Mitral valve prolapse haematuria), and comment on dental
hygiene and the presence or absence of
Mitral valve prolapse is most commonly due to
indwelling intravenous access
myxomatous degeneration, but also occurs with
inherited connective tissue diseases including
Signs of cardiac failure, including pulmonary
oedema, peripheral oedema, raised JVP and
Marfan syndrome, EhlersDanlos syndrome,
S3 gallop, which if present would indicate
pseudoxanthoma elasticum and osteogenesis
cardiac decompensation
imperfecta. The auscultatory features of mitral
valve prolapse include:
6 Chapter 1 Cardiovascular system (station 3)

State the most likely diagnosis include breathlessness, orthopnoea, paroxysmal


nocturnal dyspnoea, and pedal oedema. Any
on the basis of these findings history of palpitations should also be elicited,
The most likely diagnosis in this patient, who specifically considering the increased likelihood
has a pan-systolic murmur and is in atrial of AF.
fibrillation, is mitral regurgitation.
Demonstrate an understanding
Offer relevant differential of the value of further
diagnoses investigation
The differential diagnoses of a pansystolic
The most important investigations and their
murmur include:
relevant findings include:
Mitral regurgitation
A 12-lead electrocardiogram (ECG):
Ventricular septal defect
P mitrale (broad, bifid P waves) of left
Tricuspid regurgitation
atrial dilatation
Aortic stenosis can be mistaken for mitral Atrial fibrillation
regurgitation, especially when it radiates to the Look for ischaemic changes as a possible
apex (termed the Gallavardin phenomenon). aetiology of mitral regurgitation
The mitral valve consists of 2 leaflets A plain chest radiograph (CXR):
(3 scallops each) contained within an annulus, Cardiomegaly due to left ventricular
anchored by chordae tendinae to papillary dilatation
muscles that originate from the myocardium. Evidence of left atrial dilatation
The differential diagnosis of the underlying Pulmonary oedema
cause of mitral regurgitation can therefore be Echocardiography: both transthoracic and
considered in terms of disorders of these various transoesophageal echocardiography are
parts of the valve apparatus. Clues suggesting important. They will demonstrate:
such aetiology can be identified during the The severity of the valve lesion:
clinical examination (Table 1.3). determined by colour Doppler,
regurgitant orifice area, regurgitant
volumes and fractions, and by jet width
Demonstrate the importance Clues which point towards the
of clinical context suggest mechanism of the valve dysfunction:
looking at all structures that make up the
relevant questions that would be mitral valve apparatus
taken in a patient history The physiological consequences of
As with any valve pathology, it is important the valve lesion, namely regarding
to identify symptoms of heart failure. These left ventricular cavity size and systolic

Table 1.3 Aetiology of mitral regurgitation and associated clinical signs


Acute Chronic
Leaflets Endocarditis: review Degenerative (the most common cause)
dentition, intravenous Valvotomy for previous MS: check for scars
access, peripheral MV prolapse: check for mid systolic click and movement with manoeuvres
stigmata Rheumatic fever: check for concomitant MS
Connective tissue diseases: such as rheumatoid arthritis, check the joints
Anorectic drugs (fenfluramine, phentermine)
Annulus LV dilatation
Subvalvular Ruptured papillary Flail chordae
apparatus muscle during an
(chordae/ acute ischaemic event
papillary muscles) Flail chordae
Left ventricular Chronic ischaemia (circumflex territory causing lateral wall
posterior wall hypokinesis and hence posterior mitral valve leaflet tethering)
MS, mitral stenosis.
Case 2 Mitral regurgitation 7

function. Systolic function should be The indications for surgery (preferably repair,
hyperdynamic if considered normal in otherwise replacement) in mitral regurgitation
severe MR, as the left ventricle offloads vary depending on whether the patient is
into both the aorta and the left atrium. symptomatic or asymptomatic:
Left atrial dimensions and pulmonary
artery systolic pressure are also evaluated
Symptomatic severe MR: surgery is indicated.
In cases of significant left ventricular
To guide the nature of surgical dysfunction (ejection fraction 30%, end-
intervention and the feasibility of repair
systolic diameter >5.5 cm), response to
versus replacement
medical therapy, likelihood of valve repair,
Cardiac catheterisation acts to investigate
and comorbid burden are all considered
concomitant coronary artery disease in pre-
prior to a making a decision for surgical
operative patients, or to investigate a suspected
intervention
ischaemic aetiology of mitral regurgitation
Asymptomatic severe MR: asymptomatic
patients with severe MR and normal
Always offer a management plan left ventricular size and function can be
The management of mitral regurgitation can followed up, but certain cases should be
be considered with regard to the medical and considered individually, e.g. females of
surgical options: child-bearing age with severe MR who
When discussing medical management the want to consider pregnancy (and thus
indications for specific therapies should be expose the cardiovascular system to
considered: increased circulating volume). Otherwise,
in asymptomatic MR, surgery is indicated in
Antibiotic prophylaxis: (see Infective the following circumstances:
endocarditis, p.40) Resting left ventricular ejection fraction
In acute severe MR, nitrates, diuretics, and 60%
inotropes (to optimise prior to surgery) may Left ventricular dilatation (end-systolic
be used diameter > 4.5 cm)
In chronic severe MR with evidence of left Patients with AF
ventricular dysfunction: ACE inhibitors, beta- Patients with resultant pulmonary
blockers and potassium-sparing diuretics in hypertension
accordance with conventional cardiac failure Patients undergoing other concomitant
management cardiovascular surgery, such as coronary
Patients with AF should receive anticoagulation artery bypass grafting (CABG) or other
and rate control. Pursuit of sinus rhythm valve surgery
following AF in severe MR is unlikely to be In selected cases depending upon anatomical
successful in the long-term without correction suitability and patient profile, there are
of the underlying valvular lesion percutaneous options for the treatment
of severe MR. These would be considered
following specialist referral, and include the
Mitral regurgitation and its various
Mitraclip and Cardioband devices.
presentations
Remember that MR can be acute, chronic and Further reading
compensated, or chronic and decompensated. In
PACES you will see the latter two presentations, Vahanian A, Baumgartner H, Bax J, et al. Guidelines
but remember to mention during your present on the management of valvular heart disease:
that cases of acute MR present as a medical the task force on the management of valvular
emergency with pulmonary oedema. heart disease of the European Society of
Cardiology. Eur Heart J 2007; 28:230268.
Case 3: Ventricular septal defect

Instruction to the candidate Follow with a summary of


This 35-year-old woman feels well and is under relevant negative findings
cardiology follow-up. Please examine her The following represent clinical complications of
cardiovascular system. a VSD and it is, therefore, important to comment
on the absence of the relevant clinical findings:
Clinical context of ventricular septal Infective endocarditis: comment on the
defects (VSDs) absence of peripheral stigmata, dental
hygiene and the presence or absence of
VSDs are the second most commonly encountered
indwelling intravenous access
congenital cardiac lesion after bicuspid aortic
valves.
Arrhythmia: either tachyarrhythmias,
or bradyarrhythmias from complete
atrioventricular block
Begin with a summary of Signs of pulmonary hypertension
positive findings Signs of left ventricular failure
Cerebral (paradoxical) embolism due to the
The positive clinical findings in a patient with a right and left heart communication
VSD include: Concomitant AR due to prolapse of the right
coronary cusp can occur particularly in
Peripheral signs perimembranous VSDs
The morphologic features of known disease
It is also important to comment on
associations:
associated intracardiac lesions such as patent
Downs syndrome. ductus arteriosus, tetralogy of Fallot, and aortic
Turners syndrome coarctation.
The finding of a thoracic scar can be
indicative of a childhood surgical closure
A systolic precordial thrill may be felt on State the most likely diagnosis
palpation
on the basis of these findings
The following is an example presentation
Auscultatory features
to the examiner, relating to a patient with
A pansystolic murmur loudest at the left sternal a haemodynamically insignificant and
edge between the 3rd and 4th intercostal spaces. uncomplicated VSD:
Note that the loudness of the murmur is not
indicative of shunt size but rather of the pressure This patient has signs consistent with a
gradient. ventricular septal defect, likely a small,
To provide further clinical context, consider haemodynamically insignificant left-to-right
the various ways in which an adult can present shunt (Maladie de Roger). She has no evidence
with a chronic VSD: of pulmonary hypertension, left ventricular
failure or infective endocarditis. I would like to
Surgically repaired without residual shunt take a full history to establish her symptomatic
With residual shunt, which can then lead to status, and my investigation of choice would
the following groups: be an echocardiogram in the first instance to
A well, young adult with a likely confirm the diagnosis, to quantitate the degree
haemodynamically insignificant muscular of shunting and to estimate left ventricular
VSD, with a large pressure gradient, function and pulmonary arterial pressures.
and hence a loud murmur. (Maladie de
Roger)
An adult patient with varying degrees of Offer relevant differential
pulmonary arterial hypertension and/or diagnoses
left ventricular volume overload
The differential diagnosis of a VSD can be
A cyanotic adult patient who has considered either in relation to the underlying
developed Eisenmengers physiology
Case 3 Ventricular septal defect 9

causes of VSDs, or to their anatomical The location and dimensions of the shunt
classification. Quantification and direction of the shunt
The causes of a VSD include: ratio, and the pressure gradient between
the ventricles
A cardiac lesion as part of an underlying
An estimation of pulmonary arterial
syndrome (Downs or Turners syndrome)
systolic pressure
A congenital lesion, either alone or
Indices of left and right ventricular size
associated with other lesions (such as in
and function
tetralogy of Fallot)
Ischaemic (acquired) VSDs, which typically
Any associated lesions, such as aortic
regurgitation, which can be present with
occur approximately 5 days following a
perimembranous VSDs
full-thickness septal myocardial infarction
Cardiac catheterisation is not routinely
Anatomically, VSDs are divided as follows: required, but can be used to determine
pulmonary vascular resistance if pulmonary
Perimembranous (80%) arterial pressure is suggested to be raised
Muscular (1520%) on echocardiography, and for shunt
Outlet (5%) quantitation
Inlet (<1%) Cardiac MRI can be used for further
evaluation of ventricular volumes,
Demonstrate the importance function and shunt quantification where
echocardiography has not been diagnostic
of clinical context suggest
relevant questions that would be Always offer a management plan
taken in a patient history Patients with congenital lesions should
When informing the examiner of relevant be referred to a specialist adult congenital
questions to ask in a history, include those heart disease (ACHD) centre for follow-up.
that would identify symptoms of heart failure Additionally, consider advice on antibiotic
and pulmonary hypertension. Also remark on prophylaxis (see Infective endocarditis, p.40).
questions that would shed light on the aetiology The definitive management is a VSD closure
and previous management of the patients VSD. if there is evidence of LV dilatation and/
or failure. Surgical closure is the method of
Demonstrate an understanding choice, but the procedure can also be carried
out in a percutaneous fashion if the patient
of the value of further is considered too high-risk or unsuitable for
investigation surgical, or in the case of an anatomically
suitable muscular VSD.
The following list should be mentioned to
Patients with symptoms, left ventricular
the examiner as relevant investigations in the
volume overload, mild pulmonary hypertension,
evaluation of patients with a VSD:
or a history of recurrent (more than three
An ECG may demonstrate episodes) infective endocarditis should all be
electrocardiographic findings of left, right, or considered for closure.
combined ventricular hypertrophy, depending Closure is contraindicated at the extremes
on the size and direction of the shunt. If the of the symptomatology spectrum, such as
VSD is small, a normal ECG can be expected patients who have developed Eisenmengers
A CXR may demonstrate the following syndrome, or asymptomatic patients with small,
radiographic findings: haemodynamically insignificant lesions with
Cardiomegaly due to left ventricular no evidence of left ventricular volume overload.
dilatation Severe pulmonary hypertension is also a
Evidence of left atrial dilatation contraindication to closure.
An enlarged main pulmonary artery,
with reduced peripheral vascular
markings in patients with pulmonary
Further reading
hypertension Deanfield J, Thaulow E, Warnes C, et al. Management
Echocardiography, the gold-standard of grown up congenital heart disease. Eur Heart
diagnostic investigation, will demonstrate: J 2003; 24:10351084.
Case 4: Hypertrophic obstructive
cardiomyopathy

Instruction to the candidate Valsalva can be performed in the context of


This 32-year-old man complains of palpitations diagnostic studies for these purposes, but
and exertional dyspnoea. Please examine his should not be performed as part of the clinical
cardiovascular system. examination in PACES given the potential for
symptomatic consequences.

Begin with a summary of


positive findings Follow with a summary of
The peripheral signs of hypertrophic
relevant negative findings
cardiomyopathy include: The absence of the following clinical signs
should be mentioned in cases of hypertrophic
A bifid (double) carotid pulse. This physical cardiomyopathy:
finding represents flow acceleration in early
systole, followed by a second impulse in late Signs suggestive of cardiac failure. Cardiac
systole failure may be due to diastolic dysfunction,
The jugular venous pressure will be raised or in the later stages of the natural history of
if the patient is in cardiac failure, with the disease, due to systolic dysfunction
prominent a waves, which indicate right The finding of an irregular pulse, suggestive
atrial systole against a reduced compliance of atrial fibrillation, is a marker of poorer
right ventricle due to the impact of the prognosis
hypertrophied interventricular septum Infective endocarditis: comment on the
Precordial palpation may reveal a double absence of peripheral stigmata, dental
apical impulse, represented by a heaving hygiene and the presence or absence of
(pressure-loaded) apex beat with a palpable indwelling intravenous access
S4 due to forceful atrial systole against a non-
compliant left ventricle State the most likely diagnosis
Auscultatory features of hypertrophic on the basis of these findings
cardiomyopathy include:
The following quotation relates to a standard
An ejection systolic murmur at the left sternal case of hypertrophic cardiomyopathy:
edge in mid to late systole with no radiation
This patient has signs consistent with
to the carotids
hypertrophic obstructive cardiomyopathy.
A pansystolic murmur of mitral regurgitation
The ejection systolic murmur suggests a left
due to systolic anterior movement of the
ventricular outflow tract gradient, and the
anterior mitral valve leaflet
apical pansystolic murmur indicates mitral
A fourth heart sound (S4)
regurgitation secondary to systolic anterior
motion of the anterior mitral valve leaflet.
Auscultation in a HOCM case I would like to take a full history to establish
his symptomatic status, a family history to
The left ventricular outflow tract (LVOT) gradient establish pedigree of the disease, and my
and thus the loudness of the ejection systolic initial investigation of choice would be an
murmur vary directly with preload and afterload: echocardiogram.
Increasing preload by squatting or afterload
by the handgrip manoeuvre reduces the Offer relevant differential
gradient, with a consequently softer murmur
Reducing preload by the Valsalva manoeuvre, diagnoses
standing from sitting, or use of diuretic or A suitable differential diagnosis that can be
nitrate medications, or reducing afterload by given to the examiner is that of an ejection
administration of vasodilators will exacerbate systolic murmur as given in Aortic stenosis (p. 2).
the gradient across the LVOT, resulting in a
louder murmur
Case 4 Hypertrophic obstructive cardiomyopathy 11

Demonstrate the importance The presence and quantitation of LVOT


obstruction
of clinical context suggest The presence and quantitation of mitral
relevant questions that would regurgitation due to systolic anterior
motion of the anterior mitral valve leaflet
be taken in a patient history The absence of other conditions that
Relevant history questions to mention to the might give rise to left ventricular
examiner include direct questioning to elicit the hypertrophy, such as aortic stenosis or
following important symptoms: cardiac amyloid
Palpitations Holter monitoring can show the following
Pre-syncope or syncope findings:
Exertional dyspnoea Atrial and/or ventricular ectopy
Chest pain Atrial fibrillation
Non-sustained ventricular tachycardia
A family history of unexplained syncope or On supervised treadmill exercise testing,
sudden, unexplained premature death should an attenuated or hypotensive response is
be sought. Many mutations responsible for indicative of haemodynamic instability and a
hypertrophic cardiomyopathy are of autosomal risk factor for sudden death
dominant inheritance, so siblings and children Cardiac MRI can be used for diagnosis and
have a 50% likelihood of being affected. A clear assessment of hypertrophic cardiomyopathy,
family pedigree is important with respect to especially when echocardiography has
establishing the diagnosis and the investigation been inconclusive (particularly in cases
and treatment of relatives. of apical hypertrophy). It is also useful in
The patients job and driving status need to the differentiation of alternative causes of
be sought in order to establish adherence to myocardial hypertrophy. MRI can also be
vehicle licensing regulations. used to quantify fibrosis burden
First degree relatives should be screened
Demonstrate an understanding with an ECG and echocardiogram.
Negative results in adult relatives should be
of the value of further reevaluated at 5 yearly intervals. Adolescent
investigation relatives require annual screening.
Genotyping has been used in the research
The following investigations yield important
setting and also to establish family pedigrees
diagnostic and prognostic information in
hypertrophic cardiomyopathy:
An ECG will show various combinations of
Risk stratification in hypertrophic
the following electrocardiographic features: cardiomyopathy
Left ventricular hypertrophy Following a clinical history, examination and
ST segment or T wave abnormalities investigations, the following risk factors for sudden
suggesting abnormal repolarisation death in hypertrophic cardiomyopathy can be
Axis deviation used to guide management:
Left atrial or biatrial enlargement A personal history of syncope or previous
Atrial tachyarrhythmia: atrial tachycardia, cardiac arrest
flutter or fibrillation. A family history of unexplained premature
On CXR, abnormal findings are variable, sudden death
and may include cardiomegaly due to left Abnormal haemodynamic response to
ventricular dilatation or evidence of left atrial exercise on supervised treadmill testing
dilatation Non-sustained VT on 24 hours Holter
Echocardiography is the gold-standard monitoring, or an episode of spontaneous
diagnostic modality demonstrating: sustained VT
Different patterns of left ventricular The presence of LVOT gradient on
hypertrophy, e.g. asymmetric (septal or echocardiography (30 mmHg)
apical) or concentric Severe left ventricular hypertrophy (3 cm)
A left ventricular cavity that is small in size
Atrial dilatation
Diastolic dysfunction, and systolic Always offer a management plan
dysfunction in the later stages of the It is important to consider the advice to give
disease a patient who has a diagnosis of hypertrophic
12 Chapter 1 Cardiovascular system (station 3)

cardiomyopathy. This will include family The management of AF may require


screening, which is discussed in the specialist input from cardiomyopathy and
investigation section of this case. With regard electrophysiology subspecialties
to driving, heavy goods vehicle or passenger-
Interventional options for patients with
carrying vehicle driving licence holders will not
symptomatic LVOT obstruction include surgical
be allowed to continue to hold this licence and
septal myectomy, which remains the gold
need to inform the DVLA. Patients with an ICD
standard for this problem. Percutaneous alcohol
need to inform the DVLA and adhere to this set
septal ablation is an alternative approach.
of guidelines (see The infraclavicular mass,
Implantable cardiac defibrillators (ICDs) are
p. 33).
considered according to risk stratification of the
Additional points to advise patients about
patient. Risk stratification should be performed
include telling them what they must avoid.
on an annual basis with echocardiography, 24
Dehydration may exacerbate LVOT gradients.
hour Holter monitoring and supervised exercise
Patients should also avoid intense sports,
treadmill testing.
particularly involving rapid bursts of exertion or
simulating Valsalva (e.g. weightlifting).
Medical therapy in hypertrophic Further reading
cardiomyopathy includes: Maron BJ, McKenna WJ, Danielson GK, et al.
Beta-blockers, rate-limiting calcium channel American College of Cardiology/European
blockers and disopyramide are used for Society of Cardiology Clinical Expert Consensus
symptomatic LVOT obstruction Document on Hypertrophic Cardiomyopathy.
Amiodarone should be considered for atrial A report of the American College of Cardiology
or ventricular arrhythmia suppression Foundation Task Force on clinical expert
A low threshold for anticoagulation exists consensus documents and the European Society
for AF in hypertrophic cardiomyopathy of Cardiology Committee for practice guidelines.
due to the high risk of embolic stroke. Eur Heart J 2003; 24:19651991.

Case 5: Tricuspid regurgitation

Instruction to the candidate The auscultatory feature of tricuspid


regurgitation is a pansystolic murmur heard
This 55-year-old woman presents with
loudest at the lower left sternal edge in
flushing and dyspnoea. Please examine her
inspiration (Carvallos sign).
cardiovascular system.

Begin with a summary of Follow with a summary of


positive findings relevant negative findings
The absence of signs of right heart failure (S3,
The positive findings in tricuspid regurgitation
peripheral oedema and ascites) and signs of
include a combination of peripheral findings
pulmonary hypertension (a left parasternal
and auscultatory features.
heave, a palpable and loud P2) should be noted.
The key peripheral signs to elicit and mention
As with other valvulopathies, comment
to the examiner are:
on the absence of peripheral stigmata of
The presence of giant v waves on endocarditis, remembering however that right-
examination of the JVP sided lesions such as tricuspid regurgitation
Pulsatile hepatomegaly due to reflux of tricuspid usually cause pulmonary septic emboli rather
regurgitation via the inferior vena cava than peripheral embolic phenomena. Carefully
Case 5 Tricuspid regurgitation 13

inspect for risk factors for endocarditis such as of these structures can lead to TR (Table 1.4).
poor dental hygiene and indwelling intravenous Remember that, just like MR, TR can be
access. Importantly in right-sided lesion, acute, chronic and compensated, or chronic and
comment on any signs of intravenous drug use. decompensated. In PACES you will see the latter
two presentations, but remember to mention
during your presentation that cases of acute TR
State the most likely diagnosis can present, for example, in the setting of either
on the basis of these findings RV infarction, or sepsis in the case of infective
This patient has signs consistent with severe endocarditis.
tricuspid regurgitation. The Instruction to the
candidate mentions flushing and dyspnoea, Demonstrate the importance
which could be suggestive of carcinoid heart
disease resulting from carcinoid syndrome. of clinical context suggest
She has clinical features of right heart failure, relevant questions that would be
but does not have any features of infective
endocarditis.
taken in a patient history
Potential history questions to mention to
the examiner would include those eliciting
Offer relevant differential symptomatology as a result of the valve lesion,
diagnoses such as dyspnoea and peripheral oedema.
Offer to the examiner the differential diagnosis Additionally, the clinical history can elicit features
of a systolic murmur, as discussed in previous suggesting the aetiology of the valve lesion:
cases. Infective endocarditis: intravenous drug
The anatomical structure of the tricuspid abuse or long-term indwelling venous
valve should be considered when discussing the access, fevers or rigors, and recent dental
underlying causes of tricuspid regurgitation. procedures
The tricuspid valve consists of three leaflets Carcinoid syndrome: flushing, diarrhoea,
contained within an annulus, anchored by weight loss and wheeze
chordae tendinae to papillary muscles that Rheumatic heart disease: history of
originate from the myocardium. Disorders of any childhood rheumatic fever

Table 1.4 Aetiology of tricuspid regurgitation and associated clinical signs


Acute Chronic

Leaflets Endocarditis (review for signs of Rheumatic fever (check for concomitant
intravenous drug abuse, intravenous mitral stenosis) )
access (especially in patients receiving Carcinoid syndrome (check for
renal replacement therapy), peripheral concomitant pulmonary valve and less
stigmata) commonly left-sided valve lesions, and
flushed appearance)
Leaflet malcoaptation or trauma due to
RV pacing lead
Anorectic drugs (e.g. fenfluramine and
phentermine, also referred to as fen-
phen)
Functional (annular Pulmonary hypertension
dilatation) Congestive cardiac failure
Subvalvular apparatus Right ventricular myocardial infarction: Leaflet prolapse
(chordae/papillary muscles) ruptured papillary muscle (acute
ischaemic event)
Blunt trauma causing flail chordae
Miscellaneous Ebsteins anomaly
Dysplastic tricuspid valve
Endomyocardial fibrosis
14 Chapter 1 Cardiovascular system (station 3)

Demonstrate an understanding atrial tachyarrhythmias via the accessory


pathway.
of the value of further Antibiotic prophylaxis should be taken as
investigation indicated (see Infective endocarditis, p. 40).
The following cardiac investigations offer a The definitive management of tricuspid
complete assessment of tricuspid regurgitation: regurgitation is surgery. Indications for surgery
(which include plication, annuloplasty, or
12 lead electrocardiogram: If the ECG replacement) are:
demonstrates widening of the QRS complex
with RSR morphology in the right ventricular Symptomatic severe TR (without severe
leads, this might be reflective of dilatation RV dysfunction) despite optimal medical
of this chamber. There may be delta waves treatment
suggesting pre-excitation in Ebsteins At least moderate TR in patients undergoing
anomaly left-sided valve surgery
Echocardiography: Evaluation of patient co-morbid status, right
The severity of TR is assessed by colour ventricular dysfunction, presence of pulmonary
Doppler and by jet width (vena contracta) hypertension and previous cardiothoracic
The mechanism and underlying cause surgery are all considerations prior to deciding
of TR is elucidated by looking at all upon surgical intervention.
structures that make up the tricuspid Significant TR may or may not resolve
valve apparatus following surgical correction of mitral valve
The impact of TR upon right ventricular lesions, and predicting this post-operative
cavity size and systolic function and right course is imprecise. Furthermore, redo
atrial dimensions must be assessed surgery for persistent TR following correction
Cardiac MRI is currently the gold standard of left-sided valvular lesions carries a higher
tool for determination of right ventricular risk. Therefore severe TR secondary to mitral
size and systolic function valve disease should be fully evaluated and
Cardiac catheterisation is performed in concomitantly intervened upon in patients
cases of suspected pulmonary arterial undergoing left-sided valve surgery.
hypertension The following features increase the mortality
risk of surgical intervention for symptomatic
Always offer a management severe TR:

plan Poor functional class


Medical management of tricuspid regurgitation
Left or right heart failure
is limited to diuretics for relief of symptoms
Pulmonary hypertension
and treatment directed at the underlying cause.
Previous valvular intervention (typically
mitral valve repair or replacement) with
Atrial arrhythmias, particularly atrial flutter,
unresolved TR
are common and require management of both
the rhythm itself (especially if poorly tolerated
due to underlying congenital heart disease Further reading
or pulmonary hypertension, for example) Vahanian A, Baumgartner H, Bax J, et al. Guidelines
and anticoagulation where appropriate. on the management of valvular heart disease:
Accessory pathways are also recognised in The Task Force on the Management of Valvular
Ebsteins anomaly (for which the contemporary Heart Disease of the European Society of
treatment is a Cone repair), and hence ablation Cardiology. Eur Heart J 2007; 28:230268.
should be sought to prevent conduction of
Case 6: Pulmonary stenosis

Instruction to the candidate State the most likely diagnosis


This 20-year-old woman presents with on the basis of these findings
dyspnoea. Please examine her cardiovascular This patient has signs consistent with
system. pulmonary stenosis likely due to congenital
heart disease. She has clinical features of right
Begin with a summary of heart failure, but does not have any features
of infective endocarditis. I would like to take a
positive findings full history to establish her symptomatic status,
Peripheral signs of pulmonary stenosis include: and my investigation of choice would be an
JVP may be elevated indicating raised right echocardiogram to confirm the diagnosis.
atrial pressure, with giant a waves indicating
right atrial hypertrophy Offer relevant differential
Parasternal heave indicative of right
diagnoses
ventricular hypertrophy
A thrill over the pulmonary valve area Pulmonary stenosis is either congenital (for
Presystolic hepatic pulsation example, on its own or due to congenital rubella
syndrome), with or without other associated
Auscultatory features of pulmonary stenosis lesions (tetralogy of Fallot, for example. See
include: page 50), or acquired (for example, secondary to
An ejection systolic murmur heard loudest in carcinoid heart disease).
the pulmonary area on inspiration. This can Congenital pulmonary stenosis may feature
radiate to the left shoulder and the left lung as part of an eponymous syndrome:
field posteriorly Noonans syndrome: features include
An ejection click may be present with thin, (amongst others) short stature, typical facies
pliable valves, followed by a soft P2 (webbed neck, hypertelorism, ptosis, flat
S4 nasal bridge), pectus excavatum, learning
On identifying the positive findings of difficulties
pulmonary stenosis, consider clinical signs Watsons syndrome: affected individuals
which would point towards severe pulmonary demonstrate dermatological features of
stenosis: neurofibromatosis type 1
Williams syndrome: features include
The peak of the ejection systolic murmur is transient hypercalcaemia, elfin-like facies
delayed with increasing severity and learning difficulties with an overly
Absence of an ejection click (unless the sociable persona. Aortic and pulmonary
stenosis is located at the infundibulum) stenoses are associated with this syndrome
Findings suggestive of right heart failure
Presence of right ventricular S4
Demonstrate the importance
Follow with a summary of of clinical context suggest
relevant negative findings relevant questions that would be
In addition to commenting on the absence of taken in a patient history
stigmata of infective endocarditis and on the Relevant questions in any history of pulmonary
absence of signs of right heart failure (including stenosis would include:
peripheral oedema and ascites), comment on
features of tetralogy of Fallot and congenital
Exertional dyspnoea, chest pain and/or
syncope
cyanotic heart disease such as central cyanosis
and clubbing.
Peripheral oedema
16 Chapter 1 Cardiovascular system (station 3)

Features suggesting aetiology, e.g. a history of (infundibular and subinfundibular) or


maternal rubella supravalvular
Pathophysiological consequences of
pulmonary stenosis: right ventricular
Demonstrate an understanding of wall thickness, cavity size and systolic
the value of further investigation function, and right atrial dimensions
Relevant investigations include: To exclude other coexisting congenital
cardiac lesions
Pulse oximetry: if peripheral oxygen Cardiac MRI: For accurate quantitation
saturations are low, check a full blood count of pulmonary flow, RV size and function
for secondary erythrocytosis and an iron assessment, and cardiac morphology +/-
profile associated lesions
Functional data such as 6-minute walk test
or cardiopulmonary exercise testing (CPEX)
can be informative both prognostically and to Always offer a management plan
help guide management Medical management of pulmonary stenosis
12 lead electrocardiogram: whilst an ECG is limited to diuretics for symptomatic relief.
may be normal, it may show right ventricular Antibiotic prophylaxis should also be considered
hypertrophy (with or without strain) and (see Infective endocarditis, p. 40).
right atrial enlargement Interventional options include balloon
CXR to look for dilated pulmonary arteries valvuloplasty, which in anatomically suitable
and reduced lung markings. cases of severe symptomatic pulmonary
Echocardiography will allow the following: stenosis offers excellent long term outcomes,
Doppler flow assessment of severity and valve replacement (either surgical or
Anatomical aetiology of pulmonary percutaneous).
stenosis: valvular, subvalvular

Case 7: Mitral stenosis

Instruction to the candidate sound). A tapping apex beat and an opening


snap are both clinical indicators of a pliable
This 35-year-old woman presents with exertional
mitral valve, which would be more suitable
dyspnoea. Please examine her cardiovascular
to percutaneous balloon mitral valvuloplasty
system.
Auscultatory features of mitral stenosis
include:
Begin with a summary of
positive findings A loud first heart sound and opening snap
after S2: both indicate pliable (rather than
Consider the patient demographics given in the calcified) mitral valve leaflets
candidate information. The patient is female A low pitch, rumbling mid-diastolic murmur,
(rheumatic mitral stenosis has a 2:1 prevalence best heard at the apex with the bell of the
of female:male) and she is 35 (symptoms usually stethoscope on expiration in the left lateral
present in the 3rd to 4th decade of life). position
Peripheral signs of mitral stenosis include: If the patient is in sinus rhythm, a presystolic
Mitral facies: The malar flush of pink and murmur (termed presystolic accentuation)
purple patches on the cheeks will be heard as left atrial contraction forces
Pulse: presence or absence of AF, and rate blood across the stenosed mitral valve
Palpate at the non-displaced apex for a Signs of severity in mitral stenosis include:
tapping apex beat (palpable first heart
Case 7 Mitral stenosis 17

The interval between S2 and the opening Connective tissue disease (most commonly
snap (and thus the duration of the diastolic systemic lupus erythematosus or rheumatoid
murmur) is the auscultatory feature of arthritis)
severity in mitral stenosis. This is reflective Carcinoid heart disease
of the degree of left atrial pressure. Higher Drugs: methysergide
left atrial pressure causes earlier mitral valve Mucopolysaccharidoses
opening, and hence an earlier opening snap.
Anatomic lesions that mimic mitral stenosis
Thus, the time between S2 and the opening
arise above the left ventricle and therefore also
snap is reduced with higher left atrial
reflect raised pressures back to the pulmonary
pressures, indicating more severe MS. In
venous system. All of these conditions can
such cases the diastolic murmur is therefore
manifest as pulmonary oedema and pulmonary
longer
hypertension, but the auscultatory features will
The presence of pulmonary hypertension is
differ from those described in mitral stenosis.
also a feature of severe MS. The Graham Steell
murmur is a rare early diastolic murmur of PR Cortriatriatum: a congenital abnormality in
in the pulmonary region secondary to marked which the left (or right) atrium is divided by
pulmonary hypertension a membrane, thus separating the pulmonary
veins from the mitral valve and creating a
Follow with a summary of triatrial heart
Left atrial myxoma
relevant negative findings Pulmonary vein stenosis (this was reported
Important relevant negative findings which, if more frequently following early experiences
absent, are important to note, include: of pulmonary vein isolation in AF ablation)
Arrhythmia (atrial fibrillation): comment
on the rate, embolic complications and Demonstrate the importance
peripheral stigmata of anticoagulation of clinical context suggest
Stigmata of infective endocarditis
Pulmonary oedema (due to raised left atrial relevant questions that would be
pressure) taken in a patient history
Signs of pulmonary hypertension, including: Mitral stenosis tends to present in the 3rd to 4th
Raised JVP decade of life with a stepwise decline in exercise
Left parasternal heave (pressure-loaded tolerance. The chronology of symptoms tends
right ventricle) to be revealing of the natural history of the
Loud P2 condition. Pregnancy may unmask less severe
Graham Steell murmur (rare) mitral stenosis due to the increase in circulating
volume during the second trimester. Symptoms
State the most likely diagnosis to directly ascertain include:
on the basis of these findings Exertional dyspnoea
This young woman has signs consistent with Palpitations (low index of suspicion of AF)
severe mitral stenosis with a pliable mitral Peripheral oedema
valve, likely secondary to rheumatic fever. Childhood history of rheumatic fever,
She is clinically euvolaemic, but is in atrial country of birth and childhood
fibrillation with signs of pulmonary hypertension. History of cerebral embolic events
I would like to take a full history to establish More rarely, haemoptysis has been described.
her symptomatic status and previous history of Hoarse voice secondary to left atrial dilatation
rheumatic fever, and my investigation of choice compressing the recurrent laryngeal nerve
would be an echocardiogram to confirm the has also been described in the setting of
diagnosis. mitral stenosis, termed Ortners syndrome
Dysphagia can also manifest from oesophageal
compression by a dilated left atrium
Offer relevant differential
diagnoses Demonstrate an understanding of
Any list of the differential diagnosis of mitral
stenosis should start with rheumatic fever, which
the value of further investigation
is by far the commonest cause. Less common An investigative work-up in a case of mitral
causes include: stenosis would include the following:
18 Chapter 1 Cardiovascular system (station 3)

Table 1.5 Echocardiographic features of mitral stenosis and their relationship to severity
Mild Moderate Severe
Valve area (cm2
) 1.62.0 1.01.5 <1.0

Mean gradient (mmHg) <5 510 >10

12-lead electrocardiogram: Antibiotic prophylaxis (see Infective


P mitral (broad, bifid P waves) of left atrial endocarditis, p. 40).
dilatation Anticoagulation: All patients with AF
Atrial fibrillation (paroxysmal or permanent) should receive
Plain chest radiograph: anticoagulation. Patients in sinus rhythm
Evidence of left atrial dilatation should receive anticoagulation in the setting
Pulmonary oedema of a previous history of embolism, with a
Prominent pulmonary artery in patients dilated left atrium (diameter >5 cm), or in the
with pulmonary hypertension presence of left atrial or appendage thrombus
Splayed carina or spontaneous echo contrast (signifying
Echocardiography: the gold-standard sluggish left atrial flow) on transoesophageal
diagnostic investigation. This demonstrates: echocardiography
Severity: mitral valve area can be Diuretics and nitrates to reduce left atrial
measured by Doppler flow and 2D filling pressures
planimetry. The mean transvalvular Beta-blockers to reduce heart rate and
gradient is measured by Doppler flow. See thus lengthen the duration of diastolic left
Table 1.5 for echocardiographic figures of ventricular filling across the stenosed mitral
mitral stenosis relating to their severity valve
Cusp and subvalvular mobility and Indications for intervention in mitral stenosis
calcification, especially important when
include:
considering balloon valvotomy. The
Wilkins score is a system that grades Symptomatic, severe mitral stenosis
mitral valve mobility, calcification, leaflet Asymptomatic patients but with high risk of
thickening and subvalvular thickening to embolism (e.g. previous embolism, recent
determine the suitability for percutaneous or paroxysmal AF) or haemodynamic
balloon valvuloplasty decompensation (specifically raised
Any associated mitral regurgitation pulmonary artery pressure, defined as
(if moderate or severe, this is a estimated peak systolic pressure of >50
contraindication to balloon valvuloplasty) mmHg on echocardiogram)
Left atrial dimensions Asymptomatic patients with severe MS who
The presence of left atrial or require optimisation prior to undergoing
appendage thrombus is excluded on events with a high risk of haemodynamic
transoesophageal echocardiography, disturbance, such as those considering
which also provides more detailed views pregnancy or prior to non-cardiac surgery
of the mitral valve itself.
Estimated pulmonary arterial and right The decision regarding the percutaneous
atrial pressures versus surgical (most commonly valve
Right ventricular size and systolic replacement) approaches to treating severe
function. MS rests upon factors including the patients
Cardiac catheterisation is only required when functional status and comorbid burden,
discrepancy exists after clinical assessment anatomical suitability for balloon valvuloplasty,
and echocardiography, or to rule out and local expertise in both approaches.
concomitant coronary artery disease in older
patients undergoing surgical management of Further reading
their MS
Vahanian A, Baumgartner H, Bax J, et al. Guidelines
on the management of valvular heart disease:
Always offer a management plan the task force on the management of valvular
Medical management of mitral stenosis heart disease of the European Society of
includes: Cardiology. Eur Heart J 2007; 28:230268.
Case 8: Aortic regurgitation

Instruction to the candidate De Mussets sign head nodding in time with


systole
This 40-year-old man presents with exertional
dyspnoea. Please examine his cardiovascular Of the clinical signs mentioned above, those
system. that suggest severe aortic regurgitation include:
A wide pulse pressure (noteably low diastolic
Begin with a summary of blood pressure)
positive findings A collapsing pulse
Long duration of the early diastolic murmur
Peripheral signs in aortic regurgitation include: The presence of signs of cardiac failure
Collapsing (water-hammer) pulse at the The presence of an Austin Flint murmur
brachial artery with the patients arm raised
vertically Follow with a summary of
Wide pulse pressure
Thrusting (volume-loaded), laterally relevant negative findings
displaced apex beat The absence of stigmata of infective endocarditis
Aortic thrill and signs of cardiac failure are important to
Look for associated aetiological features (see mention to the examiner. State that you would
Table 1.6 below): for example, Marfanoid also want to check bilateral blood pressures to
habitus. exclude dissection.
The auscultatory features of aortic
regurgitation include: State the most likely diagnosis
Early diastolic murmur: a high-pitched, on the basis of these findings
decrescendo murmur heard loudest at the This young patient has signs consistent with
lower left sternal edge with the patient sitting severe aortic regurgitation, likely secondary
forward in expiration to a bicuspid aortic valve given his age. He
Associated murmurs: is clinically euvolaemic, and does not have
Systolic flow murmur any features of infective endocarditis. I would
Ejection systolic murmur (mixed aortic like to take a full history to establish his
valve disease) symptomatic status and my investigation of
Austin Flint murmur: a low-pitched, choice would be an echocardiogram to confirm
rumbling mid-diastolic murmur due to the diagnosis and to rule out any associated
turbulent mixing of the retrograde AR and aortic pathology.
antegrade mitral valve forward flow in the
left ventricular cavity
An ejection click suggests a bicuspid aortic Offer relevant differential
valve diagnoses
The constellation of clinical signs in aortic The differential of the causes of aortic
regurgitation includes numerous eponymous regurgitation can be divided according to
signs. The important ones to know and seek to whether the lesion is congenital or acquired.
identify in the exam, include: The commonest cause of congenital aortic
regurgitation is a bicuspid aortic valve. Acquired
Corrigans sign a visible carotid pulsation causes of aortic regurgitation are considered in
Quinckes sign capillary pulsation in the Table 1.6.
fingernails It is also worth considering the differential
Duroziezs sign compressing the femoral diagnosis of a collapsing pulse, which can be a
artery with the stethoscope elicits murmurs manifestation of other high-output states:
of systolic forward flow and diastolic flow
reversal Aortic regurgitation
Traubes sign pistol-shot sound heard on Patent ductus arteriosus
auscultation over the femoral arteries Anaemia
20 Chapter 1 Cardiovascular system (station 3)

Table 1.6 Aetiology of aortic regurgitation and associated clinical findings


Acute Chronic

Valve leaflet Endocarditis (review dentition, Rheumatic fever (check for concomitant mitral stenosis)
intravenous access, peripheral Connective tissue diseases (e.g. rheumatoid arthritis
stigmata) check joints)
Aortic root Dissection (type A) Dilatation Idiopathic
Trauma Marfan syndrome (check arm span, palate and eyes),
EhlersDanlos syndrome, pseudoxanthoma elasticum
Hypertension (check blood pressure)
Aortitis
Syphilis (check pupils for Argyll Robertson abnormality)
Ankylosing spondylitis (check posture)

Hyperthyroidism Valve morphology: bicuspid versus


Fever tricuspid
Pagets disease Left ventricular cavity size and systolic
Pregnancy function
Left ventricular wall thickness
Aortic root and ascending aorta
Demonstrate the importance dimensions, and associated aortic
pathology, such as in the case of
of clinical context suggest dissection or aneurysm
relevant questions that would be CT: To visualise the thoracic aorta for
taken in a patient history associated abnormalities
Relevant history questions include those aimed
Cardiac MRI can be used to assess left
ventricular volumes and aortic valve
at identifying underlying cause and those regurgitant fractions when echocardiography
regarding symptoms of cardiac failure. is unclear. This can also be used to
visualise the thoracic aorta for associated
Demonstrate an understanding abnormalities
of the value of further Cardiac catheterisation: for investigation of
concomitant coronary artery disease in pre-
investigation operative patients. Aortic dimensions and
The key investigations include a 12-lead visualisation of AR on aortogram can also be
electrocardiogram, a plain chest radiograph, assessed
and a transthoracic echocardiogram. Additional
investigations include CT, cMRI and cardiac Always offer a management
catheterisation. The findings of such relevant
investigations include: plan
Patient advice is important in aortic
ECG:
regurgitation. For patients with bicuspid aortic
Left ventricular hypertrophy valve or Marfan syndrome, advice on family
Left axis deviation screening is relevant. Patients with mild and
CXR:
moderate aortic regurgitation should receive
Cardiomegaly due to left ventricular outpatient follow-up to monitor the natural
dilatation
history of the condition. For matters relating to
Pulmonary oedema if in heart failure antibiotic prophylaxis (see Infective endocarditis,
Aortic dilatation p. 40).
Echocardiography: the gold-standard
Indications for specific medical therapies
diagnostic investigation. This demonstrates:
include:
Severity by regurgitant orifice area,
pressure half-time, regurgitant volume Acute severe AR: vasodilators (e.g. nitrates)
and fraction, jet width on colour Doppler are used to optimise prior to surgery
and the presence of flow reversal in the Chronic severe AR with evidence of left
descending aorta ventricular dysfunction: ACE inhibitors
Case 9 Pulmonary regurgitation 21

Hypertension: vasodilators should be (end-diastolic diameter >7 cm,


prescribed, including ACE inhibitors end-systolic diameter >5 cm)
or dihydropyridine calcium channel Patients undergoing other concomitant
antagonists cardiovascular surgery (e.g. CABG, other
Marfan syndrome: Beta-blockers and valve surgery, aortic surgery)
angiotensin receptor blockers (e.g. losartan) Aortic root disease: Surgical intervention is
are used to slow the rate of aortic dilatation. indicated in the following groups of patients
(do not use beta-blockers in severe AR, (irrespective of AR severity) with an aortic
as reducing heart rate lengthens diastole, root diameter of:
consequently increasing regurgitant volumes) Marfan syndrome: 45 mm
Indications for surgery in cases of aortic
Bicuspid valves: 50 mm
regurgitation include:
All other patients: 55 mm
Symptomatic severe AR: requires aortic valve Further reading
replacement
Asymptomatic severe AR: aortic valve Vahanian A, Baumgartner H, Bax J, et al. Guidelines
replacement is indicated in the following on the management of valvular heart disease:
circumstances: the task force on the management of valvular
Resting left ventricular ejection fraction heart disease of the European Society of
50% Cardiology. Eur Heart J 2007; 28:230268.
Significant left ventricular dilatation

Case 9: Pulmonary regurgitation

Instruction to the candidate P2 is absent following pulmonary valvotomy


or in cases of congenitally absent pulmonic
This 42-year-old man presents with dyspnoea.
valve
Please examine his cardiovascular system.
P2 is loud and can be palpable at the upper
left sternal edge in pulmonary hypertension
Begin with a summary of Signs of severity in pulmonary regurgitation
positive findings include:
Relevant positive findings in pulmonary When not caused by pulmonary hypertension,
regurgitation include auscultatory features and a shorter duration of the early diastolic
signs of severity. murmur is related to increased severity of PR
The auscultatory features of pulmonary Examination findings of right heart failure
regurgitation include:
A brief, early diastolic decrescendo murmur Follow with a summary of
heard loudest at the upper left sternal edge
typically on inspiration. The Graham Steell relevant negative findings
murmur describes the murmur of PR due Important features which, if absent, are
to a dilated annulus secondary to a dilated important to note, include:
pulmonary artery in severe pulmonary
hypertension. It has a high-pitched quality,
Signs of right heart failure: raised JVP,
peripheral oedema and ascites
and can be present for the duration of
diastole given the pressure gradient between
Infective endocarditis: comment on the
absence of peripheral stigmata, dental
the pulmonary artery and the right ventricle
hygiene and the presence or absence of
22 Chapter 1 Cardiovascular system (station 3)

indwelling intravenous access. It should Demonstrate the importance


be noted, however, that right-sided lesions
usually cause pulmonary septic emboli of clinical context suggest
rather than peripheral embolic phenomena. relevant questions that would be
Peripheral stigmata of intravenous drug
abuse should be carefully inspected given the
taken in a patient history
propensity for right-heart valve endocarditis Patients are usually asymptomatic until right
in these patients ventricular dilatation and dysfunction manifests.
Scars indicating previous cardiothoracic Once this occurs, symptoms will include:
surgery Exertional dyspnoea is usually the first
Also note the presence or absence of symptom that is reported
concomitant tricuspid regurgitation. It can be Peripheral oedema
due to a number of factors: Palpitations: atrial and ventricular
arrhythmias can occur, and are a risk factor
Secondary to right ventricular dilatation, and for sudden cardiac death
thus consequent tricuspid annular dilatation,
in the setting of chronic PR Features suggesting aetiology of pulmonary
Clinically significant TR is found in regurgitation include:
approximately one-third of patients with History of congenital heart disease and
repaired tetralogy of Fallot (either due to associated surgical or interventional
tricuspid annular dilatation or iatrogenic treatment
pulmonic valve injury following VSD repair) Infective endocarditis: intravenous drug
The tricuspid valve is the most commonly abuse or long-term indwelling venous access,
affected valve in carcinoid heart disease, and fevers or rigors, and recent dental procedures
therefore patients with variable combinations Carcinoid syndrome: flushing, diarrhoea,
of pulmonary regurgitation and stenosis are weight loss and wheeze
likely to have concomitant TR History of childhood rheumatic fever
Patients with pulmonary hypertension
causing PR may also have clinically significant
TR due to right ventricular pressure-overload Demonstrate an understanding
with or without volume-overload of the value of further
investigation
State the most likely diagnosis The following investigations are key in the
on the basis of these findings diagnosis and evaluation of pulmonary
This patient has signs consistent with severe regurgitation:
pulmonary regurgitation secondary to A 12-lead electrocardiogram demonstrates:
pulmonary valvotomy, likely following repaired Any current arrhythmia
tetralogy of Fallot. He has clinical features Widening of the QRS complex with RSR
of right heart failure, but does not have any morphology in the right ventricular leads
features of infective endocarditis. I would like to is reflective of dilatation of this chamber,
take a full history to establish his symptomatic and has prognostic implications with
status, and my investigation of choice would be respect to the propensity for ventricular
an echocardiogram to confirm the diagnosis. arrhythmia and sudden cardiac death
Right ventricular hypertrophy
Offer relevant differential Right bundle branch block, which is
common in patients with a history of
diagnoses previous tetralogy of Fallot repair via a
The differential diagnosis can be considered in right ventriculotomy approach
the context of whether the lesion is congenital A plain chest radiograph may demonstrate
or acquired, anatomically and by acute versus evidence of right ventricular dilatation and/
chronic pathologies. Congenital causes include or enlargement of the central pulmonary
an absent pulmonary valve. For acquired causes arteries
see Table 1.7. Echocardiography with demonstrate the
presence of pulmonary regurgitation, severity
Case 9 Pulmonary regurgitation 23

Table 1.7 Aetiology of pulmonary regurgitation and associated clinical findings


Acute Chronic

Valve leaflet Endocarditis (review for signs of Rheumatic fever (check for other likely
intravenous drug abuse, intravenous concomitant valvular abnormalities)
access (especially in patients receiving Carcinoid syndrome (check for concomitant
renal replacement therapy, peripheral tricuspid valve and, less commonly, left-sided
stigmata) valve lesions, and flushed appearance)
Congenital disease: bicuspid or quadricuspid
valves
Functional (annular Pulmonary hypertension (the commonest cause
dilatation) in adults)
Following intervention for tetralogy of Fallot
(such as pulmonary valvotomy) or pulmonary
stenosis (such as balloon valvuloplasty)
Dilatation of the pulmonary trunk due to Marfan
syndrome (check arm span, palate and eyes)

of the lesion, and any subsequent structural intervention or for patients where
abnormalities: haemodynamic quantification by non-
The valve morphology and underlying invasive techniques remains suboptimal
cause of pulmonary regurgitation is Functional data provided by serial CPEX
elucidated by looking at all structures that testing can help determine the timing of
make up the pulmonary valve apparatus, surgery.
including the valve leaflets, annulus,
RVOT and pulmonary artery. However,
previous surgical intervention can confer
Always offer a management plan
extra difficulty on echocardiographic Management of pulmonary regurgitation can be
assessment of the valve considered in terms of general advice, medical
The severity of pulmonary regurgitation management, and intervention (whether
is assessed semi-quantitatively by surgical or percutaneous). Advice regarding
colour Doppler and by the duration of antibiotic prophylaxis is considered in Infective
pulmonary regurgitant flow in diastole endocarditis, p. 40). Medical management
The impact upon right ventricular cavity is limited to diuretics for relief of symptoms
size and systolic function and treatment of the underlying cause.
Cardiac MRI is currently the gold standard Surgery is indicated for moderate to severe PR
tool for quantification of pulmonary with progressive right ventricular dilatation
regurgitation, right ventricular size and irrespective of symptomatology, and following
systolic function. This technique also permits the development of ventricular arrhythmias
detailed imaging of complex treated or or syncope. Percutaneous pulmonary valve
untreated congenital lesions implantation is also a treatment option in this
Cardiac catheterisation is only required scenario.
in patients undergoing transcatheter
Case 10: Mixed aortic valve disease

Instruction to the candidate In cases of rheumatic aortic valve disease,


comment on the presence or absence of
This 40-year-old man has been complaining of
concomitant rheumatic mitral valve lesions.
breathlessness on exertion. Please examine his
cardiovascular system.
State the most likely diagnosis
Begin with a summary of on the basis of these findings
positive findings This young patient has signs consistent with
mixed aortic valve disease with predominant
The peripheral signs to identify in mixed aortic
aortic regurgitation due to a bicuspid aortic
valve disease include:
valve. He is clinically euvolaemic, and does
A bisferious (biphasic) pulse (pulsus not have any features of infective endocarditis.
bisferiens) describes a palpable double peak I would like to take a full history to establish his
in the pulse waveform during a single cardiac symptomatic status, and my investigation of
cycle. This is typically described in patients choice would be an echocardiogram to confirm
with mixed aortic stenosis and regurgitation the diagnosis and to rule out any associated
when the regurgitant lesion predominates aortic pathology.
and is haemodynamically significant.
However, it may also be found in isolated
severe AR
Offer relevant differential
It is important to consider dominance, which diagnoses
can be determined before auscultation. If Key underlying diagnoses in mixed aortic valve
aortic stenosis is dominant, there will be disease include:
a slow-rising pulse with a narrow pulse
pressure. If the regurgitation is dominant, Bicuspid aortic valve
there will be a collapsing pulse with a wide Rheumatic valve disease: note that
pulse pressure. Likewise, thrills and the concomitant mitral stenosis can cause a
nature of the apex pulsation (heaving in reduced stroke volume, thus leading to
stenosis versus thrusting in regurgitation) underestimated gradients across a stenosed
can assist to determine dominance. aortic valve. Therefore, aortic valve area
should be measured to provide a less load-
Auscultatory features of mixed aortic valve dependent parameter of aortic stenosis
disease include:
A combination of ejection systolic and early Demonstrate the importance
diastolic murmurs in the aortic region and at
the lower left sternal edge respectively
of clinical context suggest
An underlying bicuspid aortic valve can be relevant questions that would be
identified by an aortic click taken in a patient history
See Aortic stenosis (p. 2) and Aortic regurgitation
Follow with a summary of (p. 19) for relevant history questions.
relevant negative findings
Important relevant negative findings include: Demonstrate an understanding
Infective endocarditis: comment on the of the value of further
absence of peripheral stigmata, dental investigation
hygiene and the presence or absence of
Aortic stenosis (p. 2) and Aortic regurgitation
indwelling intravenous access
(p. 19), outline the relevant investigations. As
Signs of cardiac failure, including pulmonary
with all cardiology cases, the key investigations
oedema, peripheral oedema, raised JVP and
will include:
S3 gallop
Case 11 Mixed mitral valve disease 25

A 12-lead electrocardiogram lesion is severe, intervention is indicated when


A plain chest radiograph symptoms result or the mixed disease causes
Echocardiography impairment of left ventricular function.

Always offer a management plan Further reading


The management of mixed aortic valve disease Vahanian A, Baumgartner H, Bax J, et al. Guidelines
follows that of the predominant valvular lesion on the management of valvular heart disease:
as for lone aortic stenosis or regurgitation. In the task force on the management of valvular
situations in which balanced aortic stenosis and heart disease of the European Society of
regurgitation manifest but neither individual Cardiology. Eur Heart J 2007; 28:230268.

Case 11: Mixed mitral valve disease

Instruction to the candidate Follow with a summary of


This 45-year-old woman has been complaining relevant negative findings
of breathlessness on exertion. Please examine Important relevant negative findings to
her cardiovascular system. comment on include:

Begin with a summary of Arrhythmia (atrial fibrillation): comment


on the rate, embolic complications and
positive findings peripheral stigmata of anticoagulation
Peripheral signs of mixed mitral valve disease Infective endocarditis: comment on the
include: absence of peripheral stigmata, dental
hygiene and the presence or absence of
Look for a scar overlying the femoral vein indwelling intravenous access
indicating previous vascular access for a Pulmonary oedema occurs in both mitral
percutaneous balloon mitral valvuloplasty. stenosis and regurgitation, albeit with
Increased mitral regurgitation is a frequent differing underlying pathophysiology
finding immediately post-procedure, with Signs of pulmonary hypertension,
restenosis being a later complication including:
Consider dominance, which is determined Raised JVP
clinically before auscultation: Left parasternal heave (pressure-loaded
Dominant MS: a tapping, undisplaced right ventricle)
apex beat with a loud first heart sound Loud P2
Dominant MR: a thrusting, displaced apex Graham Steell murmur (rare)
beat with a soft first heart sound
In cases where there is a specific underlying
Auscultatory features of mixed mitral valve cause, further relevant negative findings
disease include: are pertinent. In cases of rheumatic mitral
Mixed pansystolic and mid-diastolic valve disease, comment on the presence or
murmurs, best heard at the apex (with the absence of concomitant rheumatic aortic valve
bell of the stethoscope for the murmur of MS) lesions. In cases of Marfan syndrome and
on expiration in the left lateral position mitral regurgitation, rule out coexisting aortic
S3 is not heard with MS, and is a finding in LV regurgitation.
dysfunction due to MR
26 Chapter 1 Cardiovascular system (station 3)

State the most likely diagnosis a transthoracic echocardiogram. Further details


can be found in Mitral regurgitation (p. 5) and
on the basis of these findings Mitral stenosis (p. 16).
This woman has mixed mitral valve disease
with predominant mitral regurgitation following
valvuloplasty for rheumatic mitral stenosis. She
Always offer a management
is in rate-controlled atrial fibrillation with signs plan
of pulmonary and peripheral fluid retention, The management of mixed mitral valve disease
but no stigmata of infective endocarditis. My follows that of the predominant valvular lesion
investigation of choice to confirm the diagnosis as for lone mitral stenosis or regurgitation.
and look for further complications is an However, percutaneous balloon mitral
echocardiogram. valvuloplasty is contraindicated in mitral
stenosis when moderate or worse regurgitation
Offer relevant differential coexists.
In situations in which balanced mitral
diagnoses stenosis and regurgitation manifest but neither
In the setting of mixed mitral valve disease, prior individual lesion is severe, intervention
rheumatic fever is the likely cause. is indicated when symptoms result or
predominant regurgitation causes impairment
of left ventricular function.
Demonstrate the importance
of clinical context suggest
Further reading
relevant questions that would be Vahanian A, Baumgartner H, Bax J, et al. Guidelines
taken in a patient history on the management of valvular heart disease:
See Mitral regurgitation (p. 5) and Mitral stenosis the task force on the management of valvular
(p. 16) for relevant history questions. heart disease of the European Society of
Cardiology. Eur Heart J 2007; 28:230268.

Demonstrate an understanding of
the value of further investigation
The key investigations will include a 12-lead
electrocardiogram, a plain chest radiograph and

Case 12: Prosthetic valves and


cardiac surgery

Instruction to the candidate Scars. When a scar is identified, run your


finger over it to demonstrate to the examiner
This 60-year-old man has recently undergone
that you have noted it. Different scars
cardiac surgery. Please examine his
include:
cardiovascular system.
Midline sternotomy (CABG, valve
replacement or repair, cardiac
Begin with a summary of transplantation)
positive findings Lateral thoracotomy:
|| Cardiac causes: mitral valvotomy,
Positive peripheral clinical findings to look for coarctation repair, PDA ligation,
include: Blalock-Thomas-Taussig shunt
Case 12 Prosthetic valves and cardiac surgery 27

|| Respiratory causes: lobectomy,


pneumonectomy, lung Prosthetic valve incompetence
transplantation, bullectomy, previous It is vital to identify auscultatory evidence of
thoracoplasty for TB prosthetic valve incompetence.
Inspect and palpate for either an In prosthetic aortic valves, a diastolic murmur
infraclavicular or an abdominal (surgically indicates incompetence
implanted epicardial device) subcutaneous In prosthetic mitral valves, a systolic murmur
pacemaker in the cardiothoracic indicates incompetence
surgical patient. In one series of over 340
patients undergoing isolated aortic valve
replacement, post-operative permanent Follow with a summary of
pacing was required in 8.5% of patients relevant negative findings
Arterial (radial) or venous (saphenous and
The following relevant negatives relate to those
femoral) graft harvest scars. Remember
of prosthetic valves:
that left internal mammary artery (LIMA)
grafts are reflected from the internal thoracic Embolic complications: approximately
wall. Therefore, a CABG is still a possibility 75% of complications experienced with
in a patient with a midline sternotomy scar prosthetic valves are due to thromboembolic
without saphenous vein or radial artery graft phenomena and bleeding secondary to
harvest scars anticoagulation. Embolic complications can
Radial or femoral puncture scars from be due to:
previous angiography. These tend to be on Atrial fibrillation
the right-hand side which is the conventional From the valve prosthesis itself
approach for performing these procedures As an immediate post-operative
complication
Before auscultating the chest, listen away
from the chest without the stethoscope for
Bleeding from anticoagulation
the audible click of a metallic heart valve
Dysfunction of the valve prosthesis can be
due to thrombosis, degeneration, dehiscence,
replacement. If this finding is present, state to
paravalvular leak or pannus formation. Valve
the examiner that a mechanical valve prosthesis
thrombosis should be suspected when the
is in situ based upon the examination findings
patient presents with acute dyspnoea or
so far prior to confirming the type of prosthesis
syncope, possibly with a recent history of
with the stethoscope.
inadequate anticoagulation. Patients may
Auscultatory findings include:
report loss of the normal valve clicking, with
For tilting-disc or bileaflet valves, a soft muffled or absent prosthetic valve closing
opening click may be audible. A high- sounds on auscultation. Valve thrombosis
frequency metallic click at S1 indicates can be treated medically (with heparin or
closure of a mechanical mitral valve thrombolysis) or surgically, depending on
replacement (MVR), or at S2 the closure of a the degree of obstruction and the size of
mechanical aortic valve replacement (AVR) the thrombus, and the clinical condition
For caged-ball valves, a loud rumbling sound of the patient. Such a decision requires a
is heard on auscultation due to the ball multidisciplinary approach. Valve failure can
rattling in the cage with forward flow of blood present acutely with cardiogenic shock, or
across the prosthesis. Opening and closing subacutely with cardiac failure and/or valve
sounds in these valves are of low-frequency haemolysis
and equal in intensity Prosthetic valve infective endocarditis: fever
Tissue valves have similar closing sounds to is the predominant clinical feature, and
native valves unexplained fever in a patient with a valve
Listen for soft flow murmurs across the valve. prosthesis should be treated as infective
Such flow murmurs will be systolic for AVRs endocarditis until proven otherwise. Other
and diastolic for MVRs presentations of prosthetic valve endocarditis
Auscultatory evidence of valvular include a new murmur, prosthetic valve
incompetence is abnormal failure, cardiac failure and embolic
28 Chapter 1 Cardiovascular system (station 3)

phenomena. Typical peripheral stigmata of appears to be well functioning with no evidence


native valve endocarditis are often absent in of infective endocarditis or cardiac failure. I
prosthetic valve infection. Ask the examiner would like to take a full history to establish the
for the temperature and a urine dip (looking details of his intervention, and my investigation
for microscopic haematuria), and comment of choice would be an echocardiogram to
on dental hygiene and the presence or review the valve function, orifice area and
absence of indwelling intravenous access as gradients.
sources of infection
Arrhythmia (atrial fibrillation): this is Case 2: Midline sternotomy with a
common post-cardiac surgery. Comment
on the rate, embolic complications and
metallic first heart sound.
peripheral stigmata of anticoagulation This woman has undergone a mechanical mitral
Anaemia (look for skin, conjunctival and valve replacement and is in rate-controlled
mucosal pallor). This may be secondary to: atrial fibrillation. A possible indication for this
Anticoagulation procedure in a woman of this age is previous
Prosthetic valve haemolysis due rheumatic mitral valve disease. Clinically,
to paravalvular leak: this requires the valve appears to be well functioning with
transoesophageal echocardiography evidence of anticoagulation treatment. There
for further evaluation, and potentially is no evidence of infective endocarditis or
reoperation if the resultant anaemia cardiac failure. I would like to take a full history
is severely symptomatic or requires to establish the details of her intervention,
repeated transfusions and my investigation of choice would be an
echocardiogram to review the valve function,
orifice area and gradients.
Prosthetic valve types
Mechanical valves can be divided into single-tilting Case 3: Midline sternotomy with a
disc, bileaflet (double-tilting disc) or caged-ball metallic second heart sound and
(Starr-Edwards) valves. The latter prosthesis is not diastolic murmur
used in the modern era (discontinued in 2007), and
was notable for a high incidence of valve - related
This man has undergone a mechanical aortic
thrombosis and haemolysis. Prostheses vary in their
valve replacement which is regurgitant as
evidenced by an early diastolic murmur at the
relative thrombogenicity, and and hence the target
lower left sternal edge. He appears to be clinically
INR for a patient with a mechanical prosthesis is
well compensated, however, with no evidence of
dependent upon the prosthesis type itself, and
infective endocarditis or cardiac failure. He has
patient-related risk factors (e.g. aortic position
also had coronary artery bypass grafting using
versus mitral position, AF, poor LV systolic function)
saphenous vein grafts either with or without a
left internal mammary artery graft. The most
State the most likely diagnosis likely underlying indication for this operation
on the basis of these findings would be degenerative calcific aortic stenosis and
coronary artery disease. I would like to take a full
The following three cases illustrate the range history to establish the details of his intervention,
of possible presentations with scars and valve and my investigation of choice would be an
prostheses in MRCP PACES. They demonstrate echocardiogram to determine if the regurgitation
the synthesis of the range of positive findings and is valvular or paravalvular, the severity of the
relevant negatives that have been outlined above. regurgitation and the impact upon left ventricular
function.
Case 1: Midline sternotomy with a
metallic second heart sound Offer relevant differential
This man has undergone a mechanical aortic
valve replacement with coronary artery bypass
diagnoses
grafting using saphenous vein grafts either Once the appropriate form of cardiac surgery
with or without a left internal mammary artery has been identified, give a differential
graft. The most likely indication for this would diagnosis of the likely underlying disease
be degenerative calcific aortic stenosis and condition(s) that would have warranted the
coronary artery disease. Clinically, the valve intervention.
Case 12 Prosthetic valves and cardiac surgery 29

Demonstrate the importance Lifelong anticoagulation is required for all


patients with mechanical prostheses and in
of clinical context suggest patients with tissue valve replacements who
relevant questions that would be have a separate indication for anticoagulation
(such as AF).
taken in a patient history Antibiotic prophylaxis (see Infective
Questions would include symptomatic response endocarditis, p. 40).
to intervention, adequacy of anticoagulation The choice between a tissue valve
in the case of mechanical prostheses, and replacement versus a mechanical prosthesis is
any new symptoms that may warrant further an individualised decision taken with the patient
investigation of valve replacements (e.g. fevers, and taking into consideration several patient-
rigors, dyspnoea, syncope). specific factors:
Patient choice: having weighed up the
Demonstrate an understanding advantages and disadvantages of each valve
of the value of further type
investigation Age and life expectancy: where life
expectancy is expected to be less than
Relevant laboratory blood tests would include: the predicted time to valve degeneration,
A full blood count to identify anaemia bioprostheses are acceptable. In contrast,
LDH, haptoglobin and blood film to identify mechanical valves are favoured in young
haemolysis individuals or patients with risks for
Clotting studies for anticoagulation accelerated tissue valve degeneration (e.g.
monitoring renal impairment, hyperparathyroidism)
Serial blood cultures and inflammatory Anticoagulation: patients with
markers if there is suspicion of prosthetic contraindications to anticoagulation or
valve endocarditis poor compliance are more preferable for
a tissue valve replacement. Patients with
Echocardiography is the central other anticoagulation indications or already
diagnostic modality for valve prostheses. receiving anticoagulation (such as AF) would
Serial transthoracic studies can be used to be more suitable for a mechanical prosthesis
follow-up prosthetic valve gradients and Women of childbearing age: this is a
compared with a baseline post-operative study. difficult decision with advantages and
Transoesophageal echocardiography is required disadvantages for and against each type. This
in suspected infected prostheses, in prosthetic is an individualised decision that is reached
valve regurgitation or paravalvular leak, and following detailed discussion with the patient
when acoustic shadowing from the metallic
prosthesis causing dropout of the ultrasound
signal limits the data obtained by transthoracic Further reading
study. Vahanian A, Baumgartner H, Bax J, et al. Guidelines
on the management of valvular heart disease:
the task force on the management of valvular
Always offer a management plan heart disease of the European Society of
Annual review by a cardiologist is required to Cardiology. Eur Heart J 2007; 28:230268.
review prosthetic valve function and rule out Vongpatanasin W, Hillis LD, Lange RA. Prosthetic
any complications, or review as soon as new heart valves. N Engl J Med 1996; 335:407416.
symptoms of valvular dysfunction develop. Dawkins S, Hobson AR, Kalra PR, et al. Permanent
Follow-up echocardiography parameters for pacemaker implantation after isolated aortic
the prosthetic valve should be compared to an valve replacement: incidence, indications, and
initial baseline post-operative echocardiogram. predictors. Ann Thorac Surg 2008; 85:108112.
Case 13: Cardiac failure

Instruction to the candidate Gallop rhythm


This 75-year-old woman presents with
Cardiac cachexia
worsening dyspnoea and reduced mobility.
Please examine her cardiovascular system. State the most likely diagnosis
on the basis of these findings
Begin with a summary of This patient has signs consistent with
positive findings congestive cardiac failure, likely secondary to
coronary artery disease as evidenced by her
Peripheral signs to identify in cases of cardiac
midline sternotomy scar and saphenous vein
failure:
graft harvest scars. She has increased right
Signs of poor perfusion such as cool and atrial pressure as evidenced by the raised JVP,
clammy peripheries, hypotension and has peripheral fluid retention to the knees.
When palpating the pulse, feel for: I would like to take a full history to establish her
AF: present in up to 3040% of patients symptomatic status, and my investigation of
with heart failure choice would be an echocardiogram to confirm
Pulsus alternans: alternating strong and the diagnosis.
weak palpable arterial pulsations, a sign Offer relevant differential diagnoses.
that confers a poorer prognosis The differential diagnosis of the causes of
Resting tachypnoea cardiac failure is listed below, divided into those
The apex beat may be displaced responsible for systolic heart failure, those
Raised JVP (state how high) that cause diastolic heart failure, and heart
An infraclavicular mass suggesting CRT, failure caused by a high-output state. They are
CRT-D, ICD alone, or bradycardia pacemaker accompanied where appropriate by clinical
(see Infraclavicular mass, p. 33) findings which may be elicited to suggest such
Bibasal reduced air entry (pleural effusions) diagnoses.
and crepitations (state the level)
Pitting peripheral and sacral oedema (state Systolic heart failure
the level) Ischaemic heart disease: signs would include
Ascites a midline sternotomy, venous graft harvest
Hepatomegaly scars, stigmata of CAD risk factors such as
Auscultatory features which should be xanthelasma or evidence of diabetes
listened for include: Cardiomyopathy: including idiopathic
dilated, alcohol-related (signs of chronic
An S3 with a gallop rhythm liver disease), peripartum cardiomyopathy
Murmurs indicative of valvular dysfunction. (young female), chemotherapy-induced
In the context of cardiac failure, mitral or (especially having received anthracyclines
tricuspid regurgitation may be causative or or trastuzumab note previous mastectomy
consequential lesions in heart failure. For and radiotherapy tattoos)
example, MR may cause heart failure or be Valvular heart disease
functional due to a dilated annulus. Similarly, Infection or inflammation: myocarditis,
TR can cause right heart failure or may occur Chagas disease
due to congestive cardiac failure Arrhythmia (tachycardiomyopathy)
S4 in restrictive disorders
Diastolic heart failure
Follow with a summary of Restrictive cardiomyopathy: including
relevant negative findings infiltrative disorders (the examplar disease
process being amyloidosis), sarcoid,
The presence or absence of signs of severity in haemochromatosis, connective tissue
cardiac failure should be commented upon, and diseases
include: Pericardial constriction (radiotherapy tattoos
Pulsus alternans over the left hemithorax)
Case 13 Cardiac failure 31

Hypertrophic cardiomyopathy Renal impairment


Hypertensive heart disease Nephrotic syndrome
Chronic venous insufficiency
High output cardiac failure A pelvic mass with impaired lymphatic
drainage
Thyrotoxicosis
Pregnancy
Nutritional deficiencies: beriberi, thiamine Demonstrate the importance
deficiency of clinical context suggest
Anaemia
relevant questions that would be
Pagets disease
Systemic arteriovenous fistulae taken in a patient history
Important questions in a history regarding
cardiac failure include:
Causes of a raised JVP
Aside from the aetiology of congestive heart Dyspnoea: ask to quantify exercise tolerance
failure, the differential diagnoses of key clinical and categorise according to NYHA class
signs need consideration, such as a raised JVP or Orthopnoea and paroxysmal nocturnal
peripheral oedema: dyspnoea
Peripheral oedema: are shoes or trousers
Congestive cardiac failure versus isolated more difficult to fit?
right-sided heart failure. Isolated acute Palpitations or syncope suggestive of
right heart failure in the context of RV arrhythmia
infarction or pulmonary embolism presents Angina
with a raised JVP but clear lung fields. In Cardiovascular risk factors
this setting, identification of isolated right History of previous cardiovascular, renal or
heart failure is key, as treatment is aimed liver disease
at fluid administration to support the right Medication history
heart circulation rather than diuresis in the
example of RV failure due to infarction
Pulmonary hypertension Demonstrate an understanding
Significant tricuspid regurgitation: giant v of the value of further
waves
Superior vena cava obstruction: non-
investigation
pulsatile, distended JVP Important investigations in the assessment of
Cardiac tamponade and pericardial cardiac failure include:
constriction: look for Kussmauls sign, Laboratory blood tests:
which is an increase in the level of the JVP Full blood count for anaemia
on inspiration. The JVP usually falls on Natriuretic peptides (BNP or NT-proBNP)
inspiration through increased venous return as markers of myocardial stretch and thus
to the right heart, but in constriction and volume overload
tamponade the right ventricle is unable to Urea, creatinine and electrolytes:
accommodate the increased inspiratory deranged renal function has implications
filling which is thus transmitted to the JVP on prognosis and may be the result of
Be careful not to confuse Corrigans sign (an cardiorenal syndrome or the deleterious
arterial pulsation) in severe AR with a raised JVP. impact of heart failure treatment. Renal
If congestive cardiac failure has resulted from impairment is an independent marker
severe AR, both signs may be present. of poor prognosis in patients with heart
failure, impacting upon both survival and
Peripheral oedema the scope for certain medical therapies with
known survival benefit. This is an important
The differential diagnosis of peripheral oedema consideration in managing cardiac failure
includes: A 12-lead electrocardiogram may some or all
Cardiac failure of the following:
Hypoalbuminaemia which can be due to Evidence of previous ischaemia,
impaired hepatic synthesis, malnutrition, arrhythmia, left ventricular hypertrophy
gastrointestinal or renal losses Poor R wave progression
32 Chapter 1 Cardiovascular system (station 3)

Left bundle branch block, the duration ACE inhibitors, or angiotensin receptor blockers
of the QRS complex being a criteria for if intolerant of ACE inhibtors, in patients with
cardiac resynchronisation therapy systolic dysfunction (ejection fraction 40%)
A normal ECG renders systolic heart Beta-blockers in patients with systolic
failure unlikely (<10%) as the cause of the dysfunction (ejection fraction 40%)
patients symptoms Mineralocorticoid receptor antagonists
A plain chest radiograph may show a variable (potassium-sparing diuretics) in patients
combination of cardiomegaly, pleural with systolic dysfunction (ejection fraction
effusions, alveolar pulmonary oedema , 35%) and NYHA class IIIV symptoms
Kerley B lines, fluid in the fissure, and upper Adding ivabradine, an inhibitor of the If ion
lobe blood diversion channel highly expressed in the sinoatrial
Echocardiography is the investigation of node, is indicated in patients in sinus rhythm
choice. It allows assessment of biventricular over 70 beats per minute who remain in
size, systolic and diastolic function, and NYHA classes IIIV with an EF 35% and on
can demonstrate evidence of a previous optimal therapy.
infarction. It also allows assessment of The combination of hydralazine and
valvular aetiologies or consequences of isosorbide dinitrate (ISDN) may be
cardiac failure and the impact of left heart considered to reduce the risk of heart failure
failure on estimated pulmonary artery or hospitalisation in patients intolerant
pressure of ACE inhibitors or angiotensin receptor
Cardiac MRI allows additional assessment blockers. One study (A-HeFT) showed
of myocardial (especially infiltrative) disease a mortality benefit of this combination
and viability when revascularisation is being specifically in African-Americans.
considered
These medications must be prescribed
Cardiac catheterisation is used to investigate
initially at an introductory dose and then
coronary lesions responsible for impaired
up-titrated to the maximum tolerated dose,
systolic function. It also allows assessment
bearing in mind side effects (especially renal
of haemodynamic parameters, such as left
dysfunction and hypotension). Trial data for
ventricular end-diastolic pressure, pulmonary
mortality benefit is derived from up-titration
capillary wedge and pulmonary arterial
of medication to optimal doses rather than
pressures. Endomyocardial biopsy is rarely
remaining on the introductory dose. This is
required but can be considered in certain cases.
where an MDT approach, with input from a
community heart failure nurse specialist, is key.
Always offer a management plan Other ongoing treatment considerations
General principles that should be applied include: which should be recognised include:

Treat the underlying cause, such as targeted Anticoagulation in the presence of AF or


therapy for hypertensive heart disease or apical thrombus
intervention on symptomatic valvular heart Devices: cardiac resynchronisation therapy
disease (CRT, also known as biventricular pacing),
Fluid and salt restriction implantable cardiac defibrillators (ICDs, with
Immunisations (influenza, pneumococcal) or without CRT) (see Infraclavicular mass,
p. 33)
Symptomatic treatment includes diuresis Referral to a specialist centre for
with either oral or intravenous preparations. transplantation assessment and bridges to
In cases of congested gut mucosa, bumetanide transplantation (ventricular assist devices)
has superior oral bioavailability compared with
furosemide. Metolazone can be combined with
intravenous diuresis in severe cases of fluid Further reading
retention, but warrants careful monitoring of Dickstein K, Cohen-Solal A, Filippatos G, et al. ESC
renal function and electrolytes. Ultrafiltration Guidelines for the diagnosis and treatment
can be used in cases of fluid overload refractory of acute and chronic heart failure 2008: the
to diuretics. There is some evidence that task force for the diagnosis and treatment of
this treatment modality may also reset the acute and chronic heart failure 2008 of the
sensitivity of the kidneys to diuretic therapy. European Society of Cardiology. Developed in
While diuretics only provide symptomatic collaboration with the Heart Failure Association
benefit, several medications have evidence of of the ESC (HFA) and endorsed by the European
mortality benefit. These include: Society of Intensive Care Medicine (ESICM). Eur
Heart J 2008; 29:23882442.
Case 14: The infraclavicular mass
pacemakers and implantable devices

Instruction to the candidate It is important to always check the


contralateral infraclavicular region for scars
This 60-year-old man previously complained
from previous device implantations
of dyspnoea and reduced exercise tolerance.
Please examine his cardiovascular system.
State the most likely diagnosis
Begin with a summary of on the basis of these findings
positive findings This patient has signs consistent
with congestive cardiac failure and a
The key positive finding in this case is an
subcutaneously implanted cardiac device
infraclavicular subcutaneous mass, the
in the infraclavicular region, the differential
differential diagnosis of which includes:
of which includes a bradycardia pacemaker,
A conventional bradycardia pacemaker CRT device with or without defibrillator, or a
(single or dual chamber) standalone ICD. He has increased right atrial
CRT-P: cardiac resynchronisation therapy pressure as evidenced by the raised JVP, and
with biventricular pacing only has peripheral fluid retention to the knees. I
CRT-D: biventricular pacemaker and an would like to take a full history to establish
incorporated defibrillator device his symptomatic status and the nature of the
ICD: implantable cardiac defibrillator implanted device.

Which device? Offer relevant differential


Examine and describe the pacemaker as you diagnoses
would any other mass found on examination. As stated in the list of positive findings, an
The interpretation of your findings will be putting implanted device can be one of the following:
the type of device into the context of the other
examination findings (e.g. with signs of heart failure).
A bradycardia pacemaker
CRT-P
Additional findings may include:
CRT-D
ICD
Signs of heart failure (such as poor perfusion Implantable loop recorder
with cool and clammy peripheries, displaced
While the size of the device cannot tell you
apex beat, raised JVP, bibasal reduced air entry
definitively which device has been implanted,
and crepitations, pitting peripheral oedema
implantable loop recorders are the size of a USB
see preceding Cardiac failure, p. 30)
stick. CRT and ICD devices tend to be larger
An abdominal subcutaneous mass in patients
than standard bradycardia pacemakers.
with evidence of previous cardiothoracic
surgery (such as a midline sternotomy scar),
indicating a surgically-implanted epicardial Demonstrate the importance
pacemaker of clinical context suggest
A left parasternal subcutaneous mass with
an approximate size of a USB memory stick: relevant questions that would be
this is an implantable recording device that taken in a patient history
continuously monitors cardiac rhythm
The exact questions which would be asked in
a clinical history would depend upon the type
Follow with a summary of of device in situ. Inform the examiner that you
would ask for general symptoms of arrhythmia
relevant negative findings such as palpitations, dizziness, pre-syncope and
Important negative findings to note in their syncope.
absence and comment on if positive include: Further to this, if an ICD or CRT-D is in situ,
Stigmata of infective endocarditis patients should be asked about any delivery of
Complications of device implantation, device therapies noticed by the patient, i.e. shocks.
such as erythema indicative of infection, For patients with CRT devices, it is important
haematoma formation or protruding wires to consider symptoms of cardiac failure and
34 Chapter 1 Cardiovascular system (station 3)

categorise according to NYHA class. Patients Always offer a management plan


with CRT will, by definition, have had severe
It is sensible to start off discussion of management
left ventricular dysfunction, so assessing their
of patients with implantable devices by discussing
symptom burden and its change since device
the indications for device implantation. CRT and
insertion is important.
ICDs both have a plethora of proven mortality
benefit upon which the NICE guidelines are
Demonstrate an understanding based, summarised below.
of the value of further
ICDs
investigation
ICDs have indications in both primary and
In a patient with an implanted device, the secondary prevention settings (primary
necessary investigations will depend upon the prevention with ICDs in heart failure are covered
clinical presentation of the patient. For example, in the CRT section below).
if a patient with an implanted device (of any
type) presents with symptoms and/or signs of
infection it would be important to take serial
Primary prevention
blood cultures and inflammatory markers with Individuals who have a familial cardiac
the suspicion of device-related infection or condition with a high risk of sudden death
device-related infective endocarditis. Hypertrophic cardiomyopathy
General investigations will include: Long QT syndrome
Brugada syndrome
A 12-lead electrocardiogram. The QRS Arrythmogenic right ventricular dysplasia
duration is important because CRT is only (ARVD)
implanted in patients with a broad QRS, Patients who have undergone surgical repair
indicating ventricular dysynchrony. Pacing of congenital heart disease
spikes vary in location (atrial or ventricular)
and amplitude (depending upon the lead Secondary prevention
implantedmore prominent in unipolar
rather than bipolar leads) Patients with proven previous serious
A plain chest radiograph will allow ventricular arrhythmia in the absence of a
assessment of the number and type of treatable cause:
leads from a device. An ICD lead and the Survivors of cardiac arrest caused by VT or VF
left ventricular lead of a CRT device both Spontaneous sustained VT causing
have a different radiological appearance haemodynamic instability and/or syncope
when compared to a conventional pacing Sustained VT (regardless of syncope or
lead. Chest radiography helps visualise lead cardiac arrest) in the presence of severe
placement and integrity, including lead left ventricular dysfunction (defined as an
fracture. AP and lateral views are required to ejection fraction 35%) and with heart failure
check position symptoms no worse than NYHA class III.
Echocardiography guides the indications
for CRT and ICD implantation through CRT-P and CRT-D
the assessment of severely impaired LV
ejection fraction (35%). LV dyssynchrony As a treatment option for patients with heart
assessment has no role in the indications for failure and severe underlying left ventricular
CRT-P. Furthermore, current evidence for systolic dysfunction (defined as an ejection
echocardiography in CRT optimisation is fraction 35%), ICDs, CRT-P and CRT-D are
uncertain and hence is not recommended as indicated depending upon NYHA class, QRS
standard practice duration, and the presence/absence of LBBB
Device interrogation is used to look for (Table 1.8).
arrhythmia (ventricular or atrial), delivery In addition to discussing the indications for
of ICD therapies (including anti-tachycardia device insertion, it is important to consider the
pacing and defibrillation episodes, and advice that a patient should be given regarding
whether therapies were inappropriate their device. For example, the patient will be
or appropriate) and satisfactory device given a pacemaker identification card to keep
parameters (such as battery life, lead after their device implantation. This will give
sensitivities and impedance) details of the device manufacturer and of the
model of device (information that is required
Case 14 The infraclavicular mass pacemakers and implantable devices 35

Table 1.8 Implantable device treatment options for patients with severe LV dysfunction
(ejection fraction 35%) stratified by NYHA class, QRS duration, and the presence/
absence of LBBB (reproduced from NICE guideline TA314)
QRS interval NYHA class
I II III IV
<120 ms ICD if there is a high risk of sudden cardiac death ICD and CRT not
clinically indicated
120149 ms without LBBB ICD ICD ICD CRT-P
120149 ms with LBBB ICD CRT-D CRTP or CRT-D CRT-P
150 ms with or without CRT-D CRT-D CRT-P or CRTD CRT-P
LBBB

to select the correct equipment for device devices from the British Heart Foundation
interrogation). (BHF) website. These contain a wide range of
Implantable devices are followed-up in pacing information covering the indications for the
clinics in order to check parameters such as device, how the device works, the implantation
battery life, satisfactory function and arrhythmia procedure and post-procedural care, and advice
burden (e.g. AF burden for dual-chamber pacing regarding daily living with a device.
or arrhythmic events recorded by ICDs). Device
optimisation is also performed on an outpatient
basis in the case of CRT devices.
Further reading
Any implantable device has implications for National Institute for Health and Care Excellence
vehicle licence holders, and all patients who have (NICE). TA314: Implantable cardioverter
undergone any form of pacing or ICD device defibrillators and cardiac resynchronisation
implantation must inform the DVLA (Table 1.9). therapy for arrhythmias and heart failure (review
Patients can download information leaflets of TA95 and TA120). NICE, June 2014.
regarding bradycardia pacing, CRT and ICD

Table 1.9 Driving guidelines for patients with pacemakers and ICDs
Car or motorcycle driving licence holders (UK Group 1) Heavy goods vehicle or
passenger-carrying vehicle
driving licence holders (UK
Group 2)
Pacemakers Patients can resume driving one week after the pacemaker has been Patients can usually resume
implanted provided that they inform the DVLA, they do not have any driving after 6 weeks provided
recurrence of bradycardia symptoms (such as syncope or dizziness), they have informed the DVLA
they do not have any other medical condition that might preclude and subsequently received
them from driving (e.g. recent myocardial infarction), and they continue clearance from them following
to have their device checked regularly by the pacemaker clinic a review of their case
ICDs Primary prevention ICD: can resume driving 1 month after implantation. Unable to hold this licence
Secondary prevention ICD: can resume driving 6 months after following ICD implantation
implantation.
Appropriate ICD device therapy:
can resume driving after 6 months provided that a new treatment
strategy (either anti-arrhythmic medication or ablation) has been
successfully implemented, or
can resume driving after 24 months if no new treatment strategy
is implemented.
Case 15: The irregular pulse: atrial
fibrillation

Instruction to the candidate Request the patients blood pressure to


exclude this
This 65-year-old man complains of palpitations
and exertional dyspnoea. Please examine his
Left ventricular dysfunction may be an
associated finding, an aetiological factor,
cardiovascular system.
or indeed a consequence of long-standing
uncontrolled ventricular rate (known as
Begin with a summary of tachycardiomyopathy)
positive findings An important complication of
anticoagulation is bleeding, the stigmata of
Comment on the pulse rate and irregular which include bruising. Additionally, while
rhythm, measured both radially by palpation signs of previous stroke in a patient with
and apically by auscultation. The pulse atrial fibrillation are most likely to be due to
difference between the apical and radial rates embolic phenomena, over-anticoagulation
is an indicator of the degree of rate control: the may also cause haemorrhagic strokes
higher the difference between the two (termed
the pulse deficit), the poorer the rate control.
Clinical examination may also help identify State the most likely diagnosis
the aetiology of atrial fibrillation. Such findings on the basis of these findings
may include:
This patient has rate-controlled atrial
A midline sternotomy scar and graft harvest fibrillation, with possible aetiologies including
scars which would indicate bypass grafting hypertension and previous cardiothoracic
for underlying ischaemic heart disease, and surgery for ischaemic heart disease. He
previous cardiac surgery as a possible cause has signs of recent venipuncture, likely in
of AF order to check adequacy of anticoagulation.
Signs of thyrotoxicosis such thyroid eye He does not have any features of embolic
disease, goitre, pretibial myxoedema complications.
Hypertension can be identified by asking for

the patients blood pressure


Findings consistent with mitral valve
Offer relevant differential
pathology, such as mitral facies, murmurs diagnoses
Inspect for a permanent pacemaker: tachy- The differential of an irregularly irregular pulse
brady syndrome, requiring pace and block includes:
treatment
Peripheral stigmata of alcoholic liver disease Atrial fibrillation
Atrial flutter or atrial tachycardia with
variable block
Follow with a summary of Sinus rhythm with frequent atrial or
relevant negative findings ventricular extrasystoles. This can be
ruled out on exercise which would abolish
The important relevant negatives to comment
extrasystoles
on in cases of atrial fibrillation pertain to the
complications of atrial fibrillation itself and to
Sinus arrhythmia may also manifest as an
irregularly irregular pulse.
complications of anticoagulation:
Complications of atrial fibrillation include: Demonstrate the importance
Embolic complications such as stroke.
Approximately 20% of strokes result from of clinical context suggest
atrial fibrillation, conferring significant relevant questions that would be
importance on instigating appropriate
anticoagulation strategies taken in a patient history
Haemodynamic compromise, e.g. in atrial Questions would relate both to any history of
fibrillation with fast ventricular response. underlying causes and to current symptoms.
Case 15 The irregular pulse: atrial fibrillation 37

Demonstrate an understanding of The second consideration is whether to


pursue a rate or rhythm control strategy.
the value of further investigation For the former strategy, there is no evidence
The 12-lead ECG is the most important tool in that strict rate control is beneficial against
diagnosing the cause of an irregular pulse. It is lenient rate control. Medical therapy for rate
best to obtain a rhythm strip to accurately assess control includes beta-blockers, rate-limiting
the cardiac rhythm. calcium channel blockers and digoxin (the
Other investigations would include: latter preferably used as an adjunctive agent
rather than as a standalone medication). In
Laboratory blood tests: thyroid function
cases whereby medical therapy fails to provide
tests as a possible underlying aetiology,
adequate rate control, AV node ablation with
and clotting studies to assess the adequacy
permanent pacemaker implantation can be
of anticoagulation. Cardiac biomarkers
considered.
(e.g. troponin) may be elevated, but
With regard to rhythm control, many trials
differentiation between arrhythmia or
have demonstrated that there is no evidence
underlying ischaemia as the aetiology require
that mortality is improved by pursuing this
further clinical evaluation.
strategy (the largest being the AFFIRM trial
A 24-hour Holter monitor allows review of
in 2002, consisting of 4,060 patients: this trial
the AF burden in cases of paroxysmal AF and
did not demonstrate any difference between
to assess for the adequacy of rate control in
rate- and rhythm-control groups for the
cases of permanent AF
primary end-point of all-cause mortality). In
Echocardiography is used to rule out
general, rhythm control should be considered
any structural cardiac lesion (e.g. mitral
particularly for young patients, individuals
regurgitation) and to assess left atrial size and
who are physically active, and patients who
left ventricular function
remain symptomatic despite adequate rate
If ischaemic heart disease is a possibility, the
control. Rhythm control may be achieved by
relevant stress test should be considered
either medication or pulmonary vein isolation
(so-called AF ablation). Both have side-
Always offer a management plan effects and potential for complications, hence
There are two separate, principle considerations the guidance for opting for rhythm control
in managing AF. The first is anticoagulation, which in individuals in whom the symptomatic
is the only demonstrable intervention that reduces burden of AF outweighs these disadvantages
cardiovascular mortality in AF. Anticoagulation of treatment. In the absence of structural heart
should be instituted according to risk profile for disease, Flecainide (a class 1c anti-arrhythmic)
embolic stroke (CHADS-VASc score for non- can be used usually in combination with
valvular AF) and weighed up against bleeding risk a beta-blocker. The latter agent is to avoid
(HAS-BLED score). Neither the nature of the AF rapidly conducted atrial flutter which is a
(paroxysmal, persistent or permanent) nor the common by-product of Flecainide when
AF burden in paroxysmal AF should influence used to treat AF. In cases of structural heart
the indication for anticoagulation: they should disease (e.g. adult congenital heart disease,
all be managed in the same way with respect to LV impairment), Amiodarone is generally
anticoagulation. A target INR of 23 is indicated recommended.
when treated with warfarin. The examiner will want to feel that they can
trust you to safely run and manage their acute
Novel strategies for thromboprophylaxis in AF unselected medical take. AF is the commonest
The novel oral anticoagulants (e.g. arrhythmia encountered in this setting, either as
Dabigatran) are effective alternatives to the primary diagnosis or as an incidental finding
warfarin that can be considered, but their or co-morbidity. Therefore, an appropriate
place in current practice is influenced by scheme should be adhered to in order to
local policy. Whilst these agents do not prioritise the key management aspects of AF in
require blood test monitoring, it should be the acute medical patient:
remembered that they also do not have Is the patient haemodynamically
specific antidotes for reversal. compromised? If so, arrange emergency DC
Left atrial appendage occlusion devices (e.g. cardioversion. Otherwise, decide how this
Watchman) are also recommended by NICE in can be managed medically.
specific circumstances of non-valvular AF in Decide upon the anticoagulation treatment
which anticoagulation is not tolerated. regime in the acute setting
38 Chapter 1 Cardiovascular system (station 3)

Choose the most appropriate the ACC/AHA/ESC 2006 guidelines for the
pharmacological strategy for either rate management of patients with atrial fibrillation:
control, or rhythm control if the onset is a report of the American College of Cardiology
within 48 hours Foundation/American Heart Association
Treat underlying cause, most commonly Task Force on Practice Guidelines developed
infection in the acute setting in partnership with the European Society
of Cardiology and in collaboration with the
Following the acute management of the patient, European Heart Rhythm Association and the
go on to investigate underlying causes and Heart Rhythm Society. J Am Coll Cardiol 2011;
decide upon long-term anticoagulation and 57:e101198.
rate/rhythm control strategies. Camm AJ, Kirchhof P, Lip GY, et al. Guidelines for the
management of atrial fibrillation: the task force
Further reading for the management of atrial fibrillation of the
European Society of Cardiology (ESC). Eur Heart
Fuster V, Rydn LE, Cannom DS, et al. 2011 ACCF/
J 2010; 31:23692429.
AHA/HRS focused updates incorporated into

Case 16: Coarctation of the aorta

Instruction to the candidate A thrill over the suprasternal notch may be


This 30-year-old woman was found to palpated
be hypertensive when attending her GP A heaving (pressure-loaded), undisplaced apex
for abdominal pains. Please examine her beat indicates left ventricular hypertrophy
cardiovascular system. An underdeveloped left arm may represent a
coarctation at the level of the left subclavian
artery. There may also be underdevelopment
Begin with a summary of of the lower limbs.
positive findings Auscultatory features of aortic coarctation
include:
Relevant peripheral signs to seek to identify
include:
A systolic murmur, typically heard posteriorly
in the left infrascapular region but can also
Simultaneous palpation of radial and be auscultated in the left infraclavicular
femoral pulses reveals the characteristic region
radio-femoral delay. Indeed, delayed and The continuous murmurs of collateral
diminished pulses distal to the coarctation vessels can be heard bilaterally across the
with differential limb blood pressures are thorax
the characteristic features of this condition. An ejection click (with or without
Auscultatory features are supportive concomitant ejection systolic and early
Upper body systolic hypertension and lower diastolic murmurs) of an associated bicuspid
body systolic hypotension with a difference aortic valve
of greater than 20 mmHg in favour of the
upper limb is significant. Also check blood
pressure in both arms as the left subclavian
Follow with a summary of
artery can be either part of the coarctation relevant negative findings
itself, or used in repair of the defect. It would be prudent to mention the absence
Visible, palpable large collateral arteries of clinical features consistent with Turners
develop over the left scapula, left chest wall syndrome, as at least 35% of these patients also
and anterior abdominal wall have aortic coarctation.
Case 16 Coarctation of the aorta 39

State the most likely diagnosis and premature coronary and cerebral artery
disease.
on the basis of these findings
This patient has signs consistent with a coarctation
of the aorta associated with a bicuspid aortic valve
Demonstrate an understanding of
and evidence of aortic regurgitation. I would like the value of further investigation
to take a full history to establish her symptomatic Relevant investigations in the diagnostic work-
status and my investigations of choice would be an up of cases of aortic coarctation include:
echocardiogram and a CT or MRI of the aorta in
the first instance to confirm the diagnosis and to A 12-lead electrocardiogram demonstrating
look for associated pathology. left ventricular hypertrophy with or without
an accompanying strain pattern
A plain chest radiograph may demonstrate:
Offer relevant differential Rib notching on the inferior borders of
diagnoses the posterior ribs, which represents the
In addition to offering the differential diagnosis development of collaterals arising from
of a systolic murmur (see Aortic stenosis, p. 2) intercostal arteries
it is important to consider associated valvar The coarctation can appear as an
conditions: indentation in the proximal descending
thoracic aorta, giving rise to the so-called
Bicuspid aortic valve: up to 85% of patients figure of 3 sign
with aortic coarctation also have a bicuspid Echocardiography is important in
aortic valve with or without associated confirming the diagnosis of coarctation
stenosis or regurgitation with measurement of the pressure gradient
Other cardiac defects include VSD, patent across the lesion which can be made by
ductus arteriosus, aortic stenosis and mitral 2D echo, colour Doppler and continuous-
stenosis due to parachute mitral valve, and wave Doppler. Aortic dimensions and left
other left ventricular outflow tract obstructive ventricular function and wall thickness are
lesions (which can be part of the Shone also assessed and associated cardiac lesions
complex of left-sided obstructive lesions) are ruled out
Note from the relevant negatives that Turner
Cardiac catheterisation can also be used to
measure the pressure gradient across the
syndrome is a genetic condition associated with
coarctation.
aortic coarctation so this should be mentioned
with regard possible aetiologies.
CT and MRI are appropriate imaging
modalities. The narrowing of the aorta
occurs most commonly just distal to the left
Demonstrate the importance subclavian artery. These imaging modalities
allow visualisation and measurement of the
of clinical context suggest aorta in its entirety, as well as demonstration
relevant questions that would be of collateral vessels
taken in a patient history
Many adult patients are asymptomatic, being Always offer a management
diagnosed following the finding of hypertension. plan
Those patients who are symptomatic patients
may complain of: Patients with aortic coarctation should be
considered for referral to a specialist adult
Headache congenital heart disease (ACHD) centre for
Nosebleeds life-long follow-up.
Dyspnoea Hypertension persists despite coarctation
Leg claudication repair and may be due to baroreceptor
Abdominal angina dysfunction. There is also loss of aortic
Cold feet compliance. This requires lifelong monitoring.
Regular ambulatory BP monitoring and stress
Other disease associations include: ascending
testing for exercise hypertension (SBP >220
aorta aneurysms (complicated by rupture or
mmHg), should be performed. Intervention for
dissection), berry aneurysms of the circle of
lone coarctation may be performed by stenting
Willis (complicated by intracranial haemorrhage)
(method of choice) or surgically.
40 Chapter 1 Cardiovascular system (station 3)

Later complications such as aneurysm Further reading


formation and recoarctation can occur.
Deanfield J, Thaulow E, Warnes C, Webb G, et al.
Associated cardiac (such as bicuspid aortic
Management of grown up congenital heart
valve, aortic aneurysm) or neurological (such as
disease. Eur Heart J 2003; 24:10351084.
circle of Willis berry aneurysms) features would
favour a surgical approach.

Case 17: Infective endocarditis

Instruction to the candidate lesions in the fingers), Roths spots (retinal


haemorrhages), glomerulonephritis
This 50-year-old man presented with fevers and
(microscopic haematuria on urine dipstick)
dyspnoea. Please examine his cardiovascular
system.
Other signs of a more established
(historically termed subacute) infection
include clubbing, splenomegaly and anaemia
Likely cases for the PACES exam
It is important to consider right-sided valvular
Patients with active endocarditis are unlikely to lesions particularly in patients with indwelling
present as a case in PACES, but knowledge of venous catheters or stigmata of intravenous drug
the clinical signs, management, risk factors and use.
guidance on prophylaxis is relevant to many other
stations where the patient may be at increased risk
of developing this complication. Follow with a summary of
relevant negative findings
Begin with a summary of If absent, comment on risk factors for the
development of infective endocarditis such as
positive findings stigmata of intravenous drug abuse, indwelling
The key clinical findings are fever and a murmur. venous catheters, poor dental hygiene, and
While the patient in the exam will unlikely have clinical evidence of high-risk cardiac conditions.
an active fever (see text box above), such clinical Additionally, it is important to show a
information has been given in the instruction consideration of the possible complications of
to the candidate. Therefore, in the clinical infective endocarditis by commenting on the
examination, the presence of a murmur will be absence of signs of heart failure and of evidence
the key clinical finding. This may either be the of systemic septic emboli, such as cerebral
substrate for infection (e.g. native valve disease events. Remember that right-sided lesions tend
with superimposed endocarditis) or indeed to cause pulmonary septic emboli.
the consequence of infection (e.g. valvular
destruction due to infection with consequent
incompetence). State the most likely diagnosis
Additional clinical signs to seek include: on the basis of these findings
Vascular phenomena. These include splinter This patient who has a history of a febrile illness
haemorrhages (nailfold infarcts), Janeway has examination findings consistent with mitral
lesions (painless macular or nodular regurgitation. This lesion is either a substrate
haemorrhagic lesions on the palms or soles) for or consequence of infective endocarditis.
Immunological phenomena, such as He is clinically euvolaemic, and has evidence
Oslers nodes (tender raised subcutaneous of poor dentition. I would like to take a full
Case 17 Infective endocarditis 41

history to establish his symptomatic status, and Investigations will include:


my initial investigations would include serial
blood cultures, inflammatory markers and Blood cultures: three separate sets of aerobic
transthoracic echocardiography. and anaerobic cultures
Serology: for Bartonella, Chlamydia psittaci
and Coxiella
Offer relevant differential Regular 12-lead electrocardiograms
diagnoses are essential to look for a prolonging
PR interval indicating atrioventricular
Report to the examiner that you would consider node involvement in cases of aortic root
the differential diagnosis of the likely causative abscesses
organisms of infective endocarditis, which Transthoracic echocardiography is the
includes Staphylococcus aureus, which is the first-line imaging modality, followed by
most common aetiological organism, followed transoesophageal echocardiography in
by Streptococcus viridans and coagulase- either positive transthoracic studies (to look
negative Staphylococci. Staphylococcus aureus is for abscesses and more accurately assess
typically associated with nosocomial acquisition vegetation dimensions) or negative studies
due to instrumentation (e.g. venous or urinary following which the clinical suspicion of
catheters) or community intravenous drug use. endocarditis remains high
Streptococcus viridans is the most frequent
causative agent in community-acquired
infective endocarditis, being part of the normal Diagnostic criteria for infective
oral flora. endocarditis
HACEK organisms also constitute part of the
oral flora, and are associated with endocarditis The modified Duke criteria (2000) establish a
in patients with poor dentition or intravenous diagnosis of infective endocarditis if 2 major
drug users due to the contamination of criteria, 1 major and 3 minor, or 5 minor criteria are
hypodermic needles with saliva. present.
Culture-negative endocarditis requires The major criteria are:
serology for Bartonella, Chlamydia psittaci and Typical serial positive blood cultures (the
Coxiella. number of cultures depending upon the
Fungal endocarditis is associated with organism identified)
immunocompromised states, intravenous Typical echocardiography features (including
drug use and indwelling venous catheters independently mobile valvular, subvalvular or
(particularly in patients receiving total device related mass or masses, new valvular
parenteral nutrition). regurgitation, prosthesis dehiscence or
abscess formation)
The minor criteria are:
Demonstrate the importance Fever > 38oC
of clinical context suggest Predisposing factor (such as valve prosthesis,
native valve lesion, intravenous drug user)
relevant questions that would be Evidence of vascular phenomena (e.g. splinter
taken in a patient history haemorrhages)
It is important to elicit whether the patient has a Evidence of immunological phenomena (such
pre-existing cardiac abnormality or pre-existing as glomerulonephritis)
endocarditis increasing the risk of subsequent Positive blood culture result or serological
episodes. evidence of a causative organism that does
not meet major criterion

Demonstrate an understanding
Always offer a management plan
of the value of further
The initial management of cases of infective
investigation endocarditis is appropriate antimicrobial
The investigative strategy in cases of infective therapy. The specific antimicrobial drugs used
endocarditis is aimed at meeting the diagnostic and the duration of treatment depends upon
criteria for the condition. The key principles the identity and sensitivities of the organism
are of gaining microbiological evidence and isolated and the case itself. Factors to consider
echocardiographic evidence of the disease. when starting empirical treatment include:
42 Chapter 1 Cardiovascular system (station 3)

Taking 3 sets of blood cultures 30 minutes Instead, the guidance focuses upon appropriate
apart prior to commencing therapy antimicrobial cover for high risk patients
The appropriate antimicrobials will differ undergoing gastrointestinal or genitourinary
between cases of native and prosthetic valve procedures where there is suspected
endocarditis concomitant infection. Furthermore, the
Local guidance regarding specific importance of good dental hygiene should be
antimicrobials will be influenced by the conveyed to the patient.
prevalence of antibiotic-resistant organisms For prophylaxis with respect to dental
procedures that involve manipulation of the
The indications for surgical management of
gingival mucosa or the periapical region of teeth
infective endocarditis are:
or perforation of the oral mucosa, the guideline
Heart failure due to severe valvular bodies differ in their advice. NICE guidance does
regurgitation or fistula formation not routinely recommend antibiotic prophylaxis
Persistent, uncontrolled infection refractory in this scenario, with the benefits and risks of
to antimicrobials prophylaxis explained to high-risk patients.
To prevent systemic embolism in appropriate ESC and AHA guidelines state that antibiotic
cases prophylaxis should, however, be considered in
high-risk patients for such dental procedures.
The pragmatic approach is to consult with local
Prophylaxis against endocarditis guidance and protocols. NICE do state that
Guidance for prophylaxis against endocarditis the guidelines do not overrule individualised
is now stratified according to the presence of treatment decisions appropriate to the patient by
pre-existing cardiac lesions that are deemed the clinician.
high-risk for the development of endocarditis,
coupled with the medical procedure or
intervention taking place. Firstly, the following
Further reading
conditions are common to NICE, ESC and AHA Wilson W, Taubert KA, Gewitz M, et al. Prevention
guidelines in being classified as high-risk for the of infective endocarditis: guidelines from the
development of infective endocarditis: American Heart Association: a guideline from
the American Heart Association Rheumatic
Prosthetic cardiac valve or prosthetic Fever, Endocarditis, and Kawasaki Disease
material used for cardiac valve repair Committee, Council on Cardiovascular Disease
Previous infective endocarditis in the Young, and the Council on Clinical
Congenital heart disease: Cardiology, Council on Cardiovascular Surgery
Unrepaired cyanotic congenital heart and Anesthesia, and the Quality of Care and
disease Outcomes Research Interdisciplinary Working
Completely repaired congenital heart Group. Circulation 2007; 116:17361754.
defect with prosthetic material or device Nishimura RA, Carabello BA, Faxon DP, et al. ACC/
prior to endothelialisation (judged AHA 2008 Guideline update on valvular
as during the first 6 months after the heart disease: focused update on infective
procedure) endocarditis: a report of the American College
Repaired congenital heart disease with of Cardiology/American Heart Association Task
residual defects at the site or adjacent to Force on Practice Guidelines endorsed by the
the site of a prosthetic patch or device Society of Cardiovascular Anesthesiologists,
Society for Cardiovascular Angiography and
In addition, NICE also classify the following
Interventions, and Society of Thoracic Surgeons.
two conditions as high-risk:
J Am Coll Cardiol 2008; 52:676685.
Acquired valvular heart disease (stenosis or Habib G, Hoen B, Tornos P, et al. Guidelines on the
regurgitation) prevention, diagnosis, and treatment of infective
Hypertrophic cardiomyopathy endocarditis (new version 2009): the task force
on the prevention, diagnosis, and treatment of
The AHA also specify cardiac transplantation infective endocarditis of the European Society
recipients who develop cardiac valvulopathy as of Cardiology (ESC). Endorsed by the European
a separate high-risk group. Society of Clinical Microbiology and Infectious
NICE no longer advises routine prophylaxis Diseases (ESCMID) and the International Society
for high-risk patients undergoing respiratory, of Chemotherapy (ISC) for Infection. Eur Heart J
gastrointestinal or genitourinary procedures. 2009; 30:23692413.
Case 18: Pulmonary hypertension

Instruction to the candidate peripheral oedema and ascites should be noted.


Additional relevant negative findings include
This 50-year-old woman complains of a one
stigmata of anticoagulation. Look for tunnelled
year history of progressive exertional dyspnoea.
lines and infusion pumps indicative of parenteral
Please examine her cardiovascular system.
prostacyclin analogues.

Begin with a summary of State the most likely diagnosis


positive findings on the basis of these findings
Peripheral signs to be identified in cases of
This woman has signs consistent with
pulmonary hypertension include a raised
pulmonary arterial hypertension, likely
JVP (quantify how high), with giant V waves if
secondary to systemic sclerosis. She has signs
significant tricuspid regurgitation is present. A
of right heart failure, and is receiving formal
parasternal right ventricular heave may also be
anticoagulation. I would like to take a full history
identified.
to establish her symptomatic status, and my
Auscultatory features of pulmonary
screening investigation of choice would be an
hypertension include:
echocardiogram followed by right heart catheter
A loud P2 to confirm the diagnosis.
Paradoxical splitting of the second heart sound
An early diastolic murmur of pulmonary Offer relevant differential
regurgitation (Graham Steell murmur)
diagnoses
Follow with a summary of The causes of pulmonary hypertension can
be divided clinically according to the WHO
relevant negative findings classification (Table 1.10).
If absent, signs of right heart failure, such as

Table 1.10 WHO classification of causes of pulmonary hypertension


WHO group Description Features to look for at examination
1 Pulmonary arterial hypertension (PAH): Stigmata of systemic sclerosis or SLE
Idiopathic Stigmata of portal hypertension
Familial Signs of congenital heart disease with a shunt
Associated with other diseases: Evidence of previous intravenous drug use
Connective tissue disease
Porto-pulmonary
Congenital heart disease with pulmonary-
systemic shunts
HIV
Drugs (e.g. anorexigens)
2 Associated with left heart disease Signs of left ventricular systolic dysfunction
Signs of diastolic dysfunction (e.g. hypertrophic
cardiomyopathy)
Signs of valvular heart disease (e.g. MS or MR)
3 Associated with lung disease Signs of:
COPD
Interstitial lung disease
Sleep apnoea
4 Chronic thromboembolic disease Previous DVTs
5 Miscellaneous
SLE, systemic lupus erythematosus; MS, mitral stenosis; MR, mitral regurgitation; DVT, deep vein thrombosis; COPD, chronic
obstructive pulmonary disease.
44 Chapter 1 Cardiovascular system (station 3)

Demonstrate the importance associated pathology (such as pericardial


fluid) can also be evaluated
of clinical context suggest Cardiac catheterisation: right heart studies
relevant questions that would be are the gold standard for diagnosis (mean
pulmonary artery pressure >25 mmHg,
taken in a patient history and pulmonary capillary wedge pressure
Report to the examiner that history questions <15 mmHg) and monitoring of response
would seek to quantify breathlessness in terms to treatment. Right heart catheterisation is
of functional class, and identify known causes of also used to assess vasoresponder status.
pulmonary hypertension. A history of palpitations This has prognostic significance and alters
should be questioned, as atrial tachyarrhythmias management (can be treated with calcium
are poorly tolerated. Syncope is an ominous sign channel blockers)
that merits urgent work-up. Bubble echocardiography, transoesophageal
echocardiography and cardiac MRI: these
Demonstrate an understanding imaging modalities can be used to look for
causative structural lesions, particularly
of the value of further intracardiac shunts
investigation Investigations directed at an underlying
Investigations in cases of pulmonary cause include an autoantibody screen and HIV
hypertension would include: test. Liver ultrasound should be requested if
A 6-minute walk test. This has prognostic porto-pulmonary hypertension is suspected.
value and also can be used to assess response Investigate pulmonary function using
to treatment pulmonary function tests, high-resolution CT
Laboratory blood tests would include chest, CTPA or V/Q scans.
measurements of natriuretic peptides (BNP
or NT-proBNP) as markers of myocardial Always offer a management plan
stretch and thus volume overload. Additional
This needs to be directed at the underlying cause
blood testing can identify an underlying
(hence the WHO classification).
cause, e.g. markers of autoimmune disease,
In pulmonary arterial hypertension (WHO
and urate and haematinics to assess for iron
Group 1), therapies include:
depletion
A 12-lead electrocardiogram may Those aimed at pulmonary arterial pressures
demonstrate: (oral, inhaled and intravenous therapies):
Atrial arrhythmia, which can be poorly Endothelin receptor antagonists: e.g.
tolerated in PAH bosentan
Right atrial enlargement Phosphodiesterase type 5 inhibitors: e.g.
Right axis deviation sildenafil
Right ventricular hypertrophy Prostacyclin analogues (administered
On a plain chest radiograph, look for dilated parenterally using a syringe pump via
central pulmonary arteries with peripheral tunneled lines): e.g. treprostinil
vascular pruning and right ventricular Also calcium channel blockers for those
enlargement responsive to vasodilator studies
Echocardiography provides estimates (but Warfarin is considered in selected patients to
not diagnostic measures) of pulmonary artery prevent pulmonary thromboembolic disease
systolic pressure and right atrial pressure. Treating symptoms of right heart failure
Echocardiography also provides information with diuretics and long term oxygen therapy
on right ventricular systolic function, the where appropriate.
response of which to raised pulmonary
For WHO Group 4, pulmonary
pressures is the essential determinant of
thromboendarterectomy is highly successful
survival (over and above the pulmonary
in appropriate patients (depending upon the
arterial pressure value itself). Any causative
suitability of the disease and the patient for
left heart disease (systolic, diastolic or valvular
surgery).
dysfunction) or congenital lesions, and any
Lung transplantation should be considered
in selected patients.
Case 19 Atrial septal defect 45

Further reading European Society of Cardiology (ESC) and the


European Respiratory Society (ERS), endorsed
Gali N, Hoeper MM, Humbert M, et al. Guidelines
by the International Society of Heart and Lung
for the diagnosis and treatment of pulmonary
Transplantation (ISHLT). Eur Heart J 2009;
hypertension: the task force for the diagnosis
30:24932537.
and treatment of pulmonary hypertension of the

Case 19: Atrial septal defect

Instruction to the candidate Associated hand findings: patients with


HoltOram syndrome have upper limb
This 55-year-old woman presents with
abnormalities associated with secundum ASDs
increasing exertional dyspnoea. Please examine
her cardiovascular system.
State the most likely diagnosis
Begin with a summary of on the basis of these findings
positive findings This patient has signs consistent with an atrial
septal defect, most commonly due to a secundum
Seek to identify peripheral findings, auscultatory
ASD. She is in rate-controlled atrial fibrillation
features and signs of severity:
and has no evidence of pulmonary hypertension.
Peripheral signs include cyanosis if I would like to take a full history to establish
Eisenmengers syndrome. The pulse may be her symptomatic status, and my investigation
sinus rhythm, AF or atrial flutter. of choice would be an echocardiogram in the
Auscultatory features include a fixed split first instance to confirm the diagnosis and to
second heart sound (meaning that it does not quantitate the degree of shunting and estimate
vary with respiration). A systolic pulmonary pulmonary arterial pressures.
flow murmur may also be present.
Signs of severity include: Offer relevant differential
Right heart failure and arrhythmias
Severe pulmonary hypertension diagnoses
Eisenmengers syndrome This anatomical nature of atrial septal defects
can be divided as follows:
Follow with a summary of Secundum: 80% of ASDs
relevant negative findings Primum: 15% of ASDs
If negative, comment on the absence of:
Sinus venosus: 5% of ASDs
Unroofed coronary sinus: <1% of ASDs
Arrhythmia (check for stigmata of

anticoagulation)
Signs of pulmonary hypertension
Demonstrate the importance
Evidence of signs of stroke which would of clinical context suggest
indicate cerebral (paradoxical) embolism relevant questions that would be
from a right-to-left shunt
Concomitant valvular heart disease. taken in a patient history
Associated lesions include all complex It is worth noting that many patients are
congenital heart disease, or isolated lesions asymptomatic until after their fourth decade,
such as pulmonary stenosis, mitral valve beyond which symptoms may include exertional
prolapse and aortic regurgitation in primum dyspnoea and palpitations. Peripheral oedema
ASDs is present if right heart failure develops,
although this tends to be a later presentation.
46 Chapter 1 Cardiovascular system (station 3)

Demonstrate an understanding information, especially in more complex


congenital lesions
of the value of further
investigation Always offer a management plan
Investigations in cases of atrial septal defect
Patients should be referred to a specialist ACHD
include:
centre for follow-up (pre- and post-closure).
A 12-lead electrocardiogram which would Closure of the ASD may be performed
demonstrate incomplete right bundle branch percutaneously (method of choice for secundum
block or RSR, pattern in lead V1 alone. The ASD) or surgically. ASD closure is recommended
axis is different depending on the anatomical if the shunt ratio is greater than 1.5:1. Patient
nature of the lesion. There is right axis suitability for closure depends upon assessment
deviation in secundum lesions, while there is of biventricular function and pulmonary arterial
left axis deviation in primum defects pressures. Closure is contraindicated in patients
A plain chest radiograph demonstrates who have developed Eisenmengers syndrome.
increased pulmonary vascular markings , Suitability for device closure depends upon
dilated pulmonary arteries and cardiomegaly anatomical considerations (e.g. suitable defect
if there is heart failure rims) on transoesophageal echocardiogram.
Echocardiography (transthoracic and Antibiotic prophylaxis following device
transoesophageal) will provide: closure or in cyanotic patients: see Infective
Diagnosis, anatomical localisation and endocarditis (p. 40).
shunt quantification Atrial arrhythmias (fibrillation, flutter or
Estimation of pulmonary arterial systolic tachycardia) can be ablated either prior to
pressure percutaneous closure or at the time of surgical
Estimate of right ventricular size and closure.
function Patients with AF should receive formal
Transoesophageal echocardiography anticoagulation.
allows visualisation of sinus venosus
defects and anomalous pulmonary
venous drainage, and improved
Further reading
characterisation of the ASD in terms of Deanfield J, Thaulow E, Warnes C, et al. Management
anatomical suitability for percutaneous of grown up congenital heart disease. Eur Heart
device closure J 2003; 24:10351084.
Cardiac catheterisation is indicated to Basson CT, Cowley GS, Solomon SD, et al. The
determine pulmonary arterial pressures and clinical and genetic spectrum of the Holt-Oram
PVR, and to determine shunt quantification syndrome (heart-hand syndrome). N Engl J Med
from sequential blood oxygen saturations 1994; 330:885891.
Cardiac MRI can provide further structural

Case 20: Patent ductus arteriosus

Instruction to the candidate Begin with a summary of


This 40-year-old man had an incidental murmur positive findings
detected on an insurance medical. Please Seek to identify peripheral findings and
examine his cardiovascular system. auscultatory features to make a diagnosis, with
additional clinical features to comment on
Clinical context for the PACES exam severity:
Peripheral signs include:
In the adult patient, PDA is usually an isolated Differential cyanosis if the patient has
finding as associated congenital lesions Eisenmengers syndrome. In such cases, the
predominate in the paediatric population. lower limbs are cyanosed whereas the upper
Case 20 Patent ductus arteriosus 47

limbs are not. There will be toe clubbing, but Demonstrate an understanding
not finger
Differential peripheral oxygen saturations in of the value of further
the hands and feet if Eisenmengers syndrome investigation
is present. Comment to the examiner that you Investigations in cases of PDAs include:
would request pulse oximetry measurements
to be taken from the fingers and toes The results of a 12-lead electrocardiogram
Evidence of left ventricular failure and plain chest radiograph relate to the
complications of the PDA. A normal ECG and
The key auscultatory feature is a continuous CXR would be expected with a small PDA
machinery murmur heard in the infraclavicular with no cardiac sequelae. Right ventricular
or infrascapular region. hypertrophy and prominent pulmonary
The clinical presentation is related to the size vessels could be evident with shunts causing
of the PDA: pulmonary hypertension. Cardiomegaly and
Small PDA: no left ventricular volume pulmonary congestion may be evident with
overload, normal pulmonary artery pressures PDAs that have given rise to left heart failure
Moderate PDA with either predominant: Echocardiography (transthoracic or
Left ventricular volume overload: left transoesophageal) provides useful
ventricular dilatation with or without information including:
impaired systolic function Diagnosis, anatomical localisation and
Right ventricular pressure overload: shunt quantification
pulmonary hypertension with ensuing Estimation of pulmonary arterial systolic
right heart failure pressure
Large PDA: Eisenmengers syndrome. The Measurement of biventricular size and
murmur disappears with Eisenmengers systolic function
syndrome, and hence differential cyanosis is Cardiac catheterisation allows quantification of
the key clinical finding in this situation shunt and determination of pulmonary artery
pressure and pulmonary vascular resistance
Cardiac MRI provides further structural
Follow with a summary of information, identification of complex
relevant negative findings congenital heart disease and ventricular
It is important to comment on the absence function quantification
of signs of pulmonary hypertension and left
ventricular dysfunction. Always offer a management plan
Patients should be referred to a specialist ACHD
State the most likely diagnosis centre for follow-up (pre- and post-closure).
Embolisation of the PDA may be performed
on the basis of these findings percutaneously (method of choice for PDA with
This patient has signs consistent with a small a device or coil) or surgically (for anatomically
patent ductus arteriosus. There are no features unsuitable or large PDAs not amenable to device
of left or right ventricular dysfunction, or closure). Closure is contraindicated at the extremes
pulmonary hypertension. I would like to take a of the symptomatology spectrum, namely
full history to establish his symptomatic status, patients who have developed Eisenmengers
and my investigation of choice would be an syndrome, or asymptomatic patients with small,
echocardiogram in the first instance to confirm haemodynamically insignificant lesions with no
the diagnosis and to confirm the absence of clinical evidence of shunt on examination.
left or right heart involvement and estimate Asymptomatic patients with clinical evidence
pulmonary arterial pressures. of a PDA, signs of left ventricular volume
overload or pulmonary hypertension should all
Demonstrate the importance be considered for closure. According to NICE
guidelines, antibiotic prophylaxis is not routinely
of clinical context suggest recommended unless cyanosis is present.
relevant questions that would be
taken in a patient history Further reading
History questions would be aimed at identifying Deanfield J, Thaulow E, Warnes C, et al. Management
symptoms of either pulmonary hypertension or of grown up congenital heart disease. Eur Heart
left ventricular dysfunction. J. 2003; 24:10351084.
Case 21: Ebsteins anomaly: the cyanosed
patient with a murmur

Instruction to the candidate sign alluding to the diagnosis. Patients may have
had corrective surgery so look for a midline
This 25-year-old man complains of exertional
sternotomy scar.
dyspnoea, ankle swelling and palpitations.
Comment on the absence of a neurological
Please examine his cardiovascular system.
deficit. Paradoxical embolic phenomena are
present if there is right-to-left shunting at the
Begin with a summary of atrial level
positive findings
Pathophysiology
Patients can have a variety of symptoms and
signs depending upon the severity of Ebsteins This is a congenital malformation of the
abnormality. Physical findings therefore can tricuspid valve with failure in the undermining
range dramatically from the well-saturated adult process of the valve leaflets and chordae so
with a murmur of mild tricuspid regurgitation they fail to reach the atrioventricular junction.
to the adolescent with severe right heart failure, This results in apical displacement of the
arrhythmia and cyanosis. tricuspid valve and atrialisation of the right
Peripheral signs include features of right ventricle. The anterior tricuspid leaflet often
heart failure and those related to associated billows out like a sail, causing regurgitation of
arrhythmia: the valve
Comment on cyanosis or clubbing, and The most frequently associated anomalies
request peripheral oxygen saturations. include secundum ASD or patent foramen
Cyanosis is indicative of a right-to-left shunt ovale with interatrial shunting, and accessory
at the atrial or, less commonly, ventricular pathways such as WolffParkinsonWhite
level and should trigger the candidate to syndrome. Other associated congenital
look for signs of an atrial or ventricular septal abnormalities include VSD, PS, pulmonary
defect atresia, tetralogy of Fallot, coarctation of the
Signs of right heart failure: the JVP will be aorta, and disorders of the mitral valve
raised with giant v waves of TR; ascites and
peripheral oedema will be present State the most likely diagnosis
Arrhythmia: The patient could be tachycardic on the basis of these findings
with either a regular or irregular rhythm
if there is coexisting supraventricular This young adult has features of right heart
tachycardia or AF respectively. Pre-excitation failure that include cyanosis, a raised JVP, a
is associated with Ebsteins anomaly. Look murmur of tricuspid regurgitation and ankle
for stigmata of anticoagulation therapy swelling. There is absence of a right ventricular
heave. These findings are consistent with
Auscultatory features of Ebsteins anomaly Ebsteins Anomaly with a right-to-left shunt.
include He has signs of venepuncture indicating
The first heart sound will be split owing that he is anticoagulated, probably for a
to late closure of the tricuspid valve. supraventricular tachycardia such as atrial
A pansystolic murmur of tricuspid flutter.
regurgitation is heard maximally at the lower
left sternal edge on inspiration
A third heart sound is heard in the presence Offer relevant differential
of right heart failure diagnoses
The two key differential diagnoses for Ebsteins
Follow with a summary of anomaly are tricuspid atresia and tricuspid
relevant negative findings regurgitation.

The absence of a right ventricular heave in the


presence of right heart failure is an important
Case 21 Ebsteins anomaly: the cyanosed patient with a murmur 49

Demonstrate the importance Cardiac catheterisation is used infrequently,


but a pressure study may be useful
of clinical context suggest preoperatively.
relevant questions that would be
taken in a patient history Always offer a management plan
Relevant questions would be those aimed at Treatment depends on the severity of the
identifying symptoms of cyanosis and of right disease and associated structural or electrical
heart failure. defects. Specialist and multi-disciplinary
team management is essential. It is important
to realise that these patients can have very
Demonstrate an understanding complex congenital heart disease and need
of the value of further to be assessed lifelong in a specialist ACHD
investigation centre. A multidisciplinary approach is essential
as often the patient is a young adult with
The relevant investigations and the appropriate
cyanosis and may require advice on pregnancy,
findings include:
contraception and genetic counselling. Other
On a 12-lead electrocardiogram one-third
specialties such as electrophysiology are vital
of patients have supraventricular tachycardia
to the management of complex arrhythmia and
or atrial flutter, and many have accessory
structural defects.
pathways leading to ventricular arrhythmia
Medical management includes diuretics
being a cause of death. Ebsteins anomaly is
and ACE inhibitors. Rate controlling agents
commonly associated with WolffParkinson
such as beta-blockers, calcium channel
White syndrome. Check for pre-excitation (e.g.
blockers and digoxin are used to control
delta waves) and right bundle branch block.
tachycardia. Anticoagulation is indicated for
P pulmonale, demonstrated by tall, peaked P
atrial flutter.
waves may be observed indicating an enlarged
Interventional techniques include
right atrium. Notching within the RBBB is
electrophysiology studies and ablation of
commonly associated with Ebsteins.
accessory pathways or arrhythmogenic
On a plain chest radiograph there is often
substrate.
gross cardiomegaly due to right atrial dilatation.
Surgical options in Ebsteins anomaly
An echocardiogram will demonstrate
include:
the degree of valve malformation, degree of
regurgitation, severity of right ventricular Repair of the tricuspid valve is more
dysfunction and cavity loss. Additional favourable than replacement. The Cone
congenital defects and the direction of shunting repair is the more contemporary method to
at atrial level (if a defect is present) can also be reduce the degree of tricuspid regurgitation.
demonstrated. Atrial reduction surgery
Assessment of functional capacity, by Palliative creation of atrial shunt
CPEX for example, is important for symptom MAZE procedure for tachyarrhythmias
quantitation, degree of desaturation on exercise, Cardiac transplantation
arrhythmia provocation, and may guide
interventional decision making.
Cardiac MRI is recommended to assess
Further reading
the valve leaflets and right heart volumes and Deanfield J, Thaulow E, Warnes C, et al. Management
function in more detail. The degree of shunting of grown up congenital heart disease. Eur Heart
can also be determined. J 2003; 24:10351084.
Case 22: Tetralogy of Fallot

Instruction to the candidate repair. If the patient has had a BlalockThomas


Taussig shunt performed, they will have an
Examine this 30-year-old mans cardiovascular
absent radial pulse.
system. He had repair of a congenital defect and
now complains of palpitations and syncope.
Follow with a summary of
Begin with a summary of relevant negative findings
positive findings Important signs to comment on in their absence,
or include as positive findings if present, include:
The four features of tetralogy of Fallot are:
Right ventricular outflow tract obstruction
Stigmata of anticoagulation
Ventricular septal defect
Stigmata of infective endocarditis. Look for
peripheral stigmata, ask the examiner for the
Aortic dextroposition
temperature and a urine dipstick (looking for
Right ventricular hypertrophy
microscopic haematuria), and comment on
Morphologically, there is anterocephalad dental hygiene and the presence or absence
deviation of the interventricular septum that of indwelling intravenous access
determines the degree of right ventricular outflow Signs of right heart failure: The JVP will be
tract obstruction and the degree of cyanosis. raised, ascites and peripheral oedema will be
Mild cases are described as pink Fallots. Severe palpated
cases are cyanosed and classically squat to Ensure you have requested the patients
increase their preload and subsequently cardiac blood pressure, as associated aortopathy can
output. Almost all Fallot lesions are repaired with lead to root dilation and regurgitation
transannular patch and pulmonary valvotomy in Features of Di George syndrome, with which
infancy. In the case of severe cyanosis, a neonate tetratology of Fallot is associated
may undergo modified BlalockThomasTaussig
shunt to augment pulmonary blood flow via a
thoracotomy. In adulthood it is therefore likely that
State the most likely diagnosis
the radial pulse will be weak. The major adverse on the basis of these findings
sequela in adulthood is pulmonary regurgitation. This man with a previous sternotomy and
This results in right ventricular dilatation and absent right radial pulse has had cardiac surgery
dysfunction. Additionally arrhythmias are the most for congenital heart disease. He has a right
common presentation in adulthood. Ventricular ventricular heave and a soft diastolic murmur
and atrial tachycardia are often poorly tolerated. consistent with pulmonary regurgitation. He is
In keeping with the above features, the key likely to have had repair of tetralogy of Fallot and
clinical findings are: now has right ventricular dysfunction. Given his
symptom of palpitations, he needs to be assessed
Atrial fibrillation or other tachyarrhythmia
further for pulmonary valve replacement.
Raised JVP
Right ventricular heave
Infraclavicular mass, likely to be an ICD Offer relevant differential
Auscultatory features include:
diagnoses
Decrescendo diastolic murmur at
the upper left sternal edge, louder on The key differential diagnoses are pulmonary
squatting or inspiration and softer by regurgitation, right heart failure and infective
Valsalva or expiration endocarditis.
A third heart sound may be heard in the
presence of right ventricular hypertrophy Demonstrate the importance
or failure, and is louder on inspiration
of clinical context suggest
The signs present on examination depend
on what management, if any, the patient has
relevant questions that would be
undergone. If unrepaired (rare) the patient will taken in a patient history
demonstrate clubbing and cyanosis. The patient Appropriate questions in a clinical history would
may have a midline sternotomy from a previous include the age at diagnosis and intervention
Case 23 Eisenmengers syndrome 51

(including what type of procedure the patient congenital heart disease. The aorta can also
had). Additionally, ask about current symptoms be monitored
of cyanosis and breathlessness.
Always offer a management plan
Demonstrate an understanding The main issues in management of tetralogy of
of the value of further Fallot are:
investigation Timing of pulmonary valve replacement, the
Important investigations and the relevant results choices including surgical bioprosthesis or
in cases of tetralogy of Fallot include: percutaneous valve implantation (usually
Melody or Edwards valves)
A 12-lead electrocardiogram to assess Arrhythmia management, including medical
rhythm and for evidence of right ventricular and interventional approaches with input
hypertrophy. It is important to measure the from electrophysiologists
QRS duration, as a duration greater than 180 Branch pulmonary artery stenosis can be
ms is strongly related to a poorer prognosis managed by surgical reconstruction
and is an indication for ICD implantation Aortopathy and aortic root dilatation with or
Regular ambulatory Holter monitoring to without valve regurgitation requires surgical
assess arrhythmia burden root replacement
Cardiopulmonary exercise testing. Serial Coronary artery disease should be evaluated
measurements of VO2max are useful for and treated surgically at the time of surgery
functional capacity and also may reveal
ventricular arrhythmias and exertional Other considerations include:
desaturation Assessment and follow-up should be
Echocardiography allows quantification conducted in a specialist ACHD centre
of pulmonary regurgitation and right Pregnancy is usually well tolerated but
ventricular function. The aortic root size specialist care needs to be sought. There is an
should be measured as aortopathy results in increased risk of cardiac defects in offspring
dilatation and possibly regurgitation of approximately 3%
Cardiac MRI. Imaging of the right ventricle Assessment of the patients condition is required
and measurement of volume and function prior to undertaking competitive sports
is paramount in determining the timing
for pulmonary valve replacement. Cardiac
MRI is currently the gold standard tool Further reading
for determination of right ventricular size Deanfield J, Thaulow E, Warnes C, et al. Management
and systolic function. This modality is also of grown up congenital heart disease. Eur Heart
very useful for identification of complex J 2003; 24:10351084.

Case 23: Eisenmengers syndrome

Instruction to the candidate Begin with a summary of


This 33-year-old man complains of severely positive findings
reduced exercise capacity, dizziness, Features of pulmonary hypertension
breathlessness and palpitations. Please examine predominate. These include:
her cardiovascular system.
52 Chapter 1 Cardiovascular system (station 3)

Central cyanosis
Exercise capacity is reduced, as oxygen uptake
Clubbing
cannot be increased through the pulmonary bed.
Plethora
There is also systemic vasodilation and this can
Scars: especially sternotomies and
lead to syncope, which is usually a poor prognostic
thoracotomies
symptom.
Pulse: may be absent if a subclavian artery
was used to create a BlalockThomasTaussig Long term prognosis can be variable.
shunt. Eisenmengers with simple lesions such as ASD can
Raised JVP survive to the sixth decade. However, if pulmonary
Right sided parasternal heave vascular resistance continues to increase rapidly
Loud S2 and P2 component and the right ventricle fails, the prognosis is
Shunt murmurs and differential cyanosis. poor. Multisystem comorbidities are common.
If shunt reversal occurs in the presence Hypoxaemia, bleeding, sepsis (in particular
of a patent ductus arteriosus there will be cerebral abscesses) and arrhythmia contribute to
differential cyanosis with marked clubbing the cause of death.
and cyanosis in the feet and normal hands.
The shunt usually occurs after the arm
vessels branch off the aorta. This is easily Follow with a summary of
demonstrated by checking the saturations relevant negative findings
in the hands and the feet, and emphasizes If no signs of intervention are present, this is
the importance of exposing feet fully. In the important to note as a negative finding.
presence of unrepaired transposition of the
great arteries with a patent ductus arteriosus,
the differential cyanosis will be reversed. Shunt State the most likely diagnosis
murmurs may become quieter and eventually on the basis of these findings
inaudible as the pulmonary vascular resistance
This patient with Downs syndrome has
increases and the shunt reverses.
Eisenmengers syndrome. The right radial pulse
Other signs, categorised by system affected, is absent and suggests the subclavian artery was
include: used to create a BlalockThomasTaussig shunt
for pulmonary blood flow augmentation in
Respiratory: breathlessness and tachypnoea lesions such as tetralogy of Fallot, for example.
Haematological: bruises, bleeding, There are established features of pulmonary
conjunctival suffusion hypertension, cyanosis and haematological
Abdominal: hepatosplenomegaly and jaundice involvement with multiple bruises and plethora.
Musculoskeletal: muscle wasting and gouty
deposits
Skin: plethoric facies. Venous changes Offer relevant differential
especially in the lower limb diagnoses
Genetic abnormalities such as Downs
The most relevant differential diagnoses are
syndrome
pulmonary hypertension and tetralogy of Fallot.

Aetiology Demonstrate the importance


Uncorrected congenital shunts and palliative of clinical context suggest
procedures with left-to-right shunt lead to
Eisenmengers syndrome. Initially there is an relevant questions that would be
increase in pulmonary blood flow, but this taken in a patient history
subsequently leads to increased pulmonary
History questions would be aimed at identifying
vascular resistance and pulmonary hypertension. a history of intervention and current and
Ultimately this results in reversal of the shunt and past symptom burden, such as those of
cyanosis. hyperviscosity.
Common lesions include:
Unrestricted VSDs
Unrestricted ASDs Demonstrate an understanding of
Unrestricted PDAs the value of further investigation
Palliative surgical lesions such as Potts,
Waterston and BlalockThomasTaussig Important investigations and the likely results
shunts include:
Case 23 Eisenmengers syndrome 53

Laboratory blood tests: Supplemental oxygen may be required when


Full blood count: for secondary travelling by commercial airlines
erythrocytosis. Expect high haemoglobin. Epoprostanil (prostacyclin analogue): A
This does not need to be corrected with continuous intravenous infusion via central
venesection unless the patient complains access. This has shown to improve symptoms
of hyperviscosity symptoms (usually and oxygenation from 6585%. It can also
occur with haematocrits >65%) reduce pulmonary vascular resistance and
Iron studies: most patients are iron improve cardiac index
deficient and require replacement either Bosentan (endothelin receptor agonist):
orally or intravenously Improves NYHA class and 6 minute walk
Bleeding time is increased due to platelet test. The BREATH-5 study showed safety in
dysfunction and thrombocytopaenia. Eisenmenger population and improvement
Liver function tests: check conjugated in WHO class and reduction in pulmonary
bilirubin artery pressure
Natriuretic peptides (e.g. BNP or NT-pro Sildenafil (phosphodiesterase inhibitor):
BNP) for heart failure Improves functional class and symptoms
Urate, as gout is common due to high cell with a reduction in pulmonary pressures
turnover
Other considerations include:
A 12-lead electrocardiogram may demonstrate
right bundle branch block, right ventricular Anticoagulation. Although thrombus
hypertrophy and/or right axis deviation formation is common, anticoagulation
A plain chest radiograph would be expected needs to be carefully considered in the
to show enlarged pulmonary vessels and an context of platelet dysfunction and bleeding
enlarged right heart tendency
Findings on echocardiogram would include: Contraception. The risk of fetal death is 25%
Assessment of intracardiac congenital and maternal mortality from pregnancy is
lesions and the degree and direction of 50%. Saturations less than 85% fare a worse
shunting outcome. Progesterone only contraception
Right ventricular function (a poor should be used either orally or with an
prognosis in context of failure) intrauterine device (IUD).
Estimation of pulmonary artery pressure Specialist review and follow-up. The
Cardiac catheterisation is useful to assess examiner will want to know that you
right heart pressures and to provide an appreciate that these are complicated
assessment of the degree of shunting patients and require ACHD and specialist
pulmonary hypertension input. Their follow
Always offer a management plan up should be 36 monthly in a specialist
centre. Patients also need education to
Inform the examiner that you would focus ensure their wellbeing. Patients should avoid:
on patient education and a multidisciplinary Sudden fluid shifts, such as dehydration
approach. Hot conditions that may promote
Secondary erythrocytosis and both vasodilation and syncope
thrombotic and bleeding tendencies are the Competitive sport and isometric exercise
main points to discuss here. Patients are no Pregnancy
longer venesected for erythrocytosis as this has Poor oral hygiene as endocarditis is
shown to render them iron deficient, unless common and prophylactic antibiotics
there are severe symptoms of hyperviscosity. should be administered
Remaining well hydrated is paramount to Non-cardiac surgery without close cardiac
their care and can avoid thrombus formation. input
Therefore intravenous fluids should be
administered with a line filter to eliminate air in
the presence of dehydration. Further reading
Options in medical therapy include: Deanfield J, Thaulow E, Warnes C, et al. Management
of grown up congenital heart disease. Eur Heart
Oxygen: supplementary nocturnal oxygen may
J 2003; 24:10351084.
help symptoms but has no effect on outcome.
Case 24: Dextrocardia

Instruction to the candidate State the most likely diagnosis


This 50-year-old woman was referred by her GP on the basis of these findings
after an incidental finding on cardiovascular This patient has dextrocardia and is clinically
examination. Please examine her and report well. I would like to examine the abdomen for
your findings to the examiner. situsinversus and my investigation of choice would
be a chest radiograph to confirm my diagnosis.
Understanding terminology in
dextrocardia Offer relevant differential
Demonstrate to the examiner that you understand diagnoses
the difference between: Dextrocardia is associated with:
Dextrocardia: the heart is in the right
hemithorax with the base-apex axis pointing Kartageners syndrome, comprising
rightward and caudad bronchiectasis, sinusitis and infertility
Dextroposition: the heart is displaced to the (50% have situs inversus)
right hemithorax (e.g. with a diaphragmatic Turners syndrome
hernia) but the apex still points to the left Asplenia
Levocardia: the heart is in the left hemithorax Demonstrate the importance of clinical
with the apex pointing leftward context suggest relevant questions that would
Mesocardia: the heart is in the middle of the be taken in a patient history
chest with the apex pointing leftward In the absence of disease associations,
Situs inversus: dextrocardia, trilobar the patient with dextrocardia is likely to be
left lung, bilobar right lung, right-sided asymptomatic.
stomach, left-sided liver, right-sided
descending colon
Demonstrate an understanding of
Begin with a summary of the value of further investigation
positive findings Relevant investigations would comprise:
The key clinical finding is a right sided apex A 12-lead electrocardiogram demonstrating
beat in the fifth intercostal space. The heart mirror-imaged electrical activity, with P, QRS,
sounds are better heard on the right side than and T waves in leads I, aVR, aVL that are
the left side of the chest, and are commonly reversed
normal. A plain chest radiograph will demonstrate the
heart in the right hemithorax with the apex
pointing to the right. Check the horizontal
Follow with a summary of fissure of the right lobe and check sidedness
relevant negative findings of the gastric bubble. The descending aorta
It is important to comment on clear lung fields position will be reversed
without crepitation. If present this may be
suggestive of ciliary dysmotility which is present Always offer a management plan
in Kartageners syndrome. Ensure there are no
Dextrocardia in itself requires no specific
stigmata of Turners syndrome.
management.
Chapter 2

Nervous system
(station 3)
Case 25: Monocular blindness

Instruction to the candidate If visual axis opacity (absent or poor red


reflex) is present:
This 64-year-old woman has been referred
by her optometrist for urgent review. Please Corneal opacity traumatic (scarring,
examine her visual fields and report your infection), corneal dystrophy
findings to the examiner. Lens opacity dense cataract
Vitreous opacity:
Begin with a summary of
Haemorrhage
positive findings Inflammation
There is complete unilateral loss of vision Infection
affecting the right eye. The visual fields of the left
If RAPD present:
eye are intact with no apparent deficit.
Additional examination findings, narrowing Inflammatory demyelination
the level of the deficit: Central retinal artery occlusion
Relative afferent pupillary defect (RAPD) the
Ischaemic optic neuropathy
(anterior/posterior)
most useful additional finding in this situation.
RAPD supports unilateral blindness secondary
Retinal detachment
to pathology affecting the lens, retina, or optic
Advanced glaucoma
nerve anterior to the optic chiasm
Traumatic optic neuropathy
Red reflex where absent would suggest
Compressive optic neuropathy (e.g. optic
nerve sheath meningioma). Interestingly this
opacity of the visual axis
may give rise to Foster-Kennedy syndrome
Fundoscopy assessing for pale or swollen
where optic atrophy is identified in the
optic disc indicative of ischaemia
ipsilateral eye due to the compressive effect
upon the optic nerve while in the contralateral
Follow with a summary of eye papilloedema occurs due to the mass
relevant negative findings effect and raised intracranial pressure

Pupillary responses, red reflex, and optic discs


would be mentioned as important negative Demonstrate the importance
findings if they are examined and found to be of clinical context suggest
normal.
relevant questions that would be
State the most likely diagnosis taken in a patient history
When faced with a visual field deficit, attempt to
on the basis of these findings establish the history of the visual loss in terms
This patient has monocular blindness. In the of onset, duration and progression. Enquire as
absence of lens pathology and the presence of to associated symptoms or visual phenomena
a relative afferent pupillary defect monocular including floaters (suggestive of retinal
blindness is indicative of a CNS lesion anterior pathology) flashing lights halos around lights,
to the optic chiasm. suggesting lens pathology), or distorted colour
vision.
Screen for systemic disease with potential
Offer relevant differential for visual involvement, particularly diabetes,
diagnoses hypertension and those that increase the
A monocular field defect will always be due likelihood of ischaemia such as sickle cell
to disease anterior to the optic chiasm, before disease. Where cataracts are suspected, consider
the nerve fibres cross. In order to confidently congenital disease and family history (Myotonic
attribute monocular blindness to a CNS lesion, Dystrophy and Lowe syndrome are good
pathology of the lens, vitreous or retina must be examples).
excluded. Thus, the differential becomes: Consider extra-occular symptoms that may
direct towards a specific diagnosis, particularly:
Case 26 Homonymous hemianopia 57

Giant cell arteritis (GCA) visual loss rule out giant cell arteritis as this can progress to
associated with headache, tender scalp over bilateral blindness if treatment is not given.
temporal artery, proximal weakness/stiffness If there is no obvious cause of blindness
of the upper girdle, and general malaise apparent in the eye, imaging of the orbit
Migraines visual loss may present with (contrast-enhanced CT or MRI with gadolinium)
headaches as an ocular migraine is indicated.
Visual-evoked potentials are useful in
What effect has the visual deficit had on the
objectively evaluating optic nerve function.
patient's life? Consider independence and effect
upon activities of daily living, work, driving, and
resultant potential for depression. Always offer a management plan
There may be no obvious underlying cause
Demonstrate an understanding apparent, in which case the candidate may
choose to discuss the management of a
of the value of further particular cause from the differential diagnosis.
investigation Focusing upon common causes, such as
If recent onset, and ischaemic optic neuropathy cataracts, or important causes where timely
suspected in a patient over 50 years of age, intervention reduces progression such as GCA,
inflammatory markers should be requested to serves as a sensible approach.

Case 26: Homonymous hemianopia

Instruction to the candidate incongruous defect will be more anterior, in the


optic tract and lateral geniculate nucleus.
This 60-year-old man complains of blurred
vision. Please examine his visual fields and
report your findings to the examiner. Follow with a summary of
relevant negative findings
Begin with a summary of Pupillary responses, particularly the absence of
positive findings a relative afferent pupillary defect.
There is a visual field defect affecting the same
side of the visual field of both eyes, namely the State the most likely diagnosis
temporal field of the left eye and the nasal field on the basis of these findings
of the right. The field defects respect the vertical
midline. The patient has visual field defects involving
When examining the visual fields, particular both eyes consistent with a left homonymous
attention should be paid to the extent of the hemianopia indicating of a lesion affecting the
visual defect affecting each eye. Despite the right optic tract. The defect is more pronounced
description of a homonymous defect implying in the right eye than the left, and as such is
an identical field defect in each eye, in reality considered incongruous suggesting the lesion is
the extent of the visual field involved may vary more anterior within the optic tract.
between each side, in which case it is said to
be incongruous. The degree of incongruity Offer relevant differential
can be used to determine the likely site of the
lesion. A lesion causing a congruous defect diagnoses
will be posterior (affecting the optic radiations, Homonymous hemianopia is attributable
occipital cortex), while a lesion causing an to a lesion of the visual pathway posterior to
58 Chapter 2 Nervous system (station 3)

the optic chiasm. The most common causes intervention based on the findings of neuro-
are considered secondary to infarction or as imaging. It is vital however to highlight the
the result of a space-occupying lesion. Rarely importance of preventing disease progression
homonymous hemianopia can be congenital, as and/or recurrence by optimising risk factors
the result of a porencephalic cyst for instance. and identifying embolic sources, thus further
investigation will include:
Demonstrate the importance Blood tests such as HbA1c and lipid profile
of clinical context suggest Carotid Doppler ultrasound
Transthoracic echocardiogram
relevant questions that would be 24-hour ECG monitoring
taken in a patient history
Focus upon extra-ocular symptoms consistent Always offer a management plan
with either: Treatment is directed towards the underlying
Stroke sudden onset with additional cause. The management of, and scope for
neurological deficit, commonly dysphagia or functional improvement or recovery from,
hemiparesis homonymous hemianopia will depend upon the
Space-occupying lesion insidious onset underlying cause.
with gradual deterioration in vision, but Be prepared to demonstrate an understanding
with the potential for acute deterioration of the principles of cerebrovascular accident in
(haemorrhage or oedema), associated with the acute setting with need for urgent imaging
postural headache and anti-platelet or thrombolysis therapy as
appropriate, compared with the principles of
rehabilitation and multi-disciplinary care in the
Demonstrate an understanding longer term.
of the value of further Any discussion relating to a space-occupying
lesion should centre upon establishing the
investigation histopathology based on biopsy versus imaging
The most important investigation in this guided diagnosis. The broad principles of
scenario is neuro-imaging with CT and/or MRI. treatment, dependent upon the nature of the
In the acute setting where clinical suspicion mass, will be within a multidisciplinary setting
is of a cerebrovascular accident, priority and directed towards either aggressive resection
will revolve around treatment options and or removal versus monitoring or palliation.

Case 27: Bitemporal hemianopia

Instruction to the candidate Such a pattern of deficit is consistent


with a lesion affecting the optic chiasm.
This 43-year-old man has consulted with
Compression of the chiasm from below,
his general practitioner repeatedly over the
classically due to a pituitary tumour, affects
preceding 12 months complaining of headaches.
the lower nasal fibres in the first instance
He has now experienced an acute deterioration
and may result in a denser upper field defect.
in vision and has been referred for urgent
Compression from above, classically due to a
assessment. Please examine his visual fields and
craniopharyngioma, affects the upper nasal
report your findings to the examiner.
fibres resulting in a denser lower field defect
early in presentation. Both may progress to
Begin with a summary of affect lower and upper fields equally and as
such presentation will be variable and clinical
positive findings discrimination subtle.
There is a defect affecting the temporal visual
field of both eyes.
Case 28 Homonymous quadrantanopia 59

Follow with a summary of associated symptoms, particularly those related


to possible underlying pituitary adenoma.
relevant negative findings Commonly:
It would be prudent to perform fundoscopy
assessing for pallor of the optic discs and assess Lactotrophic (prolactinoma) with resultant
pupillary responses for RAPD. glactorrhoea, gynaecomastia
Where pituitary pathology is suspected Somatotrophic resulting in acromegaly
relevant negatives would include acromegalic or with associated headaches, profuse
Cushingoid features on gross inspection. sweating, and changes in physical
appearance
Corticotrophic resulting in Cushings
State the most likely diagnosis disease with associated Cushingoid facies,
on the basis of these findings centripedal and intrascapular adiposity, and
thin skin
The patient has visual field defects of both
temporal fields consistent with a bitemporal Where there is no evidence of pituitary
hemianopia, due to a lesion at the optic chiasm. involvement apparent, screen for
granulomatous or neoplastic disease including
risk factors and family history.
Offer relevant differential
diagnoses Demonstrate an understanding
Consider:
of the value of further
Pituitary tumour
investigation
Craniopharyngioma
Meningioma Dedicated neuroimaging of the optic chiasm
Aneurysm of the internal carotid artery with MRI and appropriate endocrine
Metastasis from distal primary investigations where pituitary adenoma
Granuloma (tuberculosis and sarcoidosis) suspected.

Demonstrate the importance Always offer a


of clinical context suggest management plan
relevant questions that would be Management would be directed to the
underlying chiasmal lesion, but most
taken in a patient history discussions surrounding management of
Clarify onset, duration and progression of bitemporal hemianopia will centre on the
visual symptoms. Thereafter, enquire as to management of pituitary adenomas.

Case 28: Homonymous quadrantanopia

Instruction to the candidate Temporal lobe lesions (superior


quadrantanopias): pie in the sky defect
Please examine the visual fields of this
60-year-old man.
Parietal lobe lesions (inferior
quadrantanopias): pie on the floor defect
Glaucoma can cause nasal defects which
Begin with a summary of present as a quadrantanopia, sometimes
binasal, and sometimes with a RAPD if
positive findings unilateral
Examination demonstrates a visual field deficit A chiasmal lesion could give a bitemporal
of the right upper quadrants affecting both eyes. quadrantanopia
Lesions of the optic radiations:
60 Chapter 2 Nervous system (station 3)

Follow with a summary of Focus upon extra-ocular symptoms


consistent with either:
relevant negative findings
Pupillary responses and RAPD. Stroke sudden onset with additional
neurological deficit, commonly dysphagia or
hemiparesis
State the most likely diagnosis Space-occupying lesion insidious onset
on the basis of these findings with gradual deterioration in vision, but
with the potential for acute deterioration
The pattern of deficit affecting the right upper (haemorrhage or oedema), associated with
quadrants is consistent with a homonymous postural headache
quadrantanopia due to a lesion affecting the left
temporal cortex.
Demonstrate an understanding of
the value of further investigation
Offer relevant differential Neuro-imaging is useful to rule out an
diagnoses intracranial lesion
Infarction Automated perimetry can more accurately
Tumour map the visual field defect
Trauma
Always offer a management plan
Demonstrate the importance Management is directed to the underlying cause.
of clinical context suggest
relevant questions that would be
taken in a patient history
Clarify onset, duration and progression of visual
symptoms.

Case 29: Sixth nerve palsy

Instruction to the candidate a compensatory posture, turning their head


towards the side of the palsy.
Please examine this 38-year-old woman
complaining of blurred vision. Her husband
reports that she has been squinting recently Follow with a summary of
and has been experiencing difficulties relevant negative findings
driving.
There is no ptosis or proptosis, the uniocular
visual acuity is normal, and there is no optic disc
Begin with a summary of swelling. There is no evidence of fatiguability on
positive findings repeated or sustained testing.
The eye is adducted in the primary position with
horizontal diplopia. There is impaired abduction State the most likely diagnosis
on the affected side. on the basis of these findings
Diplopia may be subtle when examined and
apparent only when viewing distant objects. The patient has impaired eye abduction
To limit diplopia patients will often assume indicative of lateral rectus weakness. This is
consistent with an isolated sixth nerve palsy.
Case 29 Sixth nerve palsy 61

Offer relevant differential Diplopia: monocular or binocular?


diagnoses Binocular diplopia is due to misalignment of the
Differential diagnosis of a sixth eyes. It disappears when one eye is covered.
nerve palsy Monocular diplopia is due to a structural problem
within one eye causing two focused images to be
Consider the differential in relation to the course formed on the retina simultaneously. Occluding
of the sixth nerve. the fellow eye has no effect.
Where onset is spontaneous and sudden in the
absence of other symptoms including headache
and sensory disturbance the differential diagnosis Demonstrate the importance
is likely to include a microvascular occlusion of clinical context suggest
commonly associated with poorly controlled
diabetes or hypertension. Isolated palsies often relevant questions that would be
indicate ischaemic pathology of the nucleus in the taken in a patient history
pons. That said the nucleus of the facial nerve is Clarify onset, duration, and ensure no associated
in close proximity and a combination of signs is deficit of visual acuity. In this case where the
often seen. patient is relatively young, it would be important
The nerve travels form its nucleus, through to be seen to explore symptoms consistent with
the subarachnoid space before passing over the raised intracranial pressure. Thereafter, focus
petrous apex of the temporal bone and entering on common co-morbidities associated with a
the cavernous sinus. Peripheral deficits at these sudden onset sixth nerve palsy - explore a history
points can be caused by anything that either of diabetes and hypertension.
directly compresses or stretches the nerve.
Papilloedema may suggest a space occupying
lesion and raised intracranial pressure, resulting in Demonstrate an understanding of
stretching of the sixth nerve in the subarachnoid the value of further investigation
space (false localising sign, particularly if bilateral).
Sixth nerve palsy associated with hearing Indications for neuro-imaging include:
loss and facial pain due to involvement of fifth, Suspicion of intracranial aneurysm
seventh or eight nerves is caused by pathology at sudden onset sixth nerve palsy associated
the petrous apex of the temporal bone. Consider with headache, particularly in the young,
trauma resulting in trauma. Also infectious, demands evaluation for aneurysm
inflammatory or neoplastic processes. Worsening signs on successive examinations
Involvement of the sixth with any combination Multiple cranial nerves involved including
of third, fourth, or the ophthalmic branch of the bilateral sixth nerve involvement
fifth cranial nerve might suggest a cavernous Swollen optic discs
sinus pathology. Look for a sixth nerve palsy Young patient (<50 years)
together with Horners syndrome and possibly
a third or fourth palsy. Common causes include If any clinical suspicion of myasthenia gravis,
thrombosis, neoplastic disease or infiltration, or antiacetylcholine autoantibodies can be
aneurysms of the carotid within the cavernous checked, and either an ice pack test or an
sinus. edrophonium test carried out to look for
reversibility of signs.
Check the blood pressure and capillary blood
Differential diagnosis of diplopia
glucose. Consider inflammatory markers and
The following conditions can cause diplopia thyroid function tests.
which can mimic cranial nerve palsies affecting
the extraocular muscles:
Always offer a
Dysthyroid eye disease
Myasthenia gravis management plan
Demyelination The underlying cause, if identified, should
Ophthalmoplegic migraine be treated. The commonest cause of third,
Decompensated squint fourth, and sixth nerve palsies is microvascular
Monocular diplopia (can be due to cataract, occlusion and the natural history is of
corneal surface disease, iris abnormality, spontaneous resolution. Spontaneous resolution
macular disease) would also be expected for an idiopathic palsy
62 Chapter 2 Nervous system (station 3)

over weeks to months. Thus, failure to improve worsening prompt reevaluation of underlying
should prompt re-evaluation for persisting pathology.
intracranial pathology.
Symptomatic management of diplopia can
be achieved through occlusion of one eye, or
Further reading
by fitting an appropriate prism to the patients Murchison AP, Gilbert ME, Savino PJ. Neuroimaging
glasses. An orthoptist can measure the angle of and acute ocular motor mononeuropathies:
deviation and fit the correct prism. The angle of a prospective study. Arch Ophthalmol 2011;
deviation should be monitored regularly and any 129:301305.

Case 30: Fourth nerve palsy

Instruction to the candidate State the most likely


Please examine this 70-year-old woman who has diagnosis on the basis
returned to clinic after successful cataract surgery
complaining of new onset blurring of vision.
of these findings
The patient has impaired vertical gaze,
predominantly in adduction, indicative of
Begin with a summary of superior oblique weakness. This is consistent
positive findings with isolated fourth nerve palsy.
The affected eye may be hypertropic (higher
than the fellow eye) in the primary position Offer relevant differential
with vertical diplopia. Diplopia may be subtle
and only be apparent on looking down or on
diagnoses
observing near objects. On examination of the Most fourth nerve palsies are either idiopathic
eye movements, there is impaired downward or traumatic. Idiopathic palsy is generally
gaze in adduction (down and in) with diplopia. expected to resolve, however while some
If long standing, there may be an over action of traumatic palsies improve with time, the
inferior oblique, with the ipsilateral eye shooting majority will not. Head injury resulting in
upwards in adduction. a fourth nerve palsy is often not associated
Congenital or chronic deficit may be with loss of consciousness and need not be
compensated by a head tilt away from the significant in force.
affected side to correct the deficit. Interestingly, Infarction or microvascular occlusion
in some cases a paradoxical tilt towards the side secondary to diabetes should be excluded as
of the deficit is employed to widen the blurred resolution with treatment is common.
images, allowing the patient to better suppress Raised intracranial pressure may cause
or ignore one image as a coping strategy. a fourth nerve palsy but may be a false
localising sign, especially if bilateral. That
said consideration should be given to the
Follow with a summary of possibility of direct compression of the fourth
relevant negative findings nerve by a space-occupying lesion. Associated
There is no ptosis or proptosis, the uniocular cerebellar signs may raise suspicion of a space-
visual acuity is normal, and there is no optic disc occupying lesion affecting the fourth nerve
swelling. There is no evidence of fatiguability on nucleus.
repeated or sustained testing.
Case 31 Internuclear ophthalmoplegia multiple sclerosis 63

Demonstrate the importance to cataract treatment can be a useful line of


enquiry.
of clinical context suggest
relevant questions that would be Demonstrate an understanding
taken in a patient history of the value of further
Collateral history or old photographs are often
useful in ascertaining a change in appearance
investigation
with development of a characteristic head tilt. Exclude medical conditions and consider neuro-
Fourth nerve palsy, of any cause can develop imaging as appropriate.
insidiously and unnoticed as elderly patients
gradually lose binocular function resulting Always offer a management plan
from cataracts. Following surgical intervention
and restoration of good vision, these patients Oculomotor exercises or prism glasses may
become aware of diplopia and present with an help restore concordant vision. In more severe
apparently new deficit. Thus, timing in relation cases, often congenital, surgical intervention is
considered.

Case 31: Internuclear ophthalmoplegia


multiple sclerosis

Instruction to the candidate Synchrony depends upon abduction of the


ipsilateral globe through the action of lateral
A 30-year-old woman complaining of blurred rectus mediated by the sixth cranial nerve
vision has been referred by her GP who together with adduction of the contralateral
is concerned by the pattern of her recent globe through the action of medial rectus
consultations in which she has complained of mediated by the third cranial nerve. The
a variety of symptoms. Please examine her eye medial longitudinal fasciculus (MLF) facilitates
movements and discuss your findings with the conjugate gaze by relaying signals between
examiner. the cranial nerves and incorporating direction
from the horizontal gaze centre of the brain,
Begin with a summary of the paramedian pontine reticular formation.
Additionally the MLF connects the vestibular
positive findings nuclei with the third cranial nerve nucleus.
There is impaired adduction of the affected Lesions affecting the MLF are responsible for
eye on looking to the opposite side (horizontal disrupting communication between the apparatus
gaze). The contralateral eye abducts fully, but required to deliver conjugate gaze giving rise to an
with nystagmus. The patient complains of internuclear ophthalmoplegia.
horizontal diplopia.
Additional finding of bilateral optic atrophy
on fundoscopy. Follow with a summary of
relevant negative findings
The neuroanatomy of conjugate gaze The other eye movements are normal and
importantly there is normal convergence to
The principle of conjugate gaze centres upon the accommodation. Adduction in the affected eye
coordinated movement of both eyes in the same is preserved in accommodation but not lateral
direction, at the same time. Gaze palsies result in a gaze as the former does not require the MLF.
lack of synchrony and result in diplopia.
64 Chapter 2 Nervous system (station 3)

State the most likely diagnosis Common clinical symptoms in multiple


on the basis of these findings sclerosis
The patient has dissociated conjugate eye
There are no clinical findings unique to multiple
movements consistent with an internuclear
sclerosis but some are highly characteristic,
ophthalmoplegia, suggesting a lesion in the
classically disseminated in space and time.
medial longitudinal fasciculus on the side of
Sensory or motor deficit of the limbs
the impaired adduction. A diagnosis of multiple
Visual disturbance including painful visual
sclerosis should be considered.
loss secondary to optic neuritis, and diplopia
due to ocular palsy or INO
Offer relevant differential Impairment of facial sensation
diagnoses Gait disturbance which may be related to poor
coordination due to cerebellar deficit or vertigo
Demyelination bilateral INO, together Bladder disturbance, usually in the first
with optic atrophy is virtually diagnostic of instance urgency
multiple sclerosis in the young patient. Generalised fatigue/exhaustion
Cerebrovascular accident commonly in the
elderly and often unilateral. Ask specifically about:
Rarely space occupying lesion such as Worsening of symptoms after a hot bath
neoplasm (brainstem glioma) or vascular Uhtoff phenomenon
malformation (ArnoldChiari) Electric shock sensation radiating with neck
Other conditions that may mimic an flexion Lhermitte sign
internuclear ophthalmoplegia include:
Dysthyroid eye disease (but adduction Demonstrate an understanding
with accommodative convergence equally of the value of further
impaired)
Orbital myositis (again accommodative
investigation
convergence equally impaired, and painful) The diagnosis of multiple sclerosis is
Exotropia predominantly a clinical one. However, imaging
and investigations can be used to support the
diagnosis, in line with the McDonald criteria.
Demonstrate the importance A diagnosis can be made on the basis of 2 or
of clinical context suggest more relapses and 2 or more objective clinical
lesions.
relevant questions that would
be taken in a patient history MRI evidence of cerebral, classically
periventricular, or cord plaques. There is no
Attempt to establish the pattern of disease on consensus on the role for serial imaging with
the basis of a history of symptoms commonly MRI to monitor the response to treatment or
encountered in multiple sclerosis: progression of disease
Relapsing- Characterised by relapses with Evoked potentials relate to electrical signals
remitting subsequent full recovery in the generated in the central nervous system in
early stages. Such remissions may response to sensory stimulation of peripheral
last for months to years. nerves. Visual evoked potentials are slowed.
Primary Steady decline without remission. Lumbar puncture with CSF analysis
progressive Generally later onset and poorer oligoclonal IgG bands
prognosis.
Secondary Initial relapsing remitting course Always offer a management plan
progressive followed by development of a
progressive decline. Relapsing-remitting disease
Progressive Disease progression is a steady Acute exacerbations are treated with
relapsing decline however clear, acute, corticosteroids. Disease modifying therapy aims
relapses are superimposed. to reduce the rate and severity of relapses.
Interferon beta
Case 32 Ptosis, large pupil and ophthalmoplegia: third nerve palsy 65

Glatiramer Fatigue: amantadine, selegiline


Natalizumab used in disease resistant to Bladder disturbance: anti-cholinergics, e.g.
interferon and glatiramer oxybutinin or tolterodine, intermittent self-
catheterisation or it may be that the switch
Progressive disease from self-catheterisation to long-term or
supraprubic catheterisation is necessary
Relapsing disease is associated with a better
prognosis than progressive disease.
Impotence: sildenafil
Progressive disease may be treated with an
Depression: tricyclic or SSRI, consider
counselling referral
immunosuppressant such as mitoxantrone.
Pain and paroxysmal features:
carbamazepine, gabapentin
Symptom control Visual problems: ophthalmology referral
Spasticity: baclofen or dantrolene, botulinum (exclude acute flare)
toxin and physiotherapy Neuropsychological referral
Tremor: clonazepam, gabapentin

Case 32: Ptosis, large pupil and


ophthalmoplegia: third nerve palsy

Instruction to the candidate Upward gaze is impaired


Please examine this 40-year-old woman who
On attempting downward gaze, the action
of superior oblique causes subtle adduction
complains of sudden onset double vision and a
and rotation
droopy eyelid.

Begin with a summary of Follow with a summary of


positive findings relevant negative findings
Ptosis of the right eye is evident, which is
The actions of superior oblique (IV) and
lateral rectus (VI) are intact
complete. To examine the remainder of the eye
requires gentle manipulation of the eyelid, held
The uniocular visual acuity is normal
open while assessing pupillary responses and
There is no proptosis or displacement of the
globe
eye movements.
The right pupil appears dilated and
There is no optic disc swelling
unreactive to light and accommodation. The
anisocoria is accentuated by bright conditions State the most likely diagnosis
and less noticeable in the dark. on the basis of these findings
In the resting position the right eye is found
in abduction, slight depression and intorsion The findings suggest weakness of the medial
down and out. On examination of eye rectus, superior rectus, inferior rectus, and
movements deficits of adduction, elevation and inferior oblique consistent with a third nerve
depression are noted: palsy. Involvement of the pupil suggests a
Adduction is limited and may not proceed compressive aetiology and raises the suspicion
past the midline of a cerebral aneurysm in the first instance.
66 Chapter 2 Nervous system (station 3)

Offer relevant differential and the pupils. Each subnucleus, with the
exception of the superior rectus subnucleus,
diagnoses supplies the ipsilateral muscle. Where due to
infarction the palsy is often associated with
The role of the pupil in determining contralateral hemiparesis and cerebellar
the cause ataxia. Other causes include space-occupying
The parasympathetic fibres supplying the lesion and demyelination
sphincter pupillae muscle originate from the Giant cell arteritis
EdingerWestphal nucleus and are carried on the Myasthenia gravis and thyroid eye disease
outside of the third nerve. The vasa nervorum may mimic pupil sparing third nerve palsy
supply blood to the nerve from the outside in. As and should be considered in all patients
such they are susceptible to external compression
and relatively spared by ischaemic lesions. It Demonstrate the importance
follows that a third nerve with a dilated pupil is
more likely to have a compressive (surgical) cause,
of clinical context suggest
such as aneurysm of the posterior communicating relevant questions that would be
artery, while preservation of the pupil is seen more taken in a patient history
often in ischaemic (medical) causes, commonly
microvascular occlusion secondary to diabetes. Pain is a common feature and is associated with
most differentials including ischaemia. In the
acute setting a third nerve palsy will usually
The differential should consider pupil sparing result in diplopia, whereas a chronic palsy
and non-sparing pathology and whether the may be asymptomatic. Screen for conditions
third nerve palsy is an isolated finding or found predisposing to ischaemia, particularly diabetes.
in the context of additional neurological deficit:
Intracranial aneurysm of posterior Demonstrate an understanding of
communicating artery of circle of Willis. Also
internal carotid and basilar artery aneurysms the value of further investigation
Cavernous sinus thrombosis associated with Investigation centres upon imaging with
other cranial nerve deficit commonly IV contrast CT or MRI and consideration of
and VI angiography. Lumbar puncture may be required
Direct compression of third nerve by space where imaging is inconclusive to rule out
occupying lesion (intracranial/intraorbital) subarachnoid haemorrhage.
Raised intracranial pressure (false localising
sign, especially if bilateral)
Peripheral ischaemic lesion due to Always offer a management plan
microvascular occlusion. Commonly as the Most cases of third nerve palsy recover. Those
result of diabetes or hypertension present at six months are likely to be persistent
Central/midbrain lesion the most deficits.
characteristic finding of a nuclear lesion Where appropriate, optimise vascular risk
is complete unilateral third nerve palsy. factors and consider anti-platelet therapy.
However, weakness affects both the Diplopia in the short term may be treated
ipsilateral and contralateral superior rectus with a patch/cover of the affected eye. Chronic,
and ptosis is bilateral due to the anatomy of mild deficits may be corrected with a prism.
the third nerve nuclei. The third nerve begins Surgical intervention is rarely advocated due
as a nucleus in the midbrain that consists to the complexity caused by the involvement
of several subnuclei that innervate the of four extra-ocular muscles and the resultant
individual extraocular muscles, the eyelids, uncertainty of the outcome.
Case 33: Ptosis and small pupil:
Horner syndrome

Instruction to the candidate of anhidrosis suggests a preganglionic lesion


(see below).
Please examine this patients pupils and proceed
as you feel appropriate.
Offer relevant differential
Begin with a summary of diagnoses
positive findings Common potential causes of Horners syndrome.
Ptosis Mllers muscle affecting both upper Central lesions affecting brainstem
and lower lids (levator palpebrae unaffected)
Miosis (constricted pupil) with dilation lag, and cervical/thoracic cord
a slow and delayed dilation of the pupil in Stroke (lateral medullary syndrome)
darkness due to the loss of active radial pull Tumour
of the dilator muscle. Demyelination
Anhidrosis (only in central/pre-ganglionic
lesions. Post-ganglionic unaffected) Pre-ganglionic spinal cord, thoracic
a difficult clinical sign to detect on
examination. outlet, lung apex
Pancoast tumour of the lung apex
Additional findings Brachial plexus lesion
There may be apparent enophthalmos an
appearance of a sunken eye. Post-ganglionic
Anisocoria associated with heterochromia Carotid artery dissection
(different coloured irides either eye) suggests Cavernous sinus mass
a congenital lesion (pigmentation in the first Middle cranial fossa tumour
months of life is under sympathetic control). Migraine
Where Horner's syndrome suspected, the
Anhidrosis is present in central or
strong candidate will look to localise the lesion
pre-ganglionic lesions. The sympathetic fibres
based on additional neurological deficit:
responsible for facial sweating and vasodilation
Brainstem diplopia, ataxia, weakness separate from the remainder of the oculo-
Spinal cord weakness, long tract signs, sympathetic pathway at the superior cervical
sensory level, sphincter involvement ganglion. Thus, anhidrosis is not a feature of
Brachial plexus hand weakness, wasting of post-ganglionic Horners syndrome.
small muscles, arm pain
Cavernous sinus VI nerve palsy with
Demonstrate the importance
diplopia in the absence of brainstem signs
Isolated Horners syndrome may suggest of clinical context suggest
head or neck pathology soft tissue masses, relevant questions that would be
lymphadenopathy
taken in a patient history
Seek to correlate the history with the presentation
Follow with a summary of to help differentiate the underlying cause.
relevant negative findings
The visual acuity and the eye movements are Central
normal. A history of possible dysarthria, dysphasia and
vertigo in the context of diplopia, ataxia and
State the most likely diagnosis weakness on examination.

on the basis of these findings Pre-ganglionic


Horners syndrome a lesion affecting the Trauma or pain associated with the brachial
sympathetic nervous supply to the eye and plexus.
orbit. The absence of diplopia and suspicion
68 Chapter 2 Nervous system (station 3)

Cough, haemoptysis, weight loss on a been proposed as an alternative test for Horners
background of significant smoking history syndrome where the diagnosis is in doubt.
suggestive of neoplasia, increasing suspicion of
an apical lung lesion. Differentiating between central/
pre-ganglionic and post-ganglionic
Post-ganglionic
lesions
VI nerve palsy with diplopia
Acute presentation with neck pain Hydroxyamphetamine testing
The Horners pupil caused by a central
As Horners syndrome has the potential to
or pre-ganglionic lesion will dilate to
be congenital or acquired, establishing onset
hydroxyamphetamine but the Horners
of symptoms is important. Comparing old
pupil caused by a post-ganglionic lesion will
photographs, particularly from childhood may
not. Hydroxyamphetamine releases stored
be of use.
norepinephrine from the postganglionic
adrenergic nerve endings, and therefore only
Demonstrate an understanding of relies on an intact post-ganglionic neurone for
effect. 24 hours must be left after the cocaine test
the value of further investigation before performing this test.
Confirming the diagnosis
Cocaine testing Identifying underlying cause
Unaffected pupils dilate in response to cocaine Clinical examination, history and testing can
drops. Cocaine inhibits noradrenaline re-uptake direct investigations, which may include:
at the sympathetic nerve synapse and causes Chest X-ray
pupillary dilation in eyes with intact sympathetic MRI brain/brainstem/cord as appropriate
innervation. Cocaine has no effect in eyes with CT/MRA/angiography to exclude carotid
impaired sympathetic innervation, regardless of dissection
the lesion location.
Apraclonidine testing Always offer a
Affected pupils dilate in response to management plan
apraclonidine, as the result of denervation
Treatment is directed to the underlying cause.
hypersensitivity, while unaffected pupils
Emphasis may be placed on discussing acute
constrict. Thus in a positive test there is a
presentations suggestive of carotid artery
reversal of the anisocoria. Apraclonidine is more
dissection or aneurysm requiring expedient
widely available than cocaine and as such has
vascular intervention.

Case 34: Bilateral ptosis with fatiguability:


myaesthenia gravis

Candidate information Begin with a summary of


This 30-year-old woman has presented positive findings
complaining of limb weakness and abnormal On inspection, the patient has classic
speech. Please inspect and proceed to an myasthenic facies:
appropriate examination. Bilateral ptosis
Case 34 Bilateral ptosis with fatiguability: myaesthenia gravis 69

Expressionless face with flat affect bilateral LambertEaton syndrome. Fatuigable proximal
lower motor neuron facial weakness with muscle weakness is also seen in myopathies.
myasthenic snarl when asked to smile Where there is predominantly ocular
manifestation, consider alternative causes of
Examination of eye movements reveals
complex ophthalmoplegia such as thyroid eye
weakness and diplopia in a pattern that does
disease.
not conform to individual nerves or muscles
suggesting a complex opthalmoparesis/plegia.
Be sure to demonstrate an appreciation Demonstrate the importance
of fatiguability (incremental weakness with
repetitive testing of muscle strength) and in
of clinical context suggest
doing so consider generalised involvement: relevant questions that would
Levator palpebrae superioris fatiguability be taken in a patient history
with sustained upward gaze
How do her symptoms affect her function and
Fatiguable, nasal speech due to weakness of activities of daily living?
pharyngeal and tongue muscles
Fatiguable proximal limb weakness (upper Involvement of speech and/or swallowing
limb more pronounced that lower limb) enquire as to episodes of dysphagia
possibly complicated by aspiration
pneumonia
Follow with a summary of Difficulty with mastication
relevant negative findings Limb weakness, classically proximal
The pattern of disease occular versus difficulty walking up stairs, rising out of a
generalised is an important consideration. chair unaided, or lifting arms above head
Thus, the presence of occular signs without Progressive worsening of symptoms towards
evidence of proximal weakness or vice versa the end of the day or after exercise?
would be important. Does she have any other autoimmune
The absence of a second-wind phenomenon diseases rheumatoid arthritis/SLE?
which, if present, would shift the likely diagnosis Ask specifically about previous myasthenic
towards LambertEaton syndrome. crises and admissions to HDU/ITU with or without
ventilatory support. Shortness of breath, indicating
State the most likely diagnosis diaphragmatic involvement is an important
consideration in neuromuscular disease.
on the basis of these findings
This patient displays bilateral ptosis,
fatiguability and proximal weakness consistent Demonstrate an understanding
with neuromuscular junction pathology. The of the value of further
likely diagnosis is one of generalised myasthenia
gravis (MG).
investigation
Blood tests
Occular versus generalised Antibodies to the post-synaptic cholinergic
receptors at the neuromuscular junction (AChR
myasthenia
antibodies) are the pathological mechanism for
There are two clinical forms of myasthenia gravis: weakness. Serological tests for these antibodies
ocular and generalised. are the key diagnostic investigation and are
In ocular myasthenia, the weakness is limited positive in the majority (>90%) of cases
to the eyelids and extraocular muscles Antibodies to muscle specific kinase (MusK)
In generalised disease, the weakness are found in up to 50% of generalised
commonly affects ocular muscles, but it also Myasthenia patients who are AChR antibody
involves a variable combination of bulbar, negative
limb, and respiratory muscles Seronegative cases, where AChR and
MusK antibodies are absent are rare but
more commonly associated with occular
Offer relevant differential myasthenia
diagnoses ESR/CRP to help exclude an inflammatory
The differential diagnosis of fatiguable myopathy
weakness includes myasthenia gravis and Thyroid function profile
70 Chapter 2 Nervous system (station 3)

Neurophysiological testing symptomatic treatment involves cholinesterase


drugs which inhibit the breakdown of
Bedside investigation with a Tensilon
acetylcholine at the NMJ. These drugs include
(edrophonium) test may be performed
neostigmine and pyridostigmine (Mestinon).
where a short-acting anticholinesterase
As MG is an auto-immune disorder,
inhibitor is administered to the patient.
corticosteroid therapy has a large role to play.
A positive result involves a transient
Steroid-sparing therapies (azathioprine and
improvement in power of affected muscle
mycophenolate) are also used in cases where
groups due to the prolonged presence
the side-effect burden of corticosteroids is large.
of acetylcholine in the neuromuscular
Biological treatment with rituximab is possible.
junction
If a thymoma or thymic hyperplasia is
Repetitive nerve stimulation (RNS) shows
present, the patient undergoes thymectomy.
decrement at high frequency stimulation
Thymectomy is also occasionally considered
with a 10% drop being significant
as a therapeutic option for symptom control in
Single fibre electromyography (EMG) shows
severe generalised MG.
jitter which is a characteristic finding
If disease is severe and/or in a myasthenic
crises, plasmapheresis and intravenous
Further consideration immunoglobulin (IVIG) can be used.
Other investigations include a CT Thorax to Plasmapheresis involves removing the causative
identify any thymic enlargement and forced auto-antibodies from the circulation. IVIG has a
vital capacity (FVC) when diaphragmatic similar mechanism of action.
involvement suspected. Avoidance of medications which worsen
weakness and predispose to a mysthenic crisis
including:
Always offer a management plan Aminoglycoside and quinolone antibiotics
Understanding of the pathophysiological Beta-blockers
mechanism of MG can act as an aide memoire Procainamide
as to the management of MG. Quinine
Due to auto-antibodies causing blockade Phenytoin
at the neuromuscular junction (NMJ),

Case 35: Relative afferent pupillary


defect (RAPD)

Instruction to the candidate only a brief pause, reveals a deficit. The affected
eye paradoxically dilates when light is moved
Please examine this patients vision.
from the fellow eye to the abnormal eye. This is
caused by dysfunction of the afferent pathway
Begin with a summary of of the affected eye (from the optic nerve to
the optic chiasm), such that the consensual
positive findings pupillary response from the fellow eye
On general inspection the pupils are equal with predominates.
no evidence of anisocoria. To help narrow the differential diagnosis test
Examination of the individual pupillary also for:
responses to light appears normal. Light shone
in each eye results in constriction with both a Impaired colour vision
direct and consensual response. Reduced visual acuity
Performing the swinging light test, moving Reduced eye movement or pain on eye
the light source quickly between each eye with movements
Case 35 Relative afferent pupillary defect (RAPD) 71

Look for: Offer relevant differential


Optic disc swelling diagnoses
Optic atrophy RAPD indicates an impaired optic pathway,
compared to the collateral side, anterior to
RAPD the chiasm. This is most commonly caused
by ischaemia, particularly in the elderly
The swinging light test compares the function (e.g. ischaemic optic neuropathy, central
of one optic nerve to that on the contralateral retinal artery occlusion), and inflammation,
side. It relies on the principle that the pupillary particularly in younger patients (e.g. optic
constriction response to light has an equal neuritis or retrobulbar neuritis, which are
relaxation response when the light source is often due to demyelinating disease).
removed. The term afferent refers to the optic Less commonly seen causes are:
nerve being the afferent limb of the pupillary light
reflex arc; the efferent limb is the parasympathetic Compressive optic neuropathy (e.g. optic
fibres of the third cranial nerve. The presence nerve sheath meningioma, other orbital
of a relative afferent pupillary defect suggests a tumour)
disrupted reflex arc due to either dysfunction of Advanced glaucoma
the retina or the optic nerve anterior to the optic Severe retinal disease (macular degeneration,
chiasm. retinal detachment, retinitis) where a
Pitfalls in assessing for RAPD: significant proportion of retinal nerve cells
Room too bright a subtle RAPD can only be are damaged
seen in a dimly lit room Traumatic optic neuropathy
Expecting RAPD in bilateral disease RAPD Radiation optic neuropathy
only indicates asymmetry in optic nerve Inherited optic neuropathy such as Lebers
function. If each eye has significant but equal Very dense amblyopia (visual acuity worse
disease, no deficit is observed than 6/60)
False positive hippus is a physiological
fluctuation in papillary constriction under Demonstrate the importance
steady illumination. It is usually bilateral
of clinical context suggest
relevant questions that would be
Follow with a summary of taken in a patient history
relevant negative findings Screen for past medical history as outlined in
the differential diagnosis.
Remember to check for signs of temporal
arteritis if it is a possibility (age >50 years,
sudden loss of vision with RAPD). Signs of Demonstrate an understanding
temporal arteritis can include:
of the value of further
Tender temporal arteries investigation
Absent temporal artery pulses
Scalp tenderness Formal slit lamp examination to assess the
Ischaemic ulceration of scalp retina
Consideration of MRI where ischaemia or
demyelination suspected
State the most likely diagnosis
on the basis of these findings Always offer a management plan
This patient has a relative afferent pupillary Treatment is directed to the underlying cause.
defect.
Case 36: Unilateral large pupil in a young
woman HolmesAdie pupil

Instruction to the candidate Follow with a summary of


Please examine this 32-year-old woman, sent by relevant negative findings
her new GP concerned about the appearance Visual acuity is intact
of her pupils on the background of a history of No restriction of eye movements, and no
minor head injury. diplopia
No ptosis
Begin with a summary of
positive findings State the most likely diagnosis
On general inspection there is anisocoria with on the basis of these findings
the appearance of a grossly dilated right pupil. Findings of a unilateral tonic pupil, impaired
The main clinical question in the patient with corneal sensation and loss of lower limb reflexes
anisocoria is deciding whether the larger pupil are consistent with HolmesAdie syndrome.
or the smaller pupil is the problem. The key Pupillary dilatation is due to parasympathetic
differentiating signs are: denervation at the ciliary ganglion with tonicity
The pupillary reaction to light as the result of iris sphincter hypersensitivity.
The pupillary response to accommodation
The reaction of relative pupillary size in the Offer relevant differential
light and in the dark
diagnoses
In this case, the right pupil constricts
poorly or not at all to light but reacts
Argyll Robertson pupils. Normally
Argyll Robertson pupils are bilateral and
better to accommodation. Having initially
constricted. HolmesAdie can rarely be
appeared larger than the left pupil, following
bilateral, and the pupils can become
constriction with accommodation the right
constricted and unreactive over time.
pupil remains tonically constricted and
smaller than the left. Subsequent relaxation is
Third nerve palsy (if ptosis and diplopia
along with dilated pupil)
slow and sustained.
Pharmacological mydriasis (is the patient
on any anticholinergics such as atropine,
Anisocoria: which pupil is abnormal? ipratropium, scolpamine)
Local disorders within the orbit affecting
In cases of anisocoria (unequal pupils), it can be the ciliary ganglion including tumor,
difficult to ascertain which is the abnormal pupil. inflammation, trauma, surgery, or infection
Measure both pupils in normal room lighting,
and then in dim lighting Demonstrate the importance
If the abnormal pupil is the larger, it will
remain large in normal room lighting while of clinical context suggest
the other constricts relevant questions that would be
If the abnormal pupil is the smaller, it will remain
small in dim lighting while the other dilates
taken in a patient history
Speed of onset and collateral history or objective
evidence with old photographs for change in
Additional findings: appearance. Screen for symptoms of blurred
On close examination (with the slit lamp, or vision or headaches.
high magnification direct ophthalmoscope)
there are vermiform movements of the iris Demonstrate an understanding of
the iris is seen to wriggle. This is attributable
to segmental palsy
the value of further investigation
Loss of corneal sensation absent corneal reflex The pupil can be tested for its response to
Loss of deep tendon reflexes in the legs pharmacological agents:
Case 37 Bilateral small pupils: Argyll Robertson pupils 73

Denervation hypersensitivity will result in reassurance once the diagnosis is confirmed.


constriction to a weak pilocarpine drop. After Although patients with Adies syndrome
the administration of dilute 0.1% pilocarpine, may complain of difficulty reading due to
a more miotic response will be seen accommodative paresis, this usually improves
compared with the fellow, normal, eye spontaneously. Where blurred vision requires
The response to mydriatic drops such as correction, contact lenses with an artificial pupil
tropicamide will be normal can be considered in the appearance conscious
patient.
Always offer a
management plan
No treatment usually required. The Adies tonic
pupil is benign, and most patients only require

Case 37: Bilateral small pupils:


Argyll Robertson pupils

Instruction to the candidate Non syphilitic causes


Please examine this 45-year-old mans pupils. Diabetes mellitus
Multiple sclerosis
Wernickes encephalopathy
Begin with a summary of Lyme disease
positive findings Sarcoidosis
The pupils are small and irregular. Varying Other midbrain pathology including tumours
degrees of anisocoria may be seen but both and ischaemia
pupils are constricted. They react poorly to light
but appropriately to accommodation. Conditions capable of mimicking
the presentation:
Follow with a summary of Miotic eye drops used for the treatment
of glaucoma may result in an iatrogenic
relevant negative findings presentation. Previous cataract surgery can
There is no evidence of ptosis and the visual result in irregular, albeit usually dilated,
acuity is intact with normal eye movements. pupils. Note that miotic pupils small,
regular pupils, are normal in the elderly
State the most likely diagnosis Uveitis (inflammation in the eye can result in
posterior synechiae, sticking the pupil edge
on the basis of these findings to the lens, giving small, irregular, unreactive
The findings suggest a diagnosis of Argyll pupils)
Robertson pupils consistent with a lesion affecting
the periaqueductal gray matter of the midbrain.
Demonstrate the importance
Offer relevant differential of clinical context suggest
diagnoses relevant questions that would
Classically Argyll Roberston pupils have been
be taken in a patient history
associated with late syphilitic infection (tabes Screen for systemic disease and behaviours
dorsalis). increasing risk of syphilis.
74 Chapter 2 Nervous system (station 3)

Demonstrate an understanding Always offer a management plan


of the value of further Directed towards underlying cause. High dose
intravenous or intramuscular penicillin is used
investigation to treat late stage syphilis infection. Treatment
General screen to reflect the range of conditions will prevent further progression, but will not
as outlined in the differential. Syphilis serology reverse existing damage.
may help to confirm the diagnosis due to the
strong association.

Case 38: Facial weakness

Candidate information on inspection. On history, see below, no


alteration of taste
Please examine this 40-year-old mans cranial
nerves. There are no vesicles around the external
auditory meatus and there is no associated
ipsilateral hearing loss.
Begin with a summary of There is no mass associated with the
positive findings ipsilateral parotid gland.
The patient has an asymmetrical face on
inspection with sagging of the eyebrow, loss State the most likely diagnosis
of naso-labial fold and drooping of the corner
of the mouth towards the right side at rest. On on the basis of these findings
testing, weakness is observed to involve the This patient has signs consistent with a lower
entirety of the right sided of the face including: motor neuron facial nerve deficit.
Paucity of blinking with difficulty closing
and/or resisting eye opening on the right Offer relevant differential
(orbicularis oculi) diagnoses
Inability to prevent deflation of the cheek
when puffed out and external pressure As with any lower motor neuron weakness,
applied (buccinator). The patient is unable to the lesion can be anywhere from the anterior
grimace, show teeth or pucker their lips horn cell to the innervated muscle, however the
Importantly, loss of ability to raise eyebrow causes include:
and wrinkle forehead on the right side
(occipitofrontalis) Common
Idiopathic Bells palsy
Follow with a summary of Infection Ramsay Hunt (herpes zoster oticus)

relevant negative findings Rare


Nerve lesions that occur proximal to the Infection Lyme disease
geniculate ganglion cause impairment of Inflammatory:
lacrimation, taste and salivation thus relevant GuillainBarr syndrome
negatives include: Sarcoidosis
Intact cornea with no evidence of ulceration Malignant parotid tumours are often
or infection associated with indolent, slowly progressive
Moist mucus membranes and tongue facial nerve palsy
Case 38 Facial weakness 75

Cerebello-pontine angle tumours possible, Altered taste sensation on anterior two-thirds


with a similar presentation, commonly of tongue
acoustic neuroma
Demonstrate an understanding
Upper neuron facial nerve weakness of the value of further
Facial nerve weakness where the forehead investigation
is spared implicates upper motor neuron
Patients with a typical presentation of Bells
pathology due to the bilateral nature of cortical
palsy with no worrying features do not
innervation from the facial nucleus in the brain
typically require further investigation unless no
stem. Orbicularis occuli may also be relatively
improvement or resolution is observed within
spared in central lesions for the same reason. The
4 months, at which point the diagnosis should
differential diagnosis should focus upon stroke,
be reconsidered.
with consideration given to a cerebellopontine
Neurophysiology is predominantly used to
angle lesion or inner ear causes where vertigo,
assess severity, identifying the degree of residual
ataxia, and hearing loss feature.
nerve function, and predict outcome.
During their examination the strong candidate
Neuro-imaging with MRI should include
will take the time to assess for any associated
brain, temporal bone and parotid gland.
limb weakness. As a common cause for upper
Inflammatory markers and autoimmune screen
motor neuron facial weakness is stroke, it is
if a systemic and/or vasculitic cause suspected.
important to identify any associated upper
Serological testing for Lyme disease where
motor neuron limb weakness. By identifying
suspected.
these, the strong candidate can present the
examiner with a diagnosis and differential that
more accurately reflects the patients underlying Always offer a management
condition including any associated risk factors plan
such as AF, carotid bruits and evidence of anti-
coagulation. Bells palsy can be expected to recover with
conservative management within 68 weeks.
There is good evidence for the use of
Demonstrate the importance corticosteroids and aciclovir within 24 hours of
of clinical context suggest onset of Bells palsy:
relevant questions that would be Steroid link to neuroanatomy with the
course of the seventh nerve through the
taken in a patient history temporal bone, and narrowing at the internal
Important questions would include: acoustic meatus. Swelling thus results in
The speed of onset of symptoms potential ischaemia, and necrosis. Reduction
Any recent flu-like symptoms in the extent of inflammation may thus be
Any pain or discharge from the ipsilateral protective
ear Aciclovir link to serological evidence
Does the patient have a past medical history suggesting herpes-simplex virus activation
of diabetes? as the cause of Bells palsy in most cases

Screen for related symptoms: Eye care is advisable with use of taping or a
patch, particularly at night, and drops to prevent
Hyperacusis drying.
Case 39: Conductive hearing loss

Candidate information Unilateral senorineural loss the sound will


Please examine the hearing of this 45-year-old localise to the unaffected ear
man who has been experiencing difficulty Rinnes test
hearing in his right ear. Conductive hearing loss is indicated where
bone conduction is perceived to be better
Begin with a summary of than air conduction as evidenced by no
perception of sound once the tuning fork is
positive findings moved from the mastoid process to close to
Gross assessment of hearing by rubbing fingers the pinna
by the patients ears in turn confirms hearing Sensorineural loss is suggested when both
loss in the right ear. bone conduction and air conduction are
Webers test localises to the right ear reduced, however normally air conduction
(the affected ear). will remain superior to bone if the deficit is
When Rinnes test is performed at the not complete
right ear, bone conduction is better than air
conduction. That is to say, the patient hears the
sound better when the tuning fork is placed on
Follow with a summary of
the mastoid process rather than when it is held relevant negative findings
close to the external auditory meatus. No evidence of associated cranial nerve
lesions.
Webers and Rinnes tests
Begin by understanding the findings of each test
State the most likely diagnosis
where no pathology exists. on the basis of these findings
This patient has signs of conductive deafness.
Webers test
A tuning fork (256 or 512Hz) is vibrated and
placed in the midline of the forehead. Where no
Offer relevant differential
pathology exists, sound is perceived equally in diagnoses
each ear. Conductive hearing loss is attributable
to pathology affecting the outer ear,
Rinnes test
predominantly the outer auditory canal, or
The tuning fork is initially placed on the mastoid the middle ear, impacting upon the tympanic
process. Patients in whom there is no deficit membrane or ossicles, preventing the
affecting the inner ear will report hearing sound transmission of sound to the functioning inner
conducted through bone, bypassing the auditory ear.
canal and tympanic membrane. When the patient
reports no further sound from the fork, which Causes of conductive deafness
is still vibrating, it is moved from the mastoid
attributable to outer ear
process to the pinna. At this point, where no
conductive problem is present sound is once again pathology include
heard, indicating air conduction superior to bone Cerumen (wax) accumulation
conduction. Infection otitis externa, often due to trauma
to the inner ear and subsequent introduction
Pathology of bacteria or fungus with increased risk in
Thereafter consider the affect of pathology on the the diabetic population
tests: Chronic psoriasis causing recurrent local
Webers test irritation with resultant scar tissue impeding
Unilateral conductive loss the sound will conduction
localise to the affected ear Local tumours (squamous cell)
Case 39 Conductive hearing loss 77

Causes attributable to middle ear with otoscopy. The external ear is inspected for
obstruction or infection. Thereafter, examination
pathology include of the tympanic membrane for perforation,
Infection otitis media (OM) chronic/ evidence of secretion or drainage suggestive
recurrent may result in permanent of otitis media, or signs consistent with
hearing loss particularly if complicated by choleasteatoma.
cholesteatoma Disorders of the tympanic membrane or
Eustachian tube dysfunction/blockage, middle ear apparatus such as chronic OM or
often secondary to an upper respiratory otosclerosis may require formal audiological
tract infection, may result in perceived testing. Tympanometry is useful for assessing
hearing impairment similar to that middle ear pathology.
associated with change in air pressure Where head trauma, cholesteatoma or a
(flying/diving) benign/malignant growth is suspected it would
Trauma to tympanic membrane be considered prudent to perform CT or MRI.
Otosclerosis where abnormal bone growth
affecting the middle ear results in reduced
mobility of the stapes impacting upon its Always offer a management plan
ability to transmit sound to the inner ear Causes are treated:
Overall the most common causes of Otitis media - upon resolution of the acute
conductive hearing loss are wax accumulation infection a large proportion of patients with
and infection. hearing deficit will continue to experience
problems for up to 6 weeks. In a small
number of patients, hearing deficit due to
Demonstrate the importance residual fluid in the middle ear preventing
of clinical context suggest free movement of the tympanic membrane
relevant questions that would and diminishing movement of the ossicular
chain, require myringotomy and grommet
be taken in a patient history (tympanostomy tube) placement or
Important questions include establishing the aspiration.
timing and onset of the hearing loss acute Cerumen impaction removal, often with
versus gradual. Thereafter look to identify suction.
precipitating events, including: Consideration can be given to the benefit of
surgical removal of masses, such as benign or
Infection malignant growths, blocking the eustachian
Head trauma tube or ear canal.
Barotrauma (recent SCUBA diving for
instance) Where the hearing deficit is permanent and/
or severe, or where reversible causes have been
Seek to elicit associated symptoms such as treated without resolution, refer to a hearing
tinnitus or headaches and assess the impact on loss specialist for further investigations and
activities of daily living. management, including adaptations to deal with
hearing loss.
Demonstrate an understanding
of the value of further
investigation
Investigation in the first instance centres upon
clinical evaluation and direct visualisation
Case 40: Sensorineural hearing loss

Instruction to the candidate Infection: childhood measles/mumps


Examine the hearing of this 40-year-old man
Autoimmune disorders: RA/SLE
recently discharged from the ITU.
Mnires
Trauma with base of skull fracture

Begin with a summary of VIII nerve


positive findings Congenital causes:
Gross assessment demonstrates gross hearing Infection: maternal rubella, CMV,
loss in the right ear. Webers test suggests that toxoplasmosis
the sound from the tuning fork is heard best in Neurofibromatosis type 2
the contralateral, left, ear. When Rinnes test is
performed, air conduction is heard better than Acquired:
bone conduction in both ears albeit reduced in Tumours affecting the cerebello-pontine
the right compared with the left. angle
Demyelinating disease
Follow with a summary of
relevant negative findings Demonstrate the importance
Cranial nerve deficits, commonly: of clinical context suggest
V Loss of facial sensation and weak jaw clench relevant questions that would
VII Weakness or asymmetry of the face and be taken in a patient history
abnormal sense of taste
Screen for medical conditions with the
Cerebellar signs, which may suggest potential for hearing loss, as above, and review
compressive lesions at the cerebellopontine medication for current of previous use of drugs
angle e.g. schwannoma. with known ototoxicity.
Look to identify associate symptoms:
State the most likely diagnosis Tinnitus
on the basis of these findings Pain/headache
Vestibular symptoms dizziness, vertigo
The patient displays signs of sensorineural often worse in the dark
hearing loss.

Demonstrate an understanding of
Offer relevant differential
the value of further investigation
diagnoses
Formal audiological testing:
Inner ear (sensory) or auditory nerve (neural)
lesions resulting in hearing deficit include: Pure tone and speech threshold testing will
quantify the level of hearing loss
Inner ear Speech discrimination, while reduced in
both, is often worse in neural than in sensory
Congenital causes:
deficit
Infection: maternal rubella, CMV, An absent acoustic reflex may indicate a
toxoplasmosis lesion of the auditory nerve
Maternal use of ototoxic drugs commonly in
Red flags indicating the need for imaging
the treatment of TB, historically thalidomide
include unilateral sensorineural hearing loss
Acquired: and abnormalities of cranial nerves other than
VIII (commonly V and VII). MRI with contrast
Noise exposure/presbycusis for soft tissue or vascular masses or CT where
Ototoxicity: aminoglycosides, vancomycin, bony lesions are suspected.
loop diuretics
Case 41 V, VII, VIII cranial nerves cerebellopontine angle 79

Always offer a management plan Minimising the impact of ototoxic drugs,


particularly antibiotics, where possible. Vigilant
Identify and treat reversible causes. Sensory
testing for peak and trough levels to guide
hearing loss, due to pathology of the inner ear, is
dosing and monitoring renal function prudent.
often reversible and rarely life-threatening unlike
Where the hearing deficit is permanent and/
lesions affecting the auditory nerve which a rarely
or severe, or where reversible causes have been
reversible and may be due to intracranial masses
treated without resolution, refer to a hearing
such as cerebello-pontine tumours.
loss specialist for further investigations and
Steroids may be beneficial in patient with
management, including adaptations to deal with
autoimmune disease.
hearing loss.

Case 41: V, VII, VIII cranial nerves


cerebellopontine angle

Instruction to the candidate auditory meatus than when held against the
mastoid process
This 74-year-old man has been referred to the
neurology outpatient clinic with difficulty in Additional signs:
balance. Please assess him and present your
findings to the examiner.
Nerves IX, X, XI and XII may be affected
Long tract signs would indicate involvement
of the brain stem
Begin with a summary of
positive findings Facial sensory loss
Examination reveals cerebellar signs: In the neurology station as in this case
abnormalities of facial sensation are most
Ataxic gait
likely to present in the context of other cranial
Nystagmus on the right
nerve lesions rather than as an isolated clinical
Past-pointing on the right
sign. When considering facial sensory loss,
Right-sided dysdiadochokinesia
the following points on anatomy should be
General inspection suggests right-sided facial considered should be considered:
weakness and formal examination reveals:
Highly focal deficits attributable to an
Right-sided lower motor neuron facial individual branch of the trigeminal nerve
weakness either ophthalmic (VI), maxillary (V2) or
Hemi-facial loss of sensation on the right. mandibular (V3) are generally distal to the
Candidates should also offer to examine for a ganglion while hemisensory loss is indicative
corneal reflex which is absent. of pathology affecting the ganglion itself.
Lesions of the lower pons, medulla, or upper
Facial weakness in this context should prompt cervical cord, will produce a dissociated
assessment of hearing, demonstrating a sensory loss of pain and temperature, with
sensorineural deafness as evidenced by: preservation of normal light touch, vibration
Assessment demonstrates gross hearing loss and proprioception in a classical balaclava
in the right ear or onion skin distribution. As the lesion
With Webers test the sound from the tuning extends up the brainstem, the sensory deficit
fork is heard best in the contralateral, left, ear spreads incrementally towards the nose.
When Rinnes test is performed, air When as part of a constellation of clinical signs,
conduction is better than bone conduction. facial sensory loss can present in a number of
The patient hears the sound better when ways:
the tuning fork is held outside the external
80 Chapter 2 Nervous system (station 3)

As in this case, lesions at the Lipomas


cerebellopontine angle are often associated Granulomas
with a disturbance in ipsilateral facial
sensation and a reduced corneal reflex. An
isolated ophthalmic division deficit is often
Demonstrate the importance
an early sign of a cerebellopontine angle of clinical context suggest
lesion prior to onset of facial weakness with relevant questions that would be
VII and VIII involvement.
Supranuclear lesions (e.g. cerebrovascular, taken in a patient history
demyelination, neoplastic) may result in Symptoms can be divided into those due
facial sensory loss associated with ipsilateral directly to cranial nerve involvement and other
pyramidal weakness, ipsilateral upper motor symptoms caused indirectly.
neuron facial weakness, or other cortical
signs such as dysphagia, inattention, apraxia, Symptoms associated with
and hemianopia.
Involvement of III, IV, and VI would suggest
individual cranial nerve lesions
a cavernous sinus lesion often associated V nerve symptoms: numbness
with ophthalmic and maxillary branch VIII nerve symptoms: slowly progressive
involvement. unilateral hearing impairment with or
Concomitant Horners syndrome would without tinnitus
suggest a lateral brainstem or upper cervical Additionally, there may be a vestibular
lesion. Think of lateral medullary syndrome. element with dizziness
Cerebellar involvement: slurred speech,
Follow with a summary of balance issues, incoordination.
relevant negative findings Indirect symptoms
Note the absence of bilateral signs.
Blurred vision and headache can occur due
to false-localising signs as a result of raised
State the most likely diagnosis intracranial pressure.
on the basis of these findings
The pattern of signs, with unilateral Demonstrate an understanding of
involvement of the V, VII and VIII cranial nerves the value of further investigation
along with cerebellar signs, is consistent with a
This patient would warrant urgent intracranial
diagnosis of a cerebello-pontine angle lesion.
imaging, likely a CT head in acute presentations,
The likely cause is an acoustic neuroma.
followed by MRI with contrast in all.
Other investigations would include:
Offer relevant differential Audiometry to confirm and grade the degree
diagnoses of sensorineural hearing loss
Any mass lesion is capable of producing CSF analysis where TB or sarcoidosis is
a cerebello-pontine angle syndrome. The suspected
commonest CPA lesion is an acoustic neuroma
(schwannoma). Note bilateral acoustic neuromas
are common in neurofibromatosis type 2.
Always offer a management plan
Other lesions at the CPA include: Management depends on the underlying cause
but any mass lesion, which is amenable to
Menigiomas surgical excision, should be removed where
Arachnoid cysts possible. It is important to look behind the
Vascular malformations patients ear for evidence of previous surgical
Cholesteatomas intervention.
Case 42: Dysphasia

Instruction to the candidate State the most likely diagnosis


Please examine this 60-year-old mans speech. on the basis of these findings
Expressive dysphagia in the context of right
Begin with a summary of hemiparesis consistent with a lesion affecting
Brocas area in the left frontal hemisphere.
positive findings
The examination is remarkable for speech that
is non-fluent with the slow production of short Offer relevant differential
phrases. It is hesitant and stutters with apparent diagnoses
significant effort required to communicate. The In any case of cerebrovascular accident be
ability to convey meaning is impaired with the prepared to discuss the likely territory affected
use of inappropriate words. Naming objects and and the relevant vascular anatomy.
repetition are both impaired. There is a poverty Causes of damage to areas of the brain
of descriptive speech and the use of neologisms required for speech can be considered generally
(made up words used to convey meaning) as one of:
evident. Infarction
Comprehension is intact as evidenced by the Tumour
ability to follow a three-stage command asked Trauma
to point to their head, pick up a pencil, and Degeneration
stand up for instance.

Demonstrate the importance


Variations in presentation of dysphasia
of clinical context suggest
Expressive (non-fluent) dysphasia
The patient demonstrates incomprehensible
relevant questions that would be
speech. Expressive dysphasia due to a deficit in taken in a patient history
Brocas (frontal lobe) speech area of the brain. Evaluate speed of onset and assess for
Receptive (fluent) dysphasia impairment of written language, which may
There is evidence of abnormal comprehension of affect both reading and writing. Understanding,
language and inability to recognise auditory or as with verbal speech, is intact.
visual stimuli. Receptive dysphasia due to a deficit Screen for cardiovascular risk and evidence
in Wernickes (posterosuperior temporal lobe) of previous transient ischaemic attacks.
language area of the brain.
Mixed/global dysphasia
There is evidence of both abnormal comprehension Demonstrate an understanding
of language and production of incomprehensible of the value of further
speech. Given that the damaged area of brain this
presentation is relatively large, one would expect to
investigation
find additional visual field or long-tract signs. Acute stroke demands urgent CT head with
Conduction dysphasia subsequent consideration of thrombolysis and
The patient displays normal comprehension of transfer to a stroke centre. Aside from imaging,
language and is capable of fluent speech but the investigations required will depend on the
appears confused and is nonsensical. There is a likely underlying cause.
failure of repetitive speech. Conductive dysphasia Ischaemic stroke
due to a defect in the arcuate fasiculus that links Blood tests to assess cardiovascular risk,
Brocas area to Wernickes area. including HbA1c and lipid profile
A 12-lead electrocardiogram to ensure to rule
out atrial fibrillation. Other, less common
Follow with a summary of pieces of useful information would include
relevant negative findings the presence of sustained ST elevation post-
Evidence of motor cortex involvement Brocas MI which may be suggestive of left ventricular
area sits anterior to the motor cortex in the aneurysm formation which can predispose to
posteroinferior frontal lobe. left ventricular mural clot formation
82 Chapter 2 Nervous system (station 3)

A trans-thoracic echocardiogram will identify Always offer a


any structural abnormalities of the heart
and, in the setting of atrial fibrillation, give management plan
information about the size of the left atrium, In the first instance, evaluation of potential
which is important for consideration of for thrombolysis in acute stroke. Thereafter,
cardioversion secondary stroke prevention anti-hypertensive,
A 24-hour tape will potentially identify any anti-platelets, anti-coagulation, statin
underlying arrhythmias therapy, and attending to reversible causes of
Carotid Doppler images will identify any thromboembolism including anticoagulation
carotid stenosis. It is worthwhile reviewing and/or rhythm control of atrial fibrillation or
the indications for carotid endarterectomy in endarterectomy for critical carotid disease
the setting of cerebro-vascular events Functional impairment is often significant
with rehabilitation and recovery dependent
MRI with contrast should be considered where upon a multi-disciplinary team.
signs of raised intracranial pressure suggest a Consideration of treatment options for
space-occupying lesion. neoplastic disease, including palliative options.

Case 43: Dysarthria bulbar palsy

Instruction to the candidate and difficult to hear. Jaw jerk is pathological. More
Please assess this 60-year-old woman who has common than bulbar palsy, pseudo-bulbar palsy is
been experiencing difficulties with her speech commonly caused by:
for over 12 months. Multiple sclerosis
Motor neuron disease
Tumours affecting the brainstem
Begin with a summary of
positive findings Follow with a summary of
On inspection wasting of the tongue, with
fasciculations, evident. Speech is quiet relevant negative findings
with a nasal quality sometimes described Examination reveals pure motor weakness with
as flaccid dysarthria. Hoarseness suggests no sensory deficit or cerebellar signs.
vocal cord involvement. Excessive salivation There is no involvement of the extra-ocular
with drooling and as a consequence regular muscles.
forced swallowing of saliva is noticeable with
occasional choking.
Examination of the upper and lower limbs
State the most likely diagnosis
suggests weakness with mixed upper and on the basis of these findings
lower motor neuron signs. Importantly, these Bulbar palsy secondary to amyotrophic lateral
mixed signs are also observed within the same sclerosis (ALS).
limb.

Offer relevant differential


Bulbar versus pseudo-bulbar palsy diagnoses
Bulbar palsy is due to diseases affecting the nuclei
of cranial nerves IXXII in the medulla. The clinical
Of bulbar palsy
manifestation is one of lower motor neuron signs. Motor neuron disease (MND)
Pseudo-bulbar (cortico-bulbar) palsy is due GuillanBarr
to lesions affecting the cortico-bulbar tracts. Syringobulbia
The tongue is small and spastic. There are no Myasthenia gravis
fasciculations. Speech is weak, slow and deliberate, Poliomyelitis
Case 43 Dysarthria bulbar palsy 83

Of motor neuron disease involve a combination of limbs, axial and


respiratory weakness over a period of months
Amyotrophic lateral sclerosis is the most
to years.
common form of MND. Variants of ALS include:
In relation to the bulbar palsy, evaluate
Progressive muscular atrophy presentation involvement of difficulties with mastication
with only lower motor neuron dysfunction, and dysphagia, reduced gag reflex increases the
which may progress to involve upper motor potential for aspiration pneumonia. The patient
neuron degeneration may complain of nasal regurgitation.
Primary lateral sclerosis presentation with Screen for emotional lability which may
only upper motor neuron dysfunction, which indicate an ALS-plus picture with cognitive
may progress to involve lower motor neuron impairment.
degeneration
Progressive bulbar palsy upper and lower
Demonstrate an understanding
motor neuron dysfunction of the cranial
nerves which often spreads to the upper of the value of further
limbs. Where mixed findings involve the investigation
cranial nerves an alternative diagnosis to
EMG and nerve conduction studies to rule out
MND is unlikely
NMJ dysfunction or demyelination.
Various disorders may resemble MND in MRI of the brain and cord to exclude
the early stages. These disorders should be alternative diagnoses where ALS is suspected,
considered, not least due to the potential for including structural lesions such as
treatment and improvement as compared spondylosis, herniation, and syrinx.
with the poor prognosis associated with MND.
Conditions with the potential for pure motor
involvement include:
Always offer a management plan
Riluzole can be beneficial. It is postulated to
Cervical myelopathy upper and lower increase survival by several months, particularly
signs within the upper limb but with a in those patients with bulbar involvement and
characteristic pattern and associated signs difficulty swallowing. Riluzole does not reverse
Nueromuscular junction disorders MG existing damage or deficit but slows progression,
Myopathies particularly in relation to the eventual need for
Polymyositis/dermatomyositis ventilatory support.
Spinal muscular atrophy lower motor Multidisciplinary support. Speech therapists
neuron signs with bulbar involvement can provide invaluable support particularly
Infections Lyme disease, HIV, syphilis may in developing strategies to improve speech
produce MND-like symptoms and prepare for potential loss with alternative
communication skills. Practical measures
Demonstrate the importance such as suction devices. Consideration should
be given to the eventual requirement for
of clinical context suggest non-invasive ventilation and palliative care
relevant questions that would be planning.
Symptomatic relief can be achieved with:
taken in a patient history
The history should be consistent with a Baclofen spasticity
progressive upper and lower motor neuron Quinine cramps
dysfunction affecting one area of the body, Anti-cholinergic salivation
such as the bulbar region, and spreading to Amitriptyline emotional lability
Case 44: Lateral medullary syndrome

Instruction to the candidate Offer relevant differential


A 58-year-old type II diabetic presented with a diagnoses
sudden onset of speech, swallowing difficulties,
Lateral medullary syndrome is the most
ataxia, hemifacial sensory and contralateral
common and important syndrome related to
hemisensory temperature loss.
intracranial vertebral artery occlusion. The
majority of occlusive lesions are atherlosclerotic.
Begin with a summary of Other possible causes of lateral medullary
syndrome include:
positive findings
Ipsilateral findings: Cardiovascular embolus sources
Hemi-facial sensory loss to pain and AF, valvular heart disease, LV aneurysm,
temperature atrial myxoma
Hemi-facial pain Rarer causes of vertebro-basilar emboli
Arm and leg ataxia including vertebral artery dissection
Gait ataxia Thrombophilic/procoagulent states (OCP
Nystagmus and pregnancy) can be contributory factors
Vertigo
Horners syndrome Non cardiovascular causes
Contralateral findings: Demyelinating plaques can occasionally occur
in exactly this anatomical distribution
Hemi-sensory loss of pain and temperature
If the lateral medullary syndrome evolves
sensation
relatively slowly, consider:

Follow with a summary of Neoplastic lesions such as primary gliomas


and secondary metastases
relevant negative findings Rarely, infectious processes leading to local
Absence of cardiovascular signs, which may abscesses
point to an embolic source:
Atrial fibrillation Demonstrate the importance
Valvular heart disease of clinical context suggest
Absence of xanthelasma which may indicate relevant questions that would be
hyperlipidaemia taken in a patient history
Speed of onset of symptoms:
State the most likely
Previous TIAs with vestibulocerebellar
diagnosis on the basis symptoms or elements of lateral medullary
of these findings syndrome. The most common symptoms is
This patient has signs consistent with lateral dizziness
medullary syndrome. Screen for cardiovascular risk
The diagnosis is evidenced by involvement
of the facial and vestibular nuclei as well Demonstrate an understanding of
as nucleus ambiguous (dysphagia). There
is additional evidence for damage to the
the value of further investigation
spinothalamic tract, the descending sympathetic Imaging
fibres (Horners syndrome) and midline MRI of the brain. There is no role for CT of the
cerebellar structures (gait ataxia). brain in these circumstances as the resolution
Case 45 Jugular foramen syndrome 85

for this technique in the posterior fossa is poor. Always offer a management
MR angiogram of extra-cranial vertebral arteries
is useful in excluding dissection. plan
In the first instance evaluation of
Cardiac investigations potential for thrombolysis in acute stroke.
Thereafter, secondary stroke prevention
Echocardiogram
including anti-hypertensive, anti-platelets,
ECG
anti-coagulation, statin therapy.
Holter monitoring
Consideration of treatment potions for
neoplastic disease, including palliative options.

Case 45: Jugular foramen syndrome

Instruction to the candidate State the most likely diagnosis


Please examine this 64-year-old man who has on the basis of these findings
complained of progressive difficulty swallowing This patient has involvement of the IX, X and
and problems with his speech, worsening over a XIth cranial nerves constituting a left sided
period of 3 months. jugular foramen syndrome. The additional
involvement of the XIIth nerve on the same side
Begin with a summary of suggests a lesion at the base of the skull.
positive findings
Examination of the cranial nerves reveals: Offer relevant differential
Decreased/absent palatal movement on the
diagnoses
left Causes of a jugular foramen syndrome include:
Decreased power of left sternocleidomastoid Glomus jugulare tumour
and trapezius with some wasting Base of skull metastases commonly from a
Left sided tongue wasting and some prostate cancer, or plasmacytoma
fasiculations. Tongue deviates to the right on Lymphoma, TB, sarcoidosis
protrusion Carotid artery dissection (rare)
Focal anterior horn cell disease (MND)
Additional findings can produce a similar presentation but the
Albeit difficult to test within a PACES presence of sensory signs exclude this
environment, information may be included in the
case vignette alluding to altered taste involving
the posterior 1/3 of the tongue on the left.
Demonstrate the importance
of clinical context suggest
Follow with a summary of relevant questions that would be
relevant negative findings taken in a patient history
No contralateral signs all signs are Speed of onset
lateralised Systemic features of disease and relevant risk
No lymphadenopathy or masses palpable factors
within the soft tissue of the neck Consequences of dysphasia
86 Chapter 2 Nervous system (station 3)

Demonstrate an understanding of Lymph node excision/biopsy, if


lymphadenopathy present
the value of further investigation
Investigation revolves around imaging:
Always offer a management plan
MRI/MR venogram to visualise the jugular Removal of glomus jugulare tumour or
bulb and venous structures treatment of metastases
CT skull base Anti-microbials for TB or other infections
Blood tests: ESR, serum electrophoresis, Corticosteroids for sarcoid
Bence Jones proteins, serum ACE, TB Haematologyoncology management of
ELISPOT, lactate dehydrogenase lymphoproliferative disorders

Case 46: Myotonic dystrophy

Instruction to the candidate the diagnosis where there is difficulty


demonstrating it
This 20-year-old man has recently been
diagnosed with diabetes. He has presented
Wasting of the intrinsic muscles of the hand
and distal muscles of the arm
complaining of weakness. Please inspect the
patient and examine as appropriate. Other positive findings may include additional
evidence of myotonia such as:
Begin with a summary of Cataracts
positive findings The presence of a pacemaker device in the
infraclavicular fossa indicative of AV node
Inspection reveals classical facial features of involvement
myotonic dystrophy including: Finger prick marks from blood glucose
Frontal balding testing indicating diabetes mellitus
Temporalis wasting Testicular atrophy
Bilateral ptosis
Focusing examination on the bilateral ptosis Follow with a summary of
may be tempting, but given the likely differential relevant negative findings
of myotonic dystrophy and myasthenia gravis
No evidence of fatiguability.
the strong candidate will consider examination
of the upper limbs in the first instance.
Examination reveals: State the most likely diagnosis
Muscle weakness with myotonia. Myotonia on the basis of these findings
is a slowed relaxation following a normal The patient has muscle weakness with evidence
muscle contraction and can be demonstrated of transiently increased tone in response to
by slow grip release after an introductory voluntary contraction or percussion. This
handshake with the patient. Alternatively, is consistent with a diagnosis of myotonic
firm percussion of the thenar eminence dystrophy.
(specifically abductor pollicis brevis)
will result in abduction of the thumb
followed by slow relaxation. Predominantly Offer relevant differential
affects the facial, jaw, tongue and hand diagnoses
muscles. Importantly, myotonia tends to Myopathies, which may be inherited or
fade as muscle weakness worsens with acquired:
progression of disease, thus avoid dismissing
Case 46 Myotonic dystrophy 87

Hereditary facio-scapulo-humeral, limb a positive family history. Confirmation centres


girdle upon electromyography, and genetic testing.
Acquired neuromyotonia (anti-voltage
gated potassium channel antibodies) Neurophysiology
paraneoplastic myopathies
EMG may be used to demonstrate myotonia
Myasthenia gravis (anti-acetylcholine
where uncertainty exists as to clinical findings,
receptor antibodies)
and to distinguish from the fatiguability seen in
Mitochondrial myopathies (these can also
myasthenia.
have diabetes and cataracts as part of the
spectrum)
Genetic testing
Specific genetic testing to demonstrate the
Demonstrate the importance presence of an expanded CTG repeat in the
of clinical context suggest myotonic dystrophy protein kinase (DMPK)
gene on chromosome 19 is the gold standard
relevant questions that would be for the diagnosis. The size of the expansion
taken in a patient history length is linked to severity of disease as well
Muscle pain is common affecting the legs more as contributing to anticipation with successive
than the arms. Myotonia is often referred to generations having more severe disease.
by patients merely as muscles stiffness and Maternal inheritance may suggest
they may not fully appreciate the nature of the mitochondrial disease which can be tested for
symptom. Symptoms may be aggravated by cold by genetic analysis of mitochondrial DNA.
and stress. Also consider specific testing to investigate
Screen for systemic symptoms: the spectrum of associated conditions:

Respiratory complications are common Oral glucose tolerance testing


screen for shortness of breath and 12-lead ECG, echocardiogram
pneumonias Formal slit lamp evaluation
Gastrointestinal symptoms due to
involvement of smooth muscle may present Always offer a management plan
as colicky abdominal pain often with bloating
and constipation as pseudo-obstruction There is no disease-modifying therapy available
Diabetes for the treatment of myotonic dystrophy. Thus,
Palpitations associated with arrhythmias, treatment is directed towards symptom control.
commonly atrial fibrillation or flutter. Aortic Pain management will often require
valve disease is also prominent neuropathic modulating agents such as
Given the autosomal dominant nature of the gabapentin and amitriptyline
disorder, be sure to explore a thorough family Physiotherapy, specifically late in disease
history (see below). where distal weakness becomes more
prominent. The use of orthoses for foot drop
and wrist weakness is common
Demonstrate an understanding Particular care should be taken in relation
of the value of further to anesthesia with the chance of respiratory
failure precipitated by sedatives and
investigation neuromuscular blocking agents
The diagnosis of myotonic dystrophy can
be readily established when there is muscle Genetic counselling is paramount since prenatal
weakness and clinical myotonia in the context of diagnosis is possible.
Case 47: Cervical myelopathy

Instruction to the candidate Additional signs on general


Please examine the upper limbs of this 80-year- inspection
old woman with severe rheumatoid arthritis. Deformity of the cervical spine or evidence of
rheumatological conditions, such as RA, OA
Begin with a summary of or ankylosing spondylitis, which may result
in degenerative change. The patient may be
positive findings wearing a supportive c-spine collar.
Examination of the upper limbs reveals:
Wasting of the deltoids and biceps evident on Follow with a summary of
general inspection with fasciculations visible relevant negative findings
from the end of the bed. Testing confirms
weakness of shoulder abduction and elbow No wasting of the small muscles of the
flexion as expected and additionally suggests hand no involvement of C8/T1
weakness of supination consistent with Absence of cerebellar signs
brachioradialis involvement
Triceps reflex is pathologically brisk whereas State the most likely diagnosis
biceps and supinator reflexes are absent.
Inversion of reflexes is noted on the basis of these findings
Dermatomal sensory loss is noted in C57 Evidence of lower motor neuron weakness and
distribution. There is profound loss of sensory disturbance in a dermatomal distribution
vibration sense and proprioception is absent of C67 with inverted reflexes is consistent with a
pseudoathetosis (writhing movements) of diagnosis of cervical myelopathy. This is further
the fingers may be observed when the patent supported by the findings of spastic paraparesis
is asked to close their eyes and dorsal column deficit in the lower limbs.
The patient is likely suffering from
The strong candidate will examine the lower limbs spondylitis of the cervical spine as a
to confirm spastic paraparesis and sensory deficit: consequence of her rheumatoid arthritis.
Markedly increased tone with clasp-knife
spasticity Offer relevant differential
Pyramidal weakness of the lower limbs
bilaterally. No evidence of muscle wasting. diagnoses
Pathologically brisk reflexes with clonus at the Cervical spondylosis
ankle and knee. Extensor plantar response Tumours
Sensory deficit evident. Impairment Trauma
of vibration sense and sensory level. A common cause of cervical myelopathy in
Proprioception, pain and temperature sense younger patients is demyelination
are spared
Demonstrate the importance
Inversion of reflexes and the of clinical context suggest
mid-cervical reflex pattern relevant questions that would
Paradoxical contraction of triceps with elbow be taken in a patient history
extension when testing biceps reflex, and flexion
of the fingers when testing supinator jerk, occurs.
Confirm an insidious onset of weakness and
Biceps and supinator reflexes in of themselves are
sensory symptoms:
absent due to a lesion affecting their roots at C56, Assess involvement of bladder/sphincter
while the reflex arcs for triceps and finger jerks lie control, albeit rare
below the level of the lesion and are pathologically Neck pain possible
brisk. Together this is termed the mid-cervical reflex
pattern.
Case 48 Unilateral arm weakness neuralgic amyotrophy 89

Demonstrate an understanding of Always offer a management plan


the value of further investigation Consultation with a neurosurgeon for
CT myelography and/or MRI to assess the consideration of surgical intervention to reduce
stenosis of the spinal column and establish the impingement or stabilise the cervical spine.
underlying cause. Conservative management with supportive
collar, analgaesia and physiotherapy/
occupational therapy input.

Case 48: Unilateral arm weakness


neuralgic amyotrophy

Instruction to the candidate Offer relevant differential


A 28-year-old female teacher presents with a diagnoses
painful right shoulder and weakness in the arm Multiple radiculopathies
on that side evolving over a few days. Please Infiltrative radiculopathies/plexopathy in the
examine them appropriately and present your context of a carcinomatous mass
findings to the examiner. Mononeurits multiplex due to vasculitis
Hereditary neuropathy with liability to
Begin with a summary of pressure palsies (HNPP)
positive findings
Peri-scapular muscle wasting and occasional Demonstrate the importance
fasiculations visible affecting the deltoid of the of clinical context suggest
right shoulder. Weakness of some but not all
muscle groups supplied by C5 and C6 on right.
relevant questions that would
Diminished right bicep and supinator jerks but be taken in a patient history
preserved tricep jerks. In addition to enquiring about the exact nature
and timing of symptoms, screen for potential
Follow with a summary of causes and triggers:

relevant negative findings Any history of malignancy such as lung,


breast, or ovarian cancer, and lymphoma
There is an absence of objective sensory Any history of a preceding upper respiratory
loss
tract or gastrointestinal infection, which may
Examination is normal in other limbs
suggest a viral cause
The absence of Horners syndrome
There are no long tract signs or suggestion of
sphincter dysfunction Demonstrate an understanding
of the value of further
State the most likely diagnosis investigation
on the basis of these findings Blood tests:
This patient has a predominantly motor
syndrome with patchy involvement of muscles
ESR
supplied by C56 roots on the right. With pain
Serum electrophoresis
also being a feature the most likely diagnosis is
Autoimmune screen
brachial neuritis (neuralgic amyotrophy).
Anti-ganglioside antibodies
90 Chapter 2 Nervous system (station 3)

EMG and nerve conduction studies will Always offer a


demonstrate post-dorsal root ganglionic nerve
dysfunction. Rarely, a nerve biopsy may be management plan
considered in the context of a vasculitis Symptomatic relief with neuropathic
MRI of the cervical spine and/or the brachial modulating agents such as gabapentin,
plexus to rule out a compressive or infiltrative pregabalin, or amitriptyline. Steroids may be
process. of limited benefit. Treatment is directed to the
CSF analysis is indicated if a more underlying cause.
generalised immune process is suspected.

Case 49: Resting tremor Parkinsonism

Instruction to the candidate Bradykinesia


This 70-year-old man has been suffering with Slowness of initiation of voluntary movement.
a worsening tremor. Please examine his upper Affecting manual dexterity of the fingers.
limbs. Movements may become less coordinated,
with frequent hesitations or arrests as disease
progresses.
Begin with a summary of Additional findings:
positive findings Paucity of facial expression:
Examination of the upper limbs reveals tremor, Hypomimia
rigidity, and bradykinesia. Reduced blink rate
Stooped posture
Tremor Characteristic shuffling gait
Unilateral resting tremor, classically pill rolling Follow with a summary of relevant negative
in nature. findings
Distraction with repetitive movements of Tremor of the legs, lips, jaw, and tongue are
the contralateral arm may accentuate/augment also commonly seen with Parkinsonism whereas
a mild tremor or uncovers a latent one. This tremor of the head is rare. Tremor of the head
phenomenon is often observed when assessing would suggest essential tremor.
gait. Excitement, stress or anxiety may also
worsen the tremor.
The tremor is predominantly resting however
Signs of a Parkinsons-plus
action and postural tremor possible, albeit less syndromes
severe. Multisystem atrophy:
The latter is associated with a re-emergent Autonomic deficit
tremor a postural tremor that manifests after a Cerebellar ataxia
latency of several seconds with identical features Severe akinesia
to the resting tremor. The re-emergent tremor Progressive supranuclear palsy:
of Parkinsons differs from that of an essential Vertical gaze palsy
tremor in which no latency is observed. Pyramidal signs
Dementia
Rigidity
Cogwheel rigidity is prominent tremor State the most likely diagnosis
superimposed upon rigidity and/or lead
piping. Tonic resistance throughout passive on the basis of these findings
movement. Idiopathic Parkinsons disease.
Case 49 Resting tremor Parkinsonism 91

Offer relevant differential Demonstrate the importance


diagnoses of clinical context suggest
Secondary Parkinsonism relevant questions that would be
Iatrogenic anti-psychotics and anti-emetics taken in a patient history
Toxin mediated Assess the impact of symptoms on function. A
Metabolic history of falls is common and would suggest
Infection postural instability.
CVA vascular Parkinsonism due to multiple Screen for non-motor symptoms:
infarcts. Postulated to be associated with
the late onset of a Parkinson-like gait due to Cognitive dysfunction and dementia
infarcts of the basal ganglia Psychosis and hallucinations
Mood disorders including depression,
Neurodegenerative disorders anxiety
Sleep disturbances
Parkinsons disease may be mimicked by other Fatigue
neurodegenerative disorders, including:
Dementia with Lewy bodies early dementia Demonstrate an understanding of
with parkinsonism
Corticobasal degeneration most the value of further investigation
closely resembles Parkinsons disease Diagnosis relies upon clinical evaluation. It is
with asymmetrical signs however often generally accepted that bradykinesia, plus one
displays limb apraxia/dystonia/alien limb of the other two cardinal manifestations, tremor
phenomenon and rigidity, must be present in order to make
Multi-system atrophy autonomic symptoms the diagnosis of idiopathic Parkinsons disease.
generally more severe, symmetrical Response to treatment with dopamine based
symptoms, lack of tremor, and poor response treatment all but confirms the diagnosis.
to levodopa. Cognitive deficit unusual Investigations may, however, give supporting
Progressive supranuclear palsy vertical evidence. Such investigations may include:
supranuclear palsy with downward gaze
abnormalities and postural instability with
MRI brain for vascular changes
unexplained falls. Again, tremor is rare.
DaT scan for dopamine content in basal
ganglia
Cognitive deficit is severe
Wilsons disease
Huntingtons disease Always offer a management plan
Management should take place in the context
Features suggestive of an alternative of a multi-disciplinary team with involvement
of specialist nursing, physiotherapy and
diagnosis to idiopathic Parkinsons
occupational therapy. The mainstay of medical
disease management is dopamine replacement.
Poor response to levodopa Drugs used in Parkinsons disease:
Symmetrical presentation of tremor, LevoDopa is given with a peripheral dopa-
bradykinesia and rigidity decarboxylase antagonist to minimise both
Rapidly progressive course the systemic side effects of dopamine and
Absence of tremor its breakdown by hepatic metabolism.
Early falls with autonomic component Systemic side effects include nausea,
Early or rapidly progressing dementia vomiting and postural hypotension.
Supranuclear gaze The main central side effect is that of
Cerebellar signs dyskinesia. In the longer term, patient's
92 Chapter 2 Nervous system (station 3)

can develop a so-called on-off syndrome, rigidity freezing is a problem. Usually


where they are rigid without L-dopa, but administered by continuous infusion
dyskinetic with it COMT inhibitors (entacapone) reduce the
Ergot dopamine agonists such as metabolism of levodopa
bromocriptine are almost never used MAO inhibitors (selegiline) act to inhibit
in modern clinical practice. Non-ergot- MAO-A and MAO-B to reduce the breakdown
derived examples, including ropinerole and of dopamine and thus increase levels
pramipexole, are less likely to cause retro-
Neurosurgical interventions in Parkinson's
peritoneal or cardiac valve fibrosis
disease include deep brain stimulation and
Amantadine preferential effect on rigidity
stereotactic surgery, however these are very
Apomorphine acts as a dopamine
much second line therapies.
agonist. Used in Parkinson's where severe

Case 50: Intention tremor and problems with


co-ordination cerebellar syndrome

Instruction to the candidate Gait disturbance is observed. The patient


is ataxic, with a wide-based, unsteady gait.
Please examine this 30-year-old woman with a
Multiple falls or loss of balance towards the right
tremor.
side. Ataxia is exacerbated, and often subtle
signs revealed, by walking heel to toe.
Begin with a summary of
positive findings Localisation of cerebellar symptoms
Intention tremor and past-pointing of the right Unilateral cerebellar symptoms can be localised to
hand best elicited by performing nose-to-finger the ipsilateral cerebellar hemisphere, which control
testing. The patient should be made to fully finely co-ordinated movements, predominantly of
extend their arm to maximally elicit any tremor the upper limbs.
present, which worsens as it approaches the Symptoms resulting from dysfunction of the
target at the extremes of reach. A postural tremor midline cerebellar vermis include truncal ataxia
is also seen with the arms held against gravity unsteadiness when sitting down.
and rebound phenomenon is observed on The archicerebellum together with the vestibular
testing with the patients eyes closed reinforcing nuclei maintains equilibrium and balance serving
dysmetria. to coordinate eye, head, and neck movement.
Thereafter, examination of the cerebellar
system reveals:
Follow with a summary of
Nystagmus of eye movements towards the
right side (the fast phase travels towards the relevant negative findings
right) There is no resting tremor
Speech disturbance highlighted by an There are no contralateral signs
inability to articulate British Constitution.
Speech is described as scanning which Rombergs test is negative imbalance does
relates to hesitancy at the beginning of a not get worse when the patient closes their eyes.
word or syllable. Also interrupted, staccato, Signs of chronic liver disease (a full abdominal
and slurred examination is not necessary, but a scan for
Dysdiadochokinesia on the right side with easily visible signs such as clubbing, jaundice,
profound dysmetria leuconychia and spider naevi will elicit sufficient
Generalised hypotonia may be subtle evidence if such signs are present).
Case 50 Intention tremor and problems with co-ordination cerebellar syndrome 93

State the most likely diagnosis Demonstrate the importance


on the basis of these findings of clinical context suggest
This patient has signs consistent with cerebellar relevant questions that would
syndrome. The ipsilateral nature of the
symptoms suggests a lesion affecting the right
be taken in a patient history
hemisphere. Clarify onset and speed of progression
Screen for associated past medical history
guided by the differential diagnosis
Offer relevant differential
diagnoses Demonstrate an understanding
Lesion of the ipsilateral cerebellar of the value of further
hemispheres Investigation
Demyelination Principles of investigation include both
Ischaemia intracranial imaging and tests to provide
Space-occupying lesion evidence of the underlying cause.

Lesions affecting the vermis or Blood testing: full blood count for MCV
(alcohol). B12 and folate levels.
global cerebellar dysfunction Imaging: MRI is the gold standard imaging
Chronic alcohol abuse investigation for cerebellar dysfunction
Iatrogenic phenytoin, amiodarone,
lithium
Vitamin deficiency B12 and vitamin E Always offer a management plan
Infections varicella (cerebellitis), Lyme Given the significant functional impairment
disease, TB caused as a result of cerebellar dysfunction,
Paraneoplastic breast, ovarian and lung management would take place in a multi-
cancer, and lymphoma. Often bilateral and disciplinary setting with input from
rapidly progressive. physiotherapy, occupation therapy, neurology
Hypothyroidism specialists and alcohol liaison teams as
Degenerative multiple system atrophy appropriate.
(MSA) The exact management depends on the
Hereditary ataxias: underlying cause, but principles include
Freidreichs the most common removing any offending toxins. If the cause is
hereditary ataxia thought to be a medication, this can be changed
Ataxia telangiectasia to an alternative. If this is impossible, the dose
Ataxia with vitamin E deficiency can be adjusted to give the maximal balance
Spinocerebellar ataxias between treatment benefits and side effect.

Case 51: Central cord syndrome


syringomyelia

Instruction to the candidate Begin with a summary of


Please examine this 30-year-old woman referred positive findings
by her GP with a 2-year history of worsening Examination of the upper limbs reveals:
neck and shoulder pain and progressive
weakness of the upper limbs. Weakness of the hands and arms bilaterally
94 Chapter 2 Nervous system (station 3)

Muscle wasting of the small muscles of the Offer relevant differential


hand and arm
Reflexes are reduced or absent diagnoses
Loss of pain and temperature sensation in a Common causes of central cord lesions
cape-like distribution over the shoulders, neck include:
and upper thorax but preservation of light
touch, vibration sense and proprioception
Syringomyelia most commonly occurs in the
setting of an ArnoldChiari malformation
Additional signs: congenital herniation of cerebellar tissue
into the spinal cord, thus associated with
Weakness of legs (spastic paraparesis) cerebellar signs.
usually less pronounced than the upper Tumours of the central cord
limbs. Look for walking aids in the room to Cervical spondylosis central cord
support involvement without the need for syndrome may be the initial presentation of a
formal examination progressive cervical myelopathy.
Multiple small cuts and burns to the pulps of Trauma
the fingers/hands suggestive of sensory loss
There may be scoliosis
Lumbar syringomyelia
Follow with a summary of Lumbar syringomyelia can occur and is
relevant negative findings characterised by atrophy of both proximal
and distal leg muscles. A dissociated sensory
Damage to the sympathetic chain may produce loss occurs, affecting the lumbar and sacral
Horners syndrome. dermatomes. Lower limb reflexes are reduced
Extension of the lesion rostrally results in or absent and commonly sphincter function is
syringobulbia, signs of which include: impaired.
Dysarthria
Wasting of the tongue Demonstrate the importance
Facial sensory disturbance with sparing of
central areas (nose and mouth) of clinical context suggest
Nystagmus relevant questions that would be
taken in a patient history
State the most likely diagnosis Seek to establish the speed of onset which
on the basis of these findings is normally insidious/slowly progressive.
The patient displays signs of limb weakness Rapid development may be suggestive of
consistent with lower motor neuron pathology. syringobulbia. Painless burns or cuts may be
There is a dissociated sensory loss consistent the first symptom alerting the patient to an
with a lesion affecting the central cord. underlying problem. Deep aching of the neck
These signs are consistent with a diagnosis of and shoulders can also be an early sign.
syringomyelia. In considering the underlying cause, rule
out recent trauma, including hyper-extension
injuries of the cervical spine.
Neuroanatomy underlying the clinical
signs in central cord syndrome Demonstrate an understanding
A syrinx arises centrally, most commonly at the of the value of further
level of C5/6, and impinges the decussating
spinothalamic tracts while the lateral dorsal
investigation
columns and posterior tracts are spared Localisation of a spinal cord lesion can usually
(Figure 2.1). be achieved through clinical evaluation and
Syrinx extension into the anterior horns of the spinal this is useful in directing the level of the cord to
cord damages motor neurons, resulting in lower be imaged. MRI should be used to differentiate
motor neuron signs, with diffuse muscle atrophy between the possible causes and fully appreciate
beginning in the hands and progressing proximally the extent of cord involvement. Gadolinium-
to include the forearms and shoulder girdles. enhanced images are indicated if a tumor is
suspected.
Case 51 Central cord syndrome syringomyelia 95

Loss of pain and


temperature in a
cape-like distribution

Central cord syndrome

Area of cord damage

Figure 2.1 Central cord syndrome. Cape-like distribution of deficit with lower motor neuron weakness of
the hands, forearms and shoulder girdle with loss of pain and temperature (spinothalamic tracts) in the same
distribution, but sparing light touch, vibration and proprioception (dorsal columns and posterior tracts). Commonly
associated with pathology affecting the cervical spine.

Always offer a Medical treatment for syringomyelia is


limited however steroid use may be beneficial,
management plan particularly with cervical spondylosis. Muscle
The clinical course is often stable and chronic relaxants and analgaesia are a prudent
however surgical decompression with consideration for symptom control.
fenestration and/or shunt placement may be
indicated dependent upon severity/progression
of symptoms.
Case 52: BrownSquard (hemi-cord)
syndrome

Instruction to the candidate Interruption of lateral spinothalamic tract results


Please examine the lower limbs of this 36-year- in contralateral loss of pain and temperature
old woman who complains of weakness in her sensation which usually manifests clinically
left leg and changes in sensation. 23 segments below the level of the lesion
(Figure 2.2).
Most cases are incomplete and as such the
Begin with a summary of clinical presentation of BrownSquard syndrome
positive findings may range from mild to severe neurologic deficit.

Ipsilateral examination findings


Pyramidal loss of power in the left leg with
Offer relevant differential
brisk knee and ankle jerks and extensor plantar diagnoses
response. Sensory examination reveals loss of Cord compression trauma, disc herniation
fine touch, proprioception and vibration. or tumour
Demyelination
Contralateral findings Inflammatory SLE or sarcoidosis
No weakness on motor examination of the right Infection Lyme disease, TB, HIV, HTLV-1
leg but loss of pain and temperature is evident.
Additional signs: Demonstrate the importance
Inspect surgical scars over the spine to of clinical context suggest
suggest a history of trauma or surgery
Loss of ipsilateral autonomic function can
relevant questions that would
result in Horners syndrome be taken in a patient history
Rule out a history of trauma
Follow with a summary of Speeds of evolution inflammatory lesions
generally occur over a period of days as
relevant negative findings compared to neoplasm typically over a
Examination of the upper limbs is period of weeks to months
unremarkable Screen for a history of previous neurological
There are no brainstem signs events consistent with demyelination and
multiple sclerosis
State the most likely diagnosis
on the basis of these findings Demonstrate an understanding
The patient has signs consistent with a right of the value of further
hemi-cord (BrownSquard) syndrome most investigation
probably secondary to demyelination.
Traumatic injury or history of acute pain
demands urgent MRI spine to visualise
Neuroanatomy underlying the clinical abnormality/lesion at a level directed by the
signs in hemi-cord syndrome clinical findings.
Non-traumatic presentation will still require
Interruption of the lateral corticospinal tract imaging but consideration of other tests may
results in ipsilateral spastic paralysis below precede MRI:
the level of the lesion
Interruption of posterior column results in Blood tests (inflammatory/immunological
ipsilateral loss of tactile discrimination, as well screen)
as vibratory and position sensation, below the Serological testing (viral antibodies)
level of the lesion CSF examination (culture, oligoclonal bands)
Case 52 BrownSquard (hemi-cord) syndrome 97

Complete loss of
motor function,
deeptouch, vibration
and position on the
same side as cord
damage
Hemicord syndrome

On the opposite
side there
is loss of pain,
temperature and
light touch
Area of cord damage

Figure 2.2 Hemi-cord syndrome. Ipsilateral motor weakness with loss of vibration sense and proprioception, and
contralateral loss of pain and temperature loss.

Always offer a management plan for demyelinating lesion or anti-microbial


therapy as appropriate (antibiotics, anti-virals,
Cord compression will require urgent
anti-retrovirals). Symptom control may revolve
referral to neurosurgeons for consideration
around pain management neuropathic
of decompression. Thereafter treatment is
modulating agents and a multi-disciplinary
directed to the cause including corticosteroid
approach to rehabilitation for persisting deficits.
Case 53: Anterior cord syndrome

Instruction to the candidate the anterior spinal artery. However, any direct or
indirect injury to the spinal cord can cause the
Please examine the lower limbs of this 69-year-
syndrome, including:
old man returning to clinic for follow up.
Trauma with crush injury
Begin with a summary of Hyperflexion injury with bony instability,
which may result in quadriplegia due to the
positive findings vulnerability of the cervical spine
Examination findings: Compression from a hematoma or disc
herniation
Bilateral spastic paraparesis with brisk Ischemia secondary to compression of the
reflexes and extensor plantar response anterior spinal artery
Bilateral loss of pain and temperature sense
Bilateral retention of fine touch,
proprioception and vibratory sense Demonstrate the importance
of clinical context suggest
Follow with a summary of relevant questions that would be
relevant negative findings taken in a patient history
Sparing of proprioception and vibratory sense Screen for underlying causes. Focus upon risk
is crucial. profiling for stroke and a history of trauma.

State the most likely diagnosis Demonstrate an understanding of


on the basis of these findings the value of further investigation
Loss of motor function and a sensory deficit MRI is the optimal modality for imaging the
consistent with a lesion affecting the anterior spinal cord.
columns of the spinal cord, while retaining those
of the posterior column is consistent with an
anterior cord syndrome (Figure 2.3).
Always offer a
management plan
Offer relevant differential Optimisation of risk factors for secondary
prevention of stroke. Treatment may involve
diagnoses stabilisation or removal of any structure that
The most common cause of anterior cord exerts increased pressure on the anterior aspect
syndrome is infarction caused by occlusion of of spinal cord demanding neurosurgical input.
Case 53 Anterior cord syndrome 99

Complete loss of
motor function Anterior cord syndrome
with preservation
of touchsense,
vibration and
postion

Area of cord damage

Figure 2.3 Anterior cord syndrome. Upper motor weakness as evidenced by bilateral spastic paraparesis with
associated loss of pain and temperature sense (spinothalamic tracts), but sparing of fine touch, vibration and
proprioception (dorsal columns and posterior tracts).
Case 54: Spastic paraparesis

Instruction to the candidate Metabolic B12 deficiency, important due


to the potential for reversibility, copper
Please examine the lower limbs of this 30-year-
deficiency
old woman.
Vascular arterial occlusion
Infectious HIV, TB, HTLV-1-associated
Begin with a summary of myelopathy, neurosyphilis
positive findings Inflammatory sarcoidosis, SLE, Behets

Examination is remarkable for: Extrinsic


Increased tone and weakness bilaterally Neoplastic compression
affecting distal and proximal muscles in a Traumatic cord injury/spondylosis
pyramidal pattern, with flexors weaker then
extensors. No evidence of wasting
Pathologically brisk reflexes, extensor Demonstrate the importance
plantars and clonus at the knee and ankle of clinical context suggest
Sensory level at the xiphisternum affecting relevant questions that would be
dorsal columns (spinothalamic sparing)
taken in a patient history
Additional findings:
Establish onset of lower limb weakness. Onset
Assessment of gait confirms spasticity with
and speed of development can be useful in
circumduction and scissoring.
establish both cause and prognosis.
Screen for features of underlying disease,
Follow with a summary of particularly in relation to multiple sclerosis,
relevant negative findings previous weakness, sensory disturbance, and
visual deficit. Ask specifically in relation to
Where demyelination forms a working sphincter disturbance.
diagnosis:
Cerebellar signs Demonstrate an understanding of
Cranial nerve involvement
Evidence of bladder dysfunction the value of further investigation
Investigation of demyelination:
Evidence of B12 deficiency
MRI spine and brain to assess for
demyelination both at the suspected spinal
State the most likely diagnosis level and of the entire neuroaxis
on the basis of these findings Evoked potentials (visual and sensory to
There is evidence of transverse myelitis with confirm central conduction delay)
a cord lesion at the level of T6. The most likely CSF analysis for oligoclonal bands
cause is demyelination with a diagnosis of Approach to spastic paraparesis where
multiple sclerosis. differential is wide:
Blood tests ESR/CRP, vitamin B12, serology
Offer relevant differential HIV/HTLV-1/VDRL, auto-antibodies, culture
diagnoses Lumbar puncture CSF analysis, culture,
Cord lesions can be either intrinsic or extrinsic. acid-fast bacilli
Imaging MRI/CT-myelogram
Intrinsic Electromyelography/nerve conduction studies

Demyelination transverse myelitis is


most commonly due to multiple sclerosis, Always offer a management plan
also consider acute disseminated Treatment is directed towards the underlying
encephalomyelitis (ADEM) and Devics cause. High dose steroid in the acute setting may
disease be of therapeutic benefit.
Case 55: Proximal myopathy

Instruction to the candidate Offer relevant differential


Please examine the lower limbs of this 50-year- diagnoses
old woman who has presented complaining of Common myopathies causing proximal
increasing difficulty climbing the stairs in recent weakness include:
months.
Polymyositis or dermatomyositis
Begin with a summary of Thyrotoxicosis
Cushings syndrome
positive findings Alcoholic myopathy
On examination of the lower limbs there is
generalised muscle weakness bilaterally, most Demonstrate the importance
marked in the hip flexors (there is weakness on
straight-leg raising). The weakness is of a lower of clinical context suggest
motor neuron pattern, with normal or absent relevant questions that would be
knee and ankle reflexes. There is a flexor plantar
response.
taken in a patient history
On walking, the patient displays a waddling Screen for conditions producing fatigue
gait, with excessive hip swing. When asked to that may be misunderstood or reported as
rise from a seated position with arms crossed, weakness. Consider cardiac failure, chronic
the patient has insufficient strength to stand. respiratory conditions, or malignancy, amongst
The pattern is suggestive of proximal others, which may cause cachexia but not true
myopathy. Strong candidates will move swiftly weakness. Patients with true weakness usually
to an assessment of the upper limbs and of neck complain not of feeling weak but of an inability
flexion to confirm a similar pattern of weakness. to perform specific tasks. Thus, explore activities
Candidates should be alert to signs of in which weakness proves most problematic.
underlying causes on general inspection: Clinical patterns of weakness are often reflected
Centripedal obesity, purple striae, in patterns of disability:
characteristic facial appearance Cushingoid Proximal weakness impairs reaching upward,
Heliotrope rash and Gottrons papules. ascending stairs, or getting up from a sitting
Muscles may be painful dermatomyositis position.
Scleroderma/SLE
Irregular pulse of atrial fibrillation, profuse As opposed to:
sweating, in a thin patient thyrotoxicosis Distal weakness which impairs tasks such as
Ptosis and fatiguability myasthenia gravis stepping over a curb often due to foot drop.
Manual dexterity is impacted and simple
actions such as holding a cup, buttoning a
Follow with a summary of shirt or writing may prove problematic.
relevant negative findings If signs pointing to a likely underlying cause
Signs consistent with upper motor neuron are elicited, then the candidate can offer questions
pathology mixed upper and lower motor relating to a focused history for that condition.
neuron signs would be highly suggestive of Candidates should be alert to screening for
motor neuron disease. underlying malignancy in a patient with proximal
There is no associated sensory loss, which myopathy and heliotrope rash in whom the most
would favour peripheral nerve pathology. likely diagnosis is dermatomyositis.

State the most likely diagnosis Demonstrate an understanding


on the basis of these findings of the value of further
This patient displays clinical signs in the lower investigation
limbs of lower motor neuron dysfunction
suggestive of a proximal myopathy. Blood work utilised as a screen for common
causes of weakness would include:
102 Chapter 2 Nervous system (station 3)

Full blood count to exclude anaemia Testing of respiratory function should always
Electrolytes including bone profile and be considered in cases of muscle weakness due
magnesium to the potential for respiratory failure.
Thyroid function
ESR
Always offer a management plan
Creatinine kinase and LDH will be raised
as evidence of muscle breakdown and In the acute setting, where true muscle
supported further by a urine dip weakness is suspected it is important to
Serology for antibodies is warranted ANA, demonstrate an appreciation of the need
ANCA and ENAs to exclude diaphragmatic involvement that
would require ventilatory support. Thereafter,
EMG and nerve conduction studies are treatment would be directed at the cause as
useful to distinguish between neuropathy, determined by relevant investigations. In
neuromuscular junction, and myopathy. general terms it would be prudent to discuss
Biopsy to differentiate between the management in the context of a multi-
myopathies where the cause is not clear. disciplinary team and stress the importance
Imaging with MRI has a limited role where of occupational therapy and physiotherapist
the suspicion is one of lower motor neuron support in adapting to potential disability
dysfunction but can be useful to direct biopsy where full function may not be expected to be
to affected muscle as evidenced by atrophy and regained despite treatment.
inflammation.

Case 56: Glove and stocking distributions


sensory polyneuropathy

Instruction to the candidate Follow with a summary of


Please examine this 50-year-old patients lower relevant negative findings
limbs and perform further neurological testing No deficit in tone or power
as appropriate. Commonly sensory peripheral neuropathy
will affect the lower extremities before the
Begin with a summary of upper extremities. Thus, an absence of upper
limb deficit may correlate with early or mild
positive findings disease and warrant discussion as a relevant
There is sensory deficit in the distal aspect of the negative finding
lower limbs bilaterally to approximately 5 cm above No sign of Charcots joint destruction or
the medial malleoli bilaterally. This applies to light deformity of a joint would be indicative
touch, pin-prick, proprioception and vibration of both the chronicity and severity of any
sense. There is areflexia (sensory limb of reflex arc sensory deficit and should be commented
interrupted) and a Rombergs test is positive. upon as such
The strong candidate, despite only being Comment on the presence or absence of
asked to examine the lower limbs, will briefly signs consistent with an underlying cause
examine the upper limbs to assess whether there
is an associated glove sensory deficit to match
the stocking sensory deficit in the lower limbs. State the most likely diagnosis
Patients with long-standing sensory on the basis of these findings
polyneuropathies may develop tropic changes The patient has signs consistent with a
such as hair loss, nail changes, and cool diagnosis of a sensory polyneuropathy.
extremities.
Case 56 Glove and stocking distributions sensory polyneuropathy 103

Offer relevant differential Infection


diagnoses HIV
It is important to appreciate that this list is not
Leprosy
exhaustive. When approaching the differential
Lyme disease (Borreliosis)
diagnosis of a sensory polyneuropathy the
aim should be to impress upon the examiner Demonstrate the importance
an understanding of the diffuse nature of the
peripheral nerve damage. A range of systemic
of clinical context suggest
diseases commonly causes symmetrical, distal relevant questions that would be
neuropathy indiscriminately. taken in a patient history
Important causes of a sensory
The past medical history is important in
polyneuropathy include:
identifying the diseases and medications listed
above that are known causes.
Metabolic Screen for symptoms, which often precede
Diabetes or accompany peripheral neuropathy, including
Alcohol pain often burning or shooting pain worse at
Amyloidosis night. Consider resultant loss of function due to
Porphyria poor co-ordination.
Speed of onset can be useful in narrowing the
Iatrogenic (drugs) differential diagnosis, acute versus insidious.
Isoniazid
Amiodarone Demonstrate an understanding of
Metronidazole
Chemotherapy (vinca alkaloids) the value of further investigation
Confirm nerve damage with nerve conduction
Nutritional studies.
Screen for common and easily identifiable
Vitamin deficiency B1, B3, B6, B12 and E
causes, such as diabetes and alcohol misuse,
with specific testing directed by clinical
Connective tissue and inflammatory suspicion.
disorders
Rheumatoid arthritis Always offer a
SLE
Sarcoidosis management plan
Treatment is directed at the underlying cause.
Hereditary The strong candidate will realise that, by
advocating cessation of a neuropathy-inducing
Hereditary sensory neuropathy (HSN)
medication, an alternative treatment for the
Freidreichs ataxia
initial condition must be sought.
Case 57: Friedreichs ataxia

Instruction to the candidate Offer relevant differential


Please examine the lower limbs of this 30-year- diagnoses
old man. The differential diagnosis of pes cavus includes:

Begin with a summary of Friedreichs ataxia


Sensorimotor neuropathy (CharcotMarie
positive findings Tooth disease)
On inspection of the patients lower limbs, there Syringomyelia
is bilateral pes cavus high-arch deformity of Hereditary spastic paraplegia
the foot. Spinal cerebellar ataxia

Examination reveals: Demonstrate the importance


Reduced power in all muscle groups spastic of clinical context suggest
paraparesis
Reduced knee and ankle jerks bilaterally
relevant questions that would
Bilateral extensor plantar reflexes be taken in a patient history
Examination of sensation reveals decreased Friedreichs ataxia has autosomal recessive
vibration sense and loss of proprioception. Pain inheritance requiring a detailed family history.
and temperature sensation are retained. The predominant clinical manifestations
Additional findings include cerebellar signs of the disease are neurological deficit,
with ataxia of all four limbs. Often patients cardiomyopathy and diabetes. Thus, screen
become wheelchair bound but an ataxic gait for cardiac failure with questions pertaining to
would be expected. breathlessness, orthopnoea, and paroxysmal
Additional signs on inspection include nocturnal dyspnoea.
marked kypho-scoliosis of the spine. Involvement Assess functional impairment.
of the autonomic nervous system may result in
bladder dysfunction and presence of a long-term Demonstrate an understanding
catheter. The upper limbs may be notable for
wasting of the small muscles of the hands.
of the value of further
investigation
Follow with a summary of Investigations would centre on genetic testing to
provide a diagnosis.
relevant negative findings
Evidence of cardiac disease cardiomyopathy
Suspicion of intellectual impairment Summary of key points
Signs of diabetes Inherited condition
Autosomal recessive
State the most likely diagnosis Due to a triplet repeat expansion affecting
the frataxin gene, 9q13
on the basis of these findings Implication of mitochondrial iron
This patient has signs consistent with a accumulation
diagnosis of Friedreichs ataxia as evidenced by: Onset of symptoms in adolescence and
progressive
Pes cavus
Pyramidal weakness and extensor plantar
MRI typically reveals spinal cord and, in
reflexes
some cases, cerebellar atrophy.
Loss of deep tendon reflexes
Posterior column deficit
Cerebellar ataxia
Case 58 CharcotMarieTooth disease 105

Always offer a management plan a multi-disciplinary team with appropriate


genetic counselling. The prognosis is poor albeit
No disease modifying treatment exists. The
improved with late onset disease.
principles of management in Friedreichs ataxia
are those of supportive care in the context of

Case 58: CharcotMarieTooth disease


(hereditary sensorimotor neuropathy type 1)

Instruction to the candidate Essential tremor


Ataxia
Please examine the lower limbs of this 40-year-
old woman. No upper motor neuron pathology.

Begin with a summary of State the most likely diagnosis


positive findings on the basis of these findings
On inspection significant symmetrical wasting This patient has signs consistent with
in the distal lower limbs is evident, giving an a diagnosis of hereditary sensorimotor
appearance of inverted champagne bottles and neuropathy, also known as CharcotMarie
the patient has gross pes cavus. Tooth disease.
On examination:
Reduced power in distal muscle groups Offer relevant differential
Absent or markedly diminished deep tendon diagnoses
reflexes
Peripheral neuropathy, which if long standing
Sensory disturbance predominantly affecting
may produce pes cavus, however weakness
proprioception and vibration sense
would be expected to be less pronounced and
Given the inherent difficulty in mobilising,
sensory deficit more significant with a stocking
requests to assess gait may be declined by the
distribution.
examiner. However on walking, the patient
Offer a differential diagnosis of neuromuscular
may demonstrate a bilateral high-stepping
disease resulting in pes cavus:
gait, due to foot drop, and is clumsy due to
muscle weakness and sensory deficit HSMN
Additional findings:
Muscular dystrophy
Friedreichs ataxia
Wasting of the small muscles of the hands/ Syringomyelia
upper limbs Cerebral palsy
Kyphosis/scoliosis Hereditary spastic paraplegia

Follow with a summary of Demonstrate the importance


relevant negative findings of clinical context suggest
Clinical signs associated with a minority of relevant questions that would be
cases of HSMN type 1 should be looked for and
commented upon where absent:
taken in a patient history
Attempt to clarify the age of onset of symptoms,
Palpable peripheral nerves, which are commonly in childhood with difficulties
present in approximately a quarter of cases of walking, frequent falls and recurrent ankle
HSMN type 1 sprains due to weakness. Obtain a detailed
106 Chapter 2 Nervous system (station 3)

family history due to inherited nature of disease. Consideration may be given to genetic
Assess the impact upon the patients functional testing be aware of the association of different
capacity. mutations within the same gene giving rise
to various clinical phenotypes/spectrum of
disease.
Demonstrate an understanding
of the value of further Always offer a management plan
investigation Principles of management include supportive
Electromyography, nerve conduction studies care in the context of a multi-disciplinary team,
and nerve biopsy. Nerve conduction studies with appropriate genetic counselling.
(NCS) show severe slowing of conduction Early implementation of physiotherapy,
velocity in both the motor and sensory nerves. stretches and foot orthoses/splints can
Histology on biopsy confirms demyelinating improve function and significantly delay ankle
disease of peripheral nerves with characteristic contracture. Surgical intervention for foot
onion bulb formation. deformity may be required later in disease.

Case 59: Shuffling gait Parkinsons disease

Instruction to the candidate Festination may develop quickening of


steps, often into a run, to avoid falling
This 70-year-old man has been suffering with
recurrent falls and difficulty mobilising. Please Postural instability is tested clinically with
examine his gait and proceed as appropriate. the pull test, firmly pulling the patient by the
shoulders from behind. Where postural reflexes
are intact, balance is normally maintained with
Begin with a summary of minimal retropulsion. Patients with postural
positive findings instability however, are likely to fall or take
The patient has difficulty standing-up from a multiple steps backwards.
chair. Thereafter, a slow shuffling gait is evident Further examination should focus on the
with a narrow base. There is difficulty starting, upper limbs.
turning, and stopping.
Follow with a summary of
Tremor
relevant negative findings
Tremor of the upper limb may be exacerbated
Postural instability is a late sign in Parkinsons
on assessment of gait either as a product of
disease. As such, where falls or instability are
distraction or due to anxiety/stress.
a prominent feature, and occur early in the
context of parkinsonism, the patient is more
Rigidity likely to be suffering from a Parkinsonian
Decreased arm swing, and the typical stooped syndrome. Look for evidence of progressive
posture, result, at least in part, from rigidity. supranuclear palsy or multiple system atrophy.

Bradykinesia and postural instability State the most likely diagnosis


Effects of bradykinesia and postural instability
on gait include: on the basis of these findings
Parkinsons disease or Parkinsonism. For
Shuffling, short steps discussion in relation to differential diagnosis,
Dragging of the legs history, investigation and management see
Freezing, particularly when initiating a turn Resting tremor (p. 90).
Case 60: Ataxic gait

Instruction to the candidate Follow with a summary of


Please examine this 57-year-old patient relevant negative findings
complaining of dizziness when walking. Perform Rule out evidence of spasticity and upper motor
a gait assessment and proceed to relevant neuron weakness.
further examination before discussing the case A paucity of cerebellar symptoms on further
with the examiner. examination may be indicative of either sensory
or vestibular ataxia as a culprit for the changes
Begin with a summary of in gait.
positive findings
A broad based unsteady gait, which at times State the most likely diagnosis
lurches to one side. Efforts to compensate by on the basis of these findings
slowing the speed of movement and shortening This patient has an ataxic gait, with associated
the length of stride may be evident. signs of cerebellar disease.
Additional findings on subsequent
cerebellum examination may include:
Differential diagnosis, history,
Nystagmus
Slurring of speech investigation and management
Intention tremor See Intention tremor (p. 92)
Past-pointing (dysmetria)
Dysdiadochokinesia
Generalised hypotonia

Case 61: Unilateral high-stepping gait

Instruction to the candidate Focused examination of the right lower limb:


A 28-year-old man has been experiencing Weakness in dorsiflexion
difficulty with his gait having recently been Weakness of foot eversion
involved in an accident. He complains of Sensory signs such as loss of sensation over
limping and tripping. Assess his gait and the dorsal and lateral aspect of the foot in
examine the lower limbs. the distribution of the superficial peroneal
nerve
Begin with a summary of
positive findings Follow with a summary of
Examination of gait reveals: relevant negative findings
On walking, the right foot is lifted higher than
No signs of bilateral disease
the left
Weakness of foot inversion would suggest an
L5 radiculopathy and the sensory loss would
There is a right-sided foot drop
not correspond to that of the superficial
When the right foot is placed back on the
peroneal nerve
ground, there is an apparent slap of the sole
of the foot on the floor
Absence of any scars around the head of the
fibula
The left foot appears to move normally
108 Chapter 2 Nervous system (station 3)

State the most likely diagnosis which may in of itself result in compression.
on the basis of these findings
This patient has a unilateral high-stepping gait
Demonstrate an understanding
as a sign of common peroneal nerve palsy. of the value of further
investigation
Offer relevant differential Nerve conduction studies will provide
diagnoses supportive evidence of the diagnosis
demonstrating superficial peroneal nerve
The causes of common peroneal nerve palsy
conduction block.
include:
Other investigations may include blood
Trauma the course of the peroneal nerve, tests to rule out causes of mono-neuropathies
wrapping around the lateral side of the including HbA1c, ESR, and auto-antibodies
fibula head, leaves in vulnerable to injury. (ANCA, dsDNA).
Compression or transection caused by the
impact of a car in road traffic accidents is
common
Always offer a
Any cause of a mononeuropathy affecting management plan
this nerve diabetes mellitus, mononeuritis In the absence of underlying diseases such
multiplex commonly as the result of vasculitis as diabetes, management is supportive in
the context of a multi-disciplinary team.
Demonstrate the importance Many patients with a foot drop and high-
stepping gait will have no objective problems
of clinical context suggest with mobility and will require no active
relevant questions that would management at all.
be taken in a patient history
Rule out trauma and enquire as to recent
immobilisation in a below or above knee cast

Case 62: Spastic hemi-paretic gait

Instruction to the candidate Further examination


Examine the gait of this 75-year-old woman. Increased tone in the left lower limb with a
pyramidal distribution of weakness. Brisk
reflexes and an extensor plantar response.
Begin with a summary of The left upper limb is held in flexion at
positive findings the elbow.
Gait examination
The patient has an obvious unilateral gait Follow with a summary of
disturbance. The left leg is held in full knee relevant negative findings
extension with the foot pointed towards the Focus upon evidence of underlying aetiologies,
ground. The left leg, instead of being lifted principally atrial fibrillation with an irregular
upwards in the course of forward movement, is pulse and signs of anticoagulation, murmurs or
swung in an arc (circumducted) without much carotid bruits.
vertical movement. The right leg moves in a
grossly normal way.
Case 62 Spastic hemi-paretic gait 109

State the most likely diagnosis Demonstrate an understanding of


on the basis of these findings the value of further investigation
This patient has a hemiparetic gait in the Acute stroke demands urgent CT head with
context of signs of left-sided upper motor subsequent consideration of thrombolysis and
neuron weakness. The most likely cause for transfer to a stroke centre. Aside from imaging,
these signs would be a right middle cerebral the investigations required will depend on the
artery stroke which may be either ischaemic or likely underlying cause.
haemorrhagic in origin. Ischaemic stroke:
Blood tests to assess cardiovascular risk,
Offer relevant differential including HbA1c and lipid profile
diagnoses A 12-lead electrocardiogram to ensure to rule
out atrial fibrillation. Other, less common
Infarction can be considered ischaemic or pieces of useful information would include
haemorrhagic. Causative lesions leading to the presence of sustained ST elevation
ischaemia can be due to: post-MI which may be suggestive of left
Embolism arising from the heart, aorta or ventricular aneurysm formation which can
proximal vessels predispose to left ventricular mural clot
Atherosclerotic plaques affecting the formation
extracranial or intracranial arteries A trans-thoracic echocardiogram will identify
Carotid artery dissection (commonest cause any structural abnormalities of the heart
of ischaemic stroke in patients under 40 years and, in the setting of atrial fibrillation, give
of age) information about the size of the left atrium,
Hypercoagulability and diseases that cause which is important for consideration of
thrombosis of small vessels cardioversion
A 24-hour tape will potentially identify any
Other causes of upper motor neuron weakness underlying arrhythmias
include: Carotid Doppler will identify any carotid
Space occupying lesions stenosis. It is worthwhile reviewing the
Trauma indications for carotid endarterectomy in the
Degenerative disease such as motor neuron setting of cerebrovascular events
disease (should only be mentioned in the MRI with contrast should be considered where
complete absence of sensory signs) or signs of raised intracranial pressure suggest a
demyelinating conditions space-occupying lesion.

Demonstrate the importance


Always offer a
of clinical context suggest
management plan
relevant questions that would be
In the first instance recommend evaluation of
taken in a patient history potential for thrombolysis in acute stroke. Then
Speed of onset sudden (stroke) versus consider, secondary stroke prevention anti-
gradual (motor neuron disease) hypertensive, anti-platelets, anti-coagulation,
Assess functional impairment statin therapy, and attending to reversible causes
Screen for cardiovascular risk: of thromboembolism including anticoagulation
Smoking and/or rhythm control of atrial fibrillation or
Hypertension endarterectomy for critical carotid disease.
Hypercholesterolaemia Functional impairment is often significant,
Diabetes with rehabilitation and recovery dependent
upon a multi-disciplinary team.
Identify symptoms consistent with previous Consider treatment options for neoplastic
transient ischaemic attacks. disease, including palliative management.
Case 63: Waddling gait

Instruction to the candidate Muscles may be painful dermatomyositis


Please examine this 50-year-old womans gait
Scleroderma/SLE
having presented complaining of increasing
Irregular pulse of atrial fibrillation, profuse
sweating, in a thin patient thyrotoxicosis
difficulty climbing the stairs in recent months.
Ptosis and fatiguability myasthenia gravis
Follow with a summary of relevant negative
Begin with a summary of findings
positive findings Signs consistent with upper motor neuron
On walking, the patient appears unsteady and pathology mixed upper and lower motor
displays a waddling gait, with excessive hip neuron signs would be highly suggestive of
swing. When asked to rise from a seated position motor neuron disease.
with arms crossed, the patient has insufficient There is no associated sensory loss, which
strength to stand. Upper limb proximal would favour peripheral nerve pathology.
weakness may be suggested by an inability to
raise arms above the head. State the most likely diagnosis
Candidates should be alert to signs of
underlying causes on general inspection: on the basis of these findings
This patient has a waddling gait, consistent with
Centripedal obesity, purple striae,
proximal myopathy. For discussion in relation to
characteristic facial appearance Cushingoid
differential diagnosis, history, investigation and
Heliotrope rash and Gottrons papules.
management see Proximal myopathy (p. 101).

Case 64: Cauda equina syndrome

Instruction to the candidate Sensory impairment in saddle/peri-anal


(S25) distribution. Highlighting the
A 36-year-old man presents with a 2-week
importance of a comprehensive assessment
history of progressive bilateral sciatica, lower
of sensation that includes the posterior
limb weakness, and sensory loss in a peri-anal
aspects of the thighs which will allude to this
distribution. Please examine the lower limbs.
deficit
Decreased anal sphincter tone should be
Begin with a summary of suspected and an assessment offered
the examiner is likely to be content with
positive findings identification of sensory disturbance and
Examination findings include: expect only a discussion in this respect
Diminished straight leg raising (with or
without positive sciatic stretch test) Follow with a summary of
Bilateral (may be asymmetrical) global
relevant negative findings
lower motor neuron weakness (flexors and
extensors affected) including foot drop UMN signs pyramidal weakness, brisk reflexes,
Decreased or absent lower limb reflexes with and extensor plantars would suggest conus
flexor plantar responses medullaris syndrome.
Case 64 Cauda equina syndrome 111

Signs suggestive of underlying causes: Incontinence of urine or faeces represent


red flag symptoms and as such must be
Vesicular rash over the lumbar spine to
emphasised both in the history and by
suggest HSV-1/2 or varicella
making a clear assessment of sensation on
Erythema nodosum to suggest sarcoid, TB or
the back of the legs and perineal region with
Behets
or without examination of anal tone. If only
Inguinal lymphadenopathy to suggest
the distal region of the cauda is compressed,
lymphoproliferative or atypical infectious
incontinence may present in the absence of
disorder
lower limb weakness.

State the most likely diagnosis Demonstrate an understanding


on the basis of these findings of the value of further
This patient has localisation of all his physical
signs to the lower sacral roots with motor,
investigation
sensory and sphincter involvement. There are Confirming the lesion
no upper motor neuron signs. The abnormality Neurophysiology nerve conduction
is in the cauda equina only. velocities and denervation changes on EMG
can differentiate demyelinating from axonal
Offer relevant differential pathology.
diagnoses
Identifying a cause
Causes of cauda equina syndrome include:
Urgent MRI lumbo-sacral spine ensuring
Neoplastic carcinoma especially breast, that the field goes high enough to include the
ovarian, and lymphoproliferative disease entirety of the cauda equina
including myeloma Blood tests ESR, CRP, serum
Infection herpetic, varicella, CMV, Lyme, electrophoresis, blood cultures, TB Elispot,
TB, Brucellosis ACE, bacterial/viral serological testing,
Inflammatory sarcoid, vasculitides immune screen
including Behets and Wegeners CSF analysis white cell count, protein, ACE
Auto-immune acute inflammatory levels, oligoclonal studies, cytology, culture,
demyelinating polyneuropathy (AIDP) PCR testing, CSF serology
(GBS), chronic inflammatory demyelinating
polyneuropathy (CIDP)
Congenital neural tube defects including Always offer a
spina bifida management plan
Central disc bulge with compression of lower This is considered a neuro-surgical emergency,
sacral roots particularly with sphincter involvement.
Imaging should differentiate structural or
Demonstrate the importance compressive (disc/tumour) lesion requiring
urgent neurosurgery, from medical (infection/
of clinical context suggest inflammation) requiring anti-microbials/steroid
relevant questions that would be treatments.
taken in a patient history
Seek to elicit onset of pain in the buttocks radiating
to the lower limbs and/or difficulty walking.
Case 65: Absent ankle jerks with ex-
tensor plantar reflexes sub-acute
degeneration of the cord

Instruction to the candidate Offer relevant differential


This 40-year-old man presents having diagnoses
experienced difficulty with co-ordination, poor
balance and an unsteady gait. Please examine Differential diagnosis of B12
the lower limbs. deficiency
Pernicious anaemia
Begin with a summary of Conditions affecting the terminal ileum
commonly Crohns disease, also coeliacs
positive findings disease
Examination of the lower limbs reveals: Dietary deficiency or malnutrition often in
the context of alcohol dependence
Stocking distribution of sensory deficit with
impaired proprioception and vibration sense
Weakness with spasticity Differential diagnosis of absent
Absent ankle reflexes in the context of normal ankle jerks and extensor plantar
knee reflexes response
The plantar reflexes exhibit an extensor
This combination of signs is most commonly
response bilaterally
caused by dual pathology such as an upper
Additional findings associated with B12 motor neuron lesion causing extensor plantars
deficiency: (cervical myelopathy spondylosis of spine)
in combination with a peripheral neuropathy
On general inspection aged appearance, (diabetes) causing absent ankle jerks.
with greying of hair and a red swollen tongue Single disease entities that can cause this
Gait assessment as suggested by the combination of signs include:
vignette, cerebellar pathology with broad
based ataxic gait Romberg positive Subacute combined degeneration of the cord
Mild cognitive impairment due to vitamin B12 deficiency
Friedreichs ataxia
Follow with a summary of Quaternary syphilis (tabo paresis).
Candidates should note that it is better to
relevant negative findings refer to tabes or treponemal infection rather
The patient does not have pes cavus. In the than syphilis
absence of cerebellar signs, and the setting of Amyotrophic lateral sclerosis (ALS) motor
absent ankle jerks and extensor plantars, pes neurone disease
cavus would suggest a diagnosis of Friedreichs Certain forms of hereditary spastic
ataxia and rarer spinocerebellar ataxias. paraplegia
No evidence of aortic regurgitation, which Copper deficiency as a mimic of subacute
could support a diagnosis of treponemal combined degeneration of the cord
infection.
Demonstrate the importance of
State the most likely diagnosis clinical context suggest relevant
on the basis of these findings questions that would be taken in a
This patient has absent ankle jerks and extensor patient history
plantar reflexes in the context of peripheral Look to identify triggers such as alcohol
neuropathy, spasticity, and a suspicion of dependency or autoimmune disorders including
cerebellar pathology. Deficits at multiple levels pernicious anaemia and diabetes. Interestingly,
of the nervous system strongly suggest a unifying metformin can further compound vitamin B12
diagnosis of sub-acute degeneration of the cord deficiency.
as the result of vitamin B12 deficiency.
Case 65 Absent ankle jerks with extensor plantar reflexes sub-acute .... 113

Demonstrate an understanding Where the aetiology is unclear, in addition to


the above consider neurophysiological testing
of the value of further and cross-sectional imaging to assess for central
investigation lesions in view of spastic paraparesis.
Blood testing B12 levels. Folate levels are
routinely checked however folate deficiency, Always offer a
while mimicking the haematolgical
manifestations, do not cause neurological
management plan
deficit. Often cases with significant neurological Intramuscular B12. Initially weekly to restore
deficit are not associated with a megaloblastic levels to normal, with monthly regimes
anaemia, as would be expected. thereafter as maintenance. Treatment reverses
Schilling test where pernicious anaemia or halts progression of most, if not all, sequelae
suspected. of B12 deficiency.
Chapter 3

Respiratory system
(station 1)
Case 66: Pulmonary fibrosis

Instruction to the candidate Follow with a summary of


Please examine this 64-year-old man who has relevant negative findings
been complaining of exertional breathlessness. Important relevant negatives include:
When you have finished, please present your
findings to the examiner. The absence of breathlessness at rest
The absence of cyanosis
Begin with a summary of The absence of signs suggestive of an
underlying cause
positive findings
Positive findings on examination of the chest State the most likely diagnosis
will include:
on the basis of these findings
Bilateral decreased expansion This patient has signs consistent with a
Bibasal dullness to percussion diagnosis of pulmonary fibrosis.
Auscultation reveals fine, end-inspiratory
crepitations bibasally with increased vocal
fremitus Pulmonary fibrosis nomenclature
There may be signs of associated pulmonary
Pulmonary fibrosis has a number of alternative
hypertension raised JVP, parasternal heave
names: interstitial lung disease (ILD); diffuse
and loud P2 on auscultation
parenchymal lung disease (DPLD). In common
Note that findings may be restricted to one clinical practice these terms are used
hemithorax (e.g. in radiotherapy-related interchangeably and candidates should not worry
fibrosis), and might be either apical (e.g. about using either term. ILD is the nomenclature
ankylosing spondylitis, tuberculosis) or basal used in the BTS guidelines. The terms idiopathic
(e.g. fibrosing alveolitis). pulmonary fibrosis and cryptogenic fibrosing
See Table 3.1 for a summary of signs that may alveolitis specifically describe fibrosis without
be seen in a patient with pulmonary fibrosis, an identifiable underlying cause, and are not
including those that may point to an underlying alternative terms for ILD.
cause for the fibrotic process.

Table 3.1 Clinical signs that may be seen in pulmonary fibrosis


Type of sign Sign Potential underlying cause of fibrosis
General Finger clubbing
Bruising
(steroid use)
Hand Symmetrical deforming polyarthropathy Rheumatoid arthritis
Sclerodactyly Systemic sclerosis
Facial Microstomia, beaked nose Systemic sclerosis
Lupus pernio Sarcoidosis
Skin Grey discolouration Amiodarone
India ink tattoo External beam
Well demarcated erythema and telangiectasia radiotherapy
Posture Question-mark spine Ankylosing spondylitis
Case 66 Pulmonary fibrosis 117

Offer relevant differential chest radiograph and to a large extent confirmed


by demonstrating a restrictive pattern on
diagnoses spirometry. High resolution CT is diagnostic and
The differential diagnosis can be considered in allows classification based on pattern which is
relation to the clinical sign, that of crepitations, further supported by an array of laboratory tests,
and relating to the aetiology of interstitial lung biopsies, and lavage where indicated.
disease itself. Peripheral oxygen saturations are typically
The differential diagnosis of lung crepitations low, with hypoxia and hypocapnia due to
includes: tachypnoea on arterial blood gases.
Consolidation
Bronchiectasis Plain chest radiograph
Pulmonary oedema congestive cardiac Reticulonodular shadowing in the distribution
failure of the crepitations on clinical examination, with
a honeycombing pattern indicative of more
The causes of interstitial lung disease can be advanced disease. Remember also that the
classified by underlying cause: plain film may also demonstrate signs pointing
Idiopathic/cryptogenic towards an underlying secondary cause for the
Dusts: fibrosis, such as peri-hilar lymphadenopathy in
Organic: bird droppings, Aspergillus pulmonary sarcoidosis or pleural plaques from
fumigatus, Micropolyspora faeni asbestos exposure.
Inorganic: asbestos, coal dust, silica
Drugs: Spirometry and lung function tests
Common: nitrofurantoin These demonstrate a restrictive pattern, with
Anti-rheumatic: methotrexate a reduction in gas transfer factor. Note that,
Anti-cancer: bleomycin in addition to providing supporting evidence
Anti-arrhythmics: amiodarone towards a diagnosis of ILD, lung function tests
Rheumatological disease/systemic disease: also provide prognostic evidence. A reduction in
Rheumatoid arthritis FVC of 10% or more and/or a reduction in TLCO
Systemic sclerosis of 15% or more in the first 612 months from
Ankylosing spondylitis diagnosis indicates a higher risk of mortality.
Sarcoidosis 6 minute walk test:
Radiotherapy This is useful to quantify exercise and
Infection: tuberculosis functional capacity, and has adverse prognostic
implications in idiopathic pulmonary fibrosis
Demonstrate the importance if post-test peripheral oxygen saturations fall to
less than 88% on air.
of clinical context suggest
relevant questions that would be Laboratory blood tests
taken in a patient history This will include ANA, ANCA, ESR and
Relevant questions would include: rheumatoid factor as part of a vasculitic screen.
In addition, if clinically indicated based on the
Drug history to identify use of a medication history, serum IgG to organic dusts are useful
known to cause ILD in confirming a diagnosis of hypersensitivity
Past medical history pneumonitis (also known as extrinsic allergic
Occupation: may elicit exposure to organic alveolitis).
and inorganic dusts
Exercise tolerance and other functional High-resolution CT of the thorax
aspects
This will confirm the extent of fibrosis and will
also allow classification of the fibrosis according
Demonstrate an understanding to the radiological pattern.
of the value of further
Lung biopsy
investigation With the advent of high-resolution CT scanning,
Suspicion of fibrosis in a breathless or hypoxic histological evidence is not always necessary.
patient can be supported by findings on a plain
118 Chapter 3 Respiratory system (station 1)

Bronchoalveolar lavage (BAL) There is ongoing research into triple


therapy with steroids, azathioprine and
This is not necessary for diagnosis, but may be
N-acetylcysteine. Interim analysis from the
supportive and has particular utility in ruling
PANTHER (Prednisolone, Azathioprine and
out alternative diagnoses. BAL fluid composition
N-acetylcysteine: A Study That Evaluates
typically includes neutrophils, a poor prognostic
Response in IPF) trial revealed that triple
marker, and less commonly eosinophils.
therapy was associated with greater mortality,
The absence of lymphocytes is indicative of
more hospitalisations and more serious adverse
idiopathic rather than nonidiopathic pulmonary
events than the placebo. Therefore, the triple
fibrosis.
therapy arm of the trial was stopped. Note that
neither steroid monotherapy, nor a steroid and
Always offer a management plan azathioprine combination, are advocated in the
Important points: BTS guidelines.
Management of pulmonary fibrosis depends Oxygen therapy for symptomatic treatment of
upon the underlying cause. Many PACES significant hypoxaemia.
discussions will centre upon the management Lung transplantation is a possibility in
of idiopathic disease in particular. In the those <65 years old and without significant
absence of clinical signs pointing towards a comorbidity. The indications for this are:
secondary cause, it is perfectly reasonable TLCO <40% of predicted
to tell the examiner the management of Progressive decline in FVC by 15% or more
idiopathic pulmonary fibrosis includes where over 6 months
a reversible or removable secondary cause
has been identified, this should be treated or The BTS guidelines stress the importance of
removed where possible. Smoking cessation a multi-disciplinary approach to ILD and you
should be actively promoted. should in turn demonstrate to the examiner that
No treatment strategy has been shown you too promote this approach.
to improve outcomes in idiopathic disease.

Case 67: Consolidation

Instruction to the candidate breathing and increased vocal fremitus


A 56-year-old man presented during the acute
Additionally, the patient may have localise
pleural rubs (arising from pleural irritation)
medical take with a productive cough and fever.
and/or a small parapneumonic effusion
Please examiner his respiratory system and
present your findings to the examiner. Possible additional findings in the patient
with consolidation include a sputum pot by the
bedside, with purulent sputum and possibly
Begin with a summary of haemoptysis. Herpetic lesion on the lips may
positive findings be noted in the patient with streptococcal
The diagnosis is made based upon the results pneumonia.
of chest examination, with additional findings
providing further information. Follow with a summary of
Findings on chest examination, in this case
indicating consolidation at the right base, relevant negative findings
include: If absent, comment on the lack of:
Decreased expansion at the right base Signs of respiratory distress
Dullness to percussion at the right base Tar staining of the fingers (usually
On auscultation, right basal coarse unilaterally as smokers tend to hold their
inspiratory crepitations with bronchial cigarettes in the same hand each time)
Case 67 Consolidation 119

State the most likely diagnosis Oxygen saturations. These may be low
and in the absence of underlying chronic
on the basis of these findings obstructive airways disease, supplementary
This patient has signs consistent with a oxygen is given, aiming for saturations of
diagnosis of consolidation at the right lung base. 9498%
Laboratory tests: A raised white blood cell
count and acute phase markers (CRP).
Offer relevant differential Atypical organism serology and blood
diagnoses cultures. Legionella urinary antigen test,
Other causes of crepitations include congestive sputum microscopy, culture and sensitivities
cardiac failure, interstitial lung disease and Plain chest radiograph: Expected
bronchiectasis. to demonstrate a localised area of
The differential diagnosis of underlying opacification in a lobar distribution with air
causes for consolidation include an infective bronchograms. Complications visible on the
process (most commonly bacterial pneumonia), radiograph may include parapneumonic
pulmonary malignancy or pulmonary effusion or abscess formation
infarction. BTS guidelines advise that a chest radiograph
The differential diagnosis of common should be repeated at 6 weeks if there is
bacteria causing lung consolidation includes: persistence of clinical symptoms or signs
Streptococcus pneumoniae despite treatment, or in individuals who are at
Haemophilus influenzae higher risk of an underlying neoplastic process
Staphylococcus can cause cavitations (especially individuals over 50 years of age and
Moraxella in those with a significant smoking history).

Legionella and Chlamydia psittaci are the


most common causes of an atypical pneumonia, Always offer a management plan
with constitutional signs and symptoms A general summary of the management of a
disproportionate to the radiographic evidence of patient with pneumonia would include:
pulmonary involvement.
Oxygen therapy in accordance with the
BTS guideline for emergency oxygen use in
Demonstrate the importance adult patients, noting to the examiner an
of clinical context suggest awareness of safe oxygen administration in
patients with concomitant COPD
relevant questions that would Antibiotics. Specific antibiotic choice would
be taken in a patient history depend on local antibiotic guidelines is a
Appropriate questioning is aimed to elicit the perfectly valid statement, but the examiner
nature, duration and acute onset of the patients will expect you to have a good knowledge of
symptoms: the standard antibiotic choices according
to the causative organism and nature and
Cough, sputum, haemoptysis severity of illness (such as whether it is
Fever, rigors, general malaise community-acquired, hospital-acquired or
Breathlessness aspiration in aetiology
Effect on daily life and function Intravenous fluids for hypovolaemia,
thromboembolic prophylaxis with low
Screen for a history of recurrent pneumonia or
molecular heparin and TEDS, and adequate
conditions that predispose to recurrent and/
nutritional support should constitute part of
or severe infection. If elderly, enquire as to
the holistic in-patient care of pneumonia
compliance with annual vaccination.
Chest physiotherapy in individuals with
difficulty expectorating or with underlying
Demonstrate an understanding of comorbid chest pathology
the value of further investigation Ongoing monitoring includes aspects such as
intravenous to oral antibiotic conversion and
Appropriate investigations in a patient with vaccination (influenza, parainfluenza and
consolidation include: pneumococcus) of at-risk groups
120 Chapter 3 Respiratory system (station 1)

of an effusion with persistent high fever


Severity grading the CURB65 score and raised inflammatory markers should
The CURB65 score is a validated scoring system trigger a high clinical suspicion of empyema.
to help grade the severity of pneumonia and Prompt thoracocentesis and, if the diagnosis
therefore stratify higher mortality risk. One point is is confirmed (e.g by a sample pH of <7.2),
attributed to each of the following: complete drainage of the effusion. Prolonged
(up to 6 weeks) courses of antibiotics, and
Confusion: Generally applied to a AMTS of in some cases surgical drainage, may be
less than 8 required
Urea: Greater than 7 mmol/L Cavity formation. Several bacteria, such
Respiratory rate: 30 respirations per minute as Staphylococcus and Klebsiella have a
or greater tendency to form lung cavities
Blood pressure: Systolic <90 mmHg, diastolic Respiratory failure. The role of non-invasive
60 mmHg or less (and subsequently invasive) ventilation
Age 65 or over should be discussed in patients who develop
A CURB65 score of 2 suggests moderate severity, respiratory failure.
usually necessitating a short in-patient course of Disseminated sepsis and septic shock.
treatment. A score of 3 and above is managed as In a straightforward case such as
high severity. This system aids to risk stratification, consolidation, add to the standard management
used to identify a likely pathway of care, but should by bringing in concepts such as the Society
be applied in conjunction with and not replace of Critical Care Medicines Surviving Sepsis
clinical judgment. guidelines or the use of community acquired
Other features increasing the risk of mortality are: pneumonia care bundles.
Coexisting disease
Bilateral or multilobar involvement
PaO2 < 8 kPa or SaO2 < 92% Further reading
Lim WS, van der Eerden MM, Laing R, et al. Defining
community acquired pneumonia severity
The appropriate management of patients
on presentation to hospital: an international
with consolidation includes monitoring for
derivation and validation study. Thorax 2003; 58:
development of possible complications. These
377382.
include:
Lim WS, Baudouin SV, George RC, et al. BTS
Parapneumonic effusion and empyema guidelines for the management of community
(see Pleural effusion, p. 122). Exudative acquired pneumonia in adults: update 2009.
effusions predominate, and development Thorax 2009; 64:iii155.

Case 68: Bronchiectasis

Instruction to the candidate auscultation, positive findings include bilateral


widespread coarse inspiratory crepitations with
This 54-year-old man complains of a chronic
associated inspiratory clicks.
productive cough. Please examine his
The key clinical signs in bronchiectasis
respiratory system.
are: Clubbing, Crepitations, and Clicks. The
presence of a sputum pot containing copious
Begin with a summary of quantities of purulent sputum is a key clue
in differentiating bronchiectasis from other
positive findings respiratory cases.
The patient is cachectic. There is evidence
of clubbing in both the hands and feet. On
Case 68 Bronchiectasis 121

Follow with a summary of Mechanical bronchial obstruction, either


by a foreign body or by a mass (e.g.
relevant negative findings tumour or lymph node) causing external
Important relevant negative findings to compression
comment on if absent include; resting In association with autoimmune disorders
tachypnoea, cyanosis and kyphoscoliosis. such as rheumatoid arthritis and
inflammatory bowel disease
State the most likely diagnosis Recurrent aspiration in gastro-oesophageal
reflux disease, in alcoholics
on the basis of these findings Yellow nail syndrome
This patient has signs consistent with a Additionally, the distribution of
diagnosis of bronchiectasis. bronchiectasis can vary according to the
underlying cause (Table 3.2).
Offer relevant differential
diagnoses Demonstrate the importance
The differential diagnosis for crepitations of clinical context suggest
includes consolidation, congestive heart failure,
and interstitial lung disease.
relevant questions that would
be taken in a patient history
Bronchiectasis can be congential Important questions in a clinical history of a
or acquired patient with bronchiectasis would include:
Congenital causes for bronchiectasis include:
Identifying the underlying cause
Cystic fibrosis Is there a known diagnosis of relevant
Kartageners syndrome (primary ciliary conditions?
dyskinesia or immotile ciliary syndrome, Is there any history of childhood infections?
characterised by bronchiectasis, infertility Is there a history of recurrent chest
and situs inversus) infections? If there is, note the number
Immunodeficiency due to of infective exacerbations and courses of
hypogammaglobulinaemia (may be antibiotics per year.
congenital or acquired)
Congenital kyphoscoliosis
Establishing the patients current
Acquired causes of bronchiectasis can be symptoms and functional status
due to:
Chronic cough with large volume, purulent
Infection: sputum on a daily basis
Viral: measles Haemoptysis
Bacterial: pertussis, tuberculosis (the Fevers, night sweats and constitutional
commonest cause worldwide) symptoms
Fungal: aspergillosis, ABPA Weight loss or inability to gain weight

Table 3.2 Localisation of bronchiectasis according to underlying cause


Location Cause
Apical Tuberculosis
Lobar distribution Mechanical obstruction, e.g. tumour or foreign body
Proximal ABPA
ABPA, allergic bronchopulmonary aspergillosis.
122 Chapter 3 Respiratory system (station 1)

The functional status alludes to exercise Always offer a management plan


tolerance and effect on activities of daily living.
The management of individual cases depends
on the underlying cause of the bronchiectasis.
Demonstrate an understanding of General principles of management include:
the value of further investigation Physiotherapy and postural drainage on a
Imaging modalities include: daily basis is key
Saline nebulisers may aid sputum
Plain chest radiograph, which would show expectoration
ring shadows and tramlines Antibiotic therapy for acute exacerbations
High resolution CT to clearly delineate tailored to the patients sputum analysis,
the distribution of disease and potentially and long-term antibiotics in cases of three or
identify a secondary cause more exacerbations per annum
Sputum analysis is used to identify infective Inhaled bronchodilators to reduce airway
organisms and antibiotic susceptibility. Patients obstruction
with bronchiectasis are often colonised with Orally administered mucolytics may be
multiple bacteria, which may be resistant to considered in complex cases
multiple antibiotics as a consequence of multiple Other therapeutic options include control
acute infectious episodes, multiple antibiotic of localised disease to prevent complications
courses and nosocomial exposure to pathogens. including surgical removal of affected tissue, or
On formal spirometry a pattern consistent embolisation in the setting of severe or recurrent
with obstructive airway disease with limited haemoptysis.
reversibility would be expected. Complications of bronchiectasis include
Investigations to identify underlying cause pleural effusions, pneumothorax, cerebral
would include: abscesses and secondary amyloidosis. Cor
CF sweat test and genetic testing pulmonale can also occur in the long term.
TB elispot The importance of childhood vaccinations
Serum immunoglobulins, electrophoresis, to prevent diseases such as measles, known to
Aspergillus precipitins, rheumatoid factor cause bronchiectasis, should be emphasised to
Bronchoscopy to exclude bronchial demonstrate importance of disease prevention.
obstruction if suggested by history (weight
loss, haemoptysis, or history of foreign object Further reading
inhalation) or by imaging (HRCT suggesting a
Pasteur MC, Bilton D, Hill AT. British Thoracic Society
bronchial malignancy). Bronchoscopy may also
guideline for non-CF bronchiectasis. Thorax
be indicated in sputum negative cases in which
atypical mycobacterial infection is suspected. 2010; 65:i158.I

Case 69: Pleural effusion

Instruction to the candidate Decreased expansion on the right


A stony dull percussion note at the right base
This 75-year-old woman has been complaining Decreased breath sounds and decreased
of worsening exertional breathlessness. Please vocal fremitus at the right base on
examine her respiratory system and present auscultation
your findings to the examiner.
Associated findings to look for include scars.
Such scars could be from chest drain insertion,
Begin with a summary of biopsies (including lymph node incisions) and
positive findings surgical interventions. If the patient in the station
In a case of a right-sided, unilateral pleural has a chest drain in situ, the candidate should
effusion, the positive chest findings would be: comment on its contents and whether it appears
Case 69 Pleural effusion 123

to be swinging with respiration or bubbling, Where there is no evidence of an infective


often elicited with a short sharp cough. process, infarction, inflammation and malignancy
should be considered. Common malignancies
that invade the pleura and give rise to a pleural
Follow with a summary of effusion include primary lung adenocarcinoma,
relevant negative findings mesothelioma, and secondary metastases.
If the following signs are not present, comment
on their absence: Demonstrate the importance
Tachypnoea of clinical context suggest
Oxygen supplementation
relevant questions that would be
Cachexia
Cervical lymphadenopathy taken in a patient history
Important questions in a clinical history from a
State the most likely diagnosis patient with a pleural effusion would include:

on the basis of these findings Questions to elicit symptoms from the


This patient has signs consistent with a right- effusion including breathlessness on exertion
sided unilateral pleural effusion. and resultant functional deficit
Questions to identify symptoms suggestive
of an infective cause. Screen for fevers,
Offer relevant differential prurulent sputum, and a history of TB
diagnoses exposure/risk
Screen for malignancy. Is there weight loss,
The differential diagnosis can be considered a new or persistent cough, haemoptysis or
either as that of dullness to percussion, or of the voice change in the context of a significant
underlying cause of the pleural effusion. The smoking history? Be sure to establish history
differential diagnosis of dullness to percussion of previous malignancy
on chest examination is: Is there a recent history of cardiac surgery,
Lobar collapse which would raise the possibility of a
Consolidation chylothorax?
Pleural thickening
Raised right hemi-diaphragm Demonstrate an understanding
The differential diagnosis can also be of the value of further
considered in the context of the cause of
the effusion. Effusions can be classified as
investigation
transudates or exudates. When the protein Principles of investigation of a pleural effusion
content of an effusion exceeds 30 mg/dL it include imaging and fluid analysis.
is classed as an exudate, of which the most
common underlying causes are pneumonia Imaging modalities include
and malignancy. When the protein content is Plain chest radiograph to confirm
less than 25 mg/dL, the effusion is a transudate, the presence of an effusion. May also
of which the most common causes are cardiac demonstrate evidence of consolidation,
failure, liver failure and renal failure. When collapse, areas of cavitation or pleural
the protein content is between 25 and 30 mg/ plaques, all of which would give supporting
dL then Lights criteria should be applied to evidence towards an underlying diagnosis
determine its classification. Ultrasound to accurately assess the size of
Lights criteria state that an effusion is likely the effusion and comment on the degree
to be an exudate when: of loculation. It is important to obtain an
Fluid:serum protein ratio >0.5 ultrasound if drainage is being considered
Fluid:serum LDH ratio >0.6
Fluid LDH greater than 2/3 the normal serum Effusion analysis
LDH level Most patients will undergo a diagnostic aspiration
Transudative pleural effusions are commonly to allow fluid analysis, with the removal of
bilateral in nature. The likely underlying process approximately 2040 ml of fluid. Therapeutic
in a patient with a unilateral effusion is exudative. aspiration is only performed in the presence of
124 Chapter 3 Respiratory system (station 1)

cardiorespiratory compromise. Aspiration is not When fluid analysis suggests empyema,


performed in the context of a bilateral pleural the effusion should be drained to dryness.
effusion unless there are atypical features or there Antibiotics for parapneumonic effusions and
is a failure to respond to treatment. empyema should take note of culture results and
Fluid is sent for: follow local guidelines.
In the absence of underlying infection,
Biochemistry: protein, LDH,
the most likely cause of a unilateral pleural
Microscopy, culture and sensitivity
effusion is a neoplastic cause. This can lead
Cytology
to a discussion with the examiner of the
pH analysis. A pH of <7.2 is suggestive of an
principles of diagnosis and management of
empyema
lung malignancies. This involves confirming the
diagnosis through cross-sectional imaging and
Always offer a management plan tissue biopsy, followed by staging. Appropriate
The management can be considered in management is decided in a multi-disciplinary
relation to that of the effusion itself and of any setting, with treatment options including
underlying cause. chemotherapy, radiotherapy and surgical
excision, with curative or palliative intent.

Case 70: Collapse

Instruction to the candidate Collapse


This 55-year-old woman complains of chronic Collapse is essentially a differential for a chest
breathlessness. Please examine her respiratory with decreased breath sounds and dullness to
system. percussion. That said, if the exact constellation of
clinical signs does arise, the PACES candidate
Begin with a summary of would be expected to arrive upon the exact
diagnosis. Collapse can be differentiated from
positive findings consolidation by the presence of tracheal deviation
Positive findings on respiratory examination towards the affected side, and the absence of
include: bronchial breathing.

Tracheal deviation towards the affected side


Unilateral decreased expansion Offer relevant differential
Ipsilateral dullness to percussion diagnoses
Ispilateral diminished breath sounds
A sensible differential diagnosis of the elicited
Ipsilateral increased vocal fremitus
signs should be given. For example, the
differential diagnosis for dullness to percussion
Follow with a summary of includes consolidation and a pleural effusion.
relevant negative findings The percussion note in a pleural effusion would
be stony dull, rather than dull and as such the
Important negative findings include: findings are more suggestive of consolidation.
Tachypnoea Commonly lung collapse is found as a
Evidence of bronchogenic malignancy tar complication of pneumonia or as the result of a
staining, clubbing, cachexia mass lesion. Mass lesions include:
External compression enlarged lymph node
State the most likely diagnosis (tuberculosis, malignancy)
on the basis of these findings Within the airway wall malignant
endobronchial lesion
This patient has signs consistent with a Within the airway lumen inhaled foreign
diagnosis of lung collapse [stating the location]. body, mucus plugs
Case 71 Chronic obstructive pulmonary disease 125

Demonstrate the importance Imaging would include a plain chest


radiograph with or without cross-sectional
of clinical context suggest imaging. On a plain chest radiograph, expect
relevant questions that would be to see a degree of opacification of the affected
lung field with a degree of tracheal deviation
taken in a patient history towards the affected lung. The Sail sign refers
Important aspects in a clinical history of a patient to a specific radiological finding in left lower
with lung collapse include establishing current lobe collapse where the collapsed lobe causes
symptoms: the appearance of a double left heart border.
Cough, haemoptysis One of the borders is the opacity produced by
Sudden, acute onset dyspnoea the collapsed lobe. For persistent collapse, a
Concomitant infection with purulent CT of the chest would be indicated to assess
sputum, fevers, and general malaise for an endobronchial lesion causing the
collapse.
Establish a smoking history and screen for Bronchoscopy may be indicated. As with
features such as weight loss, cachexia, and CT, the possibility of an endobronchial lesion
haemoptysis suggestive of malignancy. leading to the collapse is the main reason for
Identifying any past medical history of TB or this investigation. It has the added advantage of
TB exposure, or a previous diagnosis of asthma the capability to take bronchial washings and
or ABPA. biopsies at the time of the investigation.

Demonstrate an understanding Always offer a


of the value of further management plan
investigation The management plan depends on underlying
Laboratory blood tests would include cause of the collapse. Present yourself as a safe
quantification of inflammatory markers to doctor by impressing upon the examiner the
assess for active infection it is common to find importance of cross-sectional imaging and
pneumonia distal to collapses. Malignancy may ultimately bronchoscopy in the patient with
be suggested by an anaemic picture, evident on collapse to ensure early diagnosis of possible
testing of haemoglobin. malignancies.

Case 71: Chronic obstructive


pulmonary disease

Instruction to the candidate sounds are decreased globally with expiratory


wheeze and a prolonged expiratory phase on
This 68-year-old man has presented with auscultation. Vocal fremitus is reduced.
breathlessness and a productive cough. Please
examine his respiratory system and present your
findings. Follow with a summary of
relevant negative findings
Begin with a summary of Important relevant negative findings to identify
positive findings include:
There are tar stains on the fingers. The chest is Signs of respiratory distress such as a resting
hyperexpanded with a reduced sterno-cricoid tachypnea, the use of accessory muscles
distance. Expansion is symmetrically decreased or the presence of tracheal deviation (this
with hyper-resonant percussion. Breath may be indicative of an associated tension
pneumothoeax albeit not expected to arise in
126 Chapter 3 Respiratory system (station 1)

the exam setting). Additionally, central and/ tar staining, the diagnosis of a1-antitrypsin
or peripheral cyanosis, or the use of oxygen deficiency must also be considered.
Signs of previous steroid use for recurrent
exacerbations, with fragile thin skin and
bruising
Demonstrate the importance
Lymphadenopathy, a marker of possible of clinical context suggest
infection or associated malignancy relevant questions that would be
Signs of associated pulmonary hypertension
raised JVP, parasternal heave and loud taken in a patient history
P2 on auscultation Screen for symptoms, including:
Signs of active infective exacerbation
Dyspnoea: quantify breathlessness by the
scattered crepitations, presence of a sputum
pot containing purulent sputum MRC dyspnoea scale and the impact upon
Peripheral oedema that may be a feature activities of daily living (i.e. a function
of cor pulmonale, hypoalbuminaemia assessment)
(malnourished patients with chronic Cough and sputum production
disease and increased protein catabolism in Wheeze
recurrent infective exacerbations), or reflect Weight loss
fluid retention due to steroid administration Establish frequency of exacerbations, the
number of A&E attendances and hospital
State the most likely diagnosis admissions in the past year, and the total
number of ITU admissions requiring ventilation.
on the basis of these findings Review current treatment and possible use of
This patient has signs consistent with a clinical long-term-oxygen-therapy.
diagnosis of chronic obstructive pulmonary Obtain a smoking history. Smoking and
disease. the effect of cessation on retarding the natural
progression of the disease should be discussed
both in the history and management sections of
Bronchitis and emphysema your presentation respectively. Notwithstanding
Bronchitis is a clinical diagnosis that may be made the fact that it is relevant to both sections, its
when a patient has a cough productive of sputum importance in the management of COPD cannot
on most days for a minimum of three consecutive be overemphasised.
months per year over a 2-year period.
Emphysema is a histological diagnosis Demonstrate an understanding
demonstrating dilatation of the airways distal to
the terminal bronchioles. This can however be of the value of further
suggested by imaging studies. investigation
The investigations required to assess a patient
Offer relevant differential with COPD include:
diagnoses Laboratory blood tests: full blood count is
The differential diagnosis for wheeze includes useful in demonstrating active infection and
both respiratory (COPD, asthma) and cardiac secondary polycythaemia. CRP will be raised
(heart failure) causes. Strictly speaking, wheeze in active infection
is an expiratory sound caused by intrathoracic A plain chest radiograph may show
airways obstruction. hyperexpanded lung fields with flattened
Note the importance of age in that the same hemi-diaphragms. Bullae may be identified,
set of clinical signs (with the exception of the with a consequent risk of secondary
tar staining) in a younger patient could be pneumothoraces. Coexisting chest
consistent with a diagnosis of asthma. Patients pathology should also be sought, and further
with severe, difficult-to-control asthma will investigated by CT
often have received prolonged courses of oral Lung function tests demonstrate an obstructive
steroids, and therefore may have a Cushingoid deficit with an FEV1 <80% predicted and a
appearance. In the less common scenario of a post-bronchodilator FEV1:FVC ratio <70%.
young patient with signs of COPD but without Note that unlike in asthma, in COPD the
obstruction to airflow is not fully reversible
Case 71 Chronic obstructive pulmonary disease 127

Arterial blood gases are required in the


evaluation for the need of long-term oxygen Long term oxygen therapy
therapy (LTOT), see below The importance of LTOT is in the prognostic benefit
it provides.
Always offer a management plan Criteria for use:
The patient must have stopped smoking
Whilst COPD is a common case in clinical
The patient must not have an acute
practice, in PACES the discussion of
exacerbation
management is a complex one with key points
PaO2 < 7.3kPa
that must be succinctly stated. A suggested
OR
approach might be to tell the examiner that: the
PaO2 7.38 and one of the following:
management of COPD is multi-disclipinary in
Secondary polycythaemia
nature and involves encouragement of smoking
Nocturnal hypoxaemia
cessation at every patient contact. Medical
Peripheral oedema
therapy is by a step-wise approach using;
Pulmonary hypertension
short- and long-acting bronchodilators; steroids
Oxygen therapy should be maintained for at least
(inhaled for maintenance therapy and oral
15 hours per day.
for acute exacerbations); theophylline-based
drugs are an adjunctive therapy once inhaled
It is worth commenting that theophyllines
therapies have been instituted; mucolytics
are particularly prone to drug interactions
can be considered for patients with chronic
(especially with some antibiotics, such as
productive cough. Long-term oxygen therapy
macrolides, that might also be prescribed in
is used in specific indications to provide a
the acute setting). Furthermore, arrhythmia is
prognostic benefit. Surgical options are less
particularly common given the pro-arrhythmic
common, but include bullectomy and lung
effects of certain medications (theophyllines,
volume reduction surgery.
b-agonists), electrolyte depletion and in
As per NICE guidelines, management of
the presence of sepsis. Therefore, cardiac
COPD should be patient-centred and adopt a
monitoring and careful management of
multi-disciplinary approach.
electrolytes and polypharmacy should not be
Actively encourage smoking cessation. A
underestimated.
key phrase from the NICE guidelines of 2010
Non-invasive ventilation (NIV) is used in the
states that all patients should be encouraged
acute setting where COPD is complicated by
to stop, and offered help to do so at every
type 2 respiratory failure. Criteria include:
opportunity. Candidates may examiners by
presenting themselves as a proactive doctor in Hypoxia PaO2 < 8
this respect. Hypercapnia PaCO2 > 6
Medical therapy in COPD involves a Acidosis pH 7.257.35
cumulative, step-wise progression of treatment
initiation: Many patients with COPD with a pH of less
than 7.25 will still be instituted on NIV. This
Inhaled therapy guided by spirometry: reflects the fact that many patients with COPD
Bronchodilators include short- and are considered unsuitable for endotracheal
long-acting b2-agonists and muscarinic intubation and invasive ventilation, and NIV is
antagonists. Inhaled steroids are considered thus an appropriate ceiling of acute treatment.
in individuals with FEV1 < 50% or with Pulmonary rehabilitation is an evidence-
persistent exacerbations. based programme that is recommended by
Oral therapy: Oral corticosteroids in acute NICE to reduce hospital admissions and
exacerbations and theophyllines for patients possibly mortality in patients with a recent
who require a step-up in treatment once COPD exacerbation. It entails a programme of
established on inhaled therapy. education and gym based regimes to increase
Pulmonary rehabilitation exercise tolerance.
Oxygen therapy: Both controlled oxygen
therapy in acute exacerbations to treat
hypoxia whilst avoiding hypercapnia; and Further reading
LTOT in appropriately selected patients, National Institute for Health and Clinical Excellence
which is the only treatment shown to (NICE), NHS Evidence. Chronic obstructive
improve prognosis in COPD pulmonary disease: evidence update February
2012. Manchester; NICE, 2012.
Case 72: Lobectomy

Instruction to the candidate State the most likely diagnosis


This 73-year-old man is undergoing follow-up on the basis of these findings
from the respiratory physicians. Please examine This patient has signs consistent with a right
his respiratory system and present your findings upper lobectomy.
to the examiner.

Offer relevant differential


Begin with a summary of
diagnoses
positive findings Table 3.3 outlines the differential diagnoses of
The patient is comfortable at rest. The most the indications (and contraindications) for a
obvious abnormality on inspection is a lobectomy.
well-healed right-sided thoracic scar with
underlying chest wall deformity. Chest signs
included equal expansion bilaterally, but with Demonstrate the importance
dull percussion and decreased breath sounds in of clinical context suggest
the right upper zone when auscultated anteriorly.
relevant questions that would
Lobectomy scars
be taken in a patient history
Questions eliciting the history of the operation
Unlike the sternotomy, laparotomy, or should focus upon eliciting the timing and
appendicectomy scars, all of which are difficult to indication for lobectomy. In the context of
miss, the lobectomy scar can be very subtle. It runs malignancy, establish the pre-operative and
parallel with the ribs and can therefore hide in the post-operative oncological history.
natural skin crease. You must therefore actively Screen for current symptoms, particularly
look for a lobectomy or pneumonectomy scar, in relation to shortness of breath and exercise
rather than simply expecting to see it on general tolerance.
inspection.

Demonstrate an understanding
Follow with a summary of of the value of further
relevant negative findings investigation
Important negative findings include:
It is reasonable to tell the examiner that,
Signs of respiratory compromise assuming the patient has been stable since
Signs of tar staining which would indicate his operation (especially if the procedure was
heavy tobacco smoking many years ago) then no routine investigations
Signs of abnormal wound healing such as a are warranted. Take care to consider what the
relatively wide or disorganised scar, or keloid candidate information card said; if it suggests
scarring and acute presentation then give a reasonable
account of relevant investigations.

Table 3.3 Lobectomy: indications and contraindications


Indications Contraindications
Lung malignancy Poor lung function
Solitary pulmonary nodule Operative risk
Bronchiectasis: either localised disease or massive
pulmonary haemorrhage unresolved by arterial
embolisation
Pulmonary tuberculosis
Case 73 Pneumonectomy 129

If asked, a plain chest radiograph will show Always offer a management plan
absence of the resected lobe with volume loss,
Given that the definitive management, the
altered rib anatomy and tracheal shift towards
lobectomy, has already happened, it is best to
the resection side.
use the management part of the presentation
as an opportunity to discuss with the
examiner the indications, complications and
contraindications of thoracic surgery.

Case 73: Pneumonectomy

Instruction to the candidate Pneumonectomy


Please examine this patients respiratory system In a pneumonectomy, the whole lung is taken out,
and present your findings. rather than a single lobe. While the indications
remain broadly the same, given the extent of lung
Begin with a summary of involvement, they and the complications tend
to be much more severe.
positive findings
The following positive findings assume a
right-sided pneumonectomy: Offer relevant differential
Inspection: a thoracotomy scar, running from
diagnoses
x to y (describe exactly the anatomical site of Give a differential for underlying respiratory
the scar). There is deformity of the right side conditions leading to the pneumonectomy, with
of the thoracic cage lung malignancy at the top of the list. While the
Palapation: there is decreased expansion on indications for pneumectomy versus lobectomy
the right, with deviation of the trachea to the remain broadly the same, the symptoms and
right complications tend to be more severe given the
Percussion: dullness to percussion on the right extent of lung involvement.
Ausculatation: decreased breath sounds
throughout the whole right lung field, apart Demonstrate the importance
from in the apex where there is bronchial
breathing (this corresponds with air flow of clinical context suggest
through the deviated trachea). Vocal fremitus relevant questions that would
is decreased on the right
be taken in a patient history
As with the lobectomy case, history should
Follow with a summary of focus on both the initial symptoms and
relevant negative findings diagnosis that led to the initial surgery, and also
Relevant negative findings include: on the patients current symptoms, questions,
and concerns about their condition.
Signs of respiratory compromise
Signs of tobacco use
Signs of abnormal wound healing Demonstrate an understanding
of the value of further
State the most likely diagnosis investigation
on the basis of these findings Again, the investigations would be the same as
with the lobectomy case:
This patient has signs consistent with a right-
sided pneumonectomy. Plain chest radiograph
130 Chapter 3 Respiratory system (station 1)

Spirometry would demonstrate a much larger Always offer a management plan


deficit in lung function in keeping with the
As with the previous case (Lobectomy, p. 128),
much larger portion of lung removed
given that the definitive management
You would also mention to the examiner that has already happened, it is best to use
you might also perform relevant investigations the management part of the presentation
for the underlying respiratory pathology, as an opportunity to discuss with the
especially if it is one that could also affect the examiner the indications, complications and
contralateral lung. contraindications of thoracic surgery.

Case 74: Lung transplantation

Instruction to the candidate The differential diagnosis for indications for


lung transplantations includes:
This 50-year-old woman has previously had a
cardiothoracic intervention. Please examine her Diffuse parenchymal lung disease
respiratory system and present your findings. Cystic fibrosis
Bronchiectasis
Begin with a summary of a1-antitrypsin deficiency (the strong
candidate will examine for a liver edge in a
positive findings young patient)
Positive findings can be divided into the Pulmonary vascular disease idiopathic
following three categories: pulmonary hypertension
Rare diseases Langerhans cell
Signs of the lung transplantation midline granulomatosis
sternotomy
Signs of an underlying respiratory condition.
Note that these may be completely absent Demonstrate the importance
Signs that indicate immunosuppressant use of clinical context suggest
relevant questions that would be
Follow with a summary of taken in a patient history
relevant negative findings Tell the examiner that you would ask the patient
Important relevant negative findings include: about the underlying respiratory disease that led
to the transplant, any complications following
Signs of respiratory compromise
the transplant, and any current symptoms.
Tar staining to indicate tobacco use Note
that current smoking is a contraindication to
lung transplantation Demonstrate an understanding
of the value of further
State the most likely diagnosis investigation
on the basis of these findings In the post-transplant patient, routine
This patient has signs consistent with previous investigations may include:
lung transplantation. Laboratory blood tests inflammatory
markers looking for active infection and drug
Offer relevant differential levels of relevant immunosuppressants
diagnoses Spirometry looking for obstructive deficit
which may suggest bronchiolitis obliterans
The differential diagnosis for the operative scars syndrome (see below).
includes a joint heart and lung transplant.
Case 75 Plethoric face and dilated chest veins superior vena cava obstruction 131

Always offer a management plan Concommitant unresolved malignant disease


A management discussion may focus on indication
Active pulmonary infection
for transplantation and patient selection. NICE Be prepared to discuss the complications of
states that lung transplants are performed in lung transplantation:
patients with non-malignant pulmonary disease
that is unresponsive or minimally responsive to
Acute rejection
treatment and who have a life expectancy of less
Complications from immunosuppression
than 1 year. The underlying causes include cystic
Bronchiolitis obliterans syndrome (BOS) is
a form of chronic rejection which involves
fibrosis, severe pulmonary fibrosis, pulmonary
a progressive obstructive lung defect. It is a
hypertension and obliterative bronchiolitis.
process that affects most lung transplants
Contraindications to lung transplantation
include:
Further reading
From the history National Institute for Health and Care Excellence
Blood-borne viral infection (hepatitis B or C, (NICE). Living donor lung transplantation
HIV) for end-stage lung disease, IPG170. London;
Current tobacco use NICE, 2006.
Unsuitable psychological profile Todd JL, Palmer SM. Bronchiolitis obliterans
syndrome: the final frontier for lung
From investigations transplantation. Chest 2011; 140:502508.
Organ dysfunction other than respiratory
system

Case 75: Plethoric face and dilated chest


veins superior vena cava obstruction

Instruction to the candidate Remember Pembertons sign: The


development of facial flushing, disteneded head
This 45-year-old man with a significant past
and neck veins, inspiratory stridor and raised
medical history has been referred to the acute
JVP on raising both of the patients arms above
medical team from oncology clinic. Please
their head.
examine his respiratory system and comment on
your findings.
Follow with a summary of
Begin with a summary of relevant negative findings
positive findings Things to comment on if absent:
On general inspection the patient is plethoric, Signs of respiratory distress: Such signs
with rubor of the face, neck and upper chest can be due either to an underlying lung
(Figure 3.1). The face and arms are oedematous. malignancy or as a direct consequence of the
On closer inspection of the neck, the patients obstruction
neck veins are dilated, with no visible waveform Signs suggestive of an underlying primary
variation in the raised jugular venous pressure. oncological diagnosis
The face is oedematous and the eyes display Cervical or axillary lymphadenopathy
chemosis. Evidence of tobacco use (tar staining of the
fingers)
132 Chapter 3 Respiratory system (station 1)

State the most likely diagnosis compression of the SVC. If acute, can suggest
acute thrombosis within the SVC
on the basis of these findings Do the symptoms get worse on lying down or
This patient has signs consistent with a stooping?
diagnosis of superior vena cava obstruction, Is the patient experiencing headaches?
likely secondary to an underlying neoplastic
process. Questions relating to the underlying
diagnosis
Offer relevant differential Symptoms of lung cancers (cough,
diagnoses haemoptysis) and general symptoms of
malignancy (weight loss, malaise, anaemia
The differential diagnosis for prominent neck
symptoms)
veins and breathlessness includes congestive
cardiac failure and cardiac tamponade.
Given that lung cancers are the most
common causes of SVC obstruction, an exact
See Table 3.4 for secondary causes of SVCO.
and thorough smoking history is vital
Does the patient already have a primary
Demonstrate the importance oncological diagnosis or is this the first
of clinical context suggest presentation of a new cancer? If they
already have a diagnosis, what is it, what
relevant questions that would be investigations and management have they
taken in a patient history had, and when are they next due to see an
oncologist
Questions relating to the symptoms Any recent central line insertion?
of SVC obstruction Any history of non-malignant diseases
Onset of symptoms. If chronic, suggests that known to cause SVC obstruction, e.g.
the underlying process may be of gradual Bechets? Any use of medications known to
cause SVC obstruction?

Figure 3.1 Superior vena cava


obstruction (SVCO). Dilated veins
can be seen over the praecordium
and upper arms and the jugular
veins are prominent. A plethoric
appearance of the neck, although
subtle, is evident with a more
pronounced finding expected in
the face. Oedema of the face, neck
(collar of Stokes) and arms may also
be seen.

Table 3.4 Secondary causes of superior vena cava obstruction


Malignant (~90% of causes) Non-malignant
Lung cancer (non-small cell most common, followed by Indwelling central venous catheter
small cell carcinoma) Infection, including tuberculosis
Lymphoma Granulomata
Mediastinal fibrosis
Case 76 Horners syndrome: Pancoast tumour 133

Demonstrate an understanding attention to the patients airway, breathing and


circulation. Initial therapy usually involves
of the value of further the use of corticosteroids, most commonly
investigation dexamethasone. Note though, that in cases of
A plain chest radiograph may show widening possible high-grade lymphoma, steroids cause
of the mediastinum. Any causative mass a decrease in the histological yield and carry
compressing the SVC, may be visible on the the risk of precipitating tumour lysis syndrome.
radiograph as a ring lesion. A unilateral pleural Other treatment options include SVC stent
effusion is also another indicator of underlying insertion, chemotherapy, and radiotherapy
lung malignancy.
A CT of the chest with contrast will give Discussing malignancy
information on both:
Traditionally, one of the biggest concerns
The SVC, its patency, and whether the medical students have as they approach clinical
obstruction is mechanical from external examinations is how to discuss possible cancer
compression, or from thrombus within the with patients without actually using the c-word
vein (or both) itself. Despite 23 years of clinical practice, this fear
Any causative mass, whether a lung is often prevalent in PACES candidates also. It is
malignancy, or pathologically enlarged important to remember that the patient has been
lymph nodes fully prepared for the scenario by the examiners
Further investigations may include diagnostic and is unlikely to flinch at the mention of cancer.
investigations aimed at obtaining tissue samples That said most candidates will choose to use
of suspected primary malignancies. slightly more technical terms such as malignant
process or neoplastic cause. Either way, one
shouldnt worry about it, because neither the
Always offer a management plan patient nor the examiner will.
Patients with SVC obstruction should be referred
to an oncologist for urgent investigation and
MDT driven management.
Further reading
The good candidate will remind the Kanada DJ, Jung RC, Ishihara S. Superior vena cava
examiner that SVC obstruction is an syndrome due to a retained central venous
oncological emergency, with immediate pressure catheter. Chest 1979; 75:734735.

Case 76: Horners syndrome:


Pancoasts tumour

Instruction to the candidate bronchial breathing in that area. On further


examination, the patient is noted to have ptosis
Please examine the respiratory system of this and miosis of the left eye. There is also noted
82-year-old man who has presented with weight to be wasting of the intrinsic muscles of the left
loss and haemoptysis. hand.
Other possible positive findings include:
Begin with a summary of Cachexia
positive findings Cervical lymphadenopathy
On respiratory examination, there is a localised More complex neurological signs in the
area of consolidation in the left upper lobe, as ipsilateral upper limb due to brachial plexus
demonstrated by dullness to percussion and involvement
134 Chapter 3 Respiratory system (station 1)

Follow with a summary of Pain and weakness of the hands


Hemi-facial anhidrosis
relevant negative findings
Important relevant negative findings include: In addition, tell the examiner you would elicit
a history consistent with risk factors for lung
Respiratory compromise malignancy, namely a strong smoking history.
Other cranial nerve pathology
Demonstrate an understanding
State the most likely diagnosis of the value of further
on the basis of these findings investigation
This patient has signs consistent with left upper
Principles of investigation would include
lobe consolidation in the context of Horners
radiographic confirmation of a tumour,
syndrome suggestive of a Pancoasts tumour.
followed by tissue sampling and histological
confirmation of tumour type. In addition,
Pancoasts tumour radiographic staging will identify any tumour
metastasis.
Pancoasts tumour is an apical lung tumour Specific investigations would include:
which, in addition to the local lung disease and
local signs/symptoms directly attributable to the Sputum cytology for malignant cells
tumour, also causes non-respiratory symptoms, Plain chest radiography to identify a ring
namely: lesion in the apex corresponding to the
Hand signs: wasting of the intrinsic muscles palpable mass on examination
of the hands Spirometry to assess further fitness for
Eye signs: Horners syndrome with ptosis, surgery
miosis, anhidrosis, enophthalmos CT chest, abdomen and pelvis. The chest
The very reason that hand and eye signs are portion of this scan will give further
involved in Pancoasts makes it an ideal case, information on the tumour itself, local
not just for the respiratory station, but also for spread, and any metastatic mediastinal
neurology or station five. lymphadenopathy. The abdominal and
pelvic slices will help to stage the patient by
identifying any distant metastases
Offer relevant differential Histological sampling in a peripheral lesion
diagnoses such as this would likely be by a CT-guided
It would be appropriate to give a differential biopsy
diagnosis of consolidation, adjusting your list to
start with causes of apical consolidation: Always offer a management plan
Pulmonary tuberculosis Specific management strategies depend on a
Other causes of bacterial consolidation range of factors, including:
Histological subtypes. Small cell lung cancer,
Demonstrate the importance due to its high cell turnover rate, is highly
susceptible to chemotherapy
of clinical context suggest The patients performance status
relevant questions that would be Lung function testing
taken in a patient history The localisation of tumour and the tumour
staging
Appropriate questions would include those
aimed at eliciting symptoms consistent with the All patients are managed by the local lung
clinical signs: cancer multi-disciplinary team to decide on
appropriate interventions. There may also be
Progressive weight loss, chronic cough, further input by both Macmillan nurses and
haemoptysis palliative care teams.
Case 77: Cushingoid appearance

Instruction to the candidate differential of underlying respiratory diseases


which warrant steroid therapy and are therefore
This 30-year-old woman has experienced weight
prone to iatrogenic Cushings, such as:
gain over the past 4 months. Please examine her
respiratory system. Asthma
COPD
Begin with a summary of Fibrosis
Lung transplantation
positive findings
At that stage, you may want to comment on
Positive findings can be divided into signs the difference between Cushings syndrome and
suggestive of a diagnosis of Cushings syndrome Cushings disease and the individual causes.
and signs suggestive of an underlying diagnosis
that requires long-term steroid therapy.
Signs pointing toward a diagnosis of Demonstrate the importance
Cushings syndrome include: of clinical context suggest
Cushingoid facies relevant questions that would
Centripedal obesity
be taken in a patient history
Intrascapular fat pad
Abdominal striae Screen past medical history and medication,
Easy bruising current and historical.

Signs pointing towards an underlying


respiratory diagnosis necessitating long-term Demonstrate an understanding
steroid therapy include: of the value of further
Barrel chest with wheeze on auscultation (in investigation
the young patient, think of difficult-to-control A two-pronged investigative approach would
asthma, whereas in the older patient be include, investigations regarding the underlying
mindful of COPD) respiratory diagnosis and those to confirm
If the patient has fine, end-inspiratory Cushings syndrome and iatrogenic steroid
crepitations, think of fibrosis administration as the cause.
Scars, such as those related to a lung transplant
A plain chest radiograph would show hyper-
expanded lung fields in COPD and difficult-
Follow with a summary of to-control asthma, or reticulonodular
relevant negative findings shadowing if the underlying disease process
Important relevant negative findings include is pulmonary fibrosis
signs of respiratory distress and signs of chronic Peak expiratory flow rate
tobacco use.
Always offer a
State the most likely diagnosis management plan
on the basis of these findings Assuming confirmation of iatrogenic steroid
This patient has signs consistent with a administration as a cause of the Cushings, the
diagnosis of Cushings syndrome, which is likely management discussion would centre on trying
iatrogenic and secondary to their underlying to reduce the need for steroid through use of
respiratory disease. steroid-sparing agents. Strong candidates will
also discuss management of cardiovascular
risk including screening for diabetes, good
Offer relevant differential glycaemic control in those with steroid-induced
diagnoses diabetes, addressing hypertension and lipid
management.
In this case, it is sensible to not give a differential
for Cushings syndrome, but rather to give a
Case 78: Abnormal nails with pleural
effusions and lymphadenopathy yellow
nail syndrome

Instruction to the candidate Emphasising the rarity of the condition at


the outset is a sensible approach. As a general
This patient, presenting with shortness of breath,
rule, any mention of rare diseases should be put
has been referred to the respiratory clinic for
in context and be the subsequent differential
assessment. Please examine her respiratory
diagnosis should include a list of less likely, but
system and comment on your findings.
more common diseases.

Begin with a summary of Offer relevant differential


positive findings diagnoses
On examination of this middle-aged person,
Yellow nail syndrome is essentially a spot-
positive findings include:
diagnosis, with no true differential diagnoses.
On general inspection there is swelling of the Therefore, various alternative differential
lower limbs diagnoses can be offered, such as:
On inspection of the hands, there is The differential diagnosis of a pleural
discolouration and deformation of the nails effusion:
of the hand and feet, with yellowing and
abnormal curvature associated with cuticle
Transudates: cardiac, liver or renal failure
loss (Figure 3.2)
Exudates: infection (parapneumonic),
malignancy (primary or secondary)
On examination of the chest, there is
bilaterally decreased expansion, stony dull The differential diagnosis of lymphoedema:
percussion note and absent breath sounds
at the bases, with associated decreased vocal
Congenital: familial lymphoedema
fremitus
Acquired: infection (filiariasis), pelvic mass,
iatrogenic (surgical removal of lymphatics)
Alternative diagnoses of discoloured and
Yellow nail syndrome
deformed nails, such as fungal infection with
Yellow nail syndrome is based on the triad of: concomitant but unrelated respiratory pathology.
Abnormal nails with a yellow/green
discolouration, which display slow growth
Lymphoedema, most commonly of the limbs
Demonstrate the importance
Respiratory disease, classically pleural of clinical context suggest
effusions, but including bronchiectasis relevant questions that would be
taken in a patient history
Follow with a summary of History questions to elicit the clinical features
relevant negative findings include:
Important negative findings include: Questions regarding nail changes, specifically
There are no signs of respiratory compromise a slow rate of nail growth
There is no lymphadenopathy Assessing the time of disease onset. In one
The JVP is not raised, there are no murmurs study, the mean age of onset was 56 years,
and the apex beat is not displaced but with a range of 2769 years
Questions seeking to elicit any underlying
disease associations. Although rare,
State the most likely diagnosis yellow nail syndrome has been associated
on the basis of these findings with other respiratory tract involvement
such as rhinosinusitis or bronchiectasis,
The triad of yellow nails, peripheral oedema and
autoimmune conditions, and malignancy
respiratory signs is consistent with a diagnosis of
the rare condition, yellow nail syndrome.
Family history familial disease has been
described
Case 78 Abnormal nails with pleural effusions and lymphadenopathy... 137

Figure 3.2 Yellow nails. Yellow


discoloration with thickening
and deformity of the nails, and
associated swelling and loss of
cuticles evident. Similar findings
would be expected on examination
of the toes.

Demonstrate an understanding Lymphatic investigations. Given that specific


lymphatic investigations are the remit of
of the value of further vascular surgery, the candidate may choose
investigation to tell the examiner that a referral to a
Yellow nail syndrome is a clinical diagnosis, vascular surgeon would be indicated for
however a patient presenting with any or all of investigations of primary lymphoedema
the signs of yellow nail syndrome would receive Finally, indicate the possibility of ruling
an extensive investigative work-up. out alternative nail pathology given its
Investigation of pleural effusion should prominence in making a clinical diagnosis. Nail
form the predominant focus of discussion. investigations would centre upon nail scrapings
Pleural effusion analysis in yellow nail to rule out fungal infection.
syndrome classically shows a clear fluid, with a
lymphocytosis, rich in protein and LDH.
Investigation of limb swelling: Note that Always offer a management plan
although yellow nail syndrome describes Given the rarity of yellow nail syndrome, it is
lymphoedema rather than peripheral oedema, it acceptable for the candidate to tell the examiner
is appropriate to include investigations relating that a referral to a specialist with expertise in
to the more common causes of limb swelling: this condition would be made. Of course, in
the respiratory station, it would be perfectly
Uninalysis and urinary protein:creatinine
appropriate to mention the above and then
ratio for heavy proteinuria
focus on the management of a pleural effusion.
Simple blood tests including albumin, liver
function tests and renal function
Cardiological tests, including a 12-lead Further reading
electrocardiogram and an echocardiogram, Hoque SR, Mansour S, Mortimer PS. Yellow nail
to rule out a primary diagnosis of heart syndrome: not a genetic disorder? Eleven
failure, which could easily be the cause of new cases and a review of the literature.
peripheral oedema and pleural effusions Br J Dermatol 2007; 156:12301204.
Ultrasound of the abdomen would provide Norkild P, Kroman-Andersen H, Struve-Christensen E.
information on liver echotexture, thus ruling Yellow nail syndrome the triad of yellow nails,
out liver cirrhosis as a cause of lymphoedema lymphedema and pleural effusion. A review of
and in the case of lower limb swelling, the literature and a case report. Acta Med Scand
provide information on any pelvic masses 1986; 219:221227.
causing lymphatic obstruction
Case 79: Signs of right-sided heart failure
cor pulmonale

Instruction to the candidate suggested by the triad of raised JVP, bibasal


crepitations, and peripheral oedema.
This 70-year-old man has frequent chest
infections and has noted progressive shortness
of breath on exertion. Please examine his State the most likely diagnosis
respiratory system and present your findings to on the basis of these findings
the examiner.
This patient has signs consistent with a
diagnosis of chronic obstructive pulmonary
Begin with a summary of disease complicated by cor pulmonale.
positive findings
On examination of the respiratory system, Offer relevant differential
positive findings include: diagnoses
On inspection, the patient has a Cor pulmonale is simply right heart failure due
hyperexpanded chest with reduced to raised pulmonary pressures reflected from
sternohyoid distance. The patient may a primary chronic pulmonary disorder. It is a
be either cachectic (pink-puffer) or with sequela of many chronic respiratory diseases,
increased BMI (blue-bloater). If the patient including COPD, interstitial lung disease
has significant disease such that they are in and chronic thromboembolic disease. Such
the exam, will likely be short of breath at rest. irreversible, chronic and progressive conditions
On expansion, there is equal but bilaterally are ideal substrates for consequent increases in
reduced chest excursion pulmonary pressures, with the development of
There is a globally resonant or hyper- right heart dilatation and dysfunction.
resonant percussion note
On auscultation, as with uncomplicated
COPD, the range of clinical signs on Demonstrate the importance
auscultation may range from globally of clinical context suggest
reduced breath sounds, to scattered wheeze
and crepitations
relevant questions that would be
taken in a patient history
The patient with cor pulmonale will have
Relevant history questions should focus on
additional signs on examination. In the setting
eliciting current symptoms, importantly
of COPD, noting any of the following clinical
exertional breathlessness with exact
signs should lead the candidate to seek to elicit
quantification of current exercise tolerance and
them all:
any change over time.
Loud second heart sound (P2), representing Screen for symptoms suggestive of an
forceful closure of the pulmonary valve underlying cause for the cor pulmonale:
Raised jugular venous pressure
Heavy smoking history: likely in this case as
Peripheral oedema
the clinical scenario is of advanced COPD.
In COPD also assess severity and level of
Follow with a summary of current treatment including use of long-term
relevant negative findings oxygen and non-invasive ventilation
Other chronic lung disease, such a
In addition to the standard negative findings pulmonary fibrosis or bronchiectasis
for the respiratory examination, comment on A history and/or symptoms of
the absence of clinical signs suggestive of acute thromboembolic disease
exacerbation or active infection.
Additionally, in the case of cor pulmonale, As always, assess the effect of symptoms on
it is important to identify the absence of the patients daily life and offer to address the
pulmonary oedema. This is a key finding patients ideas, concerns, and expectations in
because it differentiates between right-sided relation to their condition.
heart failure and congestive heart failure
Case 79 Signs of right-sided heart failure cor pulmonale 139

Demonstrate an understanding Patients will also need spirometry and


echocardiography to assess lung and cardiac
of the value of further function respectively, and many will require
investigation right and left heart catheterisation to measure
Patients with cor pulmonale will undergo a the pulmonary pressures.
wide range of tests, possibly fewer if the patient
is known to have long-standing COPD and Always offer a management plan
therefore the likely underlying diagnosis is
It is important to communicate to the examiner
known.
that you realise that, in the context of COPD,
Imaging with high-resolution CT scan will
cor pulmonale represents an advanced state of
identify the underlying pulmonary pathology
pathology in an irreversible clinical state.
and a CT pulmonary angiogram, with or without
a ventilation-perfusion scan, will elicit any
chronic thromboembolic disease.
Chapter 4

Abdominal system
(station 1)
Case 80: The syndrome of
chronic liver disease

Instruction to the candidate Haemachromatosis:


You are the registrar in the gastroenterology
Slate grey skin pigmentation
clinic. A 53-year-old man has been referred by The character of the liver edge can also be
his GP after a recent finding of abnormal liver of use in forming a differential. A cirrhotic liver
function tests. Please examine the patients will either be impalpable, or shrunken, hard and
abdominal system and report your findings to irregular. In fatty liver disease or active hepatitis,
the examiner. the liver may be tender, smooth and enlarged.
Non-tender hepatomegaly is suggestive
of a range of causes, but if craggy should
Begin with a summary of direct investigations towards hepatocellular
positive findings carcinoma.
Seek to identify not only signs suggestive of a
diagnosis of chronic liver disease, but also signs Follow with a summary of
which would suggest an underlying cause.
relevant negative findings
Signs of chronic liver disease Advanced liver disease progresses towards
cirrhosis with implications both on portal
On general inspection cachexia with
venous pressure and reduced functional reserve
muscle wasting, scratch marks
with eventual loss of liver function. The clinical
In the hands clubbing (Figures 4.1 and 4.2),
manifestations of portal hypertension should
leukonychia (Figure 4.3), palmar erythema
not be missed, not least as they serve as a useful
In the face scleral icterus
indicator of the severity of disease and alert to
On the praecordium spider naevi (Figure 4.4),
the likelihood of varices. Eliciting the features of
paucity of body hair, gynaecomastia
hepatic failure and establishing decompensated
In the abdomen jaundice, ascites, collaterals
disease is fundamental to a basic evaluation
and caput medusae (all of which are
where liver disease is suspected and must be
suggestive of advanced or decompensated
commented upon.
disease)
Patients with portal hypertension may have
some or all of: splenomegaly, a venous hum on
Additional signs suggestive of an auscultation, abdominal collateral vessels (and
underlying cause caput medusa) and/or ascites.
Alcoholic liver disease: Decompensated hepatic failure can presents
Facial telangiectasia non-specific but with a constellation of the following signs:
common in alcohol excess Asterixis
Dupuytrens contracture predominantly Jaundice
idiopathic but with a recognised Coagulopathy. On clinical examination
association with alcoholic liver disease this may present as bruising and/or active
Bilateral swollen parotid glands bleeding
Neurological signs of alcoholism Ascites
including peripheral neuropathy, Hepatic encephalopathy. If encephalopathy
proximal myopathy and cerebellar is present, seek to grade its severity
syndrome
Signs suggestive of viral hepatitis:
Tatoos State the most likely diagnosis
Track marks suggestive of intravenous on the basis of these findings
drug use
Primary biliary cirrhosis: This patient has clinical signs consistent with
Periorbital xanthelasma, commonly with compensated chronic liver disease with no
jaundice evidence of portal hypertension.
Case 80 The syndrome of chronic liver disease 143

Figure 5.1 Clubbing of the


fingers. Loss of the normal
angle between the nail bed and
cuticle/skin is evident; normally
approximately 160 degrees, in
clubbing the angle is commonly
greater than 180 degrees. There
is an exaggerated convexity of
the nail fold. Thickening of the
distal portions of the fingers is
appreciable and sponginess would
be expected with softening and
fluctuation of the nail bed. The
skin is also taught and has a shiny
quality.

Figure 5.2 Clubbing of the toes.


The clinical findings are identical
to that described in Figure 5.1. The
process is indiscriminate, affecting
toes and fingers alike.

Figure 5.3 Leukonychia.


Whitening of the entire nail
(leukonychia totalis) reflects
hypoalbuminaemia. This should
not be confused with partial
leukonychia secondary to illness
(Mees lines) or nail bed injury,
where white lines or spots can be
observed in an otherwise normal
nail.
144 Chapter 4 Abdominal system (station 1)

Where alcoholic liver disease is suspected


screening for dependency and establishing
current level of consumption is paramount.
Risk factors for hepatitis B and C include
blood transfusions; intravenous drug use;
tattoos; and high-risk sexual intercourse. It
is important to enquire about medication
use, such as methyldopa, amiodarone and
methotrexate.
Establishing a history of previous episodes
of decompensation requiring admission and/or
treatment, particularly in the high dependency
or intensive care setting, gives important
context for the potential for deterioration.
Ask specifically about previous episodes of
ascites, and where there is a history of varcies,
haemorrhage.

Figure 5.4 Spider naevus. Telangiectasia which blanch Demonstrate an understanding


and subsequently refill centrally when pressure is of the value of further
applied; they are found within the distribution of the
superior vena cava.
investigation
A sensible approach to investigation of a
patient with chronic liver disease would
include simple blood testing and imaging tests,
Offer relevant differential with further investigations as appropriate.
diagnoses The two key aims of investigation are to assess
liver function (importantly including synthetic
From the examination findings it may be possible function) and to identify the underlying cause
to offer and justify a likely underlying aetiology if possible.
for the evident liver disease, as outlined above,
followed by a differential. Where this is not
possible, proceed directly to a sensible differential:
Blood tests
Appropriate blood testing to assess liver
Common causes: function would include:
Alcoholic liver disease
Non-alcoholic fatty liver disease Liver function tests (LFTs), including
Chronic hepatitis C infection bilirubin (conjugated and unconjugated),
Less common: transaminases and alkaline phosphatase.
Chronic hepatitis B An AST:ALT ratio of greater than two
Autoimmune hepatitis suggests alcoholic liver disease. This is due
Primary biliary cirrhosis to chronic alcohol consumption causing a
Rare: lack of vitamin B6, which is required for ALT
Primary sclerosing cholangitis function. A ratio of less than one suggests
Wilsons disease non-alcoholic liver disease. An elevated
Haemachromatosis alkaline phosphatase is suggestive of
cholestasis
Markers of synthetic function include the
Demonstrate the importance prothrombin time, serum albumin and
of clinical context suggest platelets. Impaired synthetic function is
suggested by a prolonged prothrombin time
relevant questions that would be and/or a reduced serum albumin and/or a
taken in a patient history reduced platelet count
Screening for risk factors associated with Blood tests to identify the aetiology of the
commonly recognised underlying aetiologies. chronic liver disease would include:
Case 80 The syndrome of chronic liver disease 145

Serological testing to identify viral infections Importantly, where biopsy is proposed, ensure
such as: clotting is not significantly deranged.
Hepatitis A, B, or C
CMV Always offer a management plan
EBV
Auto-antibody testing: A discussion on the management of chronic liver
Primary biliary cirrhosis positive anti- disease should involve discussion not only of
mitiochondrial antibodies in 95% general aspects of management, but also of the
Autoimmune hepatitis positive ANA, management of the underlying cause, assuming
anti-smooth muscle antibody one has been identified through clinical
Primary sclerosing cholangitis positive examination.
ANCA in 80% General management aspects include:
Other tests for miscellaneous causes such as: Dietary and lifestyle advice with B vitamin
Ferritin as a screen for haemachromatosis supplementation in patients with chronic
Caeruloplasmin for Wilsons disease alcohol consumption, and a low salt and high
Alpha-1-antitrypsin levels protein diet in those with ascites
Tumour markers if appropriate Endoscopic surveillance for varices
Screening ultrasound and a-fetoprotein levels
Imaging 6 monthly for hepatocellular carcinoma
In the acute setting it is prudent to consider Need for early referral to high-dependency/
arranging simple imaging with ultrasound to intensive care (if appropriate) in the
rule out an obstructive cause before embarking deteriorating liver patient.
upon expensive laboratory investigations. Medication prescribing in the liver patient
Abdominal ultrasound is the first line
Management dependent upon the
imaging modality of the liver. It provides
underlying cause:
information about the liver echotexture
(cirrhosis, fatty infiltration) and can identify Alcohol abstinence
masses. Additionally, ultrasound allows a Viral hepatitis (B/C) interferon, ribavirin,
Doppler assessment of portal blood flow and protease inhibitors such as telapravir and
will provide information on the presence or beceprivir
absence of splenomegaly. If ascites is present, Autoimmune hepatitis prednisolone,
this can be identified, quantified and marked for azathioprine
drainage. Haemochromatosis therapeutic
Biopsy may be considered for diagnostic phlebotomy (usually to maintain a ferritin of
purposes where the aetiology is unclear with 2050 ng/mL) and chelation
equivocal or unexplained laboratory results, or Primary biliary cirrhosis ursodeoxycholic
where there is suspicion of multiple causes of acid
liver disease such as alcohol and viral hepatitis. Wilson's disease trientine and zinc
Case 81: Organomegaly isolated
hepatomegaly

Instruction to the candidate Offer relevant differential


This 45-year-old man has been referred to the diagnoses
gastroenterology clinic following a routine There is a wide range of causes of
medical examination. Please examine his hepatomegaly and these can be classified in
abdominal system. various ways.
Congential causes of hepatomegaly include
Begin with a summary of Riedels lobe and polycystic disease.
positive findings Acquired causes include:
Hepatomegaly is suggested by a palpable mass Infections, such as the viral hepatitides,
below the right costophrenic margin which CMV, EBV, amoebiasis, toxoplasmosis,
moves towards the right iliac fossa on inspiration, malaria
and is dull to percussion. Further characteristics Drugs, such as alcohol, amiodarone,
of the hepatomegaly to identify include: methotrexate
Is it smooth or craggy? Metabolic and Infiltrative causes, such as
Is it firm/hard? NASH, haemachromatosis, Wilson's disease,
Is it tender or non-tender? amyloidosis, Gaucher's
Autoimmune conditions such as
Associated findings to look for include any autoimmune hepatitis, primary biliary
scars to indicate previous biopsy or paracentesis. cirrhosis, primary sclerosing cholangitis
Neoplastic disease, in which you would
Follow with a summary of expect to find irregular firm hepatomegaly
on clinical examination. Secondary liver
relevant negative findings metastasis are more common than primary
In patients with isolated hepatomegaly it hepatic neoplasms
is important to report the absence of signs Passive venous congestion due to right heart
to suggest chronic liver disease or hepatic failure with resistance to right ventricular
decompensation. Additionally, comment on the filling:
absence of: Tricuspid regurgitation: classically
pulsatile hepatomegaly. Examination
Concurrent splenomegaly of the JVP can be useful in this regard,
Lymphadenopathy: if present, as giant V waves would be virtually
lymphadenopathy may suggest an pathognomonic
infective cause, but the presence Constrictive pericarditis
of hepatosplenomegaly and Restrictive cardiomyopathy
lymphadenopathy would raise suspicion of a Vascular causes such as BuddChiari or
lymphoproliferative disorder sickle cell disease
Features of congestive cardiac failure
or tricuspid regurgitation. Tricuspid In view of the extensive differential, to
regurgitation classically presents with a avoid the potential of listing and appearing
pulsatile liver edge prescriptive it is useful to consider the
differential diagnosis of isolated hepatomegaly
in relation to the additional features sought
State the most likely diagnosis on examination. It is paramount that the
on the basis of these findings presentation of positive and negative findings be
This patient has signs consistent with isolated comprehensive and clear enabling the candidate
hepatomegaly. to proceed with a narrowed differential.
Case 82 The distended abdomen ascites 147

Demonstrate the importance Ultrasound of the liver would be the first


line imaging modality to confirm clinical
of clinical context suggest findings of hepatic enlargement and assess
relevant questions that would be presence of fatty infiltrate, masses, cysts or
abscesses. It can also rule out biliary duct
taken in a patient history dilatation to help distinguish parenchymal
A suitable clinical history would include liver disease from extrahepatic bililary
questions to screen for known risk factors of obstruction.
liver disease, including: Further imaging such as CT or magnetic
Alcohol: ask about average usage, including resonance cholangiopancreatography should
episodes of bingeing be considered where ultrasound scanning fails
Recent travel: has the patient been to any to provide adequate assessment, is limited by
areas known to increase risk for certain obesity or poor views or where a malignant
underlying causes? process is suspected.
Risk factors for viral hepatitis such as blood Needle aspiration or biopsy, as appropriate,
transfusions, intravenous drug use, tattoos, for evaluation of lesions suggestive of an
or unprotected or high risk sexual intercourse infectious process, a cystic mass or neoplastic
disease.

Demonstrate an understanding
Always offer a
of the value of further
management plan
investigation Directed to the cause. The focus will vary greatly
Blood testing would include inflammatory from case to case and may inevitably be guided
markers, liver function tests and liver disease by the examiner.
screen.

Case 82: The distended abdomen ascites

Instruction to the candidate cirrhosis with fatty infiltration or chronic


cirrhosis with hepatocellular carcinoma
This 57-year-old woman has presented
complaining of abdominal swelling. Please
Peripheral oedema: ascites caused by liver
disease is usually isolated or disproportionate
examine her abdominal system and present
to peripheral oedema whereas the reverse
your findings to the examiner.
is true where the cause is generalised fluid
retention attributable to congestive heart
Begin with a summary of failure
positive findings Signs of an underlying cause of liver disease

On clinical examination, the abdomen is


distended and may be tense but soft and Follow with a summary of
non-tender to palpation, with demonstrable relevant negative findings
shifting dullness. Abdominal herniae may be
Important relevant negative findings to consider
prominent or be suggested by a flattened or
include:
everted umbilicus.
Additional associated signs include:
Signs of portal hypertension: splenomegaly,
venous hum, caput medusa
Hepatomegaly, which is often difficult to Other signs of hepatic decompensation,
elicit in the distended abdomen but may importantly encephalopathy, asterixis,
indicate non-alcoholic fatty liver, early jaundice
148 Chapter 4 Abdominal system (station 1)

Signs of sepsis, which if present would raise It would also be important to ask about
the possibility of spontaneous bacterial fevers and abdominal pain, which would lead to
peritonitis (SBP) consideration of SBP.

State the most likely diagnosis Demonstrate an understanding


on the basis of these findings of the value of further
This patient has ascites, of which the most investigation
common cause is cirrhosis with portal Investigations should include an assessment
hypertension. of the ascitic fluid. This will aid both the
identification of the underlying cause of the
Offer relevant differential ascites and allow exclusion (or confirmation) of
spontaneous bacterial peritonitis.
diagnoses A diagnostic paracentesis should be
The differential diagnosis of ascites can be performed with appropriate analysis of the fluid
divided into hepatic and non-hepatic causes. according to the following parameters:
Hepatic causes include portal hypertension
due to cirrhosis. This is the underlying cause Colour and appearance: is it blood stained,
inapproximately 90% of all cases. Such patients chylous, turbid, or straw coloured?
demonstrate the stigmata of chronic liver disease. White cell count: if the polymorph count
An additional hepatic cause would include severe is >250 this is indicative of spontaneous
alcoholic hepatitis (without cirrhosis). bacterial peritonitis: Further microscopy,
Non-hepatic causes of ascites would include: culture and sensitivity will allow
identification of the causative organism
Peritoneal malignancy Fluid biochemistry: the protein level
Intra-abdominal tuberculosis differentiates transudate versus exudate.
Fluid retention due to congestive cardiac Additionally, the serum albumin to ascites
failure, which on clinical examination would albumin gradient (SAAG) differentiates
reveal peripheral oedema and a raised JVP portal hypertensive ascites (SAAG > 11g/L)
(note comparison with portal hypertension from non-portal hypertensive ascites
where, as a result of venous dilatation, the Cytological assessment allows for
cardiac filling pressure is low with a JVP identification of malignant cells in the
typically difficult to elicit) ascitic fluid
The nephrotic syndrome and generalised
hypoalbuminaemia will also give the picture
of generalised oedema and fluid overload Serum-to-ascites albumin gradient
An important vascular cause is that of hepatic Assessment of the ascitic fluid will include
vein thrombosis, known as BuddChiari comparison of the albumin content in the
syndrome ascites with the serum albumin content to give
Less common causes include, pancreatitis, the serum to ascites albumin gradient (SAAG).
SLE, hypothyroidism, Meigs syndrome and Measuring SAAG enables the classification of
chylous ascites portal hypertensive (SAAG >11 g/L) versus
non-portal hypertensive (SAAG < 11 g/L) causes
Demonstrate the importance of ascites. This is calculated by subtracting
the ascitic fluid albumin value from the serum
of clinical context suggest albumin value, from samples obtained at the
relevant questions that would be same time. The value correlates with portal
taken in a patient history pressure. While typically absolute protein levels,
transudates and exudates are rarely applied to
Questions identifying risk factors for, and ascitic fluid, the protein level in conjunction with
symptoms of, the conditions listed below: SAAG can be useful. An elevated SAAG and a
Cirrhosis and portal hypertension high protein level are observed in most cases of
Cardiac disease ascites due to hepatic congestion. However, the
Malignancy combination of a low SAAG and a high protein
TB level is characteristic of malignant ascites.
Case 83 Portal hypertension 149

Where there is clinical suspicion of of 20% albumin should be administered


non-cirrhotic aetiology, consider the following: for every 2 litres drained. Transjugular
intrahepatic porto-systemic shunting (TIPS) is
Cardiac investigations should include ECG
utilised in ascites resistant to other treatment
and echocardiogram
or where rapid reaccumulation occurs with
For malignancy consider appropriate
haemodynamic compromise. An invasive
imaging and serum tumour markers
technique not without its complications one
For pancreatic pathology assess ascitic
must ensure against a coagulopathy, and
amylase
thereafter be alert to the potential for porto-
Thyroid function tests
systemic encephalopathy and worsening of
hepatic disease.
Always offer a management plan Transplantation
In portal hypertensive ascites, initial Non-portal hypertensive ascites is largely
management should include bed rest and unresponsive to diuretics and requires recurrent
salt restriction. Salt restriction represents paracentesis.
the best conservative approach, aiming for Where SPB is suspected, it should be treated
<2 g/day. Fluid restriction should be applied as a medical emergency with broad spectrum
where hyponatraemia exists. Diuresis with antibiotics, in line with local guidance. Where
spironolactone should be considered if salt and possible, obtain ascitic, blood, and urine
fluid restriction unsuccessful. Ideally, weight cultures prior to initiation of antibiotics.
loss of 0.5 kg per day should be achieved. Where Diagnostic paracentesis should not delay
more weight is lost, it is likely to represent the start of treatment where there is strong
diuresis from the intravascular compartment, clinical suspicion. Culture yield is traditionally
not peritoneal, and can result in complications poor from ascitic fluid but diagnosis can be
including renal dysfunction and worsening made in the first instance on the presence of
electrolyte derangement. greater than 250 neutrophils per micro-litre.
Management options for refractory ascites Antibiotic prophylaxis may be considered for
include: those patients at high risk of developing SBP
Therapeutic paracentesis: ascitic drains with advanced cirrhosis and significant ascites,
should remain in situ for no longer than subsequent to successful treatment, and in those
68 hours and should not be clamped. 100 mL admitted with acute variceal bleeding.

Case 83: Portal hypertension

Instruction to the candidate Porto-systemic anastomoses such as caput


medusae and oesophageal varices
This 69-year-old man has a history of excess
alcohol consumption. Please examine his
Ascites
abdomen and present your findings to the
A venous hum on auscultation
examiner. Additional findings to identify include those
of chronic liver disease
Begin with a summary of
positive findings Follow with a summary of
Portal hypertension, a process heralding relevant negative findings
cirrhosis in the context of chronic liver disease, Important relevant negative findings would
is manifest as: include:
Splenomegaly Evidence of confusion caused by
porto-systemic encephalopathy
150 Chapter 4 Abdominal system (station 1)

Evidence of decompensated liver disease, vein patency and flow.


particularly coagulopathy given the propensity Endoscopy with direct visualisation of
for gastrointestinal bleeding from varices oesophageal and gastric fundus varices confirms
the diagnosis and allows intervention as
appropriate.
State the most likely diagnosis Where no clear precipitant for liver disease
on the basis of these findings has been identified, the patient should be
This patient has clinical signs suggestive of investigated with a full liver screen. Suspected
portal hypertension. cirrhosis, most commonly due to alcohol with
a clear history, should be investigated to assess
severity with blood tests and imaging ensuring
Offer relevant differential no signs of decompensation.
diagnoses
The most common cause of portal hypertension Always offer a management plan
in developed countries is liver cirrhosis. Prevention of bleeding is key. Acute variceal
Causes of non-cirrhotic portal hypertension bleeds carry mortality rates upwards of 50%, and
include: are associated with high rates of re-bleeding in
Schistosomiasis (in travellers from endemic survivors.
areas) and HIV are important infective causes All patients diagnosed with cirhosis should
Increased resistance to right ventricular undergo an endoscopy to screen for varices.
filling constrictive pericarditis, restrictive Primary prophylaxis is with beta-blockade
cardiomyopathy and tricuspid regurgitation (typically propranolol or carvedilol) to reduce
Hepatic vascular aetiologies such as Budd portal blood flow. The dose is titrated up to
Chiari causing a post hepatic obstruction maintain a resting heart rate of <55 beats/
Increased portal venous flow although rare, min. In addition to reducing the likelihood
this may occur in arteriovenous malformation of a variceal bleed, if successful in reducing
or as the result of massive splenomegaly portal pressure, beta-blockers may also reduce
caused by a primary haematological disorder chronic bleeding from gastric mucosal vascular
(see Splenomegaly, p. 163). congestion (portal hypertensive gastropathy).
Patients who respond poorly to medical
therapy may be considered for TIPS or porto-
Demonstrate the importance caval shunting. Liver transplantation may be
of clinical context suggest considered depending on the clinical picture.
Splenomegaly rarely causes complications
relevant questions that would be in the context of portal hypertension and thus
taken in a patient history splenectomy is avoided as a general rule.
In a clinical history, the patient should be asked The patient should be educated on avoidance
regarding whether varices have been identified of causative/contributory factors.
and also whether there has been any previous
gastrointestinal bleeding and/or intervention. Prognosis
The risk factor profile for liver disease should be
explored, specifically asking about alcohol and In the presence of cirrhosis, clinical examination
where suspected other precipitants including and the results of laboratory information can
infection and heart disease. be combined to provide a prognostic score, as
in the ChildPugh score (Table 4.1). A CPT score
greater than 10 carries a 50% 1 year mortality
Demonstrate an understanding in those with advanced cirrhosis. The Model for
of the value of further End-stage Liver Disease (MELD) scoring system
investigation has become increasingly favoured over the CPT
classification in predicting mortality although
A diagnosis of portal hypertension is inferred the modified-CPT which takes into consideration
in patients with evidence of cirrhosis by clinical creatinine levels has been shown to be as useful
examination. Direct measurement of the portal as MELD in predicting short and medium term
pressure with transjugular catheter is rarely mortality and is significantly simpler in execution.
performed due to the incumbent risks involved. Reference: Papatheodoridis et al. MELD vs
Thus, the diagnosis is confirmed with ultrasound ChildPugh and creatinine-modified ChildPugh
or CT demonstrating engorged intra-abdominal score for predicting survival in patients with
collaterals and with Doppler to assess portal decompensated cirrhosis.
Case 84 Abnormal skin pigmentation jaundice 151

Table 4.1 ChildPugh score for prognosis in liver cirrhosis


Clinical variable 1 point 2 points 3 points
Encephalopathy None Grade 12 Grade 34
Ascites Absent Slight Moderate/large
Bilirubin mg/dL <34 3450 >50
Albumin g/L >35 3528 <28
INR <1.7 1.72.3 >2.3
INR, international normalised ratio
ChildPugh classification serves as a measure of the severity of liver disease. The grades correlate with one and two year patient
survival as follows. Grade A (56 points): 100% and 85%; Grade B (79 points): 80% and 60%; Grade C (1015 points): 45% and 35%

Further reading Child-Pugh score for predicting survival in


patients with decompensated cirrhosis. World J
Papatheodoridis GV, Cholongitas E, Dimitriadou E, et Gastroenterol 2005; 11:30993104.
al. MELD vs Child-Pugh and creatinine-modified

Case 84: Abnormal skin


pigmentation jaundice

Instruction to the candidate Follow with a summary of


This 47-year-old man has presented with relevant negative findings
abnormal skin discolouration. Please examine Important relevant negatives include:
and present your findings to the examiner.
Signs of liver failure, such as confusion and
encephalopathy. Jaundice as the result of
Begin with a summary of acute on chronic liver disease, in a cirrhotic
positive findings liver, may be associated with other signs
Jaundice is demonstrated by yellow of decompensation, importantly hepatic
pigmentation of the sclera, skin and mucosa encephalopathy. Indeed, jaundice and
(Figures 4.5, 4.6, and 4.7). encephalopathy serve as sensitive indicators
Associated findings which can help form a of both severity and decompensation in
suitable differential diagnosis include: chronic disease. In the context of ascites or
portal hypertension one should consider
Signs of chronic liver disease and portal jaundice as decompensation and be
hypertension (see Portal hypertension prepared to discuss the CPT score with the
p. 149)) If these signs are present, signs examiner (see The syndrome of chronic liver
suggesting an underlying cause of the liver disease, p. 142)
disease should be sought The absence of a palpable gallbladder. This
Signs suggestive of acute onset liver disease mainly concerns surgical jaundice but one
tender hepatomegaly, abdominal discomfort should bear in mind Courvoisiers law which
Tattoos or needle track marks which would states that in the presence of a palpable
suggest an increased risk of viral hepatitis gallbladder, painless jaundice is unlikely to
152 Chapter 4 Abdominal system (station 1)

Figure 4.5 Jaundice. The normally


white sclerae are noticeably
discoloured with a yellow hue,
scleral icterus, consistent with
jaundice. There is also visible
yellowing of the peri-orbital skin.

Figure 4.6 Jaundice. Gross


yellowing of the skin is evident.
There is also praecordial spider
naevi and the paucity of body hair
consistent with underlying liver
disease.

Figure 4.7 Jaundice. Jaundice


may be appreciable on general
inspection of the hands prior to
examination of the torso and
abdomen.

be due to cholelithiasis. The implication is State the most likely diagnosis


that the clinical picture is more likely to be
due to pancreatic malignancy on the basis of these findings
Lymphadenopathy: where jaundice exists A possible presentation to the examiner in
in the context of diffuse lymphadenopathy, the case of jaundice could be: This patient
it is important to consider viral causes and appears grossly jaundiced. There are no clinical
haematological malignancy signs suggestive of a clearly identifiable cause
and thus I would like to take a full history and
Case 84 Abnormal skin pigmentation jaundice 153

investigate further to establish an underlying Demonstrate the importance


diagnosis on which to base my management.
In a jaundiced patient always attempt to of clinical context suggest
identify the underlying cause and present it relevant questions that would be
to the examiner accordingly. If the patient
clearly exhibits clinical signs consistent with
taken in a patient history
chronic liver disease the candidate may choose Important symptoms would include:
to discuss predominantly liver disease with Pruritis a common symptom in jaundiced
jaundice as a manifestation and consideration patients
of exacerbating or decompensating factors. Dark urine this often precedes icterus
Alternatively, acknowledge the finding of Risk factors for liver disease, past medical
jaundice in the presence of chronic liver history and family history
disease and thereafter consider the differential In the context of chronic liver disease, screen
diagnoses of jaundice itself. There will inevitably for triggers of decompensation
be overlap between the two approaches but the
candidate should be clear on the distinction and
commit to one or other to avoid confusion. Demonstrate an understanding
of the value of further
Offer relevant differential investigation
diagnoses Testing of the urine differentiates
The differential diagnosis of jaundice includes: between conjugated and unconjugated
hyperbilirubinaemia. Conjugated urine is
Jaundice with unconjugated soluble and thus causes the urine to be dark.
In a patient with jaundice, the serum
bilirubinaemia bilirubin will be raised. Analysis of the
Increased turn over commonly due to conjugated and unconjugated subtypes of
haemolysis or ineffective erythropoeisis bilirubin will help narrow the differential
(sickle cell disease). Less commonly due to diagnosis. Assessment of transaminase and
resorption of large haematomas alkaline phosphatase levels assists in the
Hepatic congestion with decreased uptake. differentiation between cholestasis and
Seen in congestive cardiac failure, post- hepatocellular dysfunction.
imaging with contrast, iatrogenic including Where conjugated jaundice is suspected,
rifampicin routine blood testing is likely to reveal little with
Impaired conjugation seen most commonly no significant derangement of liver function
in Gilberts. Less commonly CriglerNajjar expected. Thus, investigation should be focused
syndrome and hyperthyroidism upon establishing a cause for haemolysis
including: blood film/smear, electrophoresis,
Jaundice due to conjugated and Coombes test.
Deranged liver function and a raised
bilirubinaemia conjugated bilirubin, in the absence of
Hepatocellular dysfunction: symptoms suggestive of obstructive jaundice
Viral hepatitides, CMV, and EBV should prompt a liver screen. (See The syndrome
Alcoholic cirrhosis of chronic liver disease, p. 142).
Haemachromatosis Depending on the results of urine and blood
Autoimmune hepatitis testing, a range of imaging modalities can be used.
Wilsons disease Ultrasound of the abdomen is usually first
Alpha-1 antitrypsin deficiency line, providing important information about
Impaired hepatic excretion. Importantly, the liver and portal circulation and about
this is often associated with intractable the gallbladder and biliary tree (including
pruritis and thus excoriations serve as a good gallstones).
discriminating clinical sign allowing you Obstructive symptoms where dilatation is
to narrow the differential. Possible causes demonstrated with ultrasound but no gallstones
include: are seen should lead to magnetic resonance
Alcoholic liver disease or MRCP/ERCP. Where no dilatation is
Primary biliary cirrhosis demonstrated, consider biopsy to establish the
Ascending cholangitis cause of infiltrative liver disease or intrahepatic
Head of pancreas carcinoma cholestasis.
154 Chapter 4 Abdominal system (station 1)

A CT or MRI may be considered where there With regard management of symptoms,


is suspicion of malignancy. obstructive jaundice may cause intractable
pruritis, which can be alleviated to varying
degrees dependent upon the cause, with
Always offer a management plan cholestyramine. However, where the obstruction
The management of jaundice can be divided is complete, such as extrahepatic cholestasis
into management of the symptoms of jaundice secondary to gallstones, such medical therapy is
and of the underlying cause. The management generally unsuccessful.
of the various underlying causes can be found
elsewhere.

Case 85: Abnormal skin pigmentation


haemachromatosis

Instruction to the candidate Signs of decompensated cirrhosis


This 44-year-old man has been referred to the
Peripheral oedema and a raised jugular
venous pressure consistent with signs of
gastroenterology clinic with deranged liver
decompensated cardiomyopathy
function tests. Please perform an abdominal
examination and present your findings to the
examiner. State the most likely diagnosis
on the basis of these findings
Begin with a summary of This patient has signs consistent with a
positive findings diagnosis of haemachromatosis.
The main positive findings in a case of
haemachromatosis would involve hepatomegaly Offer relevant differential
in a patient with slate-grey skin pigmentation diagnoses
and scanty axillary hair. Additionally, there are
likely to be stigmata of concurrent diabetes Given its characteristic features,
with abdominal lipodystrophy, bruising from haemachromatosis is usually considered as a
subcutaneous insulin administration and finger spot-diagnosis. Therefore, rather than giving
pulp bruising from blood sugar monitoring. a differential diagnosis of the condition itself,
Associated signs in a case of choose a particular sign and give a differential
haemachromatosis include: diagnosis of that. This may involve giving a
differential diagnosis of hepatomegaly (see
Cardiovascular: displaced apex beat Organomegaly: isolated hepatomegaly, p. 146).
suggestive of concurrent cardiomyopathy Alternatively, a differential diagnosis of slate-grey
Musculoskeletal: pseudogout usually effects skin pigmentation would include iatrogenic
the second and third metacarpophalangeal administration of amiodarone.
joints Haemachromatosis is a disease of iron
Urogenital: testicular atrophy candidates overload, the differential diagnosis of which
should offer to examine for this but refrain includes:
from actually doing so
Primary (or hereditary) haemachromatosis
Secondary iron overload in conditions such
Follow with a summary of as sickle cell disease, thalassaemia, and
relevant negative findings sideroblastic anaemia. Due to a combination
of increased iron absorption and iron
Important relevant negative findings to note and
supplementation for anaemia and in
present to the examiner as such include:
repeated transfusions
Case 85 Abnormal skin pigmentation haemachromatosis 155

Demonstrate the importance Liver function tests are often minimally


affected
of clinical context suggest Genetic testing: HFE genotyping is used
relevant questions that would be but note incomplete penetrance of
C282Y gene and the finding of primary
taken in a patient history haemachromatosis in C282Y -/- individuals
Screen for symptoms consistent with a diagnosis Biopsy with Perls staining remains the gold
of haemachromatosis, such as lethargy, which standard for diagnosis
is the most common presentation. Arthropathy Imaging: MRI is effective in demonstrating
and erectile dysfunction are other common iron overload
symptoms.
Lethargy is the most common presentation. Tests to assess end-organ damage include:
Haemachromatosis can be considered Fasting glucose, OGTT and HbA1c to confirm
as either genetic, or as acquired due to iron and assess severity of diabetes mellitus
accumulation from repeated transfusions. Assessment of pituitary function, importantly
Demonstrate this knowledge in your differential gonadotrphin secretion
and elicit risk factors to strengthen diagnosis 12-lead electrocardiogram and
including: echocardiogram are important where
Family history suspicion of cardiomyopathy exists
A history of regular transfusions and consider
haematological disorders, commonly sickle Always offer a management plan
cell disease or thalassaemia
Principles of management in haemachromatosis
It is prudent to include a social history to include:
gauge alcohol intake.
Regular venesection with measurement
of Hb aiming for lower end of normal and
Demonstrate an understanding ferritin aiming for 2050 g/L. Venesection
of the value of further required 36 weekly for up to 2 years
initially. Where not tolerated, chelation with
investigation desferrioxamine can be considered
Investigations can be divided into those aimed Referral to endocrinology for management
at confirming the diagnosis and those aimed at of diabetes and consideration of pituitary
assessing end-organ damage. hormone replacement. Venesection will
Tests to confirm the diagnosis include: result in falsely lowered HbA1c levels
Blood tests:
Abstinence from alcohol is strongly advised,
with acceleration of cirrhosis
A raised serum ferritin >1000 g/L Cirrhotic patients have increased (>10%) risk
is highly suggestive of primary
of hepatocellular carcinoma so 6 monthly
haemachromatosis. Additionally, the
screening with ultrasound and a-fetoprotein
transferrin saturation will be increased
advisable in advanced disease
and the total iron binding capacity
decreased
Family screening
Case 86: Periorbital xanthelasma primary
biliary cirrhosis

Instruction to the candidate Offer relevant differential


This 52-year-old woman has presented with diagnoses
pruritis. Please examine her abdominal system A range of differential diagnoses can be given.
and present your findings to the examiner. The differential diagnosis of primary biliary
cirrhosis itself includes autoimmune hepatitis
Begin with a summary of and autoimmune cholangitis.
The differential diagnosis of obstructive
positive findings jaundice includes biliary obstruction (e.g.
Positive findings in primary biliary cirrhosis secondary to gallstones) and primary sclerosing
include: cholangitis.
The differential diagnosis of xanthelasma
Jaundice includes primary and secondary dyslipidaemias.
Excoriations
Peri-orbital xanthelasma (Figure 4.8)
Associated signs of chronic liver disease Demonstrate the importance
of clinical context suggest
Follow with a summary of relevant questions that would be
relevant negative findings taken in a patient history
It is important to confirm the absence of signs of The classical presentation is typically that of
cirrhosis, portal hypertension or decompensated unexplained fatigue, right upper quadrant
liver disease. Additionally, where primary biliary discomfort, and jaundice with pruritis in a
cirrhosis (PBC) is suspected, relevant negatives middle-aged woman. Questions in a clinical
would include signs consistent with other history would therefore be aimed at eliciting
commonly associated autoimmune disorders: these features.
Rheumatoid arthritis Non-specific symptoms of lethargy and
Sjgren's disease malaise are common at presentation, as are those
Systemic sclerosis of dry eyes and mouth, and cholestasis (pruritis
Thyroid disease and steatorrhoea). Other features may include
previous episodes of jaundice and/or a family
history of liver disease or autoimmune disorders.
State the most likely diagnosis
on the basis of these findings Demonstrate an understanding of
In a woman with signs of obstructive jaundice the value of further investigation
and xanthelasma a diagnosis of primary biliary
cirrhosis should be considered until proven PBC is the most common chronic cholestatic
otherwise. disease of the liver with progressive
destruction of small to medium sized bile

Figure 4.8 Periorbital


xanthelasma. Subcutaneous
deposits of lipid rich macrophages,
resulting in plaques or nodules,
often with well-demarcated borders
and yellow/orange hue, in the
peri-orbital region.
Case 87 The patient requiring renal replacement therapy 157

ducts. Investigation centres on establishing a Medical therapies include:


cholestatic picture on liver function testing,
identifying an autoimmune basis for disease and
Ursodeoxycholic acid (UDCA) is thought to
decrease hepatocellular damage and slows
imaging the bile ducts to rule out a mechanical
progression to fibrosis but does not decrease
obstruction. Confirmation of the disease is
time to transplantation
commonly made on biopsy.
Liver function tests will typically demonstrate
Pruritis may be controlled with an
antihistamine, cholestyramine rifampicin
elevated ALP and GGT with unremarkable
or UDCA however care should be taken to
transaminases. Bilirubin is not normally
avoid co-administering with ursodeoxcholic
raised in the early stages of PBC, where
acid as this will reduce absorption of the
hyperbilirubinaemia exists it is predominantly
latter. Ultraviolet light is thought to be a
conjugated and represents disease progression
beneficial adjuvant therapy
with a poorer prognosis.
Serum levels of auto-antibodies include
In advanced disease where malabsorption
is likely, vitamin replacement is advised and
positive antimitochondrial antibodies (95%).
measures to protect against osteoporosis,
Anti-nuclear (ANA), anti-smooth muscle
including bisphosphonates, prudent.
(Anti-SM) and rheumatoid factor (RF) are also
commonly present. Liver transplantation is the definitive
Suitable imaging modalities include a liver treatment. Recurrence rates are high despite
ultrasound and MRI. immunosuppression however the course of
Liver biopsy, performed early, proves recurrent disease appears to be benign with
diagnostic where characteristic bile duct lesions promising 10-year survival rates and good
are identified. Delayed biopsy in advanced quality of life. Indications for referral to a
disease may fail to differentiate more generic transplantation centre include worsening
cirrhotic changes. hyperbilrubinaemia, intractable pruritis, severe
osteoporosis, hypoalbuminaemia or evidence of
Always offer a management plan decompensated liver disease.
Conservative measures, including avoiding
alcohol and hepatotoxic drugs, are advised
where possible.

Case 87: The patient requiring renal


replacement therapy

Instruction to the candidate Arteriovenous fistula in the antecubital fossa


or forearm (brachiocephalic communication)
This 50-year-old woman has presented to the are used for haemodialysis
nephrology clinic. Please examine her abdomen Indwelling catheters that may be present
and present your findings to the examiner. include Tesio lines (long-term tunnelled
lines used for haemodialysis) or short-term
Begin with a summary of vascaths used for haemofiltration.
positive findings Patients with peritoneal dialysis will have
abdominal scars and a Tenckhoff catheter
The patient may display clinical findings
consistent with one or more mode of renal Additionally, the patient may display general
replacement therapy. Remember that many clinical signs related to renal failure, such as:
patients will, over time, undergo more than one Anorexia
mode of renal replacement. Such clinical signs Dry skin with excoriations
include:
158 Chapter 4 Abdominal system (station 1)

Anaemia Diabetes
Mobility aids necessitated by bony pain Hypertension
secondary to renal osteodystrophy. APKD
Parathyroidectomy scar
It is important to look for clinical signs which Demonstrate the importance
may indicate an underlying cause of renal of clinical context suggest
failure:
relevant questions that would
Diabetes fingertip echymoses, abdominal
lipodystrophy, ulceration and amputation be taken in a patient history
Hypertension renal bruit In a history it would be important to identify
Autosomal dominant polycystic kidney whether the patient has symptoms of renal
disease bilateral ballotable kidneys failure, and to assess risk factors for renal disease.
Symptoms of renal failure include lethargy,
nocturia, and pruritis.
Follow with a summary of Risk factors for renal disease:
relevant negative findings Diabetes
Pertaining to arteriovenous fistulas, it is Hypertension
important to palpate and auscultate for flow Family history
but be mindful that flow does not correlate with
use. Examine fistulas closely for signs of recent Demonstrate an understanding of
instrumentation consistent with ongoing use. In
the absence of needle marks the fistula is unlikely the value of further investigation
to represent an active site of dialysis delivery. Investigations can be focused on those that
It is important to assess fluid status, evaluate the renal dysfunction and its severity,
examining for a raised JVP, bibasal crepitations, and those which assess for the severity and
and peripheral oedema. Lack of fluid overload is systemic complications of the underlying
an important relevant negative to record. cause(s) of the renal dysfunction.
An additional relevant negative to note Investigations directed towards assessing the
is the absence of signs of previous renal renal dysfunction itself include:
transplantation. If present, this would suggest a Urine: dip for haematuria and proteinuria.
failed transplant requiring re-initiation of renal Protein:creatinine ratio to quantify
replacement therapy. proteinuria
Blood tests: renal function, eGFR,
State the most likely diagnosis electrolytes, haemoglobin, bicarbonate,
calcium, phosphate
on the basis of these findings Imaging: ultrasound
This patient has evidence of renal replacement
Investigations to assess underlying causes
therapy raising the suspicion of underlying renal
include:
disease.
An example presentation: The patient has Diabetes: HbA1c.
an arteriovenous fistula in the right antecubital Hypertension: 12-lead electrocardiogram and
fossa with a palpable bruit and needle marks chest radiograph assessing cardiomegaly,
consistent with recent instrumentation for left ventricular hypertrophy and where
use in haemodialysis. The need for renal appropriate echocardiogram to investigate
replacement in this patient may be secondary cardiomyopathy
to diabetic renal disease as evidenced by
abdominal lipodystrophy indicative of regular
subcutaneous insulin administration. Always offer a management plan
Renal replacement therapy is utilised to treat
both acute kidney injury (AKI) and end stage
Offer relevant differential chronic kidney disease (ESCKD). The use of
diagnoses renal replacement therapy in AKI, where a rapid
Common causes of chronic kidney disease reduction in kidney function results in a failure
leading to a requirement for renal replacement to maintain fluid, electrolyte and acidbase
therapy include: homoeostasis, represents a holding measure to
Case 88 Bilateral ballotable kidneys 159

allow reversal of the underlying pathology with Indications for dialysis in ESCKD are less
restoration of renal function. clear, but centre around the measured fall
Indications for renal replacement therapy in in glomerular filtration rate with generally
AKI include: accepted implementation at <10 mL/minute or
<15 mL/minute on a background of diabetes.
Clinical features of uraemia pericarditis,
Consideration should be given to conservative
hypothermia, encephaolpathy
management where appropriate e.g. view of
Fluid retention with pulmonary oedema
mortality and quality of life.
refractory to diuresis
Also consider:
Hyperkalaemia >6.5 mmol/L refractory to
medical management Erythropoietin
Sodium >155 mmol/L or <120 mmol/L Vitamin D
Metabolic acidosis pH <7.0 Anti-pruritics
Severe renal failure urea >30 mmol/L,
creatinine >500 mmol/L

Case 88: Bilateral ballotable kidneys

Instruction to the candidate State the most likely diagnosis


This 29-year-old man has been complaining of on the basis of these findings
abdominal pain. Please examine his abdomen Adult polycystic kidney disease.
and report your findings to the examiner.

Offer relevant differential


Begin with a summary of
diagnoses
positive findings Various differential diagnoses can be given.
Positive findings include: Bilateral, ballotable, The differential diagnosis of bilateral palpable
flank masses together with a palpable irregular, kidneys includes:
non-tender, hepatomegaly with a relative
paucity of other clinical signs. Polycystic kidney disease
Bilateral multiple simple cysts
Bilateral hydronephrosis
Follow with a summary of Bilateral renal cell carcinoma (rare)
relevant negative findings Amyloidosis
Important relevant negative findings to note The differential diagnosis of bilateral cystic
include: kidneys, where neither or only one kidney may
be palpable on examination includes:
A lack of fluid overload and/or clinical
evidence of uraemia to suggest renal Polycystic kidney disease
dysfunction Multiple simple cysts
A lack of evidence of current and/or previous von HippelLindau syndrome
renal replacement therapy or evidence of Tuberous sclerosis
transplantation LaurenceMoonBiedl syndrome
A lack of evidence of chronic liver disease MeckelGruber syndrome
(in view of the hepatomegaly)
160 Chapter 4 Abdominal system (station 1)

Demonstrate the importance Blood tests are rarely of diagnostic benefit.


Anaemia or polycythaemia are possible. Renal
of clinical context suggest function should be monitored for disease
relevant questions that would progression. Liver dysfunction is uncommon
even in presence of cysts.
be taken in a patient history Ultrasound confirms diagnosis, assesses
Appropriate questioning would include those severity and investigates for involvement of
aimed at eliciting common symptoms preceding liver, pancreas, and spleen. An MRI brain allows
diagnosis, such as: assessment of aneurysms predisposing to
Abdominal fullness or pain subarachnoid haemorrhage
Haematuria
Nocturia Always offer a management plan
Treatment of hypertension
Important aspects of management include:
Additionally, it is appropriate to include in a
history questions regarding complications of the
Close control of blood pressure, minimising
systemic effects of hypertension where
condition, such as:
possible. ACE inhibitors have a recognised
A history of subarachnoid haemorrhage role
A family history of the disease Pain management
Any history and/or treatment of hypertension Early referral for consideration of renal
replacement therapy and transplantation.
Nephrectomy may be considered at time of
Demonstrate an understanding transplantation
of the value of further Screening of relatives:
investigation Genetic screening
Ultrasound scanning from the age of
Bedside tests include an assessment of the blood 20 years
pressure, which may be raised, and a urine MRI of the brain
dipstick to look for haematuria.

Case 89: Iliac fossa scar with a palpable


mass renal transplant

Instruction to the candidate Follow with a summary of


This 39-year-old woman is under follow up in relevant negative findings
the renal clinic. Please examine her abdomen. Important relevant negative findings in a renal
transplant case include:
Begin with a summary of Signs of renal failure:
positive findings Fluid balance
There is fullness of the abdomen with a scar Uraemia: confusion, fasciculations, skin
in the right iliac fossa on general inspection. pigmentation
There is a palpable, discrete, firm mass, which Signs of current active renal replacement
is non-tender with no fluctuance and is therapy such as recent needle marks in an
non-tender to percussion which is dull in note. AV fistula, or the presence of a tunnelled
There is associated evidence of previous renal dialysis catheter
replacement therapy, notably an arteriovenous Signs of possible rejection such as a tender
fistula in the antecubital fossa with a palpable graft
bruit but no recent instrumentation marks. Signs related to immunosuppression:
Case 89 Iliac fossa scar with a palpable mass renal transplant 161

Steroids cushingoid features Full drug history including current


Ciclosporin gum hypertrophy immunosuppression and potentially
Tacrolimus excoriations from pruritis nephrotoxic drugs
Skin infections and malignancies
Demonstrate an understanding of
State the most likely diagnosis the value of further investigation
on the basis of these findings Investigations to evaluate transplant and renal
A mass below a scar in the right iliac fossa, function:
consistent with a renal transplant which
appears to functioning in view of no evidence
Blood tests: renal function, inflammatory
markers, immunosuppressant levels
of active renal replacement therapy. There
is/is not evidence of the side effects of
Urine: dip for blood and protein.
Protein:creatinine to quantify proteinuria
immunosuppression. A preliminary skin survey
does/does not reveal pathology.
Imaging: ultrasound scanning and Doppler
evaluation of transplanted vessels to ensure
patency and flow
Offer relevant differential Basic investigations to elicit common
diagnoses underlying causes:
The differential diagnosis of a right iliac fossa
mass includes:
Blood pressure hypertension.
HbA1c poor glycaemic control
Terminal ileal disease Crohns, infection, Renal ultrasound for cystic disease
TB, lymphadenopathy
Ovarian pathology
Always offer a management plan
The differential diagnosis of the underlying Important aspects of management include:
aetiology of renal disease necessitating
transplantation (along with appropriate clinical Titration of calcineurin targeted
findings) includes: immunosuppression post transplantation
balancing risk of rejection against that of
Diabetes lipodystrophy, finger pulp nephrotoxicity under corticosteroid cover
ecchymoses, offer ophthalmoscopy. Note Tight glycaemic control and blood pressure
that people with type 1 diabetes are often in patients with diabetes and hypertension
considered for dual kidney-pancreas respectively
transplantation or pancreas after kidney Surveillance for rejection or infection:
transplant mention any additional scars Rejection most commonly occurs within
Hypertension renal artery stenosis with 4 months of transplantation. Living-
audible bruit, offer ophthalmoscopy donor grafts are more successful than
Polycystic kidney disease ballotable kidneys deceased-donor grafts. Of those patients
or associated nephrectomy scar who survive 1 year post transplantation,
half die of other causes with a functioning
Demonstrate the importance graft, while the other half will suffer
chronic allograft nephropathy with graft
of clinical context suggest failure within 15 years
relevant questions that would Intensified immunosuppression with
pulsed high dose corticosteroids usually
be taken in a patient history reverses accelerated or acute rejection
Appropriate questioning would include but where unsuccessful haemodialysis is
questions regarding: required. In a rejected graft, nephrectomy
Symptoms of renal insufficiency nocturia, may be required if infection suspected on
fluid status and exercise tolerance cessation of immunosuppression
Previous history of renal replacement therapy Surveillance is best achieved through
Past medical history, specifically an serial measurement of serum creatinine
underlying cause plotted over time with some benefit from
ultrasound assessment of patency and
flow in transplanted vessels
Case 90: Flank scar nephrectomy

Instruction to the candidate Irreversible symptomatic kidney damage


secondary to infection, trauma, parenchymal
This 82-year-old man has been experiencing
disease or obstruction due to significant
weight loss. Please examine his abdomen and
calcinosis
present your findings.

Begin with a summary of Demonstrate the importance


positive findings of clinical context suggest
Cachectic elderly patient with a flank scar but relevant questions that would be
relative paucity of other clinical signs.
taken in a patient history
Symptoms consistent with renal cell carcinoma,
Follow with a summary of which would be included in a history, include:
relevant negative findings Painless haematuria
It is important to note the absence of: Abdominal discomfort and bloating
Renal replacement therapy Weight loss, cachexia, lethargy
A renal transplant Venous dilatation, lower limb oedema, and in
Contralateral kidney pathology, where men the presence of varicoceles
polycystic is suspected ensure no A screen for risk factors for renal cell
hepatomegaly carcinoma would include:
Family history, including von HippelLindau
State the most likely diagnosis disease
on the basis of these findings Occupational history and carcinogen
exposure
Summary of the findings and the pertinent
negatives with a discussion of the differential
Smoking history
diagnosis before committing to a discussion on
renal cell carcinoma. Demonstrate an understanding of
the value of further investigation
Offer relevant differential Investigation of suspected renal cell carcinoma
(RCC) after onset of symptoms and prior to
diagnoses nephrectomy:
Of the flank scar access to the retroperitoneal
space: Imaging. CT with contrast or MRI the
diagnosis is often made as an incidental
Lattisimus dorsi flap look for evidence of finding on imaging performed for other
breast reconstruction reasons. There is minimal role for needle
Adrenalectomy look for a medical alert biopsy. Where the diagnosis is not made
bracelet radiographically it will usually be confirmed
Retroperitoneal abdominal aortic aneurysm at surgery.
repair arteriopathic patient, look for Chest radiograph, where cannon ball
evidence of peripheral circulation, venous metastases suspected, further imaging with
harvesting, bypass scars and amputations CT warranted.
Abscess incision and drainage Blood tests: Where ALP or calcium raised,
Skin lesion excision bone scanning is indicated. Renal function
is not normally deranged unless there is
Of the need for nephrectomy:
bilateral renal involvement. Polycythaemia
Renal cell carcinoma (radical or partial is possible secondary to increased
nephrectomy see below). Also, transitional erythropoeitin production, more commonly
cell carcinoma, Wilms tumours and sarcoma patients are anaemic.
Polycystic kidneys due to compression of
Post nephrectomy investigations include
surrounding structures or symptom relief
surveillance with appropriate radiology and
Case 91 Splenomegaly 163

monitoring of residual renal function, full blood Late RCC palliative therapies with or
count and bone profile. without surgery:
Tumour embolisation
Always offer a management plan Nephrectomy, debulking, or resection of
Management of RCC depends on the disease metastases considered
staging (TMN). Radiotherapy typically used for bone
Early RCC surgical intervention: metastases and pain. RCC and non-bony
metastases are relatively radiotherapy
Radical nephrectomy (removal of kidney, resistant and thus this is no longer used as
adrenal gland, perirenal adipose tissue and standard adjuvant therapy
Gerots fascia) indicated for isolated local Interferon a and new, targeted biological
disease with curative intent therapies have shown promise and may
Partial nephrectomy nephron sparing, for be used increasingly as adjuvants to
small tumours or where bilateral disease exists nephrectomy with curative intent

Case 91: Splenomegaly

Instruction to the candidate Follow with a summary of


This 47-year-old woman has presented with relevant negative findings
abdominal pain. Please examine her abdomen. As mentioned above, important relevant
negatives include:
Begin with a summary of Hepatomegaly and/or signs of chronic liver
positive findings disease. Remember that, in the absence of
Middle aged patient with a fullness of the liver disease, jaundice may occur as a result
abdomen and a mass on palpation extending of haemolysis
to the right iliac fossa, with palpable notch, Signs of portal hypertension
moving with inspiration and dull to percussion Lymphadenopathy
consistent with splenomegaly. The patient may
be cachectic with signs of anaemia and plethoric State the most likely diagnosis
facies.
Associated findings commonly include on the basis of these findings
hepatomegaly. Where splenomegaly is The patient has signs consistent with isolated
encountered, given its strong correlation with splenomegaly.
liver disease it is important to exclude the
manifestation of portal hypertension heralding
cirrhosis. Where no signs of chronic liver disease
Offer relevant differential
are found, paradoxically splenomegaly will diagnoses
then more often than not be associated with With regard its various causes, splenomegaly
hepatomegaly and lymphadenopathy and and the differential diagnosis can be categorised
represent a haematological disorder or infectious according to size.
disease. Jaundice in the absence of liver disease
may occur as the result of haemolysis.
164 Chapter 4 Abdominal system (station 1)

Massive splenomegaly recent travel. Consider retroviral status and risk


factors for liver disease.
Chronic myelocytic leukaemia less
In considering haematological disorders,
commonly associated with hepatomegaly or
enquire as to evidence of marrow failure,
lymphadenopathy
including bruising and fatigue associated with
Myelofibrosis more commonly associated
thrombocytopaenia and anaemia respectively.
with hepatomegaly
Bone pain may be secondary to increased
Less commonly in temperate climates but
haematopoiesis or marrow infiltration, whilst
more common in the tropics and worldwide,
joint pain may result from hypeuricaemia.
malaria and leishmaniasis (kala-azar)
A family history of haematological disorders,
autoimmune conditions or transplantation
Moderate splenomegaly with immunosuppression should be
Portal hypertension considered.
Lymphoproliferative disorder more
commonly associated with hepatomegaly
and lymphadenopathy such as non- Demonstrate an understanding
Hodgkins lymphoma or chronic lymphocytic of the value of further

leukaemia
Haemoglobinopathies, including
investigation
thalassaemias, and sickle cell disease Appropriate investigations include:
(although commonly associated with Blood tests. Exclude pancytopaenia
autoinfarction unless early in disease) secondary to the cellular sequestration
associated with hypersplenism. This
Mild splenomegaly predisposes to anaemia, infection
Myeloproliferative disorders, although and bleeding. Thus, a basic check of
generally considered as causes of only haemoglobin, platelets, white cell
mild splenomegaly, are worth mentioning differential, inflammatory markers, and
to demonstrate an appreciation for the clotting is essential
spectrum of disease: Imaging. Ultrasound will serve to confirm
Polycythaemia rubra vera splenomegaly and assess the liver for
Essential thrombocythaemia hepatomegaly and allow duplex of portal
venous flow. Thereafter, CT or PET scanning
Infiltrative, connective tissue disease and may be required to assess degree of
storage disorders should also be considered in associated lymphadenopathy. MRI may
the differential however given the prevalence be necessary to identify portal or splenic
of haematological and infectious causes it is thromboses
sensible to clearly and comprehensively deal
with these before offering a brief synopsis of Other investigations:
these rarer causes: Thick and thin blood films if clinical
Infiltrative/inflammatory: sarcoidosis, suspicion of malaria
amyloidosis Blood film with or without progression to
Connective tissue: SLE, Felty's syndrome bone marrow biopsy
Storage disorders: Gaucher's, Langerhans cell Lymph node biopsy if concomitant
histiocytosis lymphadenopathy

Demonstrate the importance Always offer a management plan


of clinical context suggest In the case of isolated splenomegaly the
relevant questions that would be discussion with the examiner on management
may be directed towards a number of possible
taken in a patient history underlying diagnoses. For example, the
Screen for constitutional symptoms of anaemia, examiner may direct the conversation towards
infection or malignancy (B-symptoms) myeloproliferative disorders, particularly
including weight loss, lethargy, night sweats and CML.
Case 92: Feltys syndrome

Instruction to the candidate Symptoms of a long preceding period of


aggressive and poorly controlled RA
This 45-year-old woman has been referred from
the rheumatology clinic, where she was being
Weight loss is common
assessed for painful hands. Please examine their
abdomen. Demonstrate an understanding of
the value of further investigation
Begin with a summary of Feltys syndrome is characterised by the triad of
positive findings RA, splenomegaly and granulocytopaenia with
resultant susceptibility to overwhelming, often
A patient with Feltys syndrome will have signs
fatal, infections. Investigation should be directed
both in the hands and in the abdomen:
to confirm splenomegaly, assess the rheumatoid
In the hands: severe, symmetrical, arthritis and importantly establish the immune
deforming polyarthropathy, associated status of the patient.
with extra-articular manifestations of
rheumatoid arthritis, e.g. rheumatoid
Blood tests:
nodules
Full blood count and inflammatory
markers with white cell count
In the abdomen: splenomegaly
differential to assess the severity of
granulocytopaenia. Anaemia and
Follow with a summary of thrombocytopaenia are common due to
hypersplenism. ESR will be raised
relevant negative findings Anti-CCP and rheumatoid factor
It is important to exclude signs of current Anti-GCSF. Felty's syndrome has been
infection (this is important due to the finding of linked to the presence of these auto-
neutropaenia in Feltys syndrome). antibodies, possibly playing a role in the
pathology of granulocytopaenia together
State the most likely diagnosis with splenic sequestration
Imaging:
on the basis of these findings USS and CT to confirm splenomegaly and
In a patient with extensive RA and palpable assess hepatomegaly where present
spleen, a diagnosis of Feltys syndrome should Plain film radiographs of affected joints
be considered and confirmed by a finding of demonstrating destruction and loss of
neutropenia. joint space
Tissue
Offer relevant differential Bone marrow aspirate

diagnoses
Always offer a management plan
Of splenomegaly:
Management is multi-disciplinary, involving
Portal hypertension contribution from a rheumatologist, haematologist
Haematological and occasionally an infectious diseases specialist.
Infectious Focus upon achieving control of RA (see
Infiltrative/inflammatory Symmetrical deforming polyarthropathy p. 172).
Connective tissue disease Rituximab (anti-CD20) is establishing a role
Storage disorders in the treatment of Feltys.
Colony-stimulating factors, G-CSF
and GM-CSF are increasingly used where
Demonstrate the importance management of RA is poor and recurrent
of clinical context suggest infections fail to respond to antibiotics.
relevant questions that would Splenectomy in cases refractory to
improvement with medical treatment or
be taken in a patient history experiencing recurrent infections. 25% of
Pertinent questions to consider include: patients after splenectomy experience further
relapses of granulocytopaenia.
Case 93: Multiple abdominal scars

Instruction to the candidate to include other medical conditions which


may require extensive abdominal surgery
Please examine the abdomen of this 22-year-old
including complications of chronic pancreatitis,
woman who has had surgery in the past.
end-stage liver failure with transplantation
where a roof top incision is present, or with
Begin with a summary of midline laparotomy emergency access for
gastrointestinal perforation or vascular
positive findings complications such as abdominal aortic
Multiple abdominal scars, of varying age, in aneurysm rupture and mesenteric ischaemia.
a young cachectic patient should raise the The differential diagnosis of inflammatory
possibility of inflammatory bowel disease. bowel disease Crohns disease and ulcerative
Associated signs include: colitis includes:
Nutritional state growth retardation and Microscopic colitis
muscle wasting (particularly pterygoids and Colorectal cancer
triceps) Coelic disease
Anaemia tongue, palmar creases, Gastrointestinal infections (e.g. TB, Yersinia,
conjunctival pallor. Anaemia may be present amoebiasis, CMV)
from: Behets disease
Chronic disease Ischaemic colitis
Iron deficiency Radiation-induced colitis
B12/folate deficiency NSAID enteropathy
Autoimmune haemolysis Functional bowel disorders
Clubbing (Figure 4.1 and Figure 4.2)
Apthous ulceration, stomatitis, telangiectasia,
pigmentation Demonstrate the importance
Erythema nodosum or pyoderma of clinical context suggest
gangrenosum
Perianal disease abscess, fistulae, skin tags
relevant questions that would be
Metabolic bone disease with mobility aids or taken in a patient history
kyphoscoliosis from osteoporotic fracture Clinical questioning would seek to elicit a
Uveitis, iritis, episcleritis history of relapsing and remitting abdominal
pain, diarrhoea, with or without mucus and
Follow with a summary of blood.
Additional questioning would involve
relevant negative findings joint or back pain and difficulty with mobility,
Relevant negative findings include: which may suggest an underlying seronegative
arthropathy.
Abdominal pain or discomfort consistent
with active disease
Arthropathy (seronegative) Demonstrate an understanding
of the value of further
State the most likely diagnosis investigation
on the basis of these findings To support a diagnosis:
Multiple surgical interventions for underlying Blood tests:
inflammatory bowel disease. Full blood count for anaemia,
thrombocytosis
Offer relevant differential Inflammatory markers (CRP and ESR)
diagnoses Screen for electrolyte abnormalities
(including U+Es, Ca2+, PO4, vitamin B12
When considering the differential diagnosis and D)
of multiple abdominal scars it is prudent LFTs, including albumin
Case 94 Rooftop incision 167

Imaging: Maintenance therapies include:


Plain radiographs of (erect) chest and 5-ASA sulfasalazine
abdomen
Barium follow-through assessing Corticosteroids
stomach, small bowel and colon with
Steroid-sparing agents
additional plain radiographs The management of acute flares includes:
MRI enterogrpaphy
Endoscopy: sigmoidoscopy or colonoscopy Fluid balance management
with direct visualisation and biopsy Topical +/ systemic corticosteroids
Stool sample to exclude infective aetiology Ciclosporin (steroid unresponsive)
Faecal calprotectin Anti-TNF infliximab or adalumimab
considered in disease refractory to standard
To monitor disease progression, the patients treatment where no evidence of infection
symptoms are backed up by inflammatory exists
markers and markers of nutrition. There is no
role for imaging in routing monitoring of disease Complications requiring specific intervention
progression, however some radiological signs do include:
indicate severe and/or long-standing disease, Obstruction treated with nasogastric tube
such as a lead-pipe colon in long-standing UC. and intravenous fluid. Surgery may be
indicated, see below
Always offer a management plan Abscess formation requiring incision and
drainage or percutaneous drainage with
The treatment of Crohns and ulcerative colitis
cover of intravenous antibiotics
is similar and the importance of the distinction
between the two only arises from a management
Fistulae are treated with antibiotics
(including anaerobic cover) and
perspective when considering surgical
consideration of infliximab
intervention or experimental therapy.

Case 94: Rooftop incision

Instruction to the candidate Associated signs:


Please examine this 51-year-old mans abdominal Palpable liver edge
system and comment on his abdominal scar. Signs suggesting chronic liver disease
Evidence of renal insufficiency, or renal
replacement therapy
Begin with a summary of
positive findings
Follow with a summary of
When encountering a roof-top incision in
PACES it will commonly represent a liver relevant negative findings
transplant. The strong candidate will consider It is important to note the absence of signs of
the alternative indications for such a surgical hepatic decompensation.
approach, outlined in the differential diagnosis
that follows, but the focus will be placed upon
establishing the aetiology of liver disease
State the most likely diagnosis
necessitating transplantation, examining on the basis of these findings
for evidence of recurrence, and ruling out Where the scar is the predominant finding,
signs consistent with decompensation, and lead with this in a succinct summary of positive
complications of immunosuppression. and negative findings. Indicate to the examiner
168 Chapter 4 Abdominal system (station 1)

that you believe this would be consistent Demonstrate an understanding of


with a surgical approach to the liver, of which
transplantation is a possibility before discussing the value of further investigation
the differential as outlined below. Investigation may focus on establishing the
indications for transplantation, discussed below
in management, or on the post-transplant patient.
Offer relevant differential To assess the function of the graft, routine
diagnoses liver function including synthetic function with
The differential diagnosis of the underlying albumin and clotting. Formally assess for signs
indication for surgical intervention, and thus a of encephalopathy.
roof top incision, include: Inflammatory markers including white cell
count and differential, particularly in view of
Liver transplantation immunosuppression. Emphasise the potential
Extensive hepatic resection with curative for renal insufficiency, with the need for routine
intent for neoplastic disease testing of renal function.
Gastrectomy Where there is doubt with regard to the
Oesophageal cancer Ivor-Lewis diagnosis, a CT would better evaluate the
oesophagectomy, usually an abdominal and abdominal anatomy and where there was
right thoracic approach suspicion of post-operative complications such
Whipples procedure as infection, collection, anastamotic leak, or
Having established liver transplantation stricture, the threshold for performing such
as your principle diagnosis, consider the imaging would be low.
common causes of liver failure necessitating
transplantation in attempting to offer a likely Indication for liver transplantation -
cause in this case followed by a differential.
Kings College criteria
An example of an acute cause is fulminant
hepatic necrosis, which may be due to: The Kings College criteria determine prognosis
in acute liver failure and stratify patients based on
Viral hepatitis
early parameters to identify those in whom a liver
Toxins death cap mushroom
transplant would be beneficial or inevitable.
Over-the-counter and recreational
Non-acetaminophen induced:
drugs paracetamol, ecstasy
INR >6.5, or
Chronic causes include: Three of:
Age less than 11 years or greater than 40 years
Cirrhosis Serum bilirubin >300 mol/L
Primary biliary cirrhosis Time from onset of jaundice to development
Autoimmune hepatitis of coma more than 7 days
Primary sclerosing cholangitis INR >3.5
Metabolic disease: haemachromatosis, Drug toxicity, regardless of whether it was the
Wilson's disease, storage disorders cause of acute liver failure
Hepatocellular carcinoma Milan criteria: Liver transplantation in chronic liver disease is
single tumour less than 5 cm or up to three generally considered where there is evidence of
tumours each less than 3 cm end stage cirrhosis.
Biliary atresia in children

Always offer a
Demonstrate the importance
management plan
of clinical context suggest This may involve discussion of treatment
relevant questions that would be options, prior to transplantation, of the
taken in a patient history underlying cause.
In the transplant patient, there is a need
Review past medical history of liver disease,
for immunosuppression. Commonly, one or
symptoms and disease progression necessitating
combination of:
transplantation. Screen for evidence of disease
recurrence or transplant failure, enquire Calcineurin inhibitor
specifically with regard hepatitis C. Enquire as to Mycophenilate mofetil
medication regimen and compliance. Corticosteroids
Case 95 Obesity with evidence of laparoscopic procedure 169

Regular review and surveillance for rejection Raised intracranial pressure or low cerebral
with tapering of immunosuppression where perfusion pressure
possible. Severe cardiopulmonary disease including
Be prepared to discuss the indications for pulmonary hypertension
transplantation, both for acute and chronic liver Extrahepatic malignancy
failure. AIDS (HIV infection is a relative
Important absolute contraindications to liver contraindication)
transplantation include: Non-compliance with medication

Case 95: Obesity with evidence of


laparoscopic procedure

Instruction to the candidate laparoscopic surgery consistent with possible


bariatric intervention.
This 44-year-old woman has had abdominal
surgery. Please examine her abdomen and
comment on her abdominal scars. Offer relevant differential
diagnoses
Begin with a summary of Of obesity, centripetal, or abdominal fullness:
positive findings Cushingoid
Obese patient, abdominal striae with Hypothyroidism
increased sweat and skin secretions. Excess Ascites
skin folds indicative of rapid weight loss. Scars
Of the laparoscopic procedure:
consistent with laparoscopic port insertion,
looking closely in the peri-umbilical area and Cholecystectomy
flanks. Bariatric surgery restrictive procedure
(gastric banding or sleeve gastrectomy) or
Follow with a summary of bypass procedure (Roux-en-Y gastric bypass)
Nissens fundoplication
relevant negative findings
Conditions commonly related to obesity:
Demonstrate the importance
Evidence of diabetes finger pulp of clinical context suggest
ecchymoses, abdominal bruising and
lipodystrophy. relevant questions that would be
Ischaemic heart disease scars consistent taken in a patient history
with coronary bypass grafting.Thoracotomy Past medical history including childhood
with or without venous harvesting obesity and specifically diseases related to
Hepatomegaly where examination and obesity including cardiac disease, diabetes and
palpation not limited by habitus, think of depression or mental ill health.
fatty liver
Signs of excess lipids xanthelasma Enquire about reflux symptoms and biliary
colic
Rule out recent pregnancy associated weight
State the most likely diagnosis gain and rapid loss
on the basis of these findings Full drug history screening for medication
with weight gain as side effect:
Obesity commenting on distribution, evidence Corticosteroids
of weight loss and with scars of previous
170 Chapter 4 Abdominal system (station 1)

Antidepressants Always offer a management plan


Antipsychotics Of obesity:
Dietary and exercise advice
Demonstrate an understanding Medication orlistat, GLP-1 receptor agonists
of the value of further in those with diabetes
Cognitive behavioural therapy
investigation Bariatric surgery
Body mass index (BMI) weight in kilograms
Most bariatric surgery is performed
divided by the square of height in metres.
laparoscopically. It is generally indicated where
Waist circumference: It is prudent to
all reasonable efforts have been made to exhaust
investigate the possibility of metabolic
alternative methods of weight loss and where
syndrome; a clustering of risk factors for the
there are no concerns about compliance with
development of severe ischaemic cardiac
subsequent dietary requirements, mental health
disease and type 2 diabetes:
concerns or substance misuse. A BMI > 40, or
Insulin resistance measure fasting >35 with a serious complication such as diabetes
glucose or ischaemic heart disease, represents objective
Dyslipidaemia with elevated triglycerides, indications.
low HDL and increased intra-abdominal Metabolic risk factors should be treated
adiposity aggressively with diet and exercise advice,
Hypertension metformin where appropriate and optimisation
of modifiable cardiac risk factors with
In unexplained obesity consider the need to hypertension control and a low threshold for
rule out rare causes of weight gain including: statin use.
Hyperinsulinism due to pancreatic cancer
Craniopharyngioma
Chapter 5

Station 5
Case 96: Rheumatoid arthritis

Instruction to the candidate Positive findings of a focused


This 70-year-old patient has been experiencing examination
increasingly painful hands over the last 2 Always consider the benefit of initiating the
years. Please take a short history and perform examination during the history to maximise the
a relevant focused examination. Summarise consultation. During the history be confident in
your findings to the patient and explain asking to examine the hands and encourage the
your investigation and management plan. A patient to keep talking as you do so.
discussion with the examiner will then follow. When examining the hands, assess for both
deformity and for a deficit in function.
Take a focused history
Utilise open questioning to ascertain the nature Deformity
of the problem in the patients own words. Positive findings (Figure 5.1) include
Leading with a statement such as I understand symmetrical MCP and PIP involvement.
you have been experiencing some problems Tender joints, with subluxation and ulnar
with your hands? Can you tell me some more deviation. Boutonniere and swan neck
about that while open, will keep the patient on deformity are clearly demonstrable. Z-shaping
track with a focused approach and avoid initial of the thumb with 1st and 2nd MCP most
deviation from the station instruction. markedly involved. There are rheumatoid
Specific, closed questioning should cover nodules at the elbows.
both a review of hand symptoms and include
relevant systemic features of rheumatological Function
disease, ensuring to consider the importance of
Examine for difficulty unbuttoning their shirt,
any co-morbidity.
removing their watch and/or picking up a coin
In the hands, consider:
from a flat surface.
Nature of the pain Consider further examination:
Timing and duration of stiffness
To assess the extent of joint involvement
Pattern of joint involvement
large joint examination and gait examination
Associated skin changes
To evaluate systemic disease/complications
Look to specifically consider systemic of treatment such as fibrosis respiratory
features of rheumatological disease: examination
To exclude known associations such as Feltys
Eye involvement dry eyes, change in syndrome with splenomegaly abdominal
appearance (the red eye), or changes in examination
visual acuity
Lung disease shortness of breath, cough,
and haemoptysis Key clinical findings in rheumatoid
Renal involvement manifesting as lower arthritis
limb oedema or potential changes in
frequency of urination or qualitative changes History:
to the urine itself (foamy or frothy) Multiple joints
Morning stiffness
Evidence in the past medical and family Duration >6 weeks
history of a tendency towards autoimmune Effect on daily life
disorders offers important context to the case Examination:
and should not be overlooked. Thereafter Bilateral joint involvement
screen medication, specifically the use of MCP subluxation
non-steroidals in the first instance. Finally, Ulnar deviation
assess the effect on the patients daily life being Rheumatoid nodules
alert to the likelihood of depression in view of
the chronic nature of the symptoms.
Case 96 Rheumatoid arthritis 173

Relevant negatives In the meantime, I will prescribe you tablets for


pain relief.
See Table 5.1 for a summary of relevant negative
I will be writing to your GP to inform them of
findings to elicit in a case of rheumatoid
your visit today, asking them to refer you to the
arthritis. It will be important to be seen to
physiotherapist and occupational therapy team
consider alternative aetiologies, such as
who can assess your needs further.
osteoarthritis, psoariatic arthropathy, and gout.
Do you have any further concerns of
questions I can address today?
What to tell the patient
Consider the following approach: What to tell the examiner state
The pain and stiffness you describe and
the changes to the joints in your hands suggest
the most likely diagnosis on the
that you have a condition called rheumatoid basis of the history and clinical
arthritis. To confirm this, we will need to do findings
some blood tests and imaging (X-rays) of the
affected joints. Rheumatoid arthritis is usually This patient describes early morning stiffness
treated with medications both to relieve pain and pain affecting predominantly the knees and
and to stop further progression of the disease. hands. Examination confirmed a symmetrical
I would like to bring you back to clinic in 6 deforming polyarthropathy consistent with a
weeks with the results of these investigations. diagnosis of rheumatoid arthritis.

Figure 5.1 Rheumatoid hands.


The right hand of a patient with
rheumatoid arthritis showing ulnar
deviation with subluxation of the
metacarpophalangeal joints. There is
a Boutonierre deformity of the ring
finger.

Table 5.1 Relevant negative findings in rheumatoid arthritis. The findings in the joints,
skin and nails are important as they might indicate an alternate diagnosis
Joints Skin and nails Extra-articular
DIP joint involvement (OA) Erythema (psoriasis) Dry eyes/mouth (Sjogrens)

Tophi (gout) Psoriatic plaques (psoriasis) Lung fibrosis (extra-articular


disease or adverse effect of
treatment for RA - methotrexate)
Telescoping (psoriasis) Nail pitting (psoriasis) Splenomegaly (Feltys)

DIP, distal interphalangeal.


174 Chapter 5 Station 5

Imaging
British Society for Rheumatology
Plain radiographs of all affected joints should
guidelines be obtained looking for loss of joint space and
A diagnosis of RA should be made as early periarticular erosions.
as possible, on the basis of persistent joint
inflammation affecting at least three joint areas,
involvement of the metacarpophalangeal or
Always offer a management plan
metatarsophalangeal joints or early morning The management of rheumatoid arthritis
stiffness of at least 30 minutes duration. involves symptom control, arresting disease
progression, and continual disease monitoring.
Symptom control:
Offer relevant differential
Analgesia, NSAIDs
diagnoses
Of a symmetrical deforming polyarthropathy: Arrest disease progression:

Rheumatoid arthritis Early initiation of DMARD combination


Psoriatic arthropathy, which would be therapy
suggested by nail pitting and psoriatic Corticosteroids for flares
plaques Biologics in active disease with disease
Jaccouds arthropathy, which is suggested activity score (DAS-28) >3.2 after a trial of two
when, despite deformity, the patient is able or more DMARDs (including methotrexate
to make a fist, reflecting a lack of true joint unless contraindicated)
destruction Disease monitoring: DAS-28 to monitor
response to treatment.
Demonstrate an understanding Thereafter, offer management plans for any
functional impairment the patient may have:
of the value of further
investigation Involve physiotherapy and occupational
therapy early to help the patient adapt to any
Investigations should seek to reinforce the disability and maintain as much function as
clinical findings and support a diagnosis of is possible
rheumatoid arthritis. Diagnostic criteria are Surgical intervention release of tendon
available, but these are primarily for research strictures
purposes.

Blood tests Further reading


Luqmani R, Hennell S, Estrach C, et al. British
Serology: rheumatoid factor; anti-CCP
Society for Rheumatology and British Health
antibodies
Professionals in Rheumatology guideline for
Full blood count: anaemia is common. The
the management of rheumatoid arthritis (after
strong candidate should recognise that
the first 2 years). Rheumatology (Oxford) 2009;
there is a multitude of causes of anaemia
48:436439.
in RA. The patient may have anaemia
National Institute for Health and Care Excellence
of chronic disease due to the RA itself.
(NICE). CG79, Rheumatoid arthritis: NICE
Alternatively, they may have a macrocytic
guideline. London; NICE, 2009.
anaemia due to methotrexate use, or a
Deighton C, Hyrich K, Ding T, et al. BSR and BHPR
microcytic anaemia due to iron-deficiency
rheumatoid arthritis guidelines on eligibility
secondary to NSAID use. Further, in Feltys
criteria for the first biological therapy.
syndrome, they may have anaemia due to
Rheumatology (Oxford) 2010; 49:11971199.
splenic sequestration
Inflammatory markers: ESR, CRP
Case 97: Symmetrical deforming
polyarthropathypsoriatic arthritis

Instruction to the candidate mutilans with subluxation and ulnar


deviation at the MCP joints. Dactylitis with
This 40-year-old man has presented with pain
sausage digits is another possible clinical
and deformity in his hands. Please take a brief
feature of this condition
history and perform a focused examination.
When you have finished your assessment,
Function: assess function by asking the
patient to unbutton their shirt or to pick a
inform the patient of your findings and suggest a
coin off a flat surface
suitable management plan.
Consider further examination for additional
features such as:
Take a focused history
A brief history should include both specific Enthesitis: achilles tendon attachment or
questions regarding hand symptoms and plantar fascia to the calcaneus
also discriminative questions to distinguish Skin rash: symmetrical pink scaly plaques
between the possibility of rheumatoid and on extensor surfaces of elbows or knees
psoriatic arthropathy. Thus, in addition to asking (Figure 5.2)
questions focussing on hand symptoms such Nail signs: marked pitting of the nails
as pain, stiffness and reduced mobility, also affecting both hands or feet and pale pink
enquire about: discolouration of the nail beds. Onycholysis
or oil spots (pathognomonic).
The presence of a rash, including its
appearance and distribution. If the patient
does not have psoriatic plaques currently, is Relevant negatives
there a past medical history of psoriasis? Although the diagnosis of psoriatic arthropathy
Is there nail deformity? may be obvious in the setting of symmetrical
Is there a family history of either rheumatoid deforming polyarthropathy and psoriatic
arthritis or psoriasis? plaques, seek to exclude findings suggestive
Is there a history of spondyloarthropathy or of alternative diagnoses such as rheumatoid
systemic disease such as inflammatory bowel nodules or gouty tophi.
disease? Psoriatic arthropathy is one of the
seronegative spondyloarthropathies. The
others are: ankylosing spondylitis, Reiter's
What to tell the patient
syndrome, enteropathic arthritis The pattern of joint disease in the hands and
the plaques on your elbows suggest you have a
During the brief history, ensure to ascertain condition called psoriatic arthropathy. We will
function and the effect of the patients symptoms need to carry out some investigations such as
on their daily life. Enquire about common X-rays of the hands and some blood tests. This
daily activities such as opening the front door, is not a dangerous condition, and there are
opening jars of food, dressing. What can the many strategies we can implement to help you
patient not do because of their condition? What maximise hand function.
effect is this having on their daily life? Following this, ask the patient if they have
any further questions or concerns that you can
Positive findings of a focused address. Tell them that you will see them again
in clinic after an appropriate interval.
examination
Hand examination and assessment of skin -
when examining the hands, assess for both
What to tell the examiner state
deformity and for a functional deficit: the most likely diagnosis on the
Deformity: positive findings may include basis of the history and clinical
asymmetrical oligoarticular arthritis, findings
symmetrical polyarthritis (commonest type
A diagnosis of psoriatic arthropathy is suggested
and similar to rheumatoid arthritis), distal
by symmetrical joint involvement, dactylitis,
interphalangeal involvement, or arthritis
the absence of rheumatoid nodules, and DIP
176 Chapter 5 Station 5

Figure 5.2 Sites of plaques in


psoriasis. The main sites of plaques
found in chronic plauque psoriasis,
namely the extensor aspects of the
elbows and knees, the scalp and
behind the ears.

involvement in the absence of osteoarthritis. Clinical and radiological criteria are normally
This is most consistent with a symmetrical used. In blood testing, inflammatory
deforming polyarthropathy; however, alternative markers such as ESR and CRP may be raised.
presentations of psoriatic arthropathy also Rheumatoid factor is usually negative although
include: may be positive in a small number of cases.
Testing for ANA is of no diagnostic benefit.
Arthritis mutilans Plain radiographs of the hands are used to
Predominance for DIPS demonstrate bony erosive disease. In contrast to
Large joint oligoarthropathy (asymmetrical) rheumatoid arthritis, there is minimal juxta-
Spondyloarthropathy articular osteopenia.

Offer relevant differential Always offer a


diagnoses management plan
The differential diagnosis of a symmetrical Management of the disease itself involves
deforming polyarthropathy includes rheumatoid symptom control, with simple analgesia and
arthritis, psoriatic arthropathy and Jaccouds NSAIDs, and the arrest of disease progression,
arthropathy. with DMARDs and biologic treatments.
Psoriatic arthropathy can also present as Management of associated manifestations
an asymmetrical arthropathy, the differential includes topical therapies for skin disease.
diagnosis of which includes gout and Offer lifestyle advice especially with regards
osteoarthritis. to cardiovascular disease and diabetes which
are associated with chronic psoriasis. Multi-
Demonstrate an understanding of disciplinary input is important to address any
functional impairment the patient may have as a
the value of further investigation result of the condition.
No specific diagnostic tests are available.
Case 98: Jaccouds arthropathy

Instruction to the candidate to 50% of patients with SLE) and history of


photosensitivity
A 40-year-old woman who has noticed
deformity of her hands over the last 6 months.
Alopecia and livedo reticularis
Please take a brief history and perform a
Mouth ulceration
focused examination. Discuss your findings and
A cardiorespiratory examination may detect
pleural or pericardial effusions if present
management plan with the patient.

Take a focused history Relevant negatives


Important relevant negative findings include:
As with other cases of symmetrical
polyarthropathy, screen for symptoms of pain The absence of rheumatoid nodules or
and stiffness. The patient may have symptoms psoriatic plaques
ranging from painless arthropathy to pain that A unilateral swollen joint out of proportion to
is disproportionate to joint swelling. There may others may suggest a reactive arthritis, or, in
be associated myalgia. In addition to the small the presence of a fever, would raise suspicion
joints of the hands, there may be involvement of of septic arthritis
the wrists and knees. Avascular necrosis in patients taking
Screen for systemic rheumatological corticosteroids
disease. The tendency in a case of symmetrical
deforming polyarthropathy will be to focus
on rheumatoid arthritis, however this case
What to tell the patient
highlights the importance of considering The changes you have noticed in your hands
alternatives such as systemic lupus together with the other symptoms you have
erythematosus (SLE). The symptoms of SLE may mentioned suggest you have a condition called
be vague due to the widespread nature of the Jaccouds arthropathy, which is often seen
disease with involvement of a range of organs/ in the context of a systemic disease called
systems. Consider: systemic lupus erythematosis,
To confirm this, we will need to run some
Constitutional symptoms of fever, general tests, including blood tests and X-rays of the
malaise, weight loss affected joints. Jaccouds arthropathy is usually
Skin involvement with characteristic rash, managed with medications to relieve pain but
either malar or discoid in distribution the underlying condition, often needs to be
Current or past medical history of renal, treated too with systemic medication,
neuropsychiatric, pulmonary, cardiac or I would like to bring you back to clinic in 6
gastrointestinal problems. Perhaps best weeks time with the results of the tests. In the
achieved through a review of symptoms meantime, I will prescribe you some tablets for
pain relief if it is still a problem. I would advise
Positive findings of a focused you wear sunscreen and avoid sun exposure to
limit the extent of the rash that you have,
examination I will also be writing to your GP to inform
Hand examination and skin assessment. him of your visit today. Do you have any
With regard to deformity, positive findings questions or concerns that I can address today?
include a symmetrical polyarthropathy with
PIP or MCP joint involvement. Tendonitis may
be present. Look for tenderness, oedema and
What to tell the examiner state
effusions. the most likely diagnosis on the
A key finding is resolution of the apparent basis of the history and clinical
ulnar deviation on successful formation of a
clenched fist. Function is usually preserved. findings
Additional findings: This patient demonstrates symmetrical
deformation of the metacarpophalangeal
Skin manifestations rashes (malar eruption
joints that is reversible with movement. These
or butterfly rash as Jaccouds occurs in up
examination signs and history of systemic
178 Chapter 5 Station 5

lupus erythematosus make the diagnosis that of


essentially a single clinical examination technique
Jaccouds arthropathy.
that will change your diagnosis from rheumatoid
arthritis to Jaccouds. The station 5 format, with
What is Jaccouds arthropathy? elements of both history taking and clinical
examination, makes the differential slightly easier
Jaccouds arthropathy is a non-erosive, reversible because there are also marked differences in the
deforming arthropathy with dislocation of the history to help the candidate.
extensor tendons in the metacarpal fossa. In
modern clinical practice it is most commonly
associated with SLE however it has previously Demonstrate an understanding
been described in a range of conditions including of the value of further
rheumatic heart disease. It was in patients with
rheumatic heart disease that the French physician investigation
Jaccoud initially described this type of arthropathy. The investigative strategy of a suspected
Jaccouds arthropathy case would focus on the
following:
Offer relevant differential
diagnoses Plain radiographs of the hands confirming
the absence of erosive disease
In addition to Jaccouds arthropathy, the Blood tests for supportive evidence of SLE
differential diagnosis of a symmetrical including ANA, double stranded DNA
deforming polyarthropathy includes rheumatoid antibodies (anti-dsDNA) and complement,
arthritis and psoriatic arthropathy. When looking for reduced levels of C3 and C4
considering the key clinical differences between
rheumatoid arthritis and Jaccouds, consider
that the features of rheumatoid arthritis Always offer a management plan
include pain and stiffness, with an irreversible The management of this case will not focus
deformity and resulting functional impairment. on the hands themselves, but rather on the
Other joints may also be involved. Jaccouds management of the underlying SLE and its
arthropathy, however, is often painless with no complications. Briefly, management is usually
joint stiffness. There is a reversible deformity conservative and involves NSAIDs, low dose
with often no functional deficit. There is no other corticosteroids as well as methotrexate and
joint involvement, but the patient may have antimalarials in more severe cases.
other clinical features of SLE.
When considering the differential diagnosis
of the causes of Jaccouds arthropathy, consider Further reading
that, in addition to SLE, it can also be caused by Grahame R, Mitchell AB, Scott JT. Chronic post-
rheumatic heart disease, systemic sclerosis and rheumatic fever (Jaccouds) arthropathy. Ann
sarcoidosis. Rheum Dis 1970; 29:622625.
Bradley JD, Pinals RS. Jaccouds arthropathy
in scleroderma. Clin Exp Rheumatol 1984;
Jaccouds arthropathy 2:337340.
Jaccouds place in MRCP PACES is that of a Sukenik S, Hendler N, Yerushalmi B, Buskila D,
differential of rheumatoid arthritis. It is a case that Liberman N. Jaccouds type arthropathy: an
can easily catch out the PACES candidate, yet it is association with sarcoidosis. J Rheumatol 1991;
18:915917.
Case 99: Osteoarthritis

Instruction to the candidate On examination of the hands, the distal


interphalangeal (DIP) joints are predominantly
This 92-year-old woman has had difficulty
affected (Figure 5.3) with MCP sparing evident.
opening jars at home. Please take a brief history
Expect a reduced range of joint motion and
and examine as appropriate.
crepitus. Swellings of the distal and proximal
interphalangeal joints are noticeable, consistent
Take a focused history with Heberdens and Bouchards nodes,
Screen for symptoms of pain and stiffness: respectively.

Pain is usually worse with use and relieved


Relevant negatives
by rest
In osteoarthritis, the joints become stiff when Specific relevant negatives would include:
rested (gelling). In contrast to rheumatoid
arthritis, morning stiffness usually lasts less
No erythema or warmth over affected joints
than 30 minutes
Absent inflammatory changes
Absence of rheumatoid nodules, tophi or
Other important points to address include: psoriatic plaques (unless co-existent disease)
Is there other joint involvement, e.g. in the
knees and hips? Have they had any lower What to tell the patient
limb joint operations? How does the patient The symptoms you are describing today
mobilise? together with the changes to the joints suggest
What is the effect on function and daily that you have a condition called osteoarthritis.
living? Establish functional limitations such This is the commonest degenerative joint
as the ability to dress independently, wash, condition, commonly described as wear and
as related to turning taps, and prepare food tear of the joints. The diagnosis is usually
linked to the use of kitchen utensils. Be sure made on a clinical basis without the need for
to make reference to specific functional further tests although sometimes X-rays can be
limitations as outlined in the station helpful. The mainstay of management is through
instruction analgesia and physiotherapy. In severe cases of
osteoarthritis of the knees or the hips, surgical
Positive findings of a focused options might be explored, in the form of joint
replacement surgery.
examination I would like to prescribe you some tablets for
Hand examination followed by assessment of pain relief and refer you to the physiotherapy
large joints and/or gait. department for an assessment.

Figure 5.3 Osteoarthritis. This


patient has osteoarthritis in both
hands. Note the predominance
of distal interphalangeal joint
involvement with Heberdens nodes
present on both forefingers and
the middle finger of the patients
left hand.
180 Chapter 5 Station 5

I would then like to bring you back in a few Always offer a management plan
months time to see how you are getting on. Do
Symptom control includes pain alleviation with
you have any questions or concerns that I can
simple analgesia, NSAIDs (but caution in using
address today?
NSAIDs in older patients). A pain team referral
may be necessary in severe and/or uncontrolled
What to tell the examiner state symptoms.
the most likely diagnosis on the Note thatalthough intra-articular
corticosteroid injections and surgical options
basis of the history and clinical (arthroscopy/joint replacement) are features of
findings the management of osteoarthritis, in practical
terms they do not apply to disease affecting the
This patient has symptoms and signs consistent
joints of the hand. Therefore, if you mention
with a diagnosis of osteoarthritis.
them to the examiner in your discussion of
management options, make sure you discuss
Offer relevant differential them in the appropriate context.
diagnoses Chondroitin and glucosamine supplements
are commonly used by patients with the aim
The differential diagnosis includes the following: of alleviating symptoms in osteoarthritis.
Psoriatic arthropathy (associated skin There is, however, considerable controversy
lesions) surrounding their use. A recent meta-analysis of
Rheumatoid arthritis (rarely involves DIP, 10 clinical trials involving 3803 patients found
prolonged morning stiffness) that compared with placebo, glucosamine,
Crystal arthropathies (usually monoarticular, chondroitin and their combination do not
big toe or knee) reduce joint pain or have an impact on
narrowing of joint space in osteoarthritis of the
hip and knee. The MRCP PACES candidate would
Demonstrate an understanding do well to avoid getting bogged down in the
of the value of further arguments. When presenting to the examiner,
a simple statement such as: Chondroitin and
investigation glucosamine are often used in the management
With a good clinical diagnosis of OA, necessary of osteoarthritis, however their use remains
investigations are limited. Plain radiographs controversial should suffice.
of affected joints are needed, but little else. A multi-disciplinary approach is important to
Radiographic features of osteoarthritis include: maximise functional capacity.
Loss of joint space
Osteophyte formation Further reading
Sub-chondral sclerosis Wandel S, Jni P, Tendal B, et al. Effects of
Periosteal cysts glucosamine, chondroitin, or placebo in patients
with osteoarthritis of hip or knee: network meta-
analysis. Br Med J 2010; 341:c4675.
Case 100: Sclerodactyly/systemic sclerosis

Instruction to the candidate comment on potential complications of


systemic sclerosis. As such, a relevant negative
Please examine this 50-year-old woman who has
list may include, for instance, the absence of
noticed pain in her hands over the past year.
digital ulceration.

Take a focused history What to tell the patient


As the patient has presented with symptoms
The symptoms you describe together with the
of hand pain, start with an open question:
changes to your fingers and the rest of your
I understand you have pain in your hands,
skin point to the diagnosis of a condition called
can you tell me more about it? After that (in
systemic sclerosis. This is a condition that as
conjunction with inspection of the hands that
well as affecting the skin of the hands, can affect
confirms sclerodactyly), a focused history would
internal organs such as the lungs, oesophagus
include:
and kidneys. I would like to organise some tests
Symptoms of Raynauds, including whether to help confirm the diagnosis and to assess for
there is a relationship of the patients pain to any wider involvement of the organs I have just
cold and whether there is a classical pattern mentioned.
of finger skin colour change in the cold. Is The management of the condition would
there any relationship to medications such as depend on the results of the tests. There are
beta-blockers? Does the patient smoke? some general measures such as wearing gloves
Ask about other symptoms of systemic and avoiding exposure to cold that will help
sclerosis, such as problems with swallowing, with your hand symptoms. It is vital you stop
weight loss and/or exertional breathlessness smoking as it will greatly help your symptoms.
Does the patient have a confirmed diagnosis Additionally, I can prescribe some tablets that
of systemic sclerosis and if so what organ can help with the pain.
systems are involved. What investigations Do you have any further concerns or
have they had? Have they had any treatments questions that I can address today? I will be
in the past? seeing you again with the results of the tests in
68 weeks time.
Positive findings of a focused
examination What to tell the examiner state
Assessment of the hands should be followed the most likely diagnosis on the
by consideration of a focused respiratory basis of the history and clinical
examination. findings
Positive findings include sclerodactyly,
with shiny, tight skin on the fingers, associated The patient has signs and symptoms consistent
calcinosis or fingertip ulceration. The skin with systemic sclerosis.
changes may extend to the forearms but not
above the elbow. Offer relevant differential
Consider further examination for:
diagnoses
Nail changes capillary infarcts Given that sclerodactyly is often a spot
Facial features microstomia, facial diagnosis instead of offering a differential of the
telangiectasia characteristic nasal changes of diagnosis itself, offer a differential diagnosis of
systemic sclerosis (pinched or beak nose) one of the signs or symptoms you have elicited.
Respiratory system pulmonary fibrosis

Demonstrate an understanding
Relevant negatives
As this case is clearly that of systemic sclerosis,
of the value of further
relevant negatives should include other clinical investigation
signs and symptoms associated with systemic Investigations will include laboratory blood
sclerosis that are not present. In addition, tests. These may be aimed at pointing to the
182 Chapter 5 Station 5

underlying diagnosis (ANA, anti-centromere Always offer a management plan


antibodies, anti-Scl70) or at the effects of the
General lifestyle advice includes avoiding
disease process (a raised creatinine indicative
the cold and wearing gloves. The patient
of renal dysfunction; a full blood count which
should avoid medications that exacerbate
may demonstrate anaemia or a secondary
symptoms. Symptomatic relief includes
polycythaemia due to chronic hypoxia from
the use of calcium channel blockers and in
pulmonary fibrosis).
severe cases intravenous prostaglandins.
In the presence of respiratory symptoms,
Immunomodulatory strategies involve
investigations would include a plain chest
corticosteroids and methotrexate.
radiograph looking for reticulonodular
Management of complications includes:
shadowing, followed by a high-resolution CT
scan to further delineate the fibrosis. Lung Other treatment modalities include ACE
function test would seek to identify a restrictive inhibition in renal involvement
picture with raised diffusion factor. Oesophageal dysmotility is the main GI
In the presence of gastrointestinal symptoms complication of systemic sclerosis. It usually
an oesophageal-gastro-duodenoscopy (OGD) presents with dysphagia and weight loss.
with oesophageal manometry would assess for Diagnosis is made by OGD and oesophageal
oesophageal dysmotility. manometry showing disordered contraction
In the presence of abnormal renal function, of the oesophagus. Medical management
one would request a urine dipstick looking involves prokinetic agents such as
for proteinuria and haematuria and a full metoclopramide, along with proton pump
renal screen to rule out other causes of renal inhibitors
dysfunction. Nifedipine and prostacyclin may be helpful
in lung disease

Case 101: Gout

Instruction to the candidate Psoriasis


This 84-year-old man has been experiencing
Tumour lysis syndrome
recurrent attacks of pain in his forefinger. Please Take a focused social history asking about
take a brief history and focused examination. alcohol intake, specifically red wine, and intake
Report your findings to the patient and tell him of food rich in purines.
about your management plan.
Positive findings of a focused
Take a focused history examination
Ask specific questions about the presenting The hands are one of the less common sites for
symptom. Which joint is painful? Is there acute gout to occur. The MTP is the commonest
associated swelling and/or erythema? Ask about site. It is, however, possible for gout to occur in
the frequency of episodes? Is there an obvious the fingers and hands and this possibility is used
precipitant? here as a vehicle for discussion of gout. Perhaps
Take a drug history and a past medical more likely as a hand case is chronic tophaceous
history. Drugs that can precipitate gout include gout. This occurs when urate crystal deposits
thiazide diuretics and calcineurin inhibitors. form nodules around joints. The most common
Many diseases are known to increase the risk of sites for tophi are the ears, hands, toes and
gout, including: knees. Tophaceous gout is the consequence of
Hypertension chronic, poorly-controlled gout.
Diabetes, obesity and the metabolic Thus, examination should proceed with an
syndrome assessment of the hands followed by the knees
Renal failure and/or feet.
Case 101 Gout 183

Assess for deformity and function. The Demonstrate an understanding


metacarpo-phalangeal joint (MCP) of the
forefinger of the right hand is swollen, tender of the value of further
and erythematous. It is warm to touch, investigation
suggesting acute inflammation. The patient has The key in any hot, swollen joint is to rule out a
restricted range of movement of the affected septic joint.
joint, mainly due to pain. Specific investigations for gout include:
Blood tests: inflammatory markers (expect
Relevant negatives the white cell count to be raised in addition
Important relevant negative findings include to C-reactive protein, even in the absence of
the absence of symmetrical deformity and the bacterial infection). Serum urate is routinely
absence of clinical features of sepsis (important measured
in a patient with a hot, red joint). Joint aspirate: acute gout is a crystal
arthropathy. The crystals can be detected
What to tell the patient using spectrophotometry examination of
the joint aspirate. Positive aspirates contain
From the symptoms you describe and needle-like crystals that display negative
my examination findings, the most likely birefringence under polarised light. The
diagnosis is that of acute gout. Gout is where joint aspirate will also undergo gram
crystals form in the joint, causing it to become staining and culture to aid exclusion of a
painful and inflamed. It is not a dangerous septic joint
condition and the frequency of attacks can Plain radiographs of affected joints
be reduced with the right treatments and
avoidance of triggers. Today I am going to
start you on some medications to reduce the Always offer a
inflammation. It is called colchicine. One management plan
of the side-effects of colchicine is that it can
Principles of management include symptom
cause diarrhoea, so if this occurs you should
control, followed by measures to induce
stop taking it. I would like to order some
remission, and subsequently maintain
investigations and I will see you in clinic again
remission.
in 10 weeks.
Symptom control is with non-steroidal
anti-inflammatory agents with colchicine to
What to tell the examiner state reduce inflammation. Use of colchicine is often
the most likely diagnosis on the limited by diarrhoea and caution is needed in
patients with renal impairment.
basis of the history and clinical Allopurinol, a xanthine oxidase inhibitor, is
findings used to maintain remission in those patients
experiencing severe or recurrent bouts of
This patient has symptoms and signs consistent
disabling gout. It should not be started in an
with a diagnosis of acute gout.
acute attack due to precipitation of soft tissue
crystal stores, worsening and prolonging the
Offer relevant differential acute episode. The patient should avoid foods
and medications that trigger attacks.
diagnoses
The most important differential diagnosis of an
acutely inflamed joint is septic arthritis. Further reading
Roddy E, Doherty M. Epidemiology of gout. Arthritis
Res Ther 2010; 12:223.
Case 102: Carpal tunnel syndrome

Instruction to the candidate facial features with prominent brow and jaw
This 34-year-old woman has been experiencing
Neck examination where there is suspicion
of thyroid disease or inspection suggests a
tingling in her hands over a period of 6 months.
possible goitre
Please make an assessment with a brief history
and examination, as you feel appropriate. Advise
the patient as to what you think the cause of her Relevant negatives
problem is. Note the absence of scars to suggest previous
carpal tunnel decompression.
Take a focused history
As the primary symptom is that of tingling in What to tell the patient
the hand, get a full description of the tingling. The symptoms you describe to me and the fact
Specifically seek to obtain a history of tingling that they are reproduced when I tap on your
in the distribution of the median nerve. Are wrist and when I hold your wrist flexed suggest
there any specific circumstances in which the that you have a condition known as carpal
tingling occurs or is worse? Are the symptoms tunnel syndrome, where one of the nerves that
unilateral or bilateral? Is there associated pain or run to your hand gets pressed upon as it passes
numbness? through the wrist. To make a certain diagnosis,
Take a past medical history for conditions I will have to request an investigation to test
which are all associated with carpal tunnel the affected nerve. In the meantime it is
syndrome including: obesity, trauma, reasonable to start conservative treatment with
hypothyroidism, diabetes, rheumatoid arthritis, the use of a wrist splint and the avoidance of
acromegaly, or pregnancy. overuse of the wrist. It is also possible to inject
Take an occupational history. Is there steroids into the wrist. You may have heard
evidence of overuse of the wrist? that surgery is an option in this condition, but
we normally wait to see how simple measures
Positive findings of a focused work first.
Are there any other questions or concerns
examination that I can address today?
Assessing for deformity, there is wasting of the
muscle bulk of the thenar eminence. Explore the
possibility of neurological deficit, both sensory
What to tell the examiner state
and motor, in the distribution of the median the most likely diagnosis on the
nerve. Assessing for function, there is weakness basis of the history and clinical
of thumb abduction.
Utilise the following special tests to gain findings
evidence towards a diagnosis of carpal tunnel This patient has signs and symptoms consistent
syndrome: with carpal tunnel syndrome.
Tinels test: reproduction of the patient's
symptoms by repetitive percussion of a finger Offer relevant differential
over the volar aspect of the patient's wrist diagnoses
Phalens test: reproduction of the patient's
symptoms by holding the patient's wrist in Causes of carpal tunnel syndrome include:
fixed flexion for a prolonged period of time Idiopathic
(>45 seconds) Obesity
Consider further examination for clinical Hypothyroidism
signs of conditions known to cause carpal tunnel Diabetes mellitus
syndrome. In particular: Rheumatoid arthritis
Pregnancy
Visual field assessment in patients with Acromegaly
suggestive of acromegaly on general inspection Post-traumatic
spade-like hands, large stature and coarse
Case 103 Wrist drop radial nerve lesion 185

Demonstrate an understanding management of the carpal tunnel syndrome can


be divided into conservative and surgical.
of the value of further
investigation Conservative management involves splinting
and steroid injection
Investigations aim to confirm pathology of the Surgical management is second line.
median nerve: Intervention includes decompression of the
Nerve conduction studies: a positive test, carpal tunnel by division of the transverse
suggestive of median nerve pathology, will carpal ligament. A recent systematic review
show decrease conduction velocities in the has shown that both conservative and
median nerve in the context of normal nerve surgical measures confer symptomatic
conduction at other sites benefit in carpal tunnel syndrome, but that
Imaging options include ultrasound of the surgical management was more effective
carpal tunnel and MRI
Investigations to rule out secondary causes Further reading
of carpal tunnel syndrome include thyroid
Marshall S, Tardif G, Ashworth N. Local corticosteroid
function tests, -HCG if appropriate and
injection for carpal tunnel syndrome. Cochrane
IGF-1 for acromegaly (however, the clinical
Database Syst Rev 2007:CD001554.
picture should be enough to confirm or
Shi Q, MacDermid JC. Is surgical intervention more
refute a diagnosis of acromegaly)
effective than non-surgical treatment for carpal
tunnel syndrome? A systematic review. J Ortho
Always offer a management plan Surg Res 2011;6:17.
Where identified, management should be
directed to the underlying cause. Specific

Case 103: Wrist drop radial nerve lesion

Instruction to the candidate Positive findings of a focused


Please examine this 52-year-old man who has examination
been complaining of weakness and abnormal Motor deficit includes weakness of wrist
sensation in the right hand, which he noticed extension, known as wrist drop. Sensory
when he woke from sleep 3 days ago. loss is evident on the dorsum of the hand.
The area affected includes the thumb to the
Take a focused history interphalangeal joint and the fore- and middle
finger to the distal interphalangeal joint. The
The clinical history may vary. Ascertain the
lateral half of the ring finger may also be affected.
circumstances of the onset of the weakness. A
common scenario involves the saturday night
palsy where a patient has fallen asleep, under Relevant negatives
the influence of alcohol, in an awkward position Specific relevant negative findings would
with an arm straddling a chair with resultant include:
axillary compression. On waking, weakness of
the arm is noted to persist. An absence of signs of other neurological
Additionally screen for: dysfunction (therefore, this is a
mononeuropathy)
Trauma to the neck and arm No sign of neck trauma
A history of use of crutches No sign of trauma to the humerus
Any exposure to lead No crutches by the bedside
186 Chapter 5 Station 5

What to tell the patient Posterior cord of the brachial plexus


From what you have told me and from what I
Axilla: Persistent use of crutches
have found on examination, it appears that you
As the nerve runs in the spiral groove of the
humerus: Fractured humerus
have caused some damage to one of the nerves
that supplies some of the muscles in the arm the
At the elbow: nerve entrapment
radial nerve. Sustained pressure on the nerve, such
In the forearm: fractured radius
as from falling asleep with your arm draped over a
chair, causes dysfunction of the nerve. This is why Demonstrate an understanding
you have weakness and why the sensation on the of the value of further
back of the right hand is affected. In most cases
when the radial nerve is damaged in this way the investigation
damage is only temporary will usually get better by The history and examination will guide the
itself. However, I will organise some tests to check exact investigative strategy, but tests to consider
the function of the nerves of your arm and will see include neurophysiology, with assessment of
you again in 4 weeks time. Are there any other nerve conduction.
questions or concerns that I can answer for you?
Always offer a management
What to tell the examiner state plan
the most likely diagnosis on the Management depends on the underlying
basis of the history and clinical cause. If the patient is suffering pain,
analgesia may include gabapentin, pregabalin
findings or amitriptyline to target the neuropathic
This patient has signs consistent with radial component. Wrist splinting can be considered.
nerve palsy due to prolonged compression of the Depending on the level of disability caused,
radial nerve. you may need to refer to occupational therapy
and physiotherapy as part of a multi-disciplinary
Offer relevant differential strategy to minimise the effect of the functional
impairment on the patients activities of daily
diagnoses living.
Radial nerve lesions can be caused by damage
anywhere along the course of the nerve, including:
Case 104: Ulnar nerve lesion

Instruction to the candidate and pronated, to hold a piece of paper between


the straight thumb and the medial aspect of the
This 68-year-old woman has presented to clinic
forefinger. The person examining then tries to
complaining of tingling and numbness in her
pull the piece of paper out of the hand while the
left hand. Please take a brief history and perform
patient resists. A subject with normal function of
a focused examination. Inform the patient of
adductor pollicis will be able to hold the paper
your findings and formulate a management
while keeping the thumb straight. A subject with
plan.
weakness in adductor pollicis will compensate
by flexing the thumb to hold the paper in place.
Take a focused history The latter denotes a positive Froments sign.
Enquire about the type of deficit and its Sensory deficit includes sensory loss over the
distribution, i.e. sensory, motor, or both. palmar surface of the ulnar aspect of the palm,
Accurately define the type of sensory extending to involve the little finger and the
disturbance. Is it tingling, numbness or pain? Is medial aspect of the ring finger.
the sensory disturbance transient or continuous? Consider further examination of the elbow
Explore exacerbating or triggering factors. for any deformity. Also examine the neck and
Explore the past medical history: shoulder. Pain on neck movement mimicking
the patients symptoms could indicate brachial
Is there a history of carpal tunnel syndrome, plexus pathology.
nerve entrapment or mononeuropathy?
Is there a history of degenerative or traumatic
Relevant negatives
pathology at the elbow?
Does the patient have diabetes or Important relevant negative findings include:
hypertension?
There is no clawing of the hand, which
Explore the social history: would indicate severe, long-term ulnar nerve
dysfunction
What is the patients occupation? Are they a There is an absence of signs of previous
construction worker who operates vibrating trauma to the elbow
tools? There are no scars to indicated previous
Do they, or did they smoke? Take a ulnar nerve decompression see section on
systematic review of weight loss, cough and management
haemoptysis when suspecting a Pancoast
tumour
What effect do the symptoms have on the What to tell the patient
patients daily life? The symptoms you describe together with my
clinical findings suggest you have a problem
The typical ulnar nerve lesion will present as
with the ulnar nerve which supplies part of your
sensory disturbance comprising of tingling and
hand and arm. Because of its anatomic position,
numbness, felt at the lateral aspect of the palmar
it is subject to entrapment and injury by many
surface of the hand. The 5th finger is usually the
causes. Because of its superficial position at
most affected.
the elbow it is prone to injuries by excessive
pressure in this area such as leaning on the
Positive findings of a focused elbow during work, which I suspect is the case
examination in your case. With the aid of physiotherapy and
occupational therapy it should get better. Failing
Motor findings include muscle wasting. There that there are surgical options we can consider
is weakness of flexor carpi ulnaris and flexor for your symptomatic relief. I would like to run
digitorum profundus weakness. Check intrinsic some nerve studies to confirm how the nerve is
muscle function. functioning and would like to bring you back in
In ulnar nerve palsy, there is weakness of 6 weeks time to discuss the results and discuss
thumb adduction. Testing for Froments sign options. Are there any questions or concerns
involves asking the patient, with their hand flat that I can address?
188 Chapter 5 Station 5

What to tell the examiner state Demonstrate an understanding of


the most likely diagnosis on the the value of further investigation
basis of the history and clinical Nerve conduction studies will confirm nerve
pathology.
findings
This patient has symptoms and signs consistent
with a diagnosis of ulnar nerve palsy. Always offer a management plan
The management of ulnar nerve dysfunction
due to compression at the cubital tunnel is
Offer relevant differential undertaken in the context of a multi-disciplinary
diagnoses team setting. It includes initial conservative
The most common cause of ulnar nerve palsy is management with regular review and
due to entrapment in the cubital tunnel at the involvement of occupational therapy and
elbow. Other causes include: physiotherapy as appropriate.
If conservative measures fail, then surgical
Cervical disc disease intervention is indicated. Surgical options include:
Brachial plexus abnormalities
Thoracic outlet syndrome Cubital tunnel decompression
Pancoast tumour Medial epicondylectomy
Elbow abnormalities, such as epicondylitis Transposition of the ulnar nerve
Tumours
Wrist fractures Further reading
Compression within Guyons canal
Cutts S. Cubital tunnel syndrome. Postgrad Med J.
2007; 83:2831.

Case 105: Cataracts

Instruction to the candidate Does the patient suffer from glare


particularly noticeable when driving at night?
This 72-year-old woman has been complaining
of declining vision over the past year. Please Explore the past ophthalmic history. Has
take a short history and perform a focused there ever been trauma to eyes, either blunt
examination. Summarise your findings to the or penetrating? Is there any history of ocular
patient and explain your investigation and inflammation? Is there a history of squint (turn
management plan. A short discussion with the in eye) or amblyopia?
examiner will then follow. Enquire about the patients past medical
history, seeking to elicit a history of diabetes
mellitus or other conditions associated with
Take a focused history cataract. Do they use medications such as
Attempt to gain more information on the nature of corticosteroids?
the visual loss. Ask about whether it is lateralised Assess the effect of the visual loss on the
to one eye in particular. How severe is it? What has patients activities of daily living. Make sure to
been its speed of onset (the candidate information elicit the patients occupation and driving status.
suggests one year)? Is there associated pain
(cataracts is usually painless)?
Ask regarding associated features: Positive findings of a focused
Is there any scalp tenderness or jaw
examination
claudication? If present, these symptoms The key positive findings are of reduced visual
would suggest giant cell arteritis acuity with a diminished red reflex.
Case 105 Cataracts 189

Examination reveals reduced visual acuity. If Medication, such as corticosteroids


possible test with Snellen chart (distance) and Previous ocular trauma or intraocular surgery
Near Vision chart. Ensure appropriate glasses. Ocular inflammation
Using a pinhole helps to negate the effect of Chronic retinal detachment
uncorrected refractive error. A posterior segment tumour
There is a diminished red reflex you may see
Age related macular degeneration might also
generalised dimming of red reflex, spokes, flecks,
cause gradual decline in vision over many years
dots. In advanced cataract, the lens may be seen
and should be considered in such a history.
as white or brown, obscuring the red reflex.

Relevant negatives Demonstrate an understanding of


Important relevant negative findings include: the value of further investigation
Cataract is a clinical diagnosis and management
Examination of pupil reactions reveals an plans are based usually on clinical findings.
absence of a relative afferent papillary defect In cases where the fundus cannot be
(patient may have had prior mydriatic drops visualised due to dense lens opacity (white or
in which case this will not be possible) brunescent cataract), B-scan ultrasonography
Visual fields to confrontation are normal can be used to check for the presence or absence
Using the +10 dioptre setting on the direct of major posterior segment abnormalities such
ophthalmoscope to provide illumination and as retinal detachment or tumour (choroidal
magnification , the cornea is clear and there melanoma or choroidal metastasis).
is no obvious pupillary or iris deformity
Fundoscopy is normal
Always offer a management plan
What to tell the patient Early cataract may cause visual loss due to
refractive change that may be amenable to a
The reduction in vision you describe appears to change in glasses prescription; therefore, a
be due to cataract. This is a misting of the lens referral to an optometrist is necessary.
in the eye that occurs over time, and is often Surgical treatment of cataract is indicated
helped with surgery. when the patients corrected visual acuity no
If you wish to consider an operation to help longer meets their visual demand and they
your vision I can ask your GP to refer you to an are unable to perform their activities of daily
eye surgeon who can discuss this further with living. The lens is removed and an artificial lens
you. In the meantime it might help to attend implanted. Most cataract surgery in the United
your optometrist to check if a new pair of glasses Kingdom is carried out as a day case under local
would improve your vision. anaesthetic. The main complication is visual loss
The RNIB might be able to offer you some with an incidence of 1 in 800 approximately.
practical advice and gadgets to make things Many hospitals in the United Kingdom have
easier for you at home. a liaison officer from the Royal National Institute
Do you have any questions or concerns? of the Blind (RNIB) who can assist patients with
visual impairment with their practical needs.
What to tell the examiner state Remember that in the United Kingdom the
legal visual standard for driving is to be able to
the most likely diagnosis on the read a standard number plate at 20 metres in good
basis of the history and clinical daylight. This translates approximately to 6/10 on
findings the Snellen chart. If the patient does not meet this
with either eye they should be advised of this.
The patient has lens opacity consistent with
senile cataracts.
Further reading
Offer relevant differential The Royal College of Ophthalmologists (RCOphth).
Cataract Surgery Guidelines. London; RCOphth,
diagnoses 2010.
A differential diagnosis of secondary causes of Driver and Vehicle Licensing Agency (DVLA). For
cataracts includes: medical practitioners. At a glance guide to the
current medical standards of fitness to drive.
Diabetes mellitus Swansea; DVLA, 2011.
Case 106: Diabetic retinopathy

Instruction to the candidate Posterior segment examination


This 30-year-old diabetic man complains of There may be any of the following:
reduced vision in his left eye. Please take a short
history and perform a focused examination.
Microaneurysms
Summarise your findings to the patient and
Blot haemorrhages
explain your investigation and management
Hard exudates
plan. A short discussion with the examiner will
Cotton-wool spots
then follow.
Venous beading
Intra-retinal microvascular abnormalities
New vessels at the disc or elsewhere
Take a focused history Vitreous haemorrhage (large dark floaters/
As with all visual loss, seek to characterise it red haze/no view to retina at all)
further by asking questions to elicit its nature, State the location of any haemorrhages or
laterality, severity and speed of onset. When hard exudates, paying particular attention to
asking about any associated pain, remember whether the macula is affected.
that diabetic retinopathy and vitreous
haemorrhage are usually painless.
Screen for any prodromal features: Relevant negatives
Check the pupillary reactions for a relative
Preceding floaters, cobwebs or a red tinge
afferent papillary defect but this may not be
to the vision would indicate vitreous
possible if mydriatic drops have already been
haemorrhage
instilled.
Preceding visual distortion indicates a
macular pathology
Ask about a past medical history of diabetes.
What to tell the patient
Characterise the sub-type of diabetes. How My examination today has shown a small
long is it since the patient was diagnosed? amount of bleeding inside the eye, preventing
Assess blood sugar control. Ask if the patient you from seeing through it. Hopefully it will
is aware of their most recent HbA1c level. clear over the next few weeks, making the vision
Take a medication history. Does the patient brighter. I will refer you to an ophthalmologist
have a history of macro- and/or microvascular who may recommend laser treatment to prevent
complications of their diabetes? further bleeding and permanent loss of vision.
Explore the past medical history. Does This might be a good time to look at helping
the patient have a history of hypertension or you to improve your blood sugars and blood
hyperlipidaemia? pressure. I will arrange for you to see the
Take a brief social history, assessing whether diabetes team who can give you more advice.
or not the patient smokes. What is the effect of
the visual loss on the patients lifestyle? What is What to tell the examiner state
their occupation? Are they currently driving?
the most likely diagnosis on the
Positive findings of a focused basis of the history and clinical
examination findings
The patient has proliferative diabetic
retinopathy with vitreous haemorrhage.
Anterior segment examination
Examine with +10 dioptre lens on
ophthalmoscope. There may be lens opacity or Grading diabetic retinopathy
rubeosis iridis (fine new vessel formation on iris There are many systems for classifying diabetic
a sign of advanced proliferative disease). retinopathy. The RCOphth guidelines recommend
this simple system for clinical use:
Case 106 Diabetic retinopathy 191

Low risk (background), non-proliferative diabetic If loss of vision but no obvious vitreous
retinopathy is characterised by mildly dilated haemorrhage, consider diabetic macular
veins, microaneurysms, dot haemorrhages, oedema
exudates and the occasional cotton-wool spot. If loss of vision but no obvious vitreous
High risk (pre-proliferative), non-proliferative haemorrhage or macular oedema, consider
diabetic retinopathy is characterised by cataract or ischaemic optic neuropathy
intraretinal microvascular abnormalities, venous
beading and loops, clusters of large blot or Demonstrate an understanding
blotch haemorrhage, and multiple cotton-wool
spots.
of the value of further
A patient with proliferative diabetic retinopathy investigation
will have new vessels at disc or elsewhere. This HbA1c should be measured to gauge
may be accompanied by vitreous haemorrhage glycaemic control
and tractional retinal detachment. B-scan ultrasonography can be used to
See Figures 5.45.8 for examples of diabetic exclude retinal detachment if no view to
retinopathy at various stages. fundus
Slit lamp biomicroscopy, OCT and/or fundus
Offer relevant differential fluorescein angiography can be used to
ascertain if there is any macular oedema, if it
diagnoses is suspected
If vitreous haemorrhage is present, remember
there are other causes apart from proliferative
diabetic retinopathy:
Always offer a management plan
Offer the patient lifestyle advice. Tell the patient
Retinal tear or detachment to avoid vigorous activity or heavy lifting to allow
Proliferative retinopathy secondary to the vitreous haemorrhage to clear. This can take
vascular occlusion or ocular ischaemic several weeks. Smoking cessation advice should
syndrome be offered if necessary.

Figure 5.4 Diabetic retinopathy.


The left fundus with dot and blot
haemorrhages in the inferonasal
quadrant.
192 Chapter 5 Station 5

Figure 5.5 Diabetic retinopathy.


The macula of the right eye
showing exudates adjacent to
the fovea (the darker area with no
vessels at the centre of the macula).
Dot and blot haemorrhages can be
seen in the temporal macula.

Figure 5.6 Diabetic retinopathy.


The macula of the right fundus
showing new vessels at the disc.
There are circinate exudates
surrounding blot haemorrhages in
the temporal macula.

Refer the patient to an ophthalmologist for endothelial growth factor agents may have a role
consideration of laser photocoagulation therapy in diabetic retinopathy treatment.
(Figure 5.8), although laser treatment may Risk factor modification is important. The
have to wait until a clear view of the fundus is patients presentation affords an opportunity to
possible. Laser treatment can reduce the risk address systemic risk factors, including blood
of severe visual loss over 2 years by 5070%. sugar levels. Any tightening of blood sugar
Non-clearing vitreous haemorrhage or tractional control should be gradual, as rapid reductions
detachment may require surgery (a vitrectomy). in blood sugar levels can be associated with
Intravitreal injections of steroid or anti-vascular worsening of retinopathy. Both the DCCT
Case 106 Diabetic retinopathy 193

Figure 5.7 Diabetic retinopathy.


Neovascularisation at the optic
disc, with venous loops on the
inferonasal retinal veins.

Figure 5.8 Laser treated diabetic


retinopathy. The optic disc of
the left eye showing treated
proliferative diabetic retinopathy.
There are fibrovascular bands in
front of the disc and pigmented
laser scars on the periphery of the
retina.

and UKPDS trials have shown risk reductions Further reading


associated with improved glycaemic control.
The Royal College of Ophthalmologists (RCOphth).
Good hypertensive control and addition of a
Diabetic retinopathy guidelines. London;
statin are indicated if appropriate.
RCOphth, 2012.
Vitreous haemorrhage is not a
Photocoagulation treatment of proliferative diabetic
contraindication to continue antiplatelet therapy
retinopathy: the second report of diabetic
if it is required for cardiovascular risk reduction.
retinopathy study findings. Ophthalmology
Consider referral to low vision services or
1978; 85:82106.
RNIB liaison officer.
194 Chapter 5 Station 5

The effect of intensive treatment of diabetes Intensive blood-glucose control with sulphonylureas
on the development and progression of or insulin compared with conventional
long-term complications in insulin-dependent treatment and risk of complications in patients
diabetes mellitus. The Diabetes Control and with type 2 diabetes (UKPDS 33). UK Prospective
Complications Trial Research Group. N Engl Diabetes Study (UKPDS) Group. Lancet 1998;
J Med 1993; 329:977986. 352:837853.

Case 107: Hypertensive retinopathy

Instruction to the candidate Relevant negatives


This 58-year-old woman with a history of Relevant negative findings include the absence
hypertension complains of headaches and of signs suggestive of diabetic retinopathy,
transiently blurred vision. Her blood pressure although remember that the two conditions may
is 210/110 mmHg. Please take a history and co-exist.
perform a focused examination. Summarise
your findings to the patient and explain your
investigation and management plan. A short
What to tell the patient
discussion with the examiner will then follow. Your blood pressure is very high today and this
is causing some changes at the back of your eyes
affecting your vision. I need to arrange some
Take a focused history tests to check if there is any particular reason
Most patients with hypertension are that your blood pressure is so high, and I will
asymptomatic. Worrying symptoms in this change your medication to help bring your
context include a headache which is worse in blood pressure down. Usually there will be no
the morning or on lying down, associated with lasting effects on your eyesight if your blood
nausea and transient blurring of vision or visual pressure is treated.
obscurations.
Take a history of hypertension including
its duration, what current medications (if any)
What to tell the examiner state
are being used to control it, and any symptoms the most likely diagnosis on the
suggestive of a systemic cause. basis of the history and clinical
findings
Positive findings of a focused The patient has retinal changes in keeping with
examination hypertensive retinopathy.
Fundoscopy in hypertensive retinopathy can
show a constellation of signs, including:
Grading hypertensive retinopathy
Narrowed retinal arterioles
Hypertensive retinopathy can be classified into
Arteriovenous nipping
either mild, moderate, or malignant disease.
Flame shaped haemorrhages
Mild retinopathy, which carries a modest
Hard exudates
association with risk of stroke, coronary heart
Cotton wool spots
disease and mortality, is characterised by the
Swollen optic discs
presence of one or more of the following arteriolar
If possible, measure the patients blood signs:
pressure yourself. Generalised arteriolar narrowing
Focal arteriolar narrowing
Case 108 Retinitispigmentosa 195

If you have not measured the patients blood


Arteriovenous nipping
pressure as part of the focused examination,
Arteriolar wall opacity (silver wiring)
mention it as a necessary investigation.
Moderate retinopathy, which has a strong risk
of stroke, cognitive decline and cardiovascular
mortality, is characterised by one or more of the Always offer a management plan
following retinal signs: Acute management of patients with malignant
Haemorrhage (blot, dot, or flame shaped) hypertension with optic disc swelling involves
Microaneurysm urgent but controlled reduction of blood
Cotton-wool spot pressure, as a rapid reduction can result in end
Hard exudates organ ischaemia. There should be screening for
Malignant retinopathy denotes any patient causes of secondary hypertension.
displaying moderate retinopathy plus optic disc Ophthalmic referral is advised in cases of
swelling. It has a strong association with mortality. moderate hypertensive retinopathy or worse,
or in cases where the retinal findings are out of
Offer relevant differential keeping with the degree of hypertension. An
ophthalmologist can examine for signs of other
diagnoses retinal vasculopathies. Usually observation of
Both central retinal vein occlusion and the fundal changes is all that is required; there
diabetic retinopathy can present with florid is seldom any need for ophthalmic surgical
haemorrhages, cotton wool spots and swollen intervention.
optic discs. Bilateral ischaemic optic neuropathy The patient will require long-term
presents with bilateral swollen discs, loss of management of blood pressure and attention
vision and sectoral visual field defects without paid to other cardiovascular risk factors.
peripheral retinal vascular changes and
haemorrhages.
Further reading
Wong TY, Mitchell P. Hypertensive retinopathy.
Demonstrate an understanding of N Engl J Med 2004; 351:23102317.
the value of further investigation Grosso A, Veglio F, Porta M, Grignolo FM, Wong
Consider a CT scan of head if any concern TY. Hypertensive retinopathy revisited: some
regarding other causes of raised intracranial answers, more questions. Br J Ophthalmol 2005;
pressure. 89:16461654.

Case 108: Retinitispigmentosa

Instruction to the candidate Take a focused history


A 30-year-old man has been referred to you Ask regarding the nature of his clumsiness,
by his general practitioner. He has become including enquiring about symptoms that may
increasingly clumsy and prone to bumping suggest cerebellar or vestibular disease.
into things of late. There is a family history of Elicit a history of poor peripheral vision and
poor vision. Please take a history and perform a not seeing the objects being bumped into.
focused examination. Summarise your findings Ask regarding nyctalopia (poor night vision).
to the patient and explain your investigation and In children this may manifest as fear of dark
management plan. A short discussion with the environments. Does the patient need to sleep
examiner will then follow. with the light on? Do they bump into things
when walking outside at night (particularly if the
patient lives in the country)?
196 Chapter 5 Station 5

Ask about associated symptoms: Relevant negatives


Glare and blurred reading vision are Important relevant negative findings include:
important secondary symptoms
History of hearing loss
The absence of a relative afferent papillary
defect
History suggestive of other developmental
The absence of nystagmus
abnormalities
Normal colour vision (tested using Ishihara
Is there a family history of visual impairment? plates if available)
If so, who was affected, at what age? Characterise
the type of visual impairment suffered by family
members. Try to go back as far as grandparents
What to tell the patient
in the family history, and do ask about aunts, It is possible that you have a condition called
uncles and cousins. Beware, some family trees retinitis pigmentosa, meaning that the retina
can be complicated! Try to draw out a family does not work as well as it should do. This can
tree if possible, and mark on it the affected cause your eyesight to fail over time, particularly
individuals to ascertain the mode of inheritance. at the edges of your vision or when it is dark.
Take a social history, including the patients Sometimes this runs in families, and sometimes
employment and driving status. it can be passed on to any children you might
have. Sometimes it is made worse by treatable
things such as cataract and fluid accumulating
Positive findings of a focused in the retina and so I will refer you to an eye
examination doctor.
Test visual acuity if facilities available central
acuity may be normal though. What to tell the examiner state
Visual fields to confrontation show reduced
peripheral visual field in a concentric and
the most likely diagnosis on the
symmetrical pattern. basis of the history and clinical
Fundoscopy shows a pale disc, attenuated findings
retinal blood vessels, and pigment spicules in
peripheries (Figure 5.9). The patient has signs in keeping with retinitis
There may be cataract diminished red pigmentosa, an inherited rod dystrophy.
reflex, speckles/spots in red reflex.

Figure 5.9 Retinitis pigmentosa.


Characteristic pigmented spicules
in the peripheral retina.
Case 109 CMV retinitis 197

Offer relevant differential If central acuity is poor, both fundus


fluorescein angiography (FFA) and optical
diagnoses coherence tomography (OCT) can be used
There are a number of conditions which have to identify the presence of cystoid macular
retinitis pigmentosa as a feature, including: oedema.
Ushers syndrome retinitis pigmentosa
associated with hearing loss Always offer a
LaurenceMoonBardetBiedl syndrome management plan
retinitis pigmentosa associated with truncal
obesity, short stature and polydactyly If cataract or macular oedema suspected, refer
to ophthalmologist for consideration of further
Diabetic retinopathy with heavy laser medical or surgical management.
photocoagulation treatment can look similar to If the patient has younger siblings, or is
retinitis pigmentosa, and peripheral fields and considering starting a family, a referral to a
night vision can be affected in a similar way. geneticist for counselling is advised.
Consider referral to low vision services or
Demonstrate an understanding of RNIB liaison officer.
Consider eligibility to hold a driving license
the value of further investigation if applicable. The patient should be advised
Automated visual field testing can more to inform the licensing authority of their
accurately plot and quantify peripheral field condition.
loss, and provide a baseline measure.

Case 109: CMV retinitis

Fundoscopy shows creamy retinal infiltrates


Instruction to the candidate with scattered retinal haemorrhages
This 46-year-old man is an inpatient on scrambled egg and tomato ketchup. There is
the infectious diseases ward. He has been sheathing of the vessels indicative of retinal
complaining of blurred vision. Please take a vasculitis. Changes are usually present in both
history and perform a focused examination. eyes.
Summarise your findings to the patient and
explain your investigation and management
plan. A short discussion with the examiner will Relevant negatives
then follow. It would be prudent to comment on the patients
general appearance and state of health.
Take a focused history
Characterise the nature and duration of visual What to tell the patient
loss. Is there a painful or red eye? You have an infection affecting your eyes,
Assess the general health of the patient. which is a sign of a weak immune system. It
Why are they in the infectious diseases ward? will require intensive treatment with strong
Is there a history of immunocompromise or drugs to try to kill the infection and help boost
immunosuppression? What current medications your immune system. The eye is delicate
are they taking? and can be severely damaged by infections
such as this. I will refer you to a doctor who
specialises in these infections. Do you have
Positive findings of a focused any questions?
examination
Test the visual acuity if facilities are available. It
will likely be reduced.
198 Chapter 5 Station 5

What to tell the examiner state Cultures can be taken from throat and urine
for cytomegalovirus
the most likely diagnosis on the Serological tests can be performed for HIV,
basis of the history and clinical toxoplasmosis, Toxocara and syphilis if there
exists any doubt regarding the underlying
findings diagnosis
The symptoms of this patient together Vitreous sampling may be performed for
with the clinical findings are suggestive of microscopy, culture and polymerase chain
cytomegalovirus chorioretinitis. reaction (PCR) DNA analysis but is not
usually necessary
Offer relevant differential
diagnoses Always offer a management plan
Possible causes of chorioretinitis include: HIV Advice should be sought from an infectious
infection, toxoplasmosis, Toxocara, syphilis, disease physician and a medical retina
Borrelia. specialist. Management involves treatment of
any HIV infection along with systemic antiviral
agents. Antiviral agents can be administered
Demonstrate an understanding directly into the vitreous cavity by injection, or
of the value of further by slow-release implant.
investigation
Investigations would include:

Case 110: Optic atrophy

Instruction to the candidate Is there scalp tenderness (does it hurt to


brush your hair?)?
This 70-year-old woman has a history of poor
vision in their right eye. Their optometrist noted
Is there jaw claudication (do you get tired
chewing your food?)?
an unusual appearance to their right optic disc.
Please take a history and perform a focused
Are there more generalised symptoms of
weight loss, anorexia and general malaise?
examination. Summarise your findings to the
patient and explain your investigation and
management plan. A short discussion with the Positive findings of a focused
examiner will then follow. examination
Look for reduced visual acuity. There may be
Take a focused history reduced visual acuity in the right eye. With the
When and how did the visual loss happen? Does other unaffected.
it continue to progress, or was it a sudden event? There is a right relative afferent papillary
Ask about any associated neurological defect, or possibly no direct reaction to light at
symptoms. all.
Explore a past medical history for evidence Colour vision in the right eye is poor.
of existing cardiovascular disease. Enquire of a (Test with Ishihara plates, or ask about red
family history of glaucoma. desaturation).
In any patient over the age of 50 years with There is a reduced visual field to
visual loss, ask about symptoms of temporal confrontation on the right side (large central
arteritis: scotoma, or possibly a reduced peripheral field),
the left is normal.
Case 110 Optic atrophy 199

Positive findings on fundoscopy include disc Optic nerve compression (tumour,


pallor in the affected eye. The unaffected eye aneurysm)
may be normal. Demyelination
Advanced glaucoma (optic disc will be
cupped)
Relevant negatives Previous optic nerve trauma/transection
Relevant negative findings include:
Causes of bilateral optic atrophy include:
There is a preserved consensual papillary
reaction Toxic and nutritional optic neuropathies
The optic disc cup is normal on the right Long-standing papilloedema, e.g. in
There is no disc swelling idiopathic intracranial hypertension (the disc
margins will be blurred)
Lebers optic atrophy is an inherited cause of
What to tell the patient optic atrophy with visual loss
The nerve at the back of the eye is pale due to
the blockage in the blood vessels you had many
years ago. We will check your blood pressure
Demonstrate an understanding
and cholesterol to make sure we are doing all we of the value of further
can to prevent the same thing happening again. investigation
Do you have any questions?
Investigations will be guided by the likely
underlying cause, e.g. neuroimaging if there
What to tell the examiner state is suspicion of optic nerve compression or
the most likely diagnosis on the demyelination.
In cases of cardiovascular aetiology,
basis of the history and clinical cardiovascular risk factors should be quantified.
findings If there is concern regarding cardioembolic
disease, holter monitoring can assess for
The symptoms of the patient together with the
arrhythmia.
examination findings are suggestive of optic
atrophy.
Always offer a management plan
Offer relevant differential The cause in this case is presumed to be a
previous vascular event and so no immediate
diagnoses intervention is required.
The causes of optic atrophy include: Ensure secondary modifiable cardiovascular
risk factors are identified and treated.
Previous vascular event (ischaemic optic
neuropathy, central retinal artery occlusion,
temporal arteritis)
Case 111: Papilloedema

Instruction to the candidate What to tell the patient


A 20-year-old woman is referred to the The nerves at the back of your eyes appear
emergency department with headaches. swollen, which can be a sign of increased
Please take a history and perform a focused pressure inside your head. There are many
examination. Summarise your findings to the causes of this. We will need to arrange some
patient and explain your investigation and tests, including a brain scan, to find out what is
management plan. A short discussion with the the cause. Do you have any questions?
examiner will then follow.
What to tell the examiner state
Take a focused history the most likely diagnosis on the
Ask specific questions about the headaches,
including the site of pain, radiation of the
basis of the history and clinical
pain, the periodicity, duration and severity. findings
Are there any aggravating or relieving factors? This patient has papilloedema. I suspect this
Ascertain if the pain is unilateral and pulsating patients presentation is secondary to raised
like migraine? Is the pain worse in the morning intracranial pressure leading to bilateral disc
and associated with vomiting like a headache of swelling. Note the term papilloedema refers
raised intracranial pressure? Is it band-like and specifically to disc swelling secondary to
worse towards the end of the day like a tension raised intracranial pressure. The key sign is the
headache? presence of normal visual acuity. Reduced visual
Assess for associated features such as: acuity implies optic neuropathy.
nausea, vomiting (particularly in mornings),
preceding aura or fortification spectra
(flashing lights and jagged lines in visual Offer relevant differential
field). diagnoses
Take a past medical history, asking about The differential diagnosis of the causes of
head injury, hypertension. papilloedema includes:
Take a drug history, asking about oral
contraceptive pill use, or tetracycline use. Intracranial tumour
Idiopathic intracranial hypertension
(pseudotumour cerebri)
Positive findings of a focused Intracranial abscess
examination Dural sinus thrombosis
Fundoscopy is likely to show bilateral swollen Intracranial haematoma
discs, with associated haemorrhages and The differential diagnosis of bilateral disc
exudates. Central venous pulsation is absent swelling includes:
(look very carefully at the retinal veins for a
rhythmic pulsation its presence effectively Malignant hypertension
rules out raised intracranial pressure, but its Bilateral ischaemic optic neuropathy (often
absence is non-specific). seen after a major hypotensive episode)
Consider further examination for: Bilateral central retinal vein occlusion
Severe anaemia
Small central scotomata Diabetic papillitis
Other cranial nerve deficits to act as a
localising sign
Blood pressure Demonstrate an understanding of
the value of further investigation
Relevant negatives Imaging
Visual acuity and papillary reactions are Urgent neuro-imaging is indicated, usually a
normal. CT of brain to rule out an intracranial mass,
Case 112 Foster Kennedy syndrome 201

with CT venography to rule out dural sinus Always offer a management plan
thrombosis.
Management is very much dependent on the
cause of the raised intracranial pressure, and
Lumbar puncture may include neurosurgical management of any
If the CT is normal, a lumbar puncture may intracranial mass.
be performed to establish the cerebrospinal Dural sinus thrombosis requires systemic
fluid pressure and to allow analysis of the anticoagulation.
cerebrospinal fluid. Idiopathic intracranial hypertension can
Automated visual field analysis, while not be managed with acetazolamide treatment
diagnostic, can be used to monitor chronically (a carbonic anhydrase inhibitor, which
raised intracranial pressure such as in idiopathic inhibits production of cerebrospinal fluid),
intracranial hypertension. therapeutic lumbar puncture and, if necessary,
ventriculoperitoneal shunting.

Case 112: Foster Kennedy syndrome

Instruction to the candidate vision on the right. Fundoscopy shows a pale


optic disc on the right, and a swollen optic disc
A 68-year-old man is referred to your clinic
on the left.
with headaches associated with a gradual
Consider further cranial nerve examination,
deterioration in vision in the right eye. His wife
particularly looking for an absent or diminished
has noticed that he has become very forgetful.
sense of smell and for further localising signs.
Please take a history and perform a focused
examination. Summarise your findings to the
patient and explain your investigation and Relevant negatives
management plan. A short discussion with the Visual acuity in the left (contra-lateral) eye is
examiner will then follow. normal, with no visual loss (or possibly a small
central scotoma).
Take a focused history
Take a brief history, focusing on both the What to tell the patient
headaches and the visual loss. Foster Kennedy There is a problem with the nerve at the back
syndrome is usually associated with a gradual of your eye preventing you from seeing so well.
painless loss of vision. We will arrange for a brain scan to try to check if
Explore the patients forgetfulness. What is there is anything in particular causing it. I would
the degree of cognitive impairment and does the like to see you straight after the scan to discuss
patient demonstrate insight? Is there associated the results and consider treatment options as
incontinence (recent onset dementia with these will very much depend on what is causing
associated incontinence should be assumed the nerve damage. Do you have any questions at
to be due to an organic cause)? Are there any this stage?
other neurological symptoms, specifically any
disturbance of the sense of smell?
What to tell the examiner state
Positive findings of a focused the most likely diagnosis on the
examination basis of the history and clinical
There is reduced visual acuity in the right eye. In findings
addition, there is peripheral visual field loss in The patients presentation together with clinical
the right eye. There may be a right-sided relative findings point to a possible diagnosis of Foster
afferent papillary defect, and reduced colour Kennedy syndrome optic atrophy due to
202 Chapter 5 Station 5

a frontal tumour causing compressive optic Demonstrate an understanding


atrophy, with contralateral papilloedema due to
raised intracranial pressure. of the value of further
investigation
Offer relevant differential A CT scan of brain and orbits should be
performed to exclude an intracranial mass.
diagnoses
Possible causes of Foster Kennedy syndrome
include a subfrontal olfactory groove Always offer a management plan
meningioma (the classical presentation) or a Neurosurgical advice on the management a
glioma of frontal lobe or corpus callosum. causative intracranial mass should be sought.
Pseudo-Foster Kennedy syndrome describes
sequential (most often ischaemic in aetiology)
optic neuropathy or optic neuritis. The more
Further reading
recently affected nerve is swollen, the nerve Kennedy F. Retrobulbar neuritis as an exact
affected in the past is atrophic and pale. diagnostic sign of certain tumors and abscesses
Pseudo-pseudo-Foster Kennedy syndrome in the frontal lobes. Am J Med Sci 1911;
describes the history and appearance of 142:355368.
ischaemic optic neuropathy, with co-existing Gelwan MJ, Seidman M, Kupersmith MJ.
meningioma. Pseudo-pseudo-Foster Kennedy syndrome.
J Clin Neuroophthalmol 1988; 8:4952.

Case 113: Thyrotoxicosis

Instruction to the candidate particularly dryness and double vision.


Explore the past medical history and family
Please examine this 40-year-old woman who
history for autoimmune conditions.
complains of heat intolerance.

Take a focused history Positive findings of a focused


Elicit the exact nature of the patients symptoms.
examination
What does the patient or their GP mean by heat There is a wealth of positive findings in
intolerance? thyrotoxicosis. Look for a thin, flushed patient
Are there associated symptoms of wearing light clothing. Sweaty palms, noticed
hyperthyroidism, such as: when shaking hands with the patient.
Examine the neck for a goitre and make
Sweating a point of examining the eyes for proptosis,
Palpitations exophthalmos and lid lag (Figures 5.105.12).
Diarrhoea
Change in mood, anxiety and insomnia
As the patient is female, ask about menstrual Graves ophthalmopathy
symptoms such as amenorrhoea
This is the clinical syndrome of proptosis,
Questions relating to the patients
exophthalmos and lid lag. There is increased risk in
appearance
smokers. Complications include:
Weight loss, reduction in dress size
Optic neuropathy with decreased visual
Has the patient noticed a lump in their neck? acuity
Has anyone commented on the patients eyes, Ophthalmoplegia with diplopia
specifically bulging eyes? Go on to ask about Keratitis due to failure of eye lid closure
pain in the eyes and any visual disturbance,
Case 113 Thyrotoxicosis 203

Management options for Graves ophthalmopathy known as a beta-blocker which will help with
range from simple measures such as artificial some of your symptoms.
tears (to treat grittiness), to steroid use and orbital If the patient has eye signs or symptoms,
decompression. explain the complication of Graves disease
and tell them you will also refer them to the
Examine the cardiovascular system for a ophthalmologists.
tachycardia and any signs of cardiac failure.
Ask the patient to stand from sitting (without What to tell the examiner state
using their hands) to elicit any myopathy.
Examine the shins for pre-tibial myxoedema
the most likely diagnosis on the
and the toes for thyroid acropachy. basis of the history and clinical
findings
Relevant negatives This patient has symptoms and signs consistent
The lack of an irregular pulse suggestive of atrial with a diagnosis of thyrotoxicosis.
fibrillation is an important relevant negative
finding in the setting of thyrotoxicosis. Offer relevant differential
diagnoses
What to tell the patient The common causes of hyperthyroidism
From your symptoms and the clinical findings include:
I can elicit I suspect you have a condition known
as thyrotoxicosis. You may have heard of it Graves disease
as an over-active thyroid. I will request some Solitary toxic nodule
investigations and refer you to the specialist Multinodular goitre
endocrinology clinic for further assessment. In In the case of a tender thyroid think of
the meantime I will start you on a medication thyroiditis and ask about recent viral
infection

Figure 5.10 Thyroid eye disease.


This patient, with hyperthyroidism,
has forward protrusion of the
eyeball. It is best appreciated either
laterally (see Figure 5.11) or from
above. Indeed, formal examination
should involve standing behind the
seated patient and looking down
over the top of their head to see if
the proptotic eyeballs can be seen
in front of the forehead. Note also
the peri-orbital oedema.

Figure 5.11 Thyroid eye disease.


In this lateral view of the same
patient seen in Figure 5.12, the
forward protrusion of the eyeball is
visible. In such patients, further eye
examination would involve testing
for lid lag and for any complex
ophthalmoplegia, which can occur
in thyroid eye disease.
204 Chapter 5 Station 5

Figure 5.12 How to examine for lid lag. The patient is asked to focus on the examiners finger. The finger is raised
so the patients eyes are looking toward the ceiling. The finger is then rapidly moved vertically downward. A positive
finding is when the pupils move down ahead of the eye lids (there is lid lag) such that the superior aspect of the
sclera is transiently more visible than normal.

Demonstrate an understanding Always offer a management plan


of the value of further Beta-blockade can help symptoms of tremor
and palpitations.
investigation Treatment options specific to hyperthyroidism
Blood testing would include: include:
Thyroid profile including TSH, T4 and T3 Medical therapy with carbimazole or
Thyroid autoantibody profile including propylthyrouracil. It is important to
Anti-TSH receptor antibodies recognise the risk of agranulocytosis on these
Anti-thyroid peroxidase antibodies medications, so tell the patient to present to
Imaging options include an USS of the hospital if they feel unwell or have a fever. A
thyroid to identify any structural abnormality sore throat is a classical presenting symptoms
such as goitre or nodules, and an uptake scan in patients with neutropaenia
to show the pattern of activity. Homogenous Radioiodine therapy is an ablative treatment
increased uptake suggests autoimmune for thyrotoxicosis. It is contraindicated in
thyrotoxicosis, a focal area of activity is pregnant and breastfeeding women. The use
consistent with a toxic nodule and absent or of radioiodine in autoimmune thyrotoxicosis
globally reduced uptake suggests a thyroiditis. with eye disease can worsen active
If there is ophthalmopathy associated ophthalmopathy. Due to the radioactive
with Graves disease, the patient will need nature of radioiodine, patients receiving this
formal assessment by an ophthalmologist. treatment will be given advice on how to limit
Investigations may include CT or MRI of the contact with other individuals depending on
orbits. the dose of radioiodine they are given
Surgical options include thyroidectomy
Case 114: Hypothyroidism

Instruction to the candidate Examine the neck for goitre. Are there any
scars to suggest previous surgery such as a
Please assess this 40-year-old woman who has
thyroidectomy?
presented with fatigue and weight gain. Discuss
your findings with the patient and agree a
management plan. Relevant negatives
Examine the cardiovascular system and
Take a focused history comment on the absence of bradycardia,
cardiomegaly and signs of cardiac failure.
Fatigue has a wide differential, as does weight
gain. But the combination of the two, in the
setting of station 5, places hypothyroidism high What to tell the patient
on the list of possible diagnoses. From what you tell me and from the signs I can
Ask about the symptoms. Screen for elicit on examination, I think you may have a
alternative causes of fatigue such as anaemia, condition known as hypothyroidism, where the
depression, and Addisons. thyroid gland in the neck is underactive. I will
Ask about other symptoms of hypothyroidism request some blood tests to confirm this and
such as: I will refer you to the endocrinology clinic for
Intolerance to the cold. At the same time further management.
as being a history question, this is also an
examination point: Is your PACES exam in What to tell the examiner state
the summer heat of August yet the patient is
wearing a heavy jumper? the most likely diagnosis on the
Constipation basis of the history and clinical
Dry skin findings
Screen for complications of hypothyroidism This patient has signs and symptoms consistent
such as carpal tunnel syndrome. with a diagnosis of hypothyroidism.
Explore the past medical and family history
of the patient:
Offer relevant differential
Is this a patient who has previously
diagnoses
undergone treatment for hyperthyroidism?
Is the patient on thyroid replacement The differential diagnosis of hypothyroidism
therapy? If so, when was their TSH last includes:
measured? Autoimmune (Hashimotos) thyroiditis
Any past or family history of autoimmune Iatrogenic
conditions? Treatments for hyperthyroidism
Has the patient ever been on amiodarone? Amiodarone
Ensure to ask about mood as depression can Hypothalamic or pituitary causes (will have
occur insidiously in hypothyroidism. decreased TSH in addition to decreased
free T4)

Positive findings of a focused


Demonstrate an understanding
examination
On general inspection, note increased body
of the value of further
habitus. investigation
On closer inspection of the head and face, Investigations would include a thyroid profile
look for hair loss, dry skin, loss of outer third of including TSH, fT3, fT4 and autoantibodies. A
the eyebrows. full blood count should be performed to exclude
206 Chapter 5 Station 5

anaemia as an alternative cause for symptoms. Always offer a management plan


No radiology is required unless there is a goitre
The mainstay of management is thyroid
or palpable nodules.
hormone replacement with levothyroxine.
Monitoring is via TSH levels. Once stable, all
patients should have at least once yearly TSH
measurements.

Case 115: Acromegaly

Instruction to the candidate On examining the face, you may notice:


Please examine this 53-year-old man who A prominent supra-orbital ridge
presents with headaches and sweating. Discuss (Figure 5.13)
your findings with the patient and agree a Coarse facial features
management plan with appropriate follow-up. Prognathism
Macroglossia and wide inter-dental spaces
Take a focused history On examination of the visual fields, seek to
Ask first about the symptoms mentioned in the elicit a bi-temporal hemianopia.
candidate instruction. How long have they been
going on for? Relevant negatives
Go on to ask about:
If not present, comment on the absence of a
Stature, hand size and facial appearance. displaced apex suggestive of cardiomegaly.
Have they noticed a change? In many cases,
the patient will have been prompted by a
relative. Ask if they have brought an old
What to tell the patient
photograph to clinic and if not, ask them to It appears from what you tell me and from what
bring one when they next attend. Have they clinical signs I can elicit on examination that
gone up in shoe or glove size? Do they wear you have a condition known as acromegaly, a
jewelry? If so, have they had to increase the condition where the body produces too much
size of any rings? of a hormone called growth hormone. The usual
Have they noticed any changes in their vision? cause is a benign tumour of a small gland in
the head known as the pituitary gland. I think
Screen for other associated conditions: it is important that we do some tests to confirm
Do they have diabetes? that this is the case. These will involve some
Ask about hypertension and symptoms of blood tests and an MRI scan of the head. We
cardiac failure will see you again in 4 weeks time with the
Obstructive sleep apnoea results of those investigations. I will also write
Does the patient complain of any lower to your GP to let them know about the outcome
gastrointestinal symptoms (acromegaly of our consultation today. Are there any further
carries an increased risk of colonic polyps questions or concerns that I can answer for you?
and malignancy)?
What to tell the examiner state
Positive findings of a focused the most likely diagnosis on the
examination basis of the history and clinical
On general inspection, expect the patient to findings
have large, puffy (spade-like) hands and to be
This patient has symptoms and signs consistent
of tall stature.
with a diagnosis of acromegaly.
Case 115 Acromegaly 207

Figure 5.13 Prominent


supra-orbital ridge. This patient
has a clearly prominent supra-
orbital ridge, as seen from the
lateral view. This patient would
be expected to be tall in stature
and have coarse facial features,
with large hands. Further
examination would include testing
for a bitemporal hemianopia,
cardiovascular examination and
abdominal examination.

Offer relevant differential If symptoms were suggestive of


cardiovascular complications of acromegaly,
diagnoses a transthoracic echocardiogram would give
The overwhelming majority of cases of evidence on cardiac structure and function.
acromegaly are due to a growth hormone
secreting pituitary adenoma.
Always offer a management plan
In cases of growth hormone secreting pituitary
Demonstrate an understanding tumours causing acromegaly, transsphenoidal
of the value of further hypophysectomy is considered first line
therapy. Medical therapy involves somatostatin
investigation analogues, growth hormone receptor
Blood tests would include: IGF-1, fasting antagonists, or dopamine agonists. Pituitary
glucose, HbA1c, and tests of pituitary function external beam radiotherapy exists as a second
to screen for loss of function in other pituitary line for failed surgical resection.
axed. Candidates should remember that the
Dynamic testing is via the oral glucose management of acromegaly does not solely
tolerance test. A paradoxical rise in growth involve addressing causative pituitary tumours.
hormone level is diagnostic of acromegaly. There is increased risk from cardiovascular
Due to the likelihood of a pituitary adenoma, disease, colonic polyps (complicated by
patients with acromegaly require intracranial malignancy), and the complications of diabetes,
imaging. The modality of choice is an MRI. with these potential problems deserving of
Visual field testing will accurately assess any attention.
visual field defects.
Case 116: Weight gain and easy bruising
Cushingoid appearance

Instruction to the candidate Hirsutism


Please assess this 43-year-old woman who has
Pigmented striae (Figure 5.14)
presented complaining of easy bruising and
Examine for proximal myopathy by asking
the patient to stand from sitting with their
recent weight gain.
arms crossed.
Examine the visual fields for a bitemporal
Take a focused history hemianopia
How long have the symptoms been present for?
Consider causes of easy bruising. Is the Relevant negatives
patient on any medications which would put
Make a note of the absence of signs of chronic
her at higher risk such as anti-platelet agents?
liver disease. Chronic alcohol excess can lead to
Has the patient had any active bleeding such as
a cushingoid appearance.
epistaxis?
Screen for other symptoms of Cushings such
as amenorrhoea, increased hair growth, change What to tell the patient
in mood, or loss of libido. From our consultation today, I suspect you have
Ask about important complications of a condition called Cushings syndrome which is
Cushings including hypertension, diabetes, and when there is too much steroid hormone in the
osteoporosis. body. This is likely what has caused your weight
What medications is the patient on? Have gain and easy bruising. I am going to request
they been exposed to any exogenous steroids? some investigations and arrange for you to be
seen by one of my endocrinology specialist
Positive findings of a focused colleagues, who will see you promptly.
examination
On inspection, you note that the patient has
What to tell the examiner state
centripedal obesity and a rounded plethoric the most likely diagnosis on the
facial appearance. You further inspect for: basis of the history and clinical
Interscapular fat pad (avoid using the term findings
buffalo hump)
This patient has a Cushingoid appearance
Bruising in the context of thin skin
consistent with steroid excess.

Figure 5.14 Pigmented


abdominal striae. Pigmented
abdominal striae are visible on the
patients abdomen. The patient has
central adiposity. The infra-unblilical
scar is also pigmented.
Case 117 Gynaecomastia 209

Offer relevant differential test, confirm the diagnosis with low dose
dexamethasone suppression
diagnoses Assess whether is ACTH dependent or
The differential diagnosis includes: independent with serum ACTH value
ACTH-secreting pituitary adenoma
A high dose dexamethasone suppression
tests differentiates pituitary ACTH from
(Cushings disease)
ectopic ACTH production
Excess endogenous steroid from an adrenal
adenoma (Cushings syndrome) Imaging includes a pituitary MRI to identify
Pseudo-Cushings from conditions such as any pituitary adenoma. If an adrenal tumour is
depression and alcohol excess suspected, abdominal imaging is indicated.
Exogenous steroid use Inferior petrosal sinus or adrenal vein sampling
supplies functional anatomical information.
Demonstrate an understanding
of the value of further Always offer a management plan
The management of Cushings depends on the
investigation underlying cause. If the cause is found to be
Initially, perform baseline investigations and exposure to exogenous steroid, this should be
confirm steroid excess: stopped.
Full blood count, renal function, liver In the final diagnosis is that of Cushings
function, glucose, lipids, HbA1c, baseline disease, options include transsphenoidal
cortisol and pituitary function hypophysectomy, pituitary radiotherapy, and
24-hour urine collections for free cortisol and adrenalectomy. Medical management includes
overnight dexamethasone suppression test drugs such as metyrapone and ketoconazole.
If urinary free cortisol elevated and failed If a cause such as an adrenal adenoma is
overnight dexamethasone suppression found, surgical removal would be warranted.

Case 117: Gynaecomastia

Instruction to the candidate cause gynaecomastia. Rather, ask the patient


what medications he takes and if he had started
You have been asked to see this 30-year-old
any new medications around the time he noted
man, who has been referred by his general
the gynaecomastia.
practitioner after complaining of gynaecomastia.
The general practitioner notes that this young
man has a high alcohol intake. Please assess the Drugs causing gynaecomastia
patient and inform him of your findings.
Oestrogens
Hormonal medications, including those used
Take a focused history to treat prostate cancer
The patient reports that he has noticed Growth hormone
gynaecomastia over the past year. Consider the Ketoconazole
patients age. There will not be a child in the Spironolactone
exam, but the patient may be in the late teens, Many anti-cancer agents such as cisplatin,
when gynaecomastia may be physiological. vinblastine and belomycin
Take a past medical history, seeking to Calcium-channel blockers
identify alcohol use and a history of liver Digoxin
disease. Neuroleptic medications such as
Take a drug history. Given that this is a prochlorperazine
focused history, you do not need to directly ask Alcohol
the patient about every drug that is known to Marijuana
210 Chapter 5 Station 5

As this is a male with gynaecomastia, ask if What to tell the patient


he has noted any testicular lumps, thinking of
Thank you for coming to see me today. The
testicular cancer. Ask about libido and erectile
problem you are having, with enlargement of the
function.
male breast tissue, is known as gynaecomastia.
At all stages, it is important to recognise that
There is a wide range of causes for this, including
the patient may feel a sense of embarrassment
excess alcohol consumption. There are some
at having gynaecomastia. Explore his concerns
serious causes of gynaecomastia, but from
about his symptoms.
what you tell me and from what I can find on
examination I do not think these apply to you.
Positive findings of a focused However, I will order some blood tests just to be
examination sure. I will see you again in a couple of weeks
with the results of these tests. I will also refer
Examine the patients precordium. Palpate for you to our community alcohol liaison team
dense sub-areolar tissue, which should feel like for advice on how to cut down your alcohol
a coiled rope (Figure 5.15). consumption. Do you have any other questions
Offer to examine the testicles. Make a note of or concerns that I can address today?
secondary sexual characteristics.

What to tell the examiner state


Relevant negatives
The absence of signs of chronic liver disease is
the most likely diagnosis on the
an important relative negative to note. basis of the history and clinical
findings
This patient has gynaecomastia.

Offer relevant differential


diagnoses
The differential diagnosis of the causes of
gynaecomastia include:
Physiological
Due to other conditions such as liver disease,
chronic kidney disease, endocrine disease
Hormone-secreting tumours such as
testicular malignancies
Medications such as omeprazole,
spironolactone and hormonal medications
for prostate cancer
Recreational drugs such as alcohol and
marijuana

Demonstrate an understanding
of the value of further
investigation
Bloods tests including -HCG, liver function,
renal profile and prolactin level.
Further investigations such as imaging will be
Figure 5.15 Gynaecomastia. Excess breast tissue determined by the history and investigations. If
can be seen clearly in the lateral view. Findings on there is concern about testicular malignancy, an
palpation are discussed in the positive findings section. ultrasound would be warranted.
Depending on the circumstances surrounding the
patients presentation, abdominal examination, including
offering to examine the external genitalia in men, should Always offer a management plan
be performed. In any drug history, common iatrogenic Treatment is directed to the underlying causes.
causes of gynaecomastia should be ruled out. Offending drugs are removed or reduced
Case 118 Pseudohypoparathyroidism Albrights hereditary osteodystrophy 211

where appropriate on balance of need versus Further reading


acceptable and tolerable side effects. Alcohol
Tyrell CJ. Gynaecomastia: aetiology and treatment
liaison and or drug advice should be offered
options. Prostate Cancer Prostatic Dis 1999;
where appropriate. Medical management
2:167171.
includes use of oestrogen receptor blockers such
Mahoney CP. Adolescent gynaecomastia. Differential
as tamoxifen and androgens but these are used
diagnosis and management. Paediatr Clin North
with caution. Mastectomy can be considered in
Am 1990; 37:13891404.
severe cases.

Case 118: Pseudohypoparathyroidism


Albrights hereditary osteodystrophy

Instruction to the candidate Trousseaus sign: carpopedal spasm brought


on by inflating a blood pressure cuff on the
This 28-year-old woman has been referred by
patients arm
her general practitioner with hypocalcaemia.
Please take a history and perform an appropriate
examination. Following that, inform the patient Relevant negatives
of your findings. If signs of hypocalcaemia were absent
they should be mentioned as relevant
Take a focused history negatives. This would move the diagnosis
from pseudohypoparathyroidism to
Ask about symptoms of hypocalcaemia:
pseudopseudohypoparathyroidism.
Tingling and numbness, which can be
peri-oral or in the peripheries What to tell the patient
Carpopedal spasm and muscular cramps
Any history of seizures, which could be due From what your GP has told me and from the
to severe hypocalcaemia findings of our discussion and examination, I
feel you are likely to have a condition known
If any of the symptoms are present, assess as pseudohypoparathyroidism. This is an
their severity and duration. inherited condition which causes low levels of
Where general inspection suggests an calcium, one of the body salts. I will arrange
inherited condition, see below, take a careful for you to be referred to the endocrinology
family history. clinic. In the meantime I will request some
blood tests and start you on some oral calcium
Positive findings of a focused supplements. Do you have any further
questions or concerns I can answer for you
examination today?
On general inspection, you note that the patient
is of short stature, with obesity and a round
facial appearance. On inspection of the hands,
What to tell the examiner state
you note that the patient has a short 4th and 5th the most likely diagnosis on the
metacarpal on each hand. basis of the history and clinical
Additionally, examine for clinical evidence
of hypocalcaemia by assessing for the following findings
signs: The combination of the phenotypic appearance
of Albrights hereditary osteodystrophy with
Chvosteks sign: hemifacial twitching due symptoms and signs of hypocalcaemia suggest a
to repetitive percussion over the ipsilateral diagnosis of pseudohypoparathyroidism.
facial nerve
212 Chapter 5 Station 5

Offer relevant differential Screen for hypogonadism and


hypothyroidism if clinically indicated
diagnoses The Ellsworth-Howard test involves
The phenotypical appearance of Albrights measurement of serum and urine phosphate
hereditary osteodystrophy, without after intravenous PTH
evidence of hypocalcaemia, is known as
pseudopseudohypoparathyroidism.
Always offer a management plan
Important points in management of this
Demonstrate an understanding of condition:
the value of further investigation Genetic counselling
Calcium, PTH, vitamin D. Expect a low Calcium and vitamin D replacement
calcium with an appropriately raised PTH Phosphate binders if necessary

Case 119: Chronic skin conditions


plaque psoriasis
Instruction to the candidate Positive findings of a focused
Examine this 42-year-old mans skin and discuss examination
diagnosis and therapeutic options. The rash in chronic plaque psoriasis is
demonstrated in Figures 5.165.19. There are
Take a focused history well-demarcated papules or plaques of varying
Begin by ascertaining the age of onset of size, which are dull-red or salmon-pink colour. A
the rash and establish the areas affected. loosely adherent silvery-white scale is noted.
Ask specifically about scalp, genital and nail Auspitz sign is demonstrated when
involvement as this information may not be removal of scale results in minute blood
volunteered. Explore the precipitants of plaque droplets (should be mentioned rather than
psoriasis, all of which may also precipitate performed).
guttate psoriasis: Regarding the distribution of rash, it tends to
be symmetrical, focused on extensor surfaces (of
Physical or emotional stress (Koebner elbows, knees, knuckles), behind the ears, and
phenomenon) the scalp hairline. The palms and soles may also
Infection (-haemolytic streptococcal be involved.
infection) The nails may be involved (Figures 5.205.21),
Sunlight (may provoke psoriasis but also demonstrating pitting, onycholysis, discolouration
make it better) (oil spots are pathognomonic: yellow-brown spots
Drugs (beta-blockers, stopping systemic under nail plate) and subungual keratosis.
glucocorticoids, lithium, anti-malarials) If the submammary, axillary and/or
Alcohol and smoking anogenital folds are involved, there may be
glistening, sharply demarcated red plaques, with
Also ask about family history and any past
scale absent.
admission to hospital with erythroderma.
Ascertain smoking history and cardiovascular
disease risk factors as psoriasis is associated Relevant negatives
with cardiovascular disease and diabetes. Do If appropriate, comment on the absence of
not forget to ask about functional impairment or arthropathy, which occurs in 1025% of patients.
disability due to the skin disease
Case 119 Chronic skin conditions Plaque psoriasis 213

Figure 5.16 Psoriatic plaques.


The psoriatic plaques are visible on
the patients hairline and behind
the ears.

Figure 5.17 Chronic plaque


psoriasis. This patient has large flat
plaques covered in scale on the
lower back

Figure 5.18 Psoriatic plaques.


The extensor surface of the elbow
demonstrates the classical silver,
scaly plaque of psoriasis. Note also
some smaller plaques elsewhere on
the upper limb.
214 Chapter 5 Station 5

What to tell the patient


The skin changes you have suggest that you
have a condition called psoriasis. It is a very
common skin condition affecting 5% of the
population. It is not infective or contagious. The
trigger appears to be an outside event, such as
a throat infection, stress or an injury to the skin.
A high alcohol intake and smoking can worsen
psoriasis too, as can medicines used for other
conditions. Psoriasis is usually easy to recognise
and no further investigations are needed at
this stage. Topical treatment for psoriasis is
usually effective but if the psoriasis is severe or
resistant to treatment there are other options
such as light therapy and systemic medication.
The skin becomes less scaly and may then
look completely normal. However, even if your
psoriasis disappears after treatment, there is a
tendency for it to return. This may not happen for
many years, but can do so within a few weeks.
I would like to bring you back to clinic in 3
months to assess your progress on the treatment
I will prescribe today. I will be writing to your GP
to inform him of your visit today.
Do you have any questions or concerns that I
can address today?

What to tell the examiner state


the most likely diagnosis on the
Figure 5.19 Psoriatic plaques. There are plaques
on the extensor aspect of both knees, with bilateral
basis of the history and clinical
plaques also on the shins. findings
The presence of erythematous, scaly plaques in
the extensor surfaces alongside the nail findings

Figure 5.20 Psoriatic nails. Pitted


nails in a patient with psoriasis.
There is also longitudinal ridging
and evidence of onycholysis at the
free edge of the nail.
Case 119 Chronic plaque psoriasis 215

Figure 5.21 Psoriatic nails. A


second example of nail pitting.

and scalp involvement is strongly suggestive of is widespread involvement likely to require


chronic plaque psoriasis. systemic medication such as methotrexate or
biologic. Liver function and lipid profile should
be taken before starting acitretin, methotrexate.
Key clinical features of psoriasis Renal function is monitored in patients on
History: ciclosporin.
Chronic
Indolent Always offer a management plan
Present for months and years
Slowly changing Management depends on site and extent of
Examination: involvement, the type of psoriasis (e.g. plaque
Silvery-white scale vs guttate) and treatment tolerance. All patients,
Well-demarcated regardless of severity, require emollients and
Salmon pink plaques soap substitutes to control scale and protect the
Extensor surfaces skin.
For localised or mild form, affecting <5% of
the body surface and can be managed in the
Offer relevant differential community: start with vitamin D analogues
diagnoses (calcipotriol/calcitriol). Not more than
100 g/week to avoid hypercalcaemia. These
The differential diagnosis of small scaly plaques are often combined with potent topical
includes: seborrhoeic dermatitis (may be corticosteroids, e.g. calcipotriol with
indistinguishable), discoid eczema, discoid betamethasone..
lupus, tinea corporis (usually solitary lesion; If the condition is unresponsive to first line
take fungal scrapes). therapy, or if the patient suffers side effects, the
Apart from psoriasis, other causes of a flaky following can be used:
scalp include seborrhoeic dermatitis and tinea
capitis. Dithranol: inhibits DNA synthesis and
oxygen free radicals. It is, however, time-
consuming, irritant, and stains. Avoid on face
Demonstrate an understanding of and skin folds
the value of further investigation Tar preparations: act by inhibiting DNA
ASO titre/throat swab in suspected recent synthesis (anti-mitotic effect). Have a strong
streptococcal infection. aroma, messy, irritant
Screening blood tests, including hepatitis Topical steroid ointments reduce scaling and
serology, HIV, and TB Elispot especially if there erythema. They can cause dermal atrophy
and show tachyphylaxis
216 Chapter 5 Station 5

For generalised or moderate-severe form for emergencies such as erythroderma or


refer to dermatology for consideration of pustular psoriasis to induce remission. Needs
phototherapy and/or systemic treatment. monitoring of blood pressure and serum
creatinine.
Phototherapy Fumaric acid esters and hydroxychloroquine
are other alternatives.
Narrowband UVB phototherapy is especially good
Methotrexate and biologics, such as
for thin plaques. It is very effective when used in
adalimumab, etanecerpt and ustekinumab and
combination with topical vitamin D analogues.
may additionally control psoriatic arthritis.
PUVA photochemotherapy involves ingestion
Patients need to have failed conventional
of 8-methoxypsoralen and exposure to UVA.
systemic treatment and have a DLQI and PASI
Beware of long-term the side-effects, i.e.
>10 to qualify for treatment.
increased incidence of skin cancers.

Systemic treatment Disease monitoring


Disease monitoring can be achieved with the
Acitretin (may be used alone or in combination
Psoriasis Area and Severity Index (PASI). The
with light treatment). Combination therapy
intensity of redness, thickness and scaling on
improves the efficacy of each and allows for
a scale of 04 is assessed for a representative
reduced dose and duration of treatment.
area for each body region. Each subtotal is then
Methotrexate: beware of potential liver
multiplied by the body surface area for that
toxicity especially in the presence of high
region and the results are added together to give
alcohol intake, intravenous drug use, and
the PASI score. The DLQI score is also helpful
abnormal baseline liver function.
in assessing the impact the disease has in the
Ciclosporin should only be used in patients
patients life.
without risk factors and is usually used

Case 120: Atopic dermatitis or


atopic eczema

Instruction to the candidate Assess the effect of the patients symptoms on


their daily life.
Examine this 27-year-old womans skin and
discuss diagnosis and therapeutic options.
Positive findings of a focused
Take a focused history examination
On the basis of the clinical findings on general The patient may have a diffuse pattern of
inspection, see below, assess for the presence of eczema, with widespread, dry, lichenified skin
the triad of atopy childhood eczema, asthma, or persistent localised eczema, affecting areas
and allergic rhinitis/hayfever. Screen for food like the hands and eyelids (Figures 5.22 and 5.23
allergies and/or a family history of atopy. for clinical photographs of eczema) The rash is
Ask about exacerbating factors of the patients characterised by:
symptoms, such as infections, allergens (e.g. Itchy erythematous dry scaly patches
dust, grass, pollen), sweating, heat, and stress. Excoriations and lichenification from chronic
Obtain an occupational history and potential scratching
exposure to irritants such as detergents or other Fissures (hands and feet)
chemicals. Atopic eczema is a major contributor
to occupational irritant contact dermatitis. Consider further examination of the hands
Enquire about previous treatments used and for nail pitting or ridging.
their impact. Consider asking about history of
cold sores (risk of eczema herpeticum).
Case 120 Atopic dermatitis or atopic eczema 217

Figure 5.22 Eczematous


rash. This patient, with ectopic
eczema, has an itchy rash on the
flexural aspect of forearm. There is
thickened, dry skin (lichenified) with
evidence of excoriation.

Figure 5.23 Eczematous rash.


Eczematous lesions involving both
the flexural aspect of the elbow and
the dorsum of the hand.

Relevant negatives Diagnostic criteria for atopic eczema


The absence of signs suggestive of
superimposed infection is an important An itchy skin condition with three or more of:
negative finding. Onset <2
Skin crease involvement
Dry skin
What to tell the patient Personal history of atopy
Atopic eczema is an inflammatory condition of Flexural dermatitis
the skin. In people with atopic eczema, a gene
alters the skin barrier function, so irritant and
allergy-inducing substances can enter the skin
Offer relevant differential
causing dryness and inflammation. Emollients diagnoses
help to protect the skin from the outside world. The The differential diagnosis would include allergic
inflammation is treated with topical steroids and, or irritant contact dermatitis, however in such
if severe, other treatments such as light therapy or cases the site of eruption usually confined to
systemic medication may be considered. I would exposure areas.
like to see you again in 3 months time to assess Seborrhoeic dermatitis, another differential,
your progress and discuss your results. I will also is more likely in areas with sebaceous glands
be writing to your GP about todays visit. such as face, scalp, sternum, body folds.
Urticaria does not tend to be scaly.
What to tell the examiner state
the most likely diagnosis on the Demonstrate an understanding of
basis of the history and clinical the value of further investigation
findings Investigations should seek to exclude skin
infection and rule out precipitating factors.
The presence of a symmetrical, itchy, dry Investigations may include:
flexural skin rash together with a history of
asthma and hayfever are strongly suggestive of a Skin swabs and skin scrapings for suspected
diagnosis of atopic eczema. superimposed infection
218 Chapter 5 Station 5

Laboratory blood tests, including total IgE induce clearance and introduce tacrolimus
and specific IgEs to comomn allergens, a as maintenance treatment. A stronger topical
full blood count to identify eosinophilia, steroid such as betamethasone, mometasone or
and iron studies (iron deficiency can lead to even clobetasone propionate may be needed for
pruritis) body eczema. A soap substitute such as Dermol
Patch testing is useful to exclude contact 500 and an emollient such as Cetraben or
dermatitis Epaderm should be used on a daily basis.
For generalised or moderate-severe
form atopic eczema, the patient may
Always offer a management plan require phototherapy or systemic treatment
All patients regardless of severity require with immunosuppressive agents such as
emollients and soap substitutes. azathioprine, methotrexate and ciclosporine.
For localised or mild disease, topical Biologic therapies can be used for recalcitrant
steroids are used for flare-ups, with topical eczema non-responsive to conventional
immunomodulators such as tacrolimus or treatment.
pimecrolimus as steroid sparing agents for Antibiotics are used for secondary bacterial
maintenance or control of minor flares. infections. Antihistamines may helo reduce
For example, a patient may use topical steroids irritation and itching and are especially helpful
such as hydrocortisone or clobetasone at nightime.
butyrate ointment to the face and neck to

Case 121: Pyoderma gangrenosum

Instruction to the candidate Disease associations with pyoderma


This 37-year-old woman has been referred gangrenosum
by her gastroenterologist for an opinion
in relation to the lesion on her right lower Crohns
leg. Take a short history and examine this Ulcerative colitis
patients skin lesion and discuss diagnosis and Diverticulosis
therapeutic options. Rheumatoid arthritis
Paraproteinaemia
Myeloma
Take a focused history Leukemia
Establish the clinical history. Pyoderma Behet syndrome
gangrenosum is an uncommon cause of very Active chronic hepatitis
painful skin ulceration. The lower legs are the PBC
commonest site. Establish the evolution of
the lesion: Pyoderma usually starts as a small
pustule, red bump or blood blister which may
Positive findings of a focused
be painful and occur either de novo or after examination
minimal trauma. The skin then breaks down, Suspect pyoderma gangrenosum in the presence
resulting in an ulcer. of a painful ulcer with characteristic purple edge
Adopt a systems-based approach to identify and an elevated border particularly on the lower
associated systemic disease, which is present leg (Figure 5.24).
in 50% of cases. Ask about gastrointestinal Pyoderma gangrenosum demonstrates
symptoms (inflammatory bowel disease), joint pathergy (exaggerated skin injury after minor
involvement (rheumatoid arthritis) and any trauma). A skin prick test causes a papule/pustule/
history of haematological disease (myeloma). ulcer. This should not be performed in the exam.
Case 121 Pyoderma gangrenosum 219

will have to take a small biopsy to confirm


the diagnosis. The ulcer is likely to heal with
treatment for your inflammatory bowel disease
but if not we have other options to consider. It is
likely to take some time to heal but is unlikely to
require surgery or skin grafting.

What to tell the examiner state


the most likely diagnosis on the
basis of the history and clinical
findings
The history of rapid evolution of the skin
ulcer in combination with a background
inflammatory bowel disease is suggestive of a
diagnosis of pyoderma gangrenosum.

Offer relevant differential


diagnoses
The differential diagnosis of pyoderma
gangrenosum includes:
Localised gangrene
Figure 5.24 Pyoderma gangrenosum. This shallow
Ecthyma
ulcer has a classical violaceous border. The ulcer bed
Atypical mycobacterial infection
is granulating. If this finding is identified on clinical
Stasis ulcer
examination, proceed to examine for signs associated
Squamous cell carcinoma
with common causes of pyoderma gangrenosum, as
discussed in the text. Demonstrate an understanding of
the value of further investigation
Pyoderma gangrenosum is usually diagnosed
Perform further examination for signs of an clinically. Investigations are aimed at identifying
underlying causative disease: underlying disease or ruling out infection. Take a
Mouth ulceration e.g. Crohns, skin swab to exclude superimposed infection. If
myelodysplastic syndromes needed, a skin biopsy can be perfomed to confirm
Conjunctival involvement the diagnosis (taken from edge, needs to be deep).
Joint involvement e.g. rheumatoid arthritis Laboratory blood tests to exclude underlying
associated disease:
Immunology: ANA/ANCA, rheumatoid factor
Relevant negatives Full blood count
It is important to note the absence of Inflammatory markers raised in inflammatory
lymphadenopathy, which might be present bowel disease
in the setting of superadded infection or
in the case of an advanced squamous cell
carcinoma (a differential diagnosis of pyoderma
Always offer a management plan
gangrenosum). Treatment of the underlying disease and topical
or systemic treatment for the ulcer.
Topical therapies include strong steroid
What to tell the patient ointments or intralesional steroids. Involve the
I suspect the ulcer you have on your leg is tissue viability nurse for help with dressings and
pyoderma gangrenosum, which is a rare consideration of supplementary treatment, such
treatable cause of skin ulcers and is commonly as potassium permanganate. Symptom control
associated with gastrointestinal disease, involves simple analgesia to manage the pain of
especially inflammatory bowel disease. We the ulcer.
220 Chapter 5 Station 5

Systemic options (Table 5.2) include oral azathioprine. Biologic therapies (infliximab,
prednisolone and immunosuppressive agents etanercept, adalimumab) are also used.
such as mycophenolate mofetil, ciclosporin and Antibiotics are indicated for superimposed
infection.

Table 5.2 Immunosuppressive and biologic agents for pyoderma gangrenosum


and a wide range of conditions
Immunosuppresant Mode of action Side effects
Mycophenolate Inhibits lymphocyte proliferation Taste disturbance, gingival hypertrophy,
mofetil (MMF) and antibody production gastrointestinal disturbance, skin cancer,
myelosuppression
Ciclosporin (CyA) Calcineurin inhibitor Hypertension
Nephrotoxicity
Hypertrichosis
gingival hyperplasia/bleeding gums
Ototoxicity
Hyperuricaemia, tremor, paraesthesiae, skin cancer
Lymphoproliferative disorders
Methotrexate (MTX) Dihydrofolate reductase inhibitor Myelosuppression
Pulmonary and hepatic fibrosis
Teratogenic
Ulcerative stomatitis
Azathioprine (AZT) Pro-drug for purine synthesis Bone marrow toxicity, liver, interstitial nephritis,
inhibitor non-Hodgkins lymphoma, small cell carcinoma,
Azoospermia
Cyclophosphamide Nitrogen mustard alkylating agent Pancreatitis, cardiotoxicity, pulmonary fibrosis, SIADH,
causing cell death hyperpigmentation
Rarely hepatotoxic/nephrotoxic
Biologics
Infliximab Chimeric monoclonal antibody Lymphoma, reactivation of hepatitis, tuberculosis,
against TNFa drug induced lupus, demyelinating CNS disease
Etanercept Fusion protein, TNF inhibitor
Adalimumab Human monoclonal antibody
against TNFa
SIADH, syndrome of inappropriate antidiuretic hormone; TNF, tumour necrosis factor.
Case 122: Vasculitis
(HenochSchnlein purpura)

Instruction to the candidate Figure 5.25


Vasculitis.
This 38-year-old patient has recently been There is
treated by his GP for suspected gastroenteritis. a purpuric
He presents with a rash on the legs. Please assess rash with
and suggest management. petechiae and
ecchymoses
Take a focused history affecting the
lower limb.
When presented with a rash and suspected
Although
systemic features always think about a
in Henoch
diagnosis of vasculitis. The following history
Shnlein
points should be checked for any type of
purpura the
suspected vasculitis:
vasculitic rash
General symptoms: general malaise, is typically
headache, anorexia, myalgia, fever distributed over
Eyes: episcleritis, visual loss the buttocks
Respiratory: dyspnoea, haemoptysis and posterior
Cardiovascular: angina, heart failure aspect of the
GI: abdominal pain, diarrhoea lower limbs,
Neurology: sensorimotor neuropathy, fitting, the diagnosis
ataxia, chorea, psychosis, mood disturbance should be
considered in
HSP usually presents with a prodromal
the presence of
illness of general malaise, headache, fever and
any vasculitic
anorexia. Associated features include:
rash. Indeed,
Arthralgia: usually poly-arthralgia in ankles, appropriate
knees elbows and small joints of hands investigations (a
GI upset: abdominal pain, colic, vomiting, vasculitic screen) should be carried out for any patient
diarrhoea who presents with such a rash.
The rash may persist after all other
symptoms have settled or recurrent attacks Positive findings of a focused
may occur examination
Ask about the rash. Did it begin as a crop? Examine the legs, buttocks, and flanks. Look for
Is it painful or pruritic? Commons sites palpable purpura (palpable petechiae, bright
include the limbs and buttocks. Is there an red well demarcated macules and papules with
urticarial component? This is very common central dot-like haemorrhage). Lesions may be
and characteristic in HSP vasculitis which scattered, discrete or confluent (Figure 5.25).
may present as localised oedema or urticarial Consider examining the abdomen.
lesions.
Screen for other causes of vasculitis, such Relevant negatives
as:
In any patient with purpura and systemic
Drugs: aspirin, non-steroidal features, it is important to note the absence of
anti-inflammatories, antibiotics signs suggestive of meningism. Suggest a bedside
Infections such as streptococcal sore throat urine dipstick to look for renal involvement as
Autoimmune connective tissue disease evidenced by proteinuria or haematuria.
(lupus, rheumatoid arthritis, Sjgrens)
Blood disorders such as cryoglobulinaemia
Renal involvement is the most serious feature What to tell the patient
of the disease, and may present as nephritic I suspect the rash on your legs and buttocks is
or nephrotic syndrome related to your recent illness with gastroenteritis.
222 Chapter 5 Station 5

We will need to run some more tests to confirm. Demonstrate an understanding


The rash should clear within 36 weeks but I
must warn you that recurrences are common of the value of further
in up to half patients. This condition, called investigation
HenochSchnlein purpura, may occasionally Investigations in vasculitis depend on an
affect other body organs, such as your kidneys adequate history taking to identify triggers
and it is important to keep a close eye on or altered host response. They are aimed at
your kidney function with weekly urine tests. identifying triggers but also factors enhancing
Treatment is symptomatic at this stage, but I will vascular injury such as coagulation as well
arrange to see you again in a weeks time and if as identification of target organ damage.
the rash is spreading or your kidneys become Candidates should be able to identify the distinct
affected, we will start you on oral steroid tablets. patterns of HSP or erythema nodosum and tailor
investigations appropriately as follows:
What to tell the examiner state Investigations to identify triggers and
the most likely diagnosis on the enhancing factors include:
basis of the history and clinical Full blood count and clotting for
thrombocytopenia and disseminated
findings intravascular coagulation
This man presents with a palpable purpuric Inflammatory markers
eruption on his legs and buttocks and a ANA (especially when very high ESR to
preceding history of abdominal pain and rule out lupus)
arthralgia, which would be consistent with Throat swab and ASO-titre to exclude
HenochSchnlein purpura. streptococcal infection
Skin biopsy if diagnosis not clear
Vasculitis Evidence of systemic involvement can
Vasculitis is a group of conditions characterised be identified through the following
by histological changes in small and medium investigations:
sized blood vessels of the dermis. Vasculitis can be Urinalysis to look for casts and
clinically divided into cutaneous and visceral or proteinuria. Red cell casts indicate
mixed forms. glomerular injury, white cell casts acute
inflammation or infection, epithelial casts
severe renal tubular damage. Patients
Offer relevant differential should have weekly urinalysis during the
diagnoses attack, at the end of the attack, and for
13 months after the attack has resolve.
The diagnosis is usually clear when all
components of the syndrome are present.
Plasma creatinine
In young children appendicitis, mesenteric
Blood pressure
adenitis and rheumatic fever may present
similarly. Always offer a management plan
Other causes of purpuric eruption can be Therapy based on eliminating triggers and
divided into vasculitic causes, thrombocytopenic supporting host defences. Prognosis depends
causes and non-thrombocytopenic causes. on systemic involvement. General measures
Thrombocytopenic causes include meningococcal include rest and elimination of trigger factor if
disease, disseminated intravascular coagulation identified. Underlying causes should be treated
and idiopathic thrombocytopenic purpura. Non- where identified. Topical steroids are given
thrombocytopenic causes include trauma, drug- for rash. Oral steroid and immunosuppressive
induced purpura, and senile purpura. therapy is indicated in systemic involvement.
Case 123: Chronic venous insufficiency

Instruction to the candidate prevent healing), rheumatoid arthritis, history of


anaemia or malnutrition.
This 65-year-old patient has been referred by his
GP with difficult to manage eczema on his lower
legs. Please assess her legs and suggest further Positive findings of a focused
management. examination
In any elderly patient presenting with lower
Take a focused history limb eczema suspect the diagnosis of chronic
Chronic venous insufficiency arises from venous insufficiency. The cardinal features that
failure of blood venous return (from damage to are sought on examination are oedema, eczema,
valves of deep veins or communicating veins) pigmentation, thickening and fibrosis with or
and raised capillary pressure. Seek to elicit the without ulceration.
following symptoms: Look for stasis eczema with scaling and
thickening of the skin, especially near the
Pain: pain secondary to venous insufficiency ankles. Acute dermatitis may also be present
will often present as leg heaviness or aching, with exudates and inflammatory papules.
worse on standing and relieved by walking Haemosiderin deposition can be found over
(in contrast to arteriopathic pain worsening varices and ankles.
with walking and relieved by rest) Lipodermatosclerosis describes thickened,
Oedema: this will typically be worse on indurated, hyperpigmentaed skin creating classic
standing, or at the end of the day and in warm inverted Champagne-bottle appearance of legs.
weather. Ask about shoe tightness and sleeping Associated features include:
arrangements (often elderly people sleep in a
chair with gravitational effects to legs) Varicose veins (superficial, enlarged,
Erythrocyte extravasation: does the patient tortuous veins, best evaluated in standing
have easy bruising after minimal trauma position)
Ulceration: this may be painful and worse on Venous ulcers (Figure 5.26)
standing Atrophie blanche: white atrophic plaques
Itching: as a result of leaky capillaries with prominent red dots within (enlarged
capillary blood vessels) and surrounding
Regarding the past medical history, is there pigmentation on lower legs and feet
a history of venous thrombosis? This can lead
to post-phlebitic syndrome with similar clinical
appearance. Is there a history of varicose Relevant negatives
veins that an damage the valves? Screen for a It is important to note the presence of peripheral
history of atherosclerotic disease (important pulses as venous ulceration can occasionally
in the differential of ulceration), diabetes (may co-exist with arterial disease.

Figure 5.26 Venous ulceration.


This patient has multiple areas of
shallow ulceration in the medial
gaiter area of the lower limb. .
Note the inverted champagne
bottle appearance, yellow crusting
suggestive of secondary infection
and shiny, woody texture of the skin
around the ankles.
224 Chapter 5 Station 5

Take care to distinguish from other types


of ulceration (Figure 5.27, Figure 5.28 and
Table 5.3). Look for signs of eczema elsewhere
which might indicate the patient has a history of
atopy and therefore predisposition to eczema
or an autosensitisation reaction, i.e eczema that
has spread to affect other areas of the body

What to tell the patient


The changes I can see today on your lower
legs are probably related to the veins in your
leg being unable to efficiently pump back the
blood to your heart. As a result the blood pools
in your legs causing these changes we can see.
We will need to run some tests to check how well
both the veins and arteries in your legs function
before choosing the best treatment, as it may
involve compression bandaging. The most
important thing you can do to help your legs
is to keep them elevated. I will prescribe some
emollients and topical steroid ointments until
I next see you. Do you have any questions?

What to tell the examiner state


the most likely diagnosis on the
basis of the history and clinical Figure 5.28 Neuropathic ulcer. Any ulcer on a
findings pressure area, as this one is, should prompt the
candidate to examine for any associate abnormal
This patient has an eczematous rash on
sensation and to consider conditions which can cause
both lower legs with signs of chronic venous
a peripheral sensory neuropathy e.g. diabetes.
insufficiency and secondary ulceration.

Figure 5.27 Arterial ulceration.


There is a punched-out ulcer on
the dorsum aspect of the foot.
There are associated atrophic
changes. This patient would
require examination of the pulses
and clinical assessment for severe
vascular disease.
Case 123 Chronic venous insufficiency 225

Table 5.3 Clinical features of venous, arterial, and neuropathic ulcers*


Venous Arterial Neuropathic
History
Pain Often painful Painful at night Painless
Worse on standing

Past medical history Varicose veins, DVT Arterial disease elsewhere Diabetes or neurological
disease
Sites affected Malleolar area Pressure and trauma sites Pressure sites
(medial>lateral) Pre-tibial, supra-malleolar Soles
(lateral) Heels
Metatarsal heads
Ulcer characteristics
Size Large Small Variable size

Depth Shallow Deep Variable depth

Contour Irregular Sharply defined Occasionally surrounded by


hyperkeratotic area
Exudative
Base granulating base Necrotic base Granulating base
Associated features
Surrounding skin Warm Cold, shiny/pale Warm

Peripheral pulses Normal Weak or absent Normal

Other Leg oedema Shiny pale skin Peripheral neuropathy


Haemosiderin deposition Hair loss
Lipodermatosclerosis
Atrophie blance
Investigations ABPI ABPI <0.8 (ABPI not reliable in patients
Swabs Doppler ultrasound with diabetes)
Doppler ultrasound Angiography X-ray to exclude OM
Management Compression bandaging Vascular reconstruction Wound debridement
No compression Regular repositioning,
footwear/nutrition
* See also Figures 5.265.28.
NO, nitric oxide; ABPI, ankle-brachial pressure index..

Offer relevant differential Demonstrate an understanding


diagnoses of the value of further
The differential diagnosis of a lower leg ulcer investigation
includes arterial and neuropathic ulcers, Investigations are aimed at searching for
neoplasia and trauma. adjuvant factors and associated disease such as
The differential diagnosis of an erythematous anaemia and malnutrition.
or oedematous leg includes cellulitis/erysipelas Laboratory blood tests:
(may be co-existent) and venous thrombosis
(usually unilateral). Full blood count for anaemia
Inflammatory markers
226 Chapter 5 Station 5

Liver function for low albumin and serum Always offer a management plan
protein
The principles of management are aimed at
Glucose as diabetes may discourage healing
reducing swelling in the leg and treating the
Ulcer swabs to rule out suprimposed venous dermatitis. Care is delivered through
infection. Patch tests to identify a contact allergy a multidisciplinary team with dermatology,
if the patient has been using topical treatments vascular surgery and tissue viability nursing.
and not improving Options include:
Imaging:
General measures: leg elevation and exercise
Plain films to rule out rheumatoid arthritis Antibiotics for secondary infection
and osteomyelitis Compression stockings (provided no
Arterial and venous Doppler ultrasound to significant arterial disease) and bandaging in
assess venous competence, thrombosis or the presence of ulceration
co-existent arterial disease that may preclude Local applications such as potassium
compression bandaging permanganate to soak up exudates, remove
Ultrasound abdomen in suspected slough
abdominal mass causing venous obstruction Emollients
Consider ABPI measurement.
Short-term topical steroids

Case 124: Cutaneous lupus erythematosus

Instruction to the candidate Thromboembolic disease and a history of


miscarriage
This 34-year-old woman complains of a facial
rash. Please examine the patient, ask any
relevant questions and provide a management Antiphospholipid syndrome
plan.
In the context of:
Thromboembolic disease
Take a focused history Stroke
In any facial rash you need to establish whether Migraine
it is photosensitive or not and ask about specific Miscarriages
precipitating factors such as sunlight and drugs. Multi-infarct dementia
Diuretics, hydralazine, procainamide, isoniazid Livedo reticularis (Figure 5.29)
and anticonvulsants can all cause drug-induced Always think of antiphospholipid syndrome and
lupus that remits on withdrawal of the offending test for: lupus anticoagulant and antiphospholipid
drug. Sulphonamides and the combined oral antibody.
contraceptive may trigger idiopathic SLE.
Screen for constitutional symptoms, such Positive findings of a focused
as fatigue, malaise, fever, weight loss, arthralgia
and flu-like symptoms. examination
Proceed to a systems-based approach: Skin
Renal disease It is of paramount importance to do a full body
Cardio-respiratory involvement with pleuritic examination after examining the presenting
pain/pericarditis (serositis) rash, otherwise important signs might be
Neurological: seizures in the absence of missed. Note that peripheral skin examination
causative drugs, psychosis may be normal.
Case 124 Cutaneous lupus erythematosus 227

Face Further examination


Malar (butterfly) rash: A sharply defined Further examination should be guided by the
erythematous, confluent, macular rash, affecting clinical history. It may involve:
cheeks and nose and sparing the nasolabial
folds (Figure 5.30). Cardiovascular examination, looking for a
Discoid lupus erythematosus (DLE) pericardial rub (pericarditis) or signs of a
predominantly affects the cheeks, nose and ears pericardial effusion
(ocasionally the upper back and V of the neck), Respiratory examination to look for evidence
with red scaly patches that heal with scarring of pleural effusion, pleural rub or lung
and may be associated with a scarring alopecia. fibrosis and lymphadenopathy
It is usually not associated with systemic Neurological examination to look for
involvement but up to 5% of patients may focal neurological signs, chorea or ataxia,
progress to systemic lupus. psychosis
Musculoskeletal examination to identify a
Discoid rash: erythematous, raised patches with peripheral arthropathy affecting at least two
scaling and follicular plugging (Figure 5.31 and peripheral joints (present in 90% of patients).
5.32) Jaccouds arthropathy may also be noted
Associated facial oedema may be noted. Also An abdominal examination may reveal
look at the mouth for oral ulceration and the splenomegaly
scalp for alopecia areata or diffuse hair shedding.
Lupus erythematosus
Trunk/limbs
Lupus erythematosus covers a group of related
Livedo reticularis, mottled bluish discolouration disorders, all of which can affect the skin. The
of skin in a net-like pattern, may also be found spectrum ranges from the purely cutaneous type
when examining the limbs. (discoid LE), intermediate type associated with
Signs in the hands may include, peri-ungual internal problems (subacute cutaneous LE) to
telangiectasia, digital erythema or Raynauds severe multi-system disease (systemic LE).
phenomenon.

Figure 5.29 Livido reticularis. On


patients upper limb,there is a blue-
purple net-like appearance of livido
reticularis. May also be found in SLE.

Figure 5.30 Malar rash. There is


confluent erythema in a butterfly
distribution. Remember to
consider diagnoses other than
systemic lupus erythematosus
when presented with a malar
rash, especially in male or in older
patients, such as the malar flush of
mitral valve disease.
228 Chapter 5 Station 5

Figure 5.31 Discoid rash. This


patient has a discoid, red rash on
the left cheek with evidence of
scarring.

Figure 5.32 Discoid lupus. A


patient with discoid lupus on the
praecordium.

Relevant negatives many treatments are able to help it. You will
require long-term follow-up. I will refer you for
The absence of systemic features of SLE (as listed
an opinion from the heart and nerve specialists
above) would be important relevant negatives
for the symptoms you have described to me to
in a patient presenting with a malar rash. Given
rule out other organ involvement. I will review
that mitral stenosis is a differential diagnosis,
your progress in a few weeks time.
the absence of auscultatory features of mitral
stenosis is also important.
What to tell the examiner state
What to tell the patient the most likely diagnosis on the
I suspect you have a condition called systemic basis of the history and clinical
lupus erythematosus. This is an auto-immune findings
condition that usually affects other body
organs but occasionally only the skin. The most This patient appears to have systemic lupus
important thing you can do is to protect your erythematosus.
skin from sunlight. There is no instant cure but
Case 124 Cutaneous lupus erythematosus 229

A raised ESR (in the context of a normal CRP)


American Rheumatism Association Renal function and urinalysis for
diagnostic criteria for SLE identification of proteinuria or haematuria or
To reach a diagnosis of SLE, a patient should meet
red cell casts
at least four of the following criteria: Auto-antibody profile: The ANA is positive in
Malar rash 95% with anti-dsDNA antibodies being highly
Discoid rash specific for SLE. Anti-Sm antibodies are also
Photosensitivity highly specific. Rheumatoid factor is positive
Oral ulcers in 40%. Anticardiolipin antibodies, which may
Arthritis give rise to false-positive tests for syphilis, are
Serositis positive in antiphospholipid syndrome. Anti-Ro
Renal disorder antibodies are characteristic in Subacute LE. If a
CNS disorder mother with LE carries the antibody the neonate
Haematological disorder is at risk of neonatal LE with congenital heart
Positive immunology block, hepatic and haematological involvement.
ANA positive Other tests include: lupus anticoagulant and
complement levels (expect to be low).
Offer relevant differential
diagnoses Always offer a management plan
The differential diagnosis of a malar rash General measures involve rest and sun-
includes mitral stenosis, sunburn, polymorphic avoidance or appropriate sun protection, with
light eruption, and rosacea. analgesia for arthralgia.
Hydroxychloroquine is indicated in mild
disease with cutaneous or joint involvement,
Demonstrate an understanding because of its anti-inflammatory properties.
of the value of further Regular renal function tests and eye checks are
necessary.
investigation In moderate/severe disease, systemic
Investigations are aimed at confirming the steroids and immunosuppressants are used,
diagnosis and monitoring disease activity. especially in the presence of central nervous
A skin biopsy with immunofluorescence may system or renal involvement, severe illness,
be carried out. Typical biopsy features include haemolytic crisis, and thrombocytopenia.
epidermal atrophy, liquefaction degeneration Other drugs may be needed to manage
of dermoepidermal junction, with fibrinoid organ-specific complications, e.g.
degeneration of connective tissue and walls of anticonvulsants, antihypertensives.
blood vessels. Immunofluorescence (lupus band Think of conception and pregnancy. Fertility is
test) will show positive antibody deposition not usually affected in people with SLE, but some
along the basement membrane (IgG, IgM and women have a higher chance of miscarriage.
C3 granular or global deposits). Blood pressure can be affected during pregnancy
Haematological blood tests may show some in the presence of kidney involvement. Most
or all of the following: women with mild or well-controlled SLE are
likely to have an uneventful pregnancy. In the
Anaemia, which may be normocytic or due to
presence of maternal anti-Ro antibodies, as these
haemolysis
cross the placenta, the baby may be born with
Thrombocytopaenia
neonatal lupus: a temporary ring-like (annular)
Lymphopaenia
rash and risk of congenital heart block.
Case 125: Discoid lupus erythematosus

Instruction to the candidate tablets. You should protect yourself from sunlight
and always use high SpF sunscreen.
This 32-year-old patient complains of a body
rash. It has not responded to treatment for
eczema and psoriasis. Please examine the What to tell the examiner state
patient and discuss diagnosis and treatment. the most likely diagnosis on the
basis of the history and clinical
Take a focused history
findings
In any facial rash you need to establish whether
it is photosensitive or not and ask about specific This patient has a scarring, discoid rash in a
precipitating factors such as sunlight and photosensitive distribution without systemic
medication. features, in keeping with discoid lupus
As with SLE screen for the presence of erythematosus.
constitutional symptoms suggestive of systemic
involvement. Offer relevant differential
diagnoses
Positive findings of a focused Psoriasis is a key differential but this is usually
examination larger, thicker scales, symmetrical, involving
The rash in discoid lupus consists of elbows, knees, scalp, and sacrum.
well-demarcated, red, atrophic, scaly plaques Fungal skin infection is another alternative
with keratin plugs in dilated follicles (grated diagnosis, but this is likely to be a solitary,
nutmeg). The rash occurs in sun-exposed areas: annular lesion with a peripheral rim of scale.
scalp, face, ears but may also affect upper back,
V-neck and dorsum of hands. This is a scarring Demonstrate an understanding of
disease therefore may leave post-inflammatory
scarring and hypopigmentation.
the value of further investigation
Consider further examination for scarring Discoid lupus has a low incidence of ANA with
alopecia or discoid lesions on the patients scalp. titres >1:16.
A skin biopsy with immunofluorescence
should be performed. This will show
Relevant negatives hyperkeratosis, epidermal atrophy and follicular
Discoid lupus usually has no systemic plugging.
involvement so a wider focused examination The Lupus Band Test (LBT) may be positive
should reveal none of the multi-system in 90% of active lesions.
clinical signs discussed in Cutaneous lupus
erythematosus (p. 226).
Always offer a management plan
The patient should administer topical steroids
What to tell the patient twice daily until clear (ensure you give advice
I suspect you have a condition called discoid regarding long-term side effects). Intralesional
lupus erythematosus (DLE), which causes this triamcinolone if topical steroid therapy fails.
kind of rash in sun-exposed areas. There is a wide Hydroxychloroquine is an option for extensive
range of related disorders which can damage or recurrent skin involvement (ensure pre-
internal organs but the type you have is usually treatment eye tests).
confided to the skin and does not cause general Patients should wear a high SpF sunblock.
ill health. The cause is not fully understood but it Cosmetic camouflage may also be offered.
is likely to be an auto-immune condition. There In severe cases when there is no response
is not a curative treatment, but the skin rash can to anti-malarials azathioprine, mycophenolate
improve with topical steroids and anti-malarial mofetil and methotrexate may also be used.
Case 126: Skin manifestations of systemic
disease dermatomyositis

Instruction to the candidate may be erythema, or blue-purple patches, of


other sun exposed areas such as the face, neck
This 62-year-old man has been referred to the
and upper trunk.
dermatology clinic with a facial rash. Please
Go on to examine the hands for the following
examine the patient, take any relevant history
clinical signs:
and discuss further management.
Gottrons papules: flat topped violaceous
papules over bony prominences, especially
Take a focused history the knuckles
In patients in whom the focused clinical Mechanics hands: rough, cracked skin on
examination (see below) reveals signs of palms and fingertips
dermatomyositis ensure you ask specific Dilated capillary loops: periungual erythema
questions on the following: with telangiectasia, thrombosis of capillary
General malaise, fevers, weight loss loops, infarctions
Muscle weakness, usually symmetrical and See Figures 5.335.36 for clinical photographs.
proximal (rash often precedes myositis in 56% Seek to elicit signs of proximal myopathy. Ask
of patients). Such proximal weakness may be the patient to rise from a seated position. Ask
demonstrated as difficulty rising from supine them to raise their arms above their head.
position, climbing stairs, or raising arms over
head
Raynauds phenomenon Relevant negatives
Lung involvement A lack of obvious clinical findings pointing
Arthralgia towards a solid organ malignancy is an
Dysphagia, nasal speech and regurgitation important negative finding to make.
from bulbar involvement
Gut motility problems
What to tell the patient
In patients >40 consider underlying I suspect you have a condition called
malignancy. Lung, breast, gastrointestinal tract, dermatomyositis. It is an inflammatory
and ovary are the commonest associations. condition affecting the skin and muscles and
usually has a good prognosis. We will need to
Positive findings of a focused run some more tests to make sure no other body
organs are affected and to find a possible cause.
examination You will require treatment with oral steroids to
The key clinical finding is a heliotrope rash, a begin with and depending on your response
periorbital and malar violaceous (lilac-blue) we will see if you require stronger agents to
flush with associated periorbital oedema. There suppress the inflammation.

Figure 5.33 Dermatomyositis.


Photosensitive rash in a patient with
dermatomyositis. Note the well-
demarcated rash in a sun exposed
area of the chest.
232 Chapter 5 Station 5

Figure 5.34 Dermatomyositis.


This patients finger demonstrates
periungual erythema and small red
patches on the bony prominence
of the distal interphalangeal joint.
The rest of the hands should be
examined to look for associated
signs such as Gottrons papules. An
examination of the hands is likely to
be prompted by the identification
of a heliotrope rash.

Figure 5.35 Heliotropic


rash. There is a peri-orbital
violaceous hue consistent with
dermatomyositis. Be sure to
examine other sun-exposed areas
and take a close look at the hands.

Figure 5.36 Hand signs in


dermatomyositis. Gottrons
papules are present on the dorsal
aspect of the metacarpophalangeal
and proximal interphalangeal joints.
There is also rough, dry skin on
the fingertips and ragged cuticles,
colloquially known as mechanics
hands.

What to tell the examiner state associated proximal muscle weakness. The most
likely diagnosis is dermatomyositis.
the most likely diagnosis on the
basis of the history and clinical Offer relevant differential
findings diagnoses
This patient presents with a heliotrope rash and a
The differential diagnosis of a periorbital rash
rash in a photosensitive distribution over the chest
includes: seborrhoeic dermatitis, periorbital
and hands, where there are also signs of Gottrons
contact allergic dermatitis, ocular rosacea.
papules and periungual erythema. There is also
Case 127 Alopecia areata 233

Demonstrate an understanding of A skin biopsy may be performed if there is


diagnostic uncertainty.
the value of further investigation Importantly, a diagnosis of dermatomyositis
Blood tests can both demonstrate autoimmune would prompt an appropriate search for
serology and give evidence of myositis. Anti-Jo1 underlying malignancy.
antibodies are positive in 80% and are highly
specific. ANA is positive in 40% of patients but
has a low specificity. Creatine kinase may be
Always offer a
elevated, demonstrating myositis. management plan
Raised CK Systemic therapy options include:
ANA positive in 40% of patients but has low
specificity Oral steroids
Anti-Jo1 (highly specific) and anti-Mi2 Methotrexate, cyclophosphamide,
(chromosomal helicase DNA binding ciclosporin
protein) Intravenous immunoglobulin

The finding of weakness would prompt Management is also targeted at any


investigation with EMG, MRI, and muscle biopsy underlying malignancy that may have been
(gold standard for diagnosis). identified. Dont forget to offer physiotherapy
A 12-lead electrocardiogram may in the presence of muscle weakness and advise
demonstrate myocarditis or AV block in the sun-protection measures.
appropriate clinical setting.

Case 127: Alopecia areata

Instruction to the candidate Positive findings of a focused


This 40-year-old woman has attended your examination
clinic complaining of hair loss. Please assess the On the scalp, look for diffuse or patchy hair loss.
patient and provide a management plan. Alopecia areata is demonstrated by a well-
circumscribed, totally bald, smooth patch(s).
Take a focused history There are exclamation-mark hairs at the borders.
Aim to establish the type of hair loss , speed The eyebrows and beard may also be affected.
of onset (in alopecia areata the hair loss is New hair growth may be white or gray. See
sudden), and any potential causes. Establish if Figures 5.375.39 for clinical photographs.
the alopecia is diffuse or localised. See Table 5.4 If alopecia areata is identified. Occasionally
for causes of hair loss according to type, which the whole scalp is affected (Alopecia totalis)
can guide the focused history according to what or the whole body hair is lost (Alopecia
the patient describes and the clinical factors universalis). Offer to examine other hair
they describe in their past medical history. As bearing areas. Consider performing a full body
alopecia areata often starts after a stressful even examination for evidence of autoimmune
dont forget to ask and also assess the emotional disease, such as:
impact the conditions is having on the patient. Vitiligo. Well-demarcated macular
Drug history (see Table 5.4), cessation of de-pigmented areas in a symmetrical
oestrogen therapy, family history of hair loss, distribution
recent surgery/pregnancy are also important to Thyroid examination (autoimmune thyroid
clarify when taking a hair loss history of any type. disease)
234 Chapter 5 Station 5

Table 5.4 Causes of hair loss according to type*


Diffuse non-scarring Localised non-scarring Localised/diffuse scarring
Androgenic (male-pattern or female- Alopecia areata Burns/irradiation
pattern) Auto-immune disorders especially Chemical or thermal
More pronounced after menopause pernicious anaemia, vitiligo, X-rays
(see Figure 5.40) autoimmune thyroid disease
Endocrine Infections Infection
Hypothyroidism Ringworm Shingles
Hypoadrenalism Secondary syphilis kerion
Hypopituitarism
Nutrition Trauma/traction Lichen planus
Iron deficiency
Zinc
Protein
Telogen effluvium Discoid lupus erythematosus
Stressors
High fever
Childbirth
Surgery
Other stress
Drugs
Allopurinol
Beta-blockers
Carbimazole
Warfarin
* See also Figures 5.375.40.

Figure 5.37 Alopecia totalis. This


patient has complete hair loss.
Case 127 Alopecia areata 235

Figure 5.38 Localised alopecia


areata. This patient has a well
circumscribed, bald, smooth patch
with small broken hairs in the
periphery (exclamation mark hairs).

Figure 5.39 Multi-site localised alopecia areata. Figure 5.40 Female pattern hair loss - androgenic
This patient has multiple areas of hair loss. alopecia. This patient has thinning over the vertex
with parting of the hairline. In contrast male pattern
baldness results in a receding frontal hairline that
progresses to a bald patch on the top of the head.
236 Chapter 5 Station 5

Addisons disease may be suggested by peripheral signs of auto-immune disease. This is


the finding of diffuse hyperpigmentation, in keeping with alopecia areata.
accentuated on exposed body parts
and maximal on face, areolae, genitalia,
knees, knuckles, palmar creases, lips,
Offer relevant differential
gums, tongue. There may also be buccal diagnoses
hyperpigmentation Hair loss has a wide differential, the different
subtypes of which are discussed in Table 5.4.
Relevant negatives
If no signs suggestive of an underlying Demonstrate an understanding of
autoimmune disease were found, this would be the value of further investigation
an important negative finding to report to the
examiner. Investigations are aimed at identifying the cause
of hair loss and at identifying any underlying
autoimmune disease.
What to tell the patient To investigate for hair loss:
It appears that you have developed a type of Iron studies and ferritin
hair loss called alopecia areata, which consists ANA
of small, round patches of baldness on the Serum zinc
scalp. This type of hair loss is associated with Thyroid function tests
an inflammatory response but the exact cause Prolactin, LH, FSH, testosterone, SHBG
is unknown. It is thought that the immune
system attacks the growing hairs. The chances In suspected cases of Addisons, perform
of hair growing back are generally good when a 9 am cortisol, check serum electrolytes and
the patch is small. Re-growth occurs from the perform stimulation tests as appropriate.
centre of the patch and is often white at first In suspected thyroid disease check thyroid
but usually regains colour. There is a chance function tests and test for the presence of
there will be complete re-growth within one thyroid auto-antibodies.
year but further episodes may occur in the
future. Topical steroids may help induce hair
growth. People who suffer with alopecia areata
Always offer a management plan
are more likely to develop other auto-immune In addition to treating any identifiable underlying
conditions such as thyroid disease or diabetes cause, management of alopecia areata involves
and as such it will be important that we screen the use of short courses of potent topical steroids
for these. followed by topical immunomodulators such
as tacrolimus. Intralesional steroids are another
option for small patches. More widespread
What to tell the examiner state disease may require oral courses of steroids.
the most likely diagnosis on the The patient should be warned that when the
treatment stops alopecia may recur. Patients can
basis of the history and clinical also obtain wigs on prescription while they are
findings waiting for re-growth to occur.
This patient has a solitary patch of hair loss with Any co-existing vitiligo is managed with
exclamation-mark hairs on the edge without topical steroid alternating with topical protopic,
phototherapy, and cosmetic camouflage.
Case 128: Sarcoidosis

Instruction to the candidate Assess for systemic involvement:


This 47-year-old man has come back to clinic for Pulmonary: ask about cough, shortness of
the results of his lung function tests, which show breath, chest pain
some reduced lung capacity. He also complains Eyes: red or watery eyes
of a rash on his lower legs. Take a brief history, Cardiac: chest pain, palpitations
perform a focused examination and explain your Central nervous system: headaches,
findings to the patient. confusion, malaise
Musculoskeletal: arthralgia, arthritis
Take a focused history Kidney stones: hypercalcaemia may lead to
nephrocalcinosis. Make sure you ask about
Look for clues in the instruction and think of loin pain, haematuria
skin problems in association with lung disease,
focusing the history accordingly. Ask about the
onset and duration of the rash and whether it Positive findings of a focused
is painful. Erythema nodosum usually presents examination
with erythematous, painful lumps on the lower Skin lesions are present in up to 35% of patients
legs but other sites such as the arms may also be with sarcoidosis (Figures 5.41, 5.42). Note
affected. that 1/3 of patients with cutaneous sarcoidosis
Ask about other causes of erythema nodosum: may not have other organ involvement. Skin
Sore throat (streptococcal infection) manifestations of sarcoid include:
TB Erythema nodosum: tender bumps on the
Pregnancy shins
Oral contraceptive pill Lupus pernio: large dusky blue or violaceous,
NSAIDs soft infiltrates (nodules and plaques) cheeks
Respiratory involvement with reduced lung and nose
capacity is suggestive of sarcoidosis. Papules and plaques: purple-red, thick,
Enquire about constitutional symptoms circular lesions
such as fatigue, weight loss, arrhythmia, or night Calcinosis cutis
sweats.

Figure 5.41 Cutaneous sarcoid


lesion on the upper eyelid. Small
cutaneous lesions on the left upper
eyelid in a patient with sarcoidosis.
Cutaneous manifestations of
sarcoidosis occur in approximately
30% of cases.
238 Chapter 5 Station 5

Figure 5.42 Cutaneous sarcoid


lesions on the right cheek.
Multiple, dusky-purple plaque-like
lesions on the patients right cheek,
consistent with lupus pernio. Similar
large bluish-red and dusky purple
infiltrated nodules and plaque-
like lesions can be found on nose,
cheeks, ears, fingers and toes.

Examine the hands for individual swelling of in most patients, sarcoidosis resolves without
digits and popular violaceous/brown lesions on relapse, but small proportion of patients are left
the dorsum. with some permanent lung damage.
Perform a systemic examination to look for In most mild-to-moderate cases, no
other features of sarcoidosis: treatment is required apart from observation.
Sometimes oral steroids are needed to reduce
Lung involvement (90% of cases): apical
the inflammation. I will see you in 3 months
fibrosis
time with the results of all the other tests. I
Eyes (30%): uveitis, conjunctivitis
would like you to see an ophthalmologist to look
Liver (40%): look for hepatomegaly that may
at your eyes.
be accompanied by splenomegaly
Heart (525%): cardiomyopathy or arrhythmias
CNS (15%): Bells palsy, neuropathy What to tell the examiner state
Musculoskeletal (238%): ankle, elbows, the most likely diagnosis on the
wrists, hands arthritis or arthralgia
Examine all patients for lymphadenopathy basis of the history and clinical
and presence of enlarged parotids findings
This patient presents with painful, tender,
Relevant negatives erythematous lumps on the lower legs,
In a patient with suspected sarcoidosis, the in keeping with erythema nodosum. The
absence of systemic features is an important most likely cause given the lung function is
negative finding. In other cases of erythema sarcoidosis.
nodosum, it would be important to exclude the
afore-mentioned causes. Offer relevant differential
diagnoses
What to tell the patient In cases where the main finding is erythema
I suspect you have a condition called nodosum, offer a range of possible alternative
sarcoidosis, a disease that can cause causes, such as:
inflammation on many body parts. It is not
cancerous or infectious. It most commonly Medications, such as the oral contraceptive
presents with a rash on the legs and joint pains, pill, sulphonamides, salicylates and NSAIDs.
but it may often affect other body organs, such Inflammatory bowel disease
as the lungs. The lung tests you had show there Tuberculosis
is mild involvement of the lung tissue. I will Infections, such as streptococcus,
organise a CT scan of your chest, with high mycoplasma
resolution to look closer at the lungs. In general Pregnancy
Case 129 Tuberous sclerosis 239

Demonstrate an understanding A 12-lead electrocardiogram will identify any


bundle branch block or arrhythmia
of the value of further Slit lamp examination
investigation
Laboratory blood tests which would support a Always offer a management plan
diagnosis of sarcoidosis would include:
Erythema nodosum usually requires no
Lymphopaenia on full blood count treatment but may take 3-6 weeks to settle.
Raised serum angiotensin converting Recommend bed rest and NSAIDs to reduce
enzyme discomfort.
Hypercalcaemia: if this is high, a 24-hour With regards to sarcoidosis, in most cases
urinary calcium excretion can be performed no treatment is required and the disease
Abnormal liver function tests may resolve spontaneously after a few
Raised ESR months. Initial treatment is with systemic
corticosteroids, especially in the presence
A skin biopsy of the cutaneous lesion may of lung disease, uveitis, hypercalcaemia,
show non-caseating granulomata. neurologic or cardiac complications. Other
Further investigations may include: options may include hydroxychloroquine,
A plain chest radiograph to look for bilateral methotrexate, axathioprine or anti-TNFa agents.
hilar lymphadenopathy. A high-resolution Cutaneous lesions may be treated with
CT can confirm lung involvement topical or intralesional steroids. Laser surgery
can be used for severe, disfiguring plaques.

Case 129: Tuberous sclerosis

Instruction to the candidate or confetti macules. If you see any ash leaf
macules you need to inspect the whole body
Please examine this 35-year-old womans skin.
to identify more. Three or more white spots at
She is also being investigated for blackouts.
birth are suggestive of tuberous sclerosis. Ash
Tell the patient what you think the most likely
leaf macules fluoresce under Woods light. In
diagnosis is.
terms of distribution, the most common place
to find Ash leaf macules is the trunk, followed
Take a focused history by the lower extremities
Tuberous sclerosis is an uncommon autosomal Adenoma sebaceum/angiofibromas: These
dominant disorder with variable expression. are 0.10.5 cm diameter dome-shaped,
50% of patients will have new mutations. The confluent, small erythematous glistening
key features may not appear until after puberty. papules. They can be skin-coloured or red
Establish the onset of the features identified on and are commonly found on the centre of the
clinical examination and positive family history. face and in the perinasal region
Enquire about associated conditions such as Shagreen patches are skin-coloured
epilepsy, learning difficulties, autism spectrum connective tissue naevi, found on the
of disorders, and renal problems. buttock and back
Periungual fibromas (Koenen tumours) are
smooth, firm, flesh-coloured nodules that
Positive findings of a focused emerge from the nailfolds
examination
Key findings in tuberous sclerosis include Relevant negatives
(Figures 5.435.45): If having identified any of the key clinical
Ash leaf macules (hypomelanotic macules): features of tuberous sclerosis any of the other
These are off-white, polygonal or thumbprint key features were absent, these would be
240 Chapter 5 Station 5

Figure 5.43 Multiple


angiofibromas. These papules are
small and coalescent. As with any
finding consistent with a diagnosis
of tuberous sclerosis, it should
prompt examination for the other
known clinical findings. If they are
present they should be reported
as positive findings. If not present,
they should be included as relevant
negative findings.

Figure 5.44 Adenoma sebaceum


as a clinical finding in tuberous
sclerosis. Multiple small, pink spots
across the cheeks and nose.

Figure 5.45 Tuberous sclerosis.


Clinical signs in tuberous sclerosis.
Case 130 Neurofibromatosis 241

mentioned as relevant negatives. Other relevant adenoma sebaceum and thus a diagnosis of
negatives would include a lack of cognitive tuberous sclerosis. The blackouts are suggestive
deficit and the absence of abdominal masses of epileptic fits which as commonly associated
(tuberous sclerosis is associated with renal, with tuberous sclerosis.
hepatic and GI hamartomas).
Offer relevant differential
What to tell the patient diagnoses
The marks I have seen on your skin today A differential diagnosis of Ash leaf macules
suggest you have tuberous sclerosis. It is a includes other white spots such as: vitiligo, tinea
complex genetic condition caused by an versicolor, post-inflammatory hypomelanosis.
alteration in a gene. We will arrange for you
to have multi-professional care and I would
therefore like you to see an ophthalmologist to Demonstrate an understanding
check your eyes, a neurologist as well as your of the value of further
GP at regular intervals to keep an eye on your
blood pressure. If you are planning to have a investigation
family you ought to be seen by a specialist for Important investigations to mention to the
genetic counselling as tuberous sclerosis is a examiner include:
condition that you can pass on to your children.
Most people with tuberous sclerosis will live a CT or MRI head for tuberous masses
normal lifespan and any problems related to the Echo to exclude cardiac hamartomas
condition will be monitored and managed. I will Ultrasound of the renal tract for cysts and
be writing to your GP to inform him of your visit hamartomas
today.
Always offer a management plan
What to tell the examiner state Principles of management would include
genetic counselling and appropriate specialist
the most likely diagnosis on the referral for system specific complications
basis of the history and clinical (in this case and in the absence of further
findings information, the patients blackouts could
be due to either cardiac or neurological
This patients skin examination has revealed complications).
multiple hypomelanotic macules and a facial Laser surgery can be performed on
rash around the cheeks and nose conistent with angiofibromas.

Case 130: Neurofibromatosis

Instruction to the candidate Duration: all children with NF-1 have caf au
lait macules by 15-years of age
This is a 23-year-old woman who is concerned
about her skin appearance. Please examine her
Precocious puberty (especially seen in
association with optic tumours)
skin and discuss the diagnosis with her.
Learning difficulties and epilepsy
Enquire about central nervous and eye
Take a focused history involvement (optic nerve tumours can cause
As the examination section below demonstrates, visual loss)
this patient has signs consistent with Hearing defects (more common in NF2)
neurofibromatosis. In such cases, seek to elicit Hypertension secondary to renal artery
the following clinical features: stenosis or phaeochromocytoma
242 Chapter 5 Station 5

Positive findings of a focused Caf au lait macules are flat, coffee-coloured


patches of skin. They usually appear in
examination the 1st year of life and increase in number
See Figures 5.465.51 for clinical with age. If you find more than 6 caf au
photographs for the following clinical signs of lait macules proceed to look for other
neurofibromatosis: manifestations of neurofibromatosis

Figure 5.46 Axillary freckling as a clinical finding Figure 5.47 Multiple caf au lait macules. This
in neurofibromatosis. Example of freckling in the patients back demonstrates multiple caf au lait
left axilla and surrounding area. In neurofibromatosis, macules. There is freckling on the whole area of the back
freckling can also occur in the groin, the base of the (non-specific) and freckling at the base of the neck.
neck and the submammary areas. It is usually present
by the age of 10.

Figure 5.48 Single


neurofibromas. An isolated, flesh-
coloured papule consistent with a
neurofibroma.
Case 130 Neurofibromatosis 243

Figure 5.49 Lisch nodules in


neurofibromatosis. This patient
has brown-coloured pigmented
lesions on the iris. On further
examination with a slit-lamp and
with clinical examination,associated
signs of neurofibromatosis would
be indicated.

Figure 5.50 Neurofibroma. Superficial, soft tumour Figure 5.51 Multiple cutaneous neurofibromata. In
of the skin on the lower back. this patient there are also caf au lait macules present.

Freckling is apparent in the axillae, groin, Lisch nodules tiny tumours on the iris of
neck base and sub-mammary areas. They the eye
usually appear by 10-year of age
Neurofibromas can take on a number of forms.
Relevant negatives
Most commonly they appear as flesh-coloured
papules, soft, button holed, easily pushed into Important relevant negative findings include
the skin. Their number increases with age. the absence of signs suggestive of tuberous
They can also be nodular or plexiform sclerosis, which as discussed below is in the
244 Chapter 5 Station 5

differential diagnosis for causes of caf au lait be a normal finding in healthy individuals
spots. (especially if they number <6).
Skin lesions which may appear to be
similar to neurofibromas include leiomyomas
What to tell the patient (red-brown smooth, firm papules or nodules,
Examination of your skin suggests a condition often painful) and common nevi (although
called neurofibromatosis. This is a genetic they are softer than neurofibromas and
condition which causes abnormalities of the compressible).
skin but also in the nervous system and other
organs. As you are concerned about your skin
appearance I will arrange for you to see a Demonstrate an understanding
dermatologist to discuss the treatment options. of the value of further
Furthermore, we will arrange for specialist
review to discuss the genetic nature of this
investigation
condition. Do you have any further questions or Slit-lamp investigation will confirm the presence
concerns I can address for you today? of Lisch nodules, which are pigmented iris
hamartomas and are part of the diagnostic
criteria for neurofibromatosis type 1. X-rays
What to tell the examiner state for bone deformities. Annual blood pressure
the most likely diagnosis on the measurement. Regular skin survey for
sarcomatous change of neurofibromas.
basis of the history and clinical Where appropriate, MRI can be used to
findings exclude optic pathway and central nervous
The clinical examination has revealed multiple system involvement. Genetic testing for
cafe au lait spots, axillary freckling and multiple confirmation (NF1 chromosome 17, NF2
neurofibromas. These findings would be in chromosome 22).
keeping with a diagnosis of neurofibromatosis
type I. Always offer a management plan
Management is multi-disciplinary involving
Diagnostic criteria for primary care, geneticist, neurologist, and
neurofibromatosis type 1 ophthalmologist. Genetic counselling and
appropriate investigation of family members is
For a diagnosis to be made, two out of the appropriate.
following seven features need to be identifies: Carbon dioxide (CO2) laser, dermabrasion
> 6 caf au lait macules or surgical excision can be used for
> 2 neurofibromas disfiguring facial neurofibromas. Consider the
Axillary or groin freckling psychological impact the condition may have,
>2 Lisch nodules the risk of isolation and loneliness.
Optic glioma
Bone deformity, e.g. sphenoid
First degree relative with two criteria Neurofibromatosis type 2key facts
Autosomal dominant
Much rarer than NF1
Offer relevant differential Caf au lait spots
Bilateral vestibular schwannomas (acoustic
diagnoses neuromas)
The differential diagnosis of disease Juvenile posterior subscapular lenticular
associations of caf au lait spots includes opacity
tuberous sclerosis, McCuneAlbright Schwannomas of other cranial nerves
syndrome and urticaria pigmentosa. Meningiomas
Remember also that caf au lait spots can
Case 131 & 132: Dermatology spot diagnoses

Case 131: OslerWeber Management is of complications. Genetic


counselling is important due to the autosomal
Rendu syndrome (hereditary dominant inheritance of this condition.
haemorrhagic telangiectasia)
The hallmark of this condition is the presence Case 132: Port-wine stains
of muco-cutaneous telangiectasia and arterio-
venous malformations (pulmonary, hepatic,
(capillary vascular malformation)
cerebral, and spinal). Red nodules and starry Port-wine stains are well-defined, large, flat
telangiectasia are found on mucous membranes patches, purple or dark red in colour, which
and skin (Figures 5.52 and 5.53). are found usually on face but can present
The patient may present with a history of anywhere in the body (Figures 5.54 and 5.55).
epistaxis and GI bleeds. They may also have If present in a trigeminal nerve supplied
neurological symptoms due to TIA or stroke. dermatome, think of SturgeWeber syndrome
Cardiac symptoms or signs may be present due (this is associated with microcephaly, brain
to high-output cardiac failure secondary to calcification, seizures). Laser treatment can be
arteriovenous malformations. There may be liver performed for cosmesis.
dysfunction.

Figure 5.52 Hereditary


haemorrhagic telangiectasia.
Mucosal telangiectasia in a
patient with the rare, autosomal
dominant condition of hereditary
haemorrhagic telangiectasia.

Figure 5.53 Tongue lesions


in hereditary haemorrhagic
telangiectasia. The same patient
as in Figure 5.58, with a further
mucocutaneous lesion, this time
demonstrated on the tongue. In
such patients the skin should be
examined carefully for cutaneous
telangiectasia. Enquire about
symptoms of anaemia. Has the
patient been given a diagnosis
of hereditary haemorrhagic
telangiectasia? Have they had to
have any interventions in the past?
246 Chapter 5 Station 5

Figure 5.54 Port-wine stain.


The large, purple red patch on the
patients left cheek is consistent
with a port-wine stain.

Figure 5.55 Port-wine stain. This


example of a port-wine stain is
situated under the eye.
Case 133: Ankylosing spondylitis

Instruction to the candidate


This 30-year-old man has been experiencing
back pain. Please take a brief history, perform
an appropriate examination and present your
findings to the examiner.

Take a focused history


Use the history to elicit whether this back pain
in a young male is mechanical, or whether it is
associated with stiffness and decreased mobility.
Ankylosing spondylitis will be high in your mind
given the presenting symptom and the patient
demographic (age <45), so take the time to ask
about rheumatological symptoms as well.
Remember to ask about function (what can
the patient not do because of the symptoms)
and explore the patients concerns and
questions.
What is their occupation? While the PACES
candidate may be focusing on detecting iritis to
demonstrate their knowledge to the examiner,
the young, previously healthy man working in
a manual job may be more concerned about
potential loss of income.

Positive findings of a focused


examination Figure 5.56 Ankylosing spondylosis. Exaggerated
Inspect the spine for deformity (Figures 5.56 thoracic kyphosis, as part of a question mark spine in a
and 5.57) and make a functional assessment by patient with ankylosing spondylosis.
asking the patient to look up towards the ceiling,
noting limitation.
Assess for reduced flexion of the lumbar
spine by performing Schobers test. Get the
Relevant negatives
patient to stand in front of you, facing away from It is important to note the absences of psoriatic
you. Find L5 level. With a pen, make two marks plaques and abdominal scars, either of which
in the midline, one 5 cm below L5 and one 10 may suggest an alternative diagnosis of back
cm above. Ask the patient to touch their toes. pain (see differential diagnosis below).
Measure the distance between the two marks
on the patients back. If the distance between What to tell the patient
the two marks is now less than 20 cm (that is to
say it has increased by less than 5 cm), this is The symptoms you describe and the clinical
indicative of reduced lumbar spine flexion. signs I have found on examination suggest you
Examine for associated findings: may have a condition known as ankylosing
spondylitis. The main manifestation of this
Examine the lungs for apical lung fibrosis condition is reduced mobility of the spine,
Examine the cardiovascular system for We will have to order some investigations to
the early-diastolic murmur of aortic look more closely at the spine. These will involve
regurgitation, with or without a collapsing plain X-rays in the first instance and then likely
pulse and other peripheral signs of aortic some more advanced imaging such as MRI scans.
regurgitation We will also take some blood tests that will tell us
Dry eyes about the levels of inflammation in the body,
248 Chapter 5 Station 5

on clinical (pain, stiffness, loss of lumbar


spinal mobility, loss of chest expansion) and
radiological criteria (the presence and grade of
sacroiliitis).

Offer relevant differential


diagnoses
The differential diagnosis for back pain in a
young male includes:
Mechanical
Traumatic
Ankylosing spondylitis
Other spondyloarthropathy: reactive arthritis,
psoriatic arthritis, enteropathic arthritis

Demonstrate an understanding
of the value of further
investigation
Initial investigations will include appropriate
imaging modalities and blood tests.
Imaging would include:
Plain spinal radiograph: the main
radiological evidence to tell the examiner
about is that of sacroiliitis, which in the
majority of cases will be bilateral. Other
Figure 5.57 Reduced flexion of the lumbar spine
features include cervical spine involvement
in ankylosing spondylitis. This can be confirmed
and syndesmophyte formation, which can
clinically by performing Schobers test.
lead to spinal fusion
MRI spine can detect inflammatory
I will arrange for you to be seen in the changes and is now included in the
rheumatology clinic with the results of these ASAS classification for diagnosis of
investigations. Until then I will prescribe you spondyloarthritis
some anti-inflammatory medications and Blood tests would include inflammatory
arrange for you to be seen by the physiotherapy markers and rheumatoid factors.
team,
Additionally, an electrocardiogram will
Are there any other questions or concerns
demonstrate any atrioventricular conduction
that I can address for you today?
abnormality, of which there is a higher risk in
ankylosing spondylitis. In addition, the presence
What to tell the examiner state of a collapsing pulse, diastolic murmur, or any
other clinical evidence of aortic regurgitation
the most likely diagnosis on the would demand a transthoracic echocardiogram
basis of the history and clinical to assess the aortic valve and root (aortic
findings regurgitation).
This patient has signs and symptoms consistent
with a diagnosis of ankylosing spondylitis. Always offer a
The British Society of Rheumatology advises management plan
the use of modified New York diagnostic
criteria for ankylosing spondylitis. The modified Management is multi-disciplinary in nature,
New York criteria divides cases into definite with physiotherapy and occupational therapy
and probable ankylosing spondylitis based teams involved to help maintain function.
Case 134 Marfan syndrome 249

Medical therapies involve: Further reading


Analgesia (WHO pain ladder), specifically Jang JH, Ward MM, Rucker AN, et al. Ankylosing
NSAIDs spondylitis: patterns of radiographic
Biologicals (anti-TNF drugs). The British involvementa re-examination of accepted
Society of Rheumatology guidelines (2004) principles in a cohort of 769 patients. Radiology
recommend anti-TNF therapy for those with 2011; 258:192198.
Disease which meets modified New York Rudwaleit M, van der Heijde D, Landew R, et al.
Criteria. Note that DMARDs are not effective The development of Assessment of Spondylo
for spinal symptoms Arthritis international Society classification
criteria for axial spondyloarthritis (part II):
Monitoring for depression remember that
validation and final selection. Ann Rheum Dis
these are young patients who may have been
2009; 68:777783.
extremely active previously so are at risk of
The British Society for Rheumatology (BSR). BSR
depression.
guidelines for prescribing TNF-alpha blockers
Use the Bath Ankylosing Spondylitis Activity
in adults with ankylosing spondylitis. London;
Index (BASAI) for disease monitoring. It
BSR, 2004.
incorporates current medications, gender, age
van der Linden S, Valkenburg HA, Cats A. Evaluation
and questions about pain and swelling over
of diagnostic criteria for ankylosing spondylitis.
the previous week to assess disease activity.
A proposal for modification of the New York
Whilst in a station 5 case where you make a de
criteria. Arthritis Rheum 1984; 27:361368.
novo diagnosis of ankylosing spondylitis it is
Garrett S, Jenkinson T, Kennedy LG, et al. A new
likely not necessary to actually use BASAI, if the
approach to defining disease status in
patient tells you they have a known diagnosis
ankylosing spondylitis: the Bath Ankylosing
of ankylosing spondylitis, it will impress the
Spondylitis Disease Activity Index. J Rheumatol
examiner if you use it as part of your focused
1994; 21:22862291.
assessment (assuming time allows).

Case 134: Marfan syndrome

Instruction to the candidate explore the family history of the patient's


ancestors. Remember to include questions
Please examine this 22-year-old woman who about descendants. A proportion of cases
has been complaining of shortness of breath. are new mutations so there may be no family
Following an examination and history, discuss history
your findings with the patient. A discussion with Visual defects: is the patient myopic (the
the examiner will then follow. most common symptom)?
Is there any arthritis due to joint laxity? The
Take a focused history candidate may miss this aspect of the disease
After having made the spot diagnosis of Marfan if they focus too much on the cardiovascular
syndrome in a patient who is tall and thin, with manifestations
long arms and arachnodactyly (see positive
findings of focussed examination below), ask the
patient about: Positive findings of a focused
Her shortness of breath (see cardiac
examination
history). Does she have any known cardiac A sensible focused examination would include:
valvular abnormalities? Has she ever had a Get the patient to stand: ask for a measuring
pneumothorax? tape and measure both their height and
Is there a confirmed diagnosis of Marfan's? their arm span. Compare the length of their
Family history: it would be a mistake to only
250 Chapter 5 Station 5

trunk to the length of their legs. Look at the complication of a congenital syndrome known
fingers: are they long and thin demonstrating as Marfan syndrome. We will need to order some
arachnodactyly? investigations to assess the heart valve lesion and
Assess for joint hyper-extensibility I will be arranging for you to see a cardiologist
Inspect the praecordium for pectus deformity in the near future. Do you have any further
(Figure 5.58) questions or concerns I can address today?
Auscultation the heart, listening specifically
for the ejection diastolic murmur of aortic
regurgitation. Examine for a collapsing pulse.
What to tell the examiner state
If a murmur or collapsing pulse is present, the most likely diagnosis on the
go on to examine for other signs of aortic basis of the history and clinical
regurgitation. Also, listen carefully for any
mitral valve prolapse/regurgitation that findings
might be present This patient has symptoms and signs consistent
Assess the eyes, is there heterochromia? with a diagnosis of aortic regurgitation as a
Do they have a high-arched palate? complication of Marfan syndrome. Marfan
(Figure 5.59) syndrome is an autosomal dominant condition
(with variable penetrance) affecting connective
tissue, due to a mutation in the FBN-1 gene on
Relevant negatives chromosome 15, which codes for the protein
It would be important to note the absence of fibrillin-1.
a midline sternotomy, which would indicate Whilst it is easy for a candidate in MRCP
previous aortic and/or cardiac complications. PACES to comment that a patient is likely
to have Marfan syndrome, the Ghent criteria
What to tell the patient for making a formal diagnosis are extensive.
They cover the following systems: Skeletal,
You have features consistent with a diagnosis Ocular, Cardiovascular, Pulmonary, Skin, Dura,
of a defect in one of the heart valves. This is family/Genetic history. While most candidates
known as aortic regurgitation. This may be a would consider it an inefficient use of time to

Figure 5.58 Pectus deformity in


Marfan syndrome. This patient,
with an underlying diagnosis of
Marfan syndrome. Note the defect
in the centre of the chest.
Case 134 Marfan syndrome 251

Offer relevant differential


diagnoses
You can choose to give the examiner any of a
range of differential diagnoses. For example, you
could give a differential diagnosis of shortness of
breath in a patient with Marfan syndrome:
Aortic regurgitation
Mitral valve disease
Spontaneous pneumothorax
Respiratory failure due to pectus deformity

Demonstrate an understanding
of the value of further
investigation
This patient will require a full cardiac work-
up, firstly to confirm aortic regurgitation and
secondly to assess its severity. See Chapter
1 for details but this will include a 12-lead
electrocardiogram, a plain chest radiograph,
and a transthoracic echocardiogram. The plain
chest radiograph will also serve to rule out any
pneumothorax.

Always offer a management plan


Figure 5.59 High-arched palate in Marfan syndrome. An important aspect of management of patients
Use a pen torch to look inside the patients mouth. with Marfans (and particularly important in
this case) is the prevention and management of
memorise the whole list of criteria, the body cardiovascular complications.
system titles can be used as an aide memoir Key factors in the management of this
during the history and examination to ensure condition include:
you are covering all the relevant points. Beta-blockade
The criteria go on to state the requirements Regular (and often) clinical and
for a diagnosis of Marfans. Note that these echocardiographic assessment to monitor for
requirements are different for the index case, progressive aortic root dilatation
and for relatives of the index case. Consideration of prophylactic aortic root
surgery when diameter >45 mm
Key clinical features of Marfan Regular ophthalmological assessment is
also an important aspect to impress upon
syndrome the examiner. This is a young woman
Tall stature so counselling must be given regarding
Arm span > height pregnancy. When one considers the issues
Trunk length > leg length surrounding Marfan's in pregnancy, it is
Arachnodactyly natural to concern oneself with the increased
Pectus deformities risk that a pregnancy confers on the
Aortic regurgitation due to cystic medial mother. She has an increased risk of aortic
necrosis dissection, an event that occurs in 4.5% of
Mitral valve prolapse or regurgitation pregnancies involving patients with Marfan's.
Upward lens dislocation (the fact that Any discussion with the examiner should
in homocystinuria the lens dislocation include mention of the multi-disciplinary
is downwards is a dichotomy loved by approach to managing a pregnant patient
examiners but almost entirely useless in the with Marfan's, with a team of obstetricians
daily clinical practice of a general physician) and cardiologists with expertise in the field.
Aortic dissection Genetic counselling will also need to be
Spontaneous pneumothorax given to the patient regarding the autosomal
252 Chapter 5 Station 5

dominant nature of inheritance and the RE. Revised Diagnostic Criteria For The Marfan
implications for children in the event of a Syndrome. Am J Med Genet 1996; 62:417426.
successful pregnancy Dean JC. Marfan Syndrome: Clinical Diagnosis
and Management. Eur J Hum Genet 2007;
15:724733.
Further reading Lind J, Wallenburg HC. The Marfan syndrome and
Yuan SM, Jing H. Marfan syndrome: an overview. pregnancy: a retrospective study in a Dutch
Sao Paulo Med J 2010; 128:360366. De Paepe population. Eur J Obstet Gynaecol Reprod Biol
A, Devereux RB, Dietz HC, Hennekam RC, Pyeritz 2001; 98: 2835.

Case 135: Turner syndrome

Instruction to the candidate Failure of growth. At what age was it noticed


that her growth was lagging behind that
Please examine this 19-year-old woman who
of her peers? Was it this that led to her
has been complaining of recurrent urinary tract
diagnosis? Has she had any treatment for
infections. Following your assessment, inform
small stature such as growth hormone
the patient of your findings. A short discussion
therapy or oxandrelone?
with the examiner will follow.
Are there any neurological or behavioural
complications?
Take a focused history
Ask about the recurrent urinary tract infections. Positive findings of a focused
How often does she get them? What antibiotics
has she taken in the past? Ask about sexual
examination
intercourse, is there a correlation? Has she been The examination should seek to elicit findings
investigated for this already with an ultrasound both to confirm your diagnosis of Turner
of the urinary tract? syndrome and to elicit any abnormality that
Ask about her past medical history. Given can help in your assessment of the patients
findings of general inspection with evidence recurrent urinary tract infections.
of small stature, webbed neck (may be hidden A brief auscultation of the heart would identify
by hair) and, a shield chest deformity, ask any concurrent aortic stenosis or coarctation.
specifically about Turner syndrome. The clinical features of Turner syndrome
Ask about other features of Turner syndrome: include:

The key cardiovascular manifestations are General inspection: short stature


the presence of a bicuspid aortic valve, Hands and arms: nail dysplasia, cubitus
aortic coarctation, and mitral regurgitation. valgus, short fourth metacarpal, single
Bicuspid aortic valve occurs in up to 30% palmar crease
of patients with Turner syndrome when Head and face: low-set ears, retrognathia
assessed by transthoracic echocardiography (posterior positioning of the mandible), Low
in conjunction with MRI. Coarctation of the posterior hairline, webbed neck, epicanthal
aorta occurs in 6.6% of patients and mitral folds, ptosis, strabismus
regurgitation occurs in 7.4% Praecordium: pectus deformity, widely-spaced
Gynaecological complications such as nipples, coarctation of the aorta, aortic stenosis
ovarian failure (secondary to bicuspid valve)
Case 135 Turner syndrome 253

Relevant negatives Demonstrate an understanding of


Important relevant negative features would the value of further investigation
include the absence of signs of active infection To investigate this patients recurrent infections,
such as fever, tachycardia and any abdominal the following tests would be requested:
tenderness (suprapubic and/or renal angle).
Urine dipstick and mid-stream urine for
microscopy, culture and sensitivity
What to tell the patient Blood tests including white cell count and
The recurrent urine infections are most likely inflammatory markers to look for active
due to kidney problems associated with a infection
condition called Turner syndrome. It will be Renal tract ultrasound scan is a key
necessary to obtain an ultrasound of your investigation as it identifies any structural
kidneys and bladder to investigate the cause abnormality of the renal tract such as those
further and discuss potential management listed in the box on renal complications
options with the results in clinic. It may be While not necessarily directly applicable to the
possible to place you on a preventative daily presenting complaint, mention to the examiner
antibiotic in the interim to reduce the symptoms that the patient would require (at some stage)
and prevent further complications. Karyotyping: likely done at a very young age
Transthoracic echocardiography to identify
What to tell the examiner state valvular or aortic abnormality
Blood pressure monitoring
the most likely diagnosis on the
basis of the history and clinical Always offer a management plan
findings Principles of management can be divided into
This patient has signs and symptoms managing the recurrent urinary tract infection
consistent with recurrent urinary tract (antibiotics where appropriate, hygiene advice,
infection, likely as a renal complication of urology and renal referral if needed) and that
Turner syndrome. of the underlying Turner syndrome. This would
include regular cardiovascular assessment,
genetic counselling, and endocrine referral for
Offer relevant differential ovarian failure or growth failure.
diagnoses
As with most obvious spot diagnoses you Further reading
can choose your differential, in this case the
Davenport ML. Approach to the patient with Turner
differential diagnosis for recurrent urinary tract
syndrome. J Clin Endocrinol Metab 2010; 95:
infection.
14871495.
The differential diagnoses for renal
Sachdev V, Matura LA, Sidenko S, et al. Aortic valve
manifestations in Turner syndrome include:
disease in Turner syndrome. J Am Coll Cardiol
Double collecting systems 2008; 51:19041909.
Hydronephrosis Carvalho AB, Guerra Jnior G, Baptista MT, et al.
Horseshoe kidney Cardiovascular and renal anomalies in Turner
syndrome. Rev Assoc Med Bras 2010; 56:655659.
Case 136: Tremor

Instruction to the candidate Perform a neurological examination to assess


for:
You are the SHO in the neurology clinic. A
48-year-old man has been referred by his GP Parkinsonian signs pill-rolling resting
complaining of tremor. Please assess him and tremor, expressionless face, clasp-knife
inform him of your management plan. rigidity with cog-wheeling
Cerebellar signs: ataxia, dysdiadochokinesia,
nystagmus, past-pointing, intention tremor,
Take a focused history slurred speech, hypotonia
Identify the characteristics of the tremor:
Onset and severity Relevant negatives
Resting versus intention or postural
Relevant negative findings would include any of
Unilateral or bilateral
the above clinical signs, which are absent (see
Ask the patient is the tremor is something positive findings).
that they have noticed, or if it has been
mentioned by a relative/carer? A patient with What to tell the patient
an essential tremor or cerebellar dysfunction
may be more likely to notice their tremor than a From what you tell me and from examination
Parkinsonian patient. findings, I believe you are suffering from what
Consider the differential diagnosis of is known as benign essential tremor, where a
tremor (see below). Does the patient have any tremor exists that can be exacerbated by social
associated symptoms that would identify an situations. I can reassure you that I can find
underlying diagnosis? no evidence of Parkinsons disease, cerebellar
Be sure to ask about alcohol intake. In benign dysfunction or any other sinister cause of
essential tremor, symptoms are often alleviated tremor. The initial management in benign
by alcohol, with a propensity for alcohol essential tremor is lifestyle modification, with
dependence. Alternatively, alcohol may be the avoidance of caffeinated drinks and ensuring
underlying cause of any cerebellar disease, with you get good quality and duration of sleep.
resultant tremor. Additionally, with regard to If you feel strongly about the severity of the
essential tremor, be sure to take a family history impact of the tremor we can consider using a
to screen for an inherited or genetic component. medication called propranolol to attempt to
Assess the impact of the tremor on the control it. This medication is from a family of
patient's quality of life. Even in benign essential drugs called beta-blockers. Side effects of this
tremor, there is considerable associated drug include feeling fatigued but are tolerated
morbidity, with potential for significant well.
impairment and disability as a consequence. It I think we should discharge you from the
is therefore important that you assess, and are clinic today, and continue care through your GP
seen to assess, the functional and psychosocial to whom I will write a letter. However, if you feel
effects of the tremor. things are getting worse I am more than happy
for the GP to refer you back to see me again. Do
you have any further questions or concerns I can
Positive findings of a focused address for you today?
examination
Inspect the patient to assess for the presence of What to tell the examiner state
a resting tremor, looking specifically for a pill- the most likely diagnosis on the
rolling Parkinsonian tremor. It may be necessary
to utilise distraction to elicit a resting tremor. basis of the history and clinical
Screen for postural or action tremor, often by findings
performing a hand or cerebellar examination.
The patient history and clinical examination are
A re-emergent tremor can often be useful in
consistent with a diagnosis of benign essential
distinguishing between a subtle essential tremor
tremor.
and the resting tremor of Parkinsons.
Case 137 Seizures 255

Offer relevant differential burden. For example, limiting the intake of


caffeinated drinks, ensuring adequate sleep
diagnoses levels and avoiding stress where possible are all
Important causes of tremor include: important.
The first-line pharmacological
Benign essential tremor
management of benign essential tremor is
Parkinsonism
beta-blockade with propranolol. If that is
Cerebellar dysfunciton
ineffective, the anticonvulsant primidone can
Remember that in a young person, in be used.
addition to considering benign essential tremor, NICE has recently released a consultation
you should consider Wilson's disease if any document relating to deep brain stimulation
cerebellar disorder is present. for essential tremor, but most candidates would
correctly wish to avoid such discussions in the
PACES examination.
Demonstrate an understanding of If the patient has reported a significant
the value of further investigation alcohol intake, you should seek to counsel them
If the history and examination are suggestive of about the dangers of using alcohol to control
benign essential tremor then no investigations their tremor and, if appropriate, refer them to
are necessary. Should there be present features community alcohol liaison services.
that are consistent with either Parkinsonism You can also inform the patient that there is
or cerebellar dysfunction then the appropriate a National Tremor Foundation to provide help
investigations should be requested (See Intention and support to those with tremor.
tremor and problems with co-ordination
cerebellar syndrome, p. 92 and Central cord Further reading
syndrome syringomyelia, p. 93).
Louis ED. Treatment of essential tremor: are there
Issues we are overlooking? Front Neurol
Always offer a management plan 2012; 2:91
It is important to counsel the patient on lifestyle National Tremor Foundation. www.tremor.org.uk.
adaptations, which can help with the symptom

Case 137: Seizures

Instruction to the candidate are more common on NF1 however all seizure
types can occur). Has the patient had any
You are asked to review a 48-year-old man on
investigations for the seizures? Have they been
the acute medical ward who was admitted
started on any anti-epileptic medications? Ask
overnight with new-onset seizures. Please assess
about the patients occupation does it involve
him and inform him of your management plan.
driving (an important point in the patient with
seizures)?
Take a focused history Confirm that the patient has already been
On introducing yourself to the patient, given a diagnosis of NF1 (this is likely). Ask him
you notice that he has multiple cutaneous about any complications that have occurred in
neurofibromas. the past.
Ask about the seizures: Are they generalised, Ask (tactfully) about academic progression,
partial, or complex-partial (complex-partial as learning difficulties can be a feature. Explore
256 Chapter 5 Station 5

the effect on the patients life, their questions diagnosis of neurofibromatosis type 1, an
and their concerns. autosomal dominant condition due to a
mutation on chromosome 17.
Positive findings of a focused
examination Offer relevant differential
Confirm on examination the presence of diagnoses
neurofibromata, being mindful of alternative The differential diagnosis for caf au lait macules
diagnoses such as multiple lipomata or includes:
Dercums disease.
Tuberous sclerosis
Ask to see the patient's axillae and note the Normal finding in healthy individuals
presence of axillary freckling (especially if <6)
Look for and comment on the number of caf McCune-Albright syndrome (caf au lait
au lait macules macules, bony deformities and fractures,
Inspect the eyes for evidence of Lisch endocrine abnormalities)
nodules affecting the iris Urticaria pigmentosa

Relevant negatives Demonstrate an understanding


Perform a full neurological examination to
of the value of further
assess for any focal neurological deficit. investigation
For the patients seizures, they will require:
What to tell the patient Electroencephalogram
From what you have told me and the findings of Intracerebral imaging: MRI is the imaging
my examination it appears that the seizures you modality of choice
have experienced are most likely a complication
of your neurofibromatosis. I will be requesting
some investigations to rule out other causes of Always offer a management
seizures and I will refer you to a neurologist for
specialist management. They may choose to
plan
start you on anti-epileptic medications. In the The patient would warrant specialist
meantime, I must inform you that the licensing neurological referral for initiation of
authorities stipulate that you should not drive anti-epileptic medications. The patient also
following a seizure. As this is likely epilepsy needs to be counselled on rules regarding
secondary to neurofibromatosis, this period of driving after seizures.
time would be of one years duration, however
the neurologist will advise you further. I can give Further reading
you some written information if you would like.
Ferner RE. The neurofibromatoses (review article).
Do you have any further questions or concerns
Pract Neurol 2010; 10:8293.
that I can address at the current time?
Brems H, Chmara M, Sahbatou M, et al. Germline
loss-of-function mutations in SPRED1 cause
What to tell the examiner state a neurofibromatosis 1-like phenotype. 2007;
the most likely diagnosis on the 39:11201126.
Gutmann DH, Aylsworth A, Carey JC, et al. The
basis of the history and clinical diagnostic evaluation and multidisciplinary
findings management of neurofibromatosis 1 and
neurofibromatosis 2. J Am Med Assoc 1997;
This patient describes seizures on a background
278:5157.
of symptoms and signs consistent with a
Chapter 6

History taking
(station 2)
Case 138: Chest pain

Candidate information exploration of the symptoms, in particular


asking about:
Scenario
You are the registrar in the Rapid Access
Site
Chest Pain Clinic. You are asked to see a
Nature of the pain
58-year-old woman who has been referred
Onset insidious/acute
by her GP due to chest pain of recent onset.
Context
Her 12-lead electrocardiogram has been
Duration
commented on by the referring GP as being
Frequency of symptoms
unremarkable. Please take a full history from
Alleviating/exacerbating factors
the patient. An examination is not required. Allow the patient to describe the nature of
Explain your working diagnosis to the patient the pain using her own language. Try to avoid
and formulate a plan of action including any giving exemplars of types of pain, to avoid
further investigations or management options. misinterpretation and maximise the diagnostic
Thereafter be prepared to discuss the case with value of the history.
the examiner. The differential diagnosis can then be
narrowed by the characteristics of the pain
revealed in the history:
Actor information
You have attended today because you are Cardiac
worried about the exertional chest discomfort
you have been having recently. You are Myocardial ischaemia pressure or crushing
particularly worried about your risk of a heart sensation, exertional, relieved by rest,
attack. radiating to the arm or jaw, associated
diaphoresis and autonomic features
Pericarditis constant, aggravated by
Prior to entering the room breathing or swallowing, relieved on sitting
The differential for chest pain is broad. It is forward
useful to consider the common causes by Myocarditis similar to pericarditis,
system: cardiac, pulmonary, gastrointestinal, systemically unwell, fever or recent viral
vascular and musculoskeletal. As ever, open illness, often with features of heart failure
questioning will allow you to best evaluate Dissection acute tearing pain, often
the value of further questioning on any given radiating to the back
topic. However, there are often clues in the
written instructions that will guide your thought Respiratory
process. Where chest discomfort is described Pulmonary embolism pleuritic with
as exertional, immediately one should be alert dyspnoea, cough and haemoptysis possible
to the possibility of ischaemic heart disease. In Pleurisy painful, difficult breathing, often
taking a cardiac history, be mindful that a strong post pneumonia or viral infection
candidate will establish a working diagnosis, risk Pneumothorax acute onset shortness of
stratify the patient, and construct a management breath and pain
plan that will involve clear explanations to
the patient avoiding jargon. Additionally, Gastrointestinal
one should explore thoroughly the patients
underlying ideas and preconceptions as to the Ulcer recurrent, epigastric, improved with
cause of their symptoms. food or antacids in a smoker or in the context
of excessive alcohol
Gastro-oesophageal reflux burning,
On entering the room begin with radiating from epigastrium to centre of chest,
open questioning worse on lying flat and post meals in patient
with risk factors
In the first instance open questioning should
be used to subsequently guide a more focused
Oesophageal dysmotility insidious onset,
related to swallowing
Case 138 Chest pain 259

Musculoskeletal stable chest pain, always ensure that there are


no crescendo or rest symptoms consistent with
Strain reproducible long-standing pain with
unstable angina or acute coronary syndromes.
history of heavy lifting or recent infection
Establish exercise tolerance as accurately
with chronic cough
as possible, usually the distance on the flat
Costochondritis focal reproducible
that the patient can tolerate before onset
tenderness aggravated with movement
of symptoms. Commonly, where it proves
Fractures history of trauma, pain with
difficult to quantify distance, patients relate
movement
better to the number of flights of stairs.
Sometimes it is more useful to establish
Infective whether the distance has decreased recently,
Herpes zoster unilateral, band-like, limited by pain or shortness of breath, thus
burning. relative as opposed to absolute distance is key.
Exercise tolerance will give both an indication
as to the severity of disease but also prove
Open questioning suggests a useful in monitoring disease progression,
diagnosis of angina particularly after initiation of management and
future clinic review.
With this diagnosis, closed Identify associated symptoms. Enquire
questioning should focus on as to the presence of autonomic features
including sweating, dry mouth and nausea. In
the following some patients shortness of breath represents
A typical history of angina will involve all of: the predominant feature of their angina
Constricting or pressure-like discomfort in and proves more troublesome than chest
the anterior chest, neck, shoulder, or arm discomfort.
Pain precipitated by a clear trigger, most Succinctly combine past medical and social
commonly physical exertion or stress history to achieve a review of the patients
Relief after a period of rest or use of cardiac risk factors. It can be useful to ask
sublingual nitrate within minutes a general question such as Other than the
episodes of chest discomfort that are clearly
Where a patient describes any of the above it
troubling you, how is the rest of your health?
is important that you are seen to clarify each of
This demonstrates empathy and moves the
the three points. Consider using the Canadian
conversation on without seeming like you
Cardiovascular Classification grading severity
are running through a prescribed list. If the
related to triggers:
patient replies that she is otherwise well,
Class I s trenuous or prolonged physical it is useful to proceed with there are a few
activity questions that I ask everyone at this stage,
Class II v igorous physical activity: walking would that be alright followed by each of the
rapidly, climbing multiple flights of modifiable and non modifiable risk factors.
stairs If she informs you that she has diabetes,
Class III s ymptoms with everyday living clarify when it was diagnosed, whether it is
activities: walking, stairs, emotional diet, tablet, or insulin controlled, what her
stress blood sugar control is like and whether she is
Class IV angina at rest experiencing any recognised complications.
Focus on precipitating and relieving factors With regard to modifiable risk factors enquire
most commonly physical exertion resulting in as to control, compliance with medication
increased myocardial oxygen demand limited and most recent review. Be prepared to swiftly
by atherosclerotic disease, but be mindful of cover hypertension, hypercholesterolaemia and
the alternatives including decubitus and variant smoking cessation in a similar fashion. A history
(also known as Prinzmetals) angina. Decubitus of recreational drug use, in particular cocaine,
angina where there is a clear history of heart is also important, and needs to be sensitively
failure, occurring on lying flat due to increased addressed.
return to the heart and resultant strain. Variant A final review of symptoms and risk factors
angina, particularly in younger women with no offers you the ability to take stock of the history
atherosclerotic risk factors, occurring at rest and thus far and allows you to summarise the
in cycles, thought to be secondary to vasospasm. information that you have gleaned.
Despite being prepared for a history of To summarise, you have been experiencing
260 Chapter 6 History taking (station 2)

central chest discomfort that sometimes moves That is why, when you rest and your heart rate
into your neck after physical exertion for the last slows down, with time the pain goes away.
6 months. These symptoms seem to be starting Check the patient understands. Angina
after two flights of stairs and force you to stop. is chronic but can be treated. There are
The discomfort passes after a couple of minutes. medications that you can take to improve
It has been happening three times a week but your symptoms and the amount of exercise
you are concerned that it is occurring more that you can do. It is important that we treat
frequently lately. You never experienced these your symptoms and try to stop the narrowing
symptoms at rest. Is there anything else that we from getting worse to prevent you from having
havent discussed that you think is important problems in the future. Importantly we want to
and that I should know about? avoid problems like a heart attack.

Explore concerns, ideas, and What to tell the examiner


expectations Summarise the clinical case
Question Have you had any thoughts or This patient described symptoms consistent
concerns about what may be causing these with a diagnosis of angina. This is most
symptoms? commonly caused by atherosclerotic disease for
Answer Is this a heart attack? or Does this which she does (or does not) have risk factors.
mean I am going to have a heart attack? The requirement for functional or anatomical
Question How have these symptoms been tests is dependent on the likelihood of coronary
affecting your life? artery disease. That likelihood is dependent on
Answer I worry that this will affect my ability how typical the history of angina is, the patients
to work and my husband was recently made age and gender and the presence of risk factors.
redundant. or Does this mean I cannot drive?
Investigations
What to tell the patient I would review a 12-lead electrocardiogram,
By exploring their ideas, concerns and and check fasting lipids and blood glucose in
expectations, often the working diagnosis will order to risk stratify their cardiac profile. Resting
be offered by the patient and you will be in a echocardiography should also be considered
position to confirm and clarify. for LV function assessment. ECGs in angina
The symptoms that you describe sound very may show evidence of previous infarction,
much like angina. Have you heard of angina/ hypertrophy, or nonspecific ST-segment and
what do you understand by the term? T-wave abnormalities. A resting ECG alone does
Offer a working diagnosis, explore not establish or refute the diagnosis.
understanding, ensure clear explanations, and In this case, I would assess their calculated
formulate a plan of action. likelihood of coronary artery disease (as per
The discomfort that you experience when NICE guidelines) and use this to decide on the
you are walking up stairs I think is coming from most appropriate investigation to perform.
your heart. This is a common condition and Options include coronary CT scanning
is due to a narrowing of the blood vessels that (calcium score with or without CT coronary
supply your heart with blood. The heart pumps angiography), functional testing, or invasive
blood to the rest of the body, supplying oxygen coronary angiography. If the patients likelihood
to allow your body to work. But the heart itself of coronary artery disease is calculated as
needs blood and oxygen which are supplied being greater than 90%, then no further tests
by the coronary arteries. Over the years, the are necessary to reach a diagnosis of angina
coronary arteries can become narrowed. The (however, further investigations may be used,
narrowing is made worse by smoking, a fatty depending on the patients clinical presentation
diet, and other conditions like diabetes. When to aid decisions on revascularisation).
you are sitting at rest, the amount of blood that
your heart needs to do its job can be supplied Management
by your narrowed arteries. But when you ask Initiation of management and choice of
your heart to pump faster or harder, when you medication should always be discussed with the
are climbing stairs for instance, the narrowed patient. Give a brief overview, and then discuss
arteries prevent sufficient blood from flowing details with the examiner.
to your heart muscle and it responds with pain. Management should be considered as
Case 139 Irregular pulse 261

optimisation of modifiable and non-modifiable channel blocker when added to a beta-blocker)


risk factors (involving both lifestyle changes and where symptoms are not controlled on mono-
medical therapies), pharmacological secondary therapy. Anti-anginal drugs prevent attacks
prevention, and PRN and regular symptom relief of angina by decreasing myocardial oxygen
medication. consumption (by lowering heart rate, blood
In addition to dietary advice, smoking pressure, myocardial loading, or myocardial
cessation, and moderation of alcohol contractility) or by increasing myocardial oxygen
consumption, it is often necessary to aid supply (by increasing coronary blood flow).
a patient in pacing their daily activities, Where patients are intolerant of beta-blockers
managing stress and anxiety, and offering or calcium channel blockers, or both are
advice with regard physical exertion and contraindicated, second line treatment with one
sexual activity. Conservative management of the following is considered (depending upon
in this regard can be complex and require heart rate, blood pressure and tolerance):
significant input from primary care.
Secondary prevention measures: Aspirin (or Isosorbide mono-nitrate (ISMN)
clopidogrel if aspirin intolerant) and statin
Ivabradine
therapy should be prescribed for all stable
Nicorandil
coronary artery disease patients, with ACE
Ranolazine
inhibitors (or angiotensin receptor blockers if In cases of chest pain of recent onset, where
intolerant) if there is coexisting hypertension, investigations suggest significant coronary artery
diabetes mellitus or heart failure. disease or evidence of inducible ischaemia,
Anti-anginal drug treatment. invasive angiography with a view to coronary
In the first instance a short-acting intervention should be considered.
sublingual nitrate should be used. In providing
GTN spray for symptom relief, one should
explain to the patient how to administer the Further reading
short-acting nitrate. Furthermore, instruct National Institute for Health and Care Excellence
her to use it immediately before any planned (NICE). CG95, Chest pain of recent onset:
exercise or exertion. Also, warn the patient assessment and diagnosis of recent onset
that side effects such as flushing, headache chest pain or discomfort of suspected cardiac
and light-headedness may occur and to sit origin. London, NICE, 2010. National Institute for
down or find something to hold on to if feeling Health and Care Excellence (NICE). CG126, The
light-headed after use. management of stable angina. London; NICE,
Beta-blockers or a calcium channel blocker 2011.
are first-line treatment for stable angina, using
synergistically (a non-rate limiting calcium

Case 139: Irregular pulse

Candidate information spell associated with his palpitations for the first
time, resulting in a fall and minor head injury,
Scenario although he denies losing consciousness. This
As the registrar on the acute medical take you are man is a stoical type and this is the first time he
asked to see the following GP referral: I would has brought these symptoms to my attention,
be grateful if you would review this 76-year-old which I am concerned may be related to atrial
man who has been experiencing palpitations fibrillation in view of an irregularly irregular
over the last year. I reviewed him in the surgery pulse of 100 beats per minute. His blood
today after he presented having suffered a dizzy pressure today was 162/90 mmHg. Many thanks.
262 Chapter 6 History taking (station 2)

Please take a history from the patient, offer an episodes last less than 24 hours. Diagnosed
explanation for his symptoms and discuss your after 2 or more discrete episodes
findings with the examiner. Persistent episodes lasting more than
7 days, requiring pharmacological or
electrical cardioversion to terminate.
Actor information Labelling someone as having persistent
You have recently been experiencing palpitations atrial fibrillation implies that there is still
and dizziness. On one occasion you have actually an intention to attempt rhythm control to
fallen over with these symptoms. Your GP has achieve sinus rhythm
told you today that you have an irregular pulse. Permanent persisting for more than
You are concerned that you have also been told 1 year. Labelling a patient as having
that irregular pulses cause strokes, and you wish permanent AF implies pursuance of a rate-
to know how likely or unlikely this is. control strategy
The duration and frequency of palpitations
Prior to entering the room will be key in attempting to classify atrial
Despite the suggestion of atrial fibrillation, fibrillation from the history alone. Useful lines of
be wary of a presumptuous diagnosis. Allow enquiry will relate to the number of palpitations
a description of the episodes or symptoms experienced in the average, day, week, or month
before attempting to narrow the differential. and whether the patient feels that their pulse
The age of the patient might bias towards a ever returns to normal.
potential diagnosis of atrial fibrillation, but all
arrhythmias should be considered especially if
there is a history of preexisting cardiac disease.
Open questioning suggests a
Be mindful that this consultation will focus diagnosis of atrial fibrillation
on the underlying causes of atrial fibrillation,
symptoms from atrial fibrillation, and With this diagnosis closed
consideration of embolic cerebrovascular event questioning should focus on
risk with the need for anticoagulation and/or
warfarin counselling. the following
In considering the patients past medical history,
medication and social history, it is possible to
On entering the room, begin swiftly cover the common causes and triggers of
with open questioning atrial fibrillation:
Encourage the patient to best describe the
palpitations in his or her own words. Often Past medical history
patients will describe fluttering, racing, or simply Atrial fibrillation is common secondary to
pounding and an awareness of their heartbeat, cardiac disease that affects the atria including
others will describe missed beats. Diagnostic yield congestive heart failure, coronary artery disease,
from description alone is poor and the need to hypertensive heart disease, and rheumatic heart
capture episodes on a 12-lead electrocardiogram disease. Such aetiologies are more common in
paramount, however one can narrow the the elderly population and generally result in
differential based on the nature, onset, rate, persistent or permanent atrial fibrillation.
duration and termination of palpitations. In this
case, focus upon the rhythm and rate. Establishing Cardiac causes
the irregularly irregular nature and the rate will be
best achieved by asking the patient to tap out the
Ischaemic heart disease
beat with his finger.
Valvular pathology, most commonly mitral
valve disease resulting in raised left atrial
In such cases, the history can serve to
pressures.
establish some fundamental features of the
diagnosis. Firstly, is this new onset atrial
Hypertension
fibrillation and further, is it possible to classify
Heart failure, whereby AF is commonly an
accompanying pathology
between paroxysmal, persistent and permanent?
Atrial fibrillation can be classified as:
Non-cardiac causes
Acute onset within 48 hours
Thyroid disease, infection, pulmonary embolism
Paroxysmal episodes that terminate
and hypothermia can all precipitate atrial
spontaneously within 7 days, most often
fibrillation, as can electrolyte abnormalities.
Case 139 Irregular pulse 263

Ensure to enquire about the patients caffeine, What to tell the patient
smoking, and alcohol intake, and also a history
I think you have a condition called atrial
of stress and anxiety.
fibrillation which is causing an erratic heart
Where a potentially symptomatic episode
rate which you are experiencing as palpitations
has been eluded (as in this case) it is vital
and may have been responsible for your recent
to understand the exact circumstances
dizzy episode. Have you ever heard of atrial
surrounding the event. Clearly define the
fibrillation, or know anything about it?
features and relationship of the symptoms
Where you are reviewing a patient referred
to the palpitations then screen for any other
by a GP or colleague it is likely that some form of
symptoms, such as breathlessness, palpitations,
discussion has already occurred in rationalising
and syncope. Symptoms in atrial fibrillation can
the need for a specialist clinic appointment. As
be classified using the EHRA classification:
such, gauging the patients understanding is a
I. No symptoms good way to start and often makes the process
II. Mild symptoms those that do not affect of explaining the need for further investigation
normal daily activities with ECG, blood tests and echocardiogram,
III. Severe symptoms those that affect normal easier.
daily activities Tell the patient that you will arrange
IV. Disabling symptoms those that cause the some investigations, starting with a 12-lead
patient to stop normal daily activities electrocardiogram.
Treatment options should be explained
Stratify risk succinctly, explaining:
Embolic risk: Ask about a history of heart
a.  The potential to treat an underlying cause
failure, hypertension, diabetes mellitus, revealed by the results of investigations, or
previous stroke or TIA, or previous vascular avoid triggers elucidated in the history
disease The potential need to slow the heart to
Bleeding risk: In addition to previous
b.  prevent further dizzy episodes
questions, specifically ask about previous The choice of medications to slow the heart
bleeding episodes or use of anticoagulation, or the potential to stop further palpitations
renal disease, liver disease, and other and restore normal sinus rhythm may be
medications such as NSAIDs influenced by the results of the investigation,
including the echocardiogram. It would
Explore concerns, ideas, and be sensible to arrange a further clinic
appointment with the results, or admit to
expectations hospital for investigation if sufficient concern
Be prepared for varying themes, introduced by about symptoms
the patient, which may change the direction Explain the risk of complications from atrial
of the consultation. With common conditions fibrillation, specifically that of an embolic
such as atrial fibrillation, often there will be cerebrovascular event and the need for
a twist for which you should be prepared. As anticoagulation. This will involve use of the
always ideas, concerns and expectations can be CHA2DS2 VASC score (see below) which is
used to lead into diagnosis, investigation and more likely to come up in discussion with the
management. The patient may offer statements examiner, but a brief explanation of risk and
such as: choice of anticoagulation with the patient is
prudent
I think this is happening because I have
been drinking too much since the death of
my wife What to tell the examiner
I was very frightened by the fall, will it
Summarise the clinical case
happen again?
My GP told me I would have a stroke, I am Reaffirm the working diagnosis and
very upset. What did he mean? classification of atrial fibrillation. Succinctly
My GP told me I would need to take reiterate the need to investigate as outlined with
medication for the rest of my life, I don't the patient to confirm the diagnosis and screen
like pills. Do I have any choice in the for underlying causes. Where ECG demonstrates
matter? sinus rhythm and paroxysmal AF is suspected,
264 Chapter 6 History taking (station 2)

discuss the need for ambulatory investigation. used in the presence of structural heart disease
The emphasis of your discussion should then or heart failure. The relative benefits and
be placed upon the principles of management, risks, particularly side-effect profiles, of these
rhythm versus rate control, and rationale for common drugs need to be considered on an
anticoagulation. individual patient basis.
Otherwise, rate control is offered. Emphasize
Investigations that rate versus rhythm control has no difference
on long-term outcome, with patients offered
The necessary investigations in a case of atrial
rhythm control more likely to experience
fibrillation may include:
side-effects of therapy or intervention (hence
A 12-lead electrocardiogram which may this is reserved for those in whom the benefit
demonstrate atrial fibrillation at that time. outweighs the risk). Rate control is achieved
There may also be evidence of ischaemia initially with a beta-blocker or a rate-limiting
and/or structural heart disease calcium-channel blocker, with digoxin as
Holter monitoring may be required if the adjunctive therapy if first line monotherapy
12-lead ECG demonstrates sinus rhythm has failed. Long-term digoxin monotherapy
(i.e. to achieve the diagnosis). Furthermore, is only considered for sedentary individuals.
ambulatory monitoring helps to assess for Amiodarone is not recommended for long-term
the adequacy of rate control in patients with rate control. Difficult cases of poorly controlled
known AF and also to help identify any other ventricular rate despite medical therapy may
concomitant arrhythmias require a pacemaker with AV node ablation
Transthoracic echocardiography can give strategy.
aetiological information such as confirming
the presence of valvular disease or the Anticoagulation
presence of LV dysfunction. Furthermore it
The need for anticoagulation relates to the
can aid in the assessment of risk stratification
inherent risk of embolic stroke related to atrial
for cardioembolic disease
fibrillation, which is increased in the presence
of other risk factors. The CHA2DS2-VASc score
Management is a scoring system which aims to risk stratify
Patients with AF should be offered an patients suffering from atrial fibrillation with
individualised, tailored package of care. The the intention of identifying those who would
main principles of management in atrial benefit most from anticoagulation. It uses
fibrillation can be summarised as: the following factors to assess stroke risk:
heart failure, hypertension, age, diabetes,
Rate control vs rhythm control
cerebrovascular disease, vascular disease,
Risk stratification for anticoagulation
patient sex. Aside from patients undergoing
If possible, identify and treat the underlying
cardioversion or intervention, long-term
cause
anticoagulation is not offered to patients
under 65 years with AF and no risk factors
Rate versus rhythm control other than their sex (i.e. a CHA2DS2-VASc
Rhythm control is offered to patients: score of 0 for men or 1 for women). Otherwise,
men with a score of 1 can be considered for
in whom it would be more suitable based anticoagulation, and anticoagulation is offered
upon clinical judgement (such as young to all patients with a score of 2 or more.
individuals, for example) In at risk patients with suitable safety profiles,
who develop heart failure secondary to the formal anticoagulation with warfarin or the novel
AF oral anticoagulants (NOACs, such as apixaban,
with new-onset AF dabigatran, and rivaroxaban) is recommended
in whom AF has a reversible cause for reducing the risk of stroke. Aspirin has no
Rhythm control strategies include drugs, role as monotherapy in the contemporary
electrical cardioversion, and ablation management of stroke prevention in AF.
procedures (pulmonary vein isolation). Bleeding risk must always be taken into
Drug therapy is individualised to the patient. account. An important additional scoring
Flecainide, for example, can be used in the system is the HAS-BLED score, which attempts
absence of underlying ischaemic or structural to quantify the risk of bleeding. This is an
heart disease, whereas amiodarone may be important tool in considering anticoagulation
Case 140 Palpitations 265

therapy particularly in older patients where with the North American Society of Pacing
there is a history of previous bleeding such and Electrophysiology. Circulation 2001;
as peptic ulcers or cerebral haemorrhage, 104:21182150.
excess alcohol intake, chronic liver or kidney Camm JA, Kirchhof P, Lip GYH, et al. Guidelines for
disease, poorly controlled hypertension or on the management of atrial fibrillation. Eur Heart
a background of poly-pharmacy. Combining J 2010; 31:2369-2429.
CHA2DS2-VASc and HAS-BLED scores allows Camm AJ, Kirchhof P, Lip GY, et al. Guidelines for the
you to consider the risk versus benefit of management of atrial fibrillation: the Task Force
anticoagulation in atrial fibrillation. for the Management of Atrial Fibrillation of the
In complex patients, or those with poor European Society of Cardiology (ESC). Eur Heart
symptom control, consider referral to an J 2010:23692429.
electrophysiology specialist service where National Institute for Health and Care Excellence
interventional options such as ablation can be (NICE). CG180, The management of atrial
considered. fibrillation. London; NICE, June 2014.
Lip GY, Nieuwlaat R, Pisters R, Lane DA, Crijns HJ,
et al. Refining clinical risk stratification for
Further reading predicting stroke and thromboembolism in
Fuster V, Rydn LE, Asinger RW, et al. ACC/AHA/ atrial fibrillation using a novel risk factor-based
ESC guidelines for the management of patients approach: the euro heart survey on atrial
with atrial fibrillation: executive summary a fibrillation. Chest 2010; 137:263272.
report of the American College of Cardiology/ Lip GY, Frison L, Halperin JL, Lane DA. Comparative
American Heart Association task force on validation of a novel risk score for predicting
practice guidelines and the European Society bleeding risk in anticoagulated patients with
of Cardiology committee for practice guidelines atrial fibrillation: the HAS-BLED (Hypertension,
and policy conferences (committee to develop Abnormal Renal/Liver Function, Stroke, Bleeding
guidelines for the management of patients with History or Predisposition, Labile INR, Elderly,
atrial fibrillation) developed in collaboration Drugs/Alcohol Concomitantly) score. J Am Coll
Cadriol 2011; 57:173180.

Case 140: Palpitations

Candidate information is an added stress on top of your upcoming


exams. You do not feel dizzy or collapse with
Scenario your palpitations. You do not take any regular
As the medical registrar on-call, you are asked to medications or drugs.
review a patient in Accident and Emergency. A
22-year-old male student has been experiencing Prior to entering the room
palpitations recently. It is a stressful time of
year with end of year examinations looming. In general when considering palpitations the
His observations are unremarkable, HR 68, BP importance is placed upon distinguishing
122/81. Resting 12-lead electrocardiogram is between those that are harmless requiring
normal. Please take a history from the patient, reassurance only versus those that carry
tell them what you think is wrong and discuss more serious adverse outcomes necessitating
your findings with the examiner. intervention. Palpitation is a non-specific
term relating to an increased and subjective
awareness of the heart beating. Palpitations
Actor information commonly reflect a change in cardiac rate,
You have recently noticed an unnerving rhythm or contractility. Accurate diagnosis
sensation of feeling your heart beat at rest. requires capture of an electrocardiogram during
You are very concerned about this, which an episode of palpitations. Thus, the history
266 Chapter 6 History taking (station 2)

should serve in the first instance to risk stratify A fast irregularly irregular heartbeat, often
patients to decide on appropriate reassurance with sudden onset and offset will require
versus referral or admission for observation, investigation, treatment and referral with the
monitoring or treatment. presumption of paroxysmal atrial fibrillation.
Symptoms of sudden onset and offset
associated with a heartbeat which is often too
On entering the room begin with rapid to count, likely regular but occasionally
open questioning difficult to ascertain due to the rate, and
Encourage the patient to describe the associated with symptoms of breathlessness,
palpitations in his own words. Often patients chest tightness or dizziness raise the suspicion of
will describe fluttering, racing, or simply supraventricular or ventricular tachycardias.
pounding and an awareness of their heartbeat,
others will describe missed beats. Diagnostic Consider a summary of differential
yield from description alone is poor and the diagnosis based on the history
need to capture electrocardiographic evidence
during symptoms is paramount, however
Extra or missed beats, isolated, thumping
sensation:
one can narrow the differential based on the
nature, onset, rate, duration and termination of
Premature atrial or ventricular beats
benign normal variant
palpitations. It is often more useful to ask the
patient to tap out the beat to better assess rate
Intermittent atrioventricular block
and rhythm.
Irregular:
For all palpitations, to narrow the differential,
Atrial fibrillation or flutter
consider the following:
Sudden onset and termination, regular,
recurrent lasting for seconds or minutes to
Nature, rate and rhythm speed of onset and hours:
offset, estimated rate, regular or irregular? Supraventricular arrhythmias
Duration and frequency || Atrial tachycardia
Termination use of valsalva, knowingly || Atrioventricular re-entry tachycardia
or unknowingly by holding their breath or (AVRT), including WolffParkinson
'popping' their ears White
Precipitants stress, anxiety, sleep || Atrioventricular nodal re-entry
deprivation, caffeine, drug use (prescription tachycardia (AVNRT)
or recreational), pregnancy Ventricular arrhythmias
|| Sustained or non-sustained ventricular
Consider associated features red flags tachycardia (VT)
prompting possible admission or expedient || Polymorphic VT
referral as certain associated symptoms raise Palpitations during exercise or stress:
suspicion of a more serious underlying pathology: Sinus tachycardia
Light-headedness, severe dizziness or Exercise can induce supraventricular
syncope (particularly if injury occurs from or ventricular tachycardia, particularly
syncope) polymorphic VT in patients with
Tunnel vision pre-existing long QT
Chest pain Ischaemia induced arrhythmia, often
Breathlessness ventricular
Medication or recreational drug use:
Patients sometimes complain of missed Drug-induced palpitations. Think of illicit
or skipped beats. On further questioning it drugs and also of drugs that prolong the
may transpire that the sensation is one of QT interval on the ECG
an isolated pounding, or of a weak or strong Palpitations associated with emotional
extra beat. This would raise the suspicion of distress or general anxiety may not have
ectopic beats. Commonly they occur at rest, an underlying arrhythmia, but should be
rarely with compromise and require firm considered as a diagnosis of exclusion
reassurance from the outset particularly on after organic arrhythmia has been ruled
a background of no previous cardiac disease out
(which is an imperative piece of past medical
history to establish, see below). Ectopics tend to Where the history does not conform to a
disappear with exercise. classical pattern, be sure to establish the key
Case 140 Palpitations 267

features with prompting where appropriate Macrolide antibiotics erythromycin,


and aim to review with the results of further clarithromycin
investigation. Antihistamines terfenadine
Anti-depressants
Open questioning suggests a In the social history, attempt to quantify
diagnosis of fast palpitations the intake of caffeine and alcohol, and where
possible establish any potential link with
With this diagnosis closed onset of symptoms to intake or increase
in consumption. Smoking habits may be a
questioning should focus on useful indicator of stress levels and serve as a
the following sensitive way in which to broach the subject
A review of symptoms should cover those of Is there any particular reason that you have
potentially causative disorders. Where anxiety been smoking more recently?, Do you smoke
and stress form a predominant feature of more when you are stressed? Do you feel more
the history relating to palpitations be sure to stressed recently?
consider hyperthyroidism and rarer diagnoses You must ask about illicit drug use, especially
such as phaeochromocytoma: in the younger patient. Explain to the patient
that it is a standard, confidential question with
Hyperthyroidism palpitations in the context no judgement or suspicion attached. Directly
of heat intolerance, weight loss, and tremor ask about common recreations drugs including
would be useful screening questions cocaine, ecstasy and amphetamines.
Phaeochromocytoma palpitations in
the context of headaches, sweating and
diaphoresis on a background of hypertension
Explore concerns, ideas, and
would be strongly suggestive expectations
Where the review of symptoms are vague What do they think is causing the palpitations?
and centre around tiredness, lethargy and Is it affecting his ability to study? Has his
breathlessness think of anaemia. Screen sporting performance decreased, is he reluctant
for predispositions such as haematological to push harder or perform to his best for fear of
disorders or for symptoms such as menorrhagia symptoms?
and rectal bleeding. Awareness of palpitations can be heightened
Consider the past medical history. Ask in depression or anxiety thus screening is
specifically about any previous history of cardiac prudent.
disease (including ischaemic, congential,
or valvular heart disease or heart failure) What to tell the patient
thyroid disorders, and previously diagnosed
Without electrocardiographic evidence of the
hypertension if not suggested by the review of
causative cardiac rhythm disturbance (if any)
symptoms. Structural heart disease, especially
it is impossible to give the patient a definitive
mitral valve disease, with which palpitations
diagnosis. It is possible, however, to inform the
are a common finding. Also congenital
patient of a range of likely differential diagnoses,
abnormalities and cardiac surgery. In the family
and to reassure them (if possible) regarding risk
history screen for arrhythmias and sudden
stratification based on the absence of red flag
cardiac death.
symptoms. Inform the patient of the necessary
Be alert to the use of medication with
investigations that will be organised with the
adrenergic effects, predisposing to palpitations,
aim to reach a diagnosis. Further advise the
such as b-agonists or theophyllines in the
patient on any lifestyle modifications which
treatment of asthma, levothyroxine in the
can be implemented with the aim of symptom
treatment of hypothyroidism, or commonly
reduction, e.g. managing stress. Inform the
over the counter remedies for flu and cold that
patient of any follow-up which is necessary.
contain pseudoephedrine. Be aware that certain
medication can inadvertently induce palpitations
and prolong the QT interval predisposing to What to tell the examiner
ventricular tachycardia. These include:
Summarise the clinical case
Anti-arrhythmia medication amiodarone, The patient has intermittent palpitations
quinidine, sotalol without suggestion of red flag symptoms. Apart
268 Chapter 6 History taking (station 2)

from stress due to examinations, there are no Implantable loop recorders are typically used
obvious precipitating features. in situations where a diagnosis has not been
reached despite extensive ambulatory monitoring
Investigations and/or where the time period between symptoms
is sufficiently long such that a 24-hour or 1 week
Standard screening of all palpitations will
long period of monitoring is thought to have a low
involve a resting 12-lead electrocardiogram (to
diagnostic yield.
assess the resting cardiac rhythm and to identify
Electrophysiological studies are also an
signs of structural heart disease or arrhythmia
option, typically utilised following review at an
potential such as long QT interval), together
arrhythmia clinic.
with echocardiography particularly in cases of
suspected underlying structural heart disease.
Routine blood testing with full blood count to Management
rule out anaemia, quantification of electrolytes, The specific management of palpitations
and importantly thyroid function testing. Where varies widely depending on the underlying
appropriate, urinary catecholamines dependent diagnosis. For the patient in this specific case,
upon clinical context. no drug treatments are necessary based on the
Ambulatory electrocardiogram monitoring evidence provided to the candidate. The patient
is used to identify the cardiac rhythm at the should be reassured and advised regarding
time of symptoms. The duration of monitoring stress management. Any specific treatments
is variable, usually between 24 hours to 1 week. would depend on reaching a specific diagnosis
following the results of investigations.

Case 141: Ankle swelling

Candidate information given the diagnosis, the pitfalls of a greater


depth of patient understanding and complex
Scenario management issues typically makes these
You are the cardiology SHO. A 68-year-old encounters harder. That said, PACES does
woman, with an established diagnosis of heart not assume in-depth specialist knowledge,
failure, attends for her 6-monthly review in the and the approach to the station should cover
outpatient clinic complaining of increasing common themes, in a generalist fashion,
ankle swelling. Please make an assessment and adopting a holistic approach. Be mindful that
formulate a management plan. you have been specifically asked to formulate
a management plan and thus taking even
the most thorough history alone will not be
Actor information sufficient. Indeed, regardless of whether it
You were diagnosed with heart failure is specifically stated in the instruction, the
approximately 2 years ago. Your symptoms have strong candidate will always strive to offer a
deteriorated slightly in the past few months. You management plan and suitable time frame for
can only manage to walk about half a mile on further review or specialist opinion.
the flat and you would like to be able to do more.
On entering the room begin with
Prior to entering the room open questioning
In considering a patient review, it will be
Greet the patient and open the dialogue in
important to attempt to gather a past history
such a way that allows them to direct you to
of the condition and use this in gauging the
areas of concern whilst potentially offering
severity and importance of any new symptoms.
you useful information. If, in starting the
Although this type of station appears simpler
conversation you ask specifically about their
Case 141 Ankle swelling 269

ankle swelling, it prohibits the patient from The question worth considering in this
giving you general insight into the potential case will be whether the symptoms reflect a
aetiology of their heart failure, and time worsening or progression of disease requiring
since last review or diagnosis. For example, optimisation or whether this is non-compliance
a question such as How have you been and with medication resulting in symptoms but no
how can help you today may be met with a worsening of disease.
response such as Well doctor, life since my Establish current management and look
heart attack 6 months ago has been terrible, to consider common side effects to explain
and now my ankles have started to swell up non-compliance. Tactfully approach this
too, I just cannot cope, or Things have been subject. Many patients will resent the position
going wonderfully since I last saw you in in which they have been placed, with chronic
December doctor, I am back playing bowls illness, limitation on lifestyle and the burden
again and have even become involved with the of polypharmacy. Fairly innocuous side effects
Cardiomyopathy Society like you suggested. I may have a significant bearing on quality of
went to a fundraiser last week but fear I over life in this context. Be mindful to screen for
did it and now I have noticed my ankles are depression and end-of-life decisions. It is not
swelling somewhat. uncommon for this scenario to end up in a my
Where possible aim to establish: lifes not worth living any more discussion.
Alternatively consider recent changes in
The nature of the heart failure relating
medication, withdrawal or altered dosing, or
to ischaemic heart disease; hypertension;
addition of new medication in relation to onset
valvular pathology; arrhythmias; infiltrative
of symptoms.
disease or cardiomyopathy
The time since diagnosis or last review
The patients general state of health and Open questioning suggests a
whether symptoms improving, staying the diagnosis of heart failure
same or worsening
After that, probe with regard to the ankle With this diagnosis closed
swelling. Enquire as to the onset of symptoms, questioning should focus on
insidious or acute, and duration, intermittent or
persisting over a long-period of time. Be sure to
the following
gain a clear picture of the symptoms including Of medication side effects
the extent of the swelling: Is it limited to the
ankles or does it extend to the knee above? Is
ACE inhibitors cough, hypotension
the swelling pitting? Are there associated signs?
Diuretics urgency, night time waking, and
incontinence. Dizziness and leg cramps
Establish whether this is a recurrent problem
and seek to identify associated symptoms of
Beta-blockers lethargy, wheeze
(contraindicated in asthma)
heart failure including:
Nitrates headache and dizziness
Shortness of breath Aldosterone antagonists fatigue and
Chest pain headaches
Palpitations
Paroxysmal nocturnal dyspnoea Of alternative causes of ankle swelling
Orthopnea Be mindful, despite the known diagnosis of
Be seen to relate the symptoms to exercise heart failure, that where there is doubt you
tolerance and gauge her disease severity in should consider the alternative causes of lower
terms of the New York Heart Association limb swelling. Does the swelling appear to be
(NYHA) classification. unilateral or bilateral? Where the former is
NYHA defining severity in relation to exercise prominent and acute in onset, rule out deep
limitation by symptoms: vein thrombosis. Venous insufficiency may
be suggested by dependent oedema and by a
I no symptoms and no limitation in history of symptoms worsening throughout the
ordinary physical activity day. Screen for causes of hypoalbuminaemia,
II mild symptoms of shortness of breath hepatic disease, or renal insufficiency. Consider
or stable angina with slight limitation on on the basis of pitting versus non-pitting, in the
ordinary activity latter thinking about lymphoedema. Finally
III marked limitation in activity screen for inflammatory processes with resultant
IV severe limitation, symptoms at rest oedema including infection or arthritis.
270 Chapter 6 History taking (station 2)

Explore concerns, ideas, and Investigations


expectations Initial investigations in cases of suspected heart
failure will include:
Where you are reviewing a chronic condition
the emphasis will be placed on identifying Serum BNP, which should be raised
potential reasons for exacerbations and A 12-lead electrocardiogram: In symptomatic
the patients ideas will be key in offering ally and echocardiographically-severe heart
insight to this. Medication compliance will failure it is important to note the QRS duration
be central to this as outlined above. It will be as this has management implications (cardiac
important to address the patients concerns resynchronisation therapy)
of a worsening of their condition, outline the A plain chest radiograph will demonstrate the
potential for investigation and understand their cardio-thoracic ratio and also demonstrate
expectations for management. Establishing any active pulmonary oedema
whether the patient expects more medication, A transthoracic echocardiogram is diagnostic
is prepared to try a change in regime or In severe systolic dysfunction, with or without
titration of dosing, or whether they have heard a dilated left ventricle, a period of ambulatory
about the potential for implantable devices ECG monitoring will help to exclude the
will guide the conversation about further presence of ventricular arrhythmia
management. Cardiac MRI for tissue characterisation in the
evaluation of cardiomyopathy or infiltrative
What to tell the patient diseases

Inform the patient of the working diagnosis. Management


Explain to them the need for investigations to
both confirm the diagnosis and to influence In addition to addressing aetiological and
management decisions. Tell the patient you exacerbating factors, the management of heart
would like to perform some blood tests, repeat failure due to left ventricular systolic dysfunction
the ECG to assess for any new arrhythmias, should involve:
and review the function of her heart with an Institution of prognostic therapy where
echocardiogram. possible with beta-blockade and ACE
Inform the patient that they will need to start inhibition
some medications to treat the heart failure. Symptomatic therapy with diuretics
Summarise the case, and reaffirm the need In patients with severe systolic dysfunction,
for review with the results of the investigations mineralocorticoid receptor antagonists
outlined. Titration of a loop diuretic dose may be such as spironolactone (or eplerenone in
prudent as an initial management option where ischaemic cardiomyopathy) are indicated
renal function is not of concern. Alternatively,
management change may centre on compliance, Biventricular pacing with cardiac
and areas such as increasing morning, and resynchronisation may relieve symptoms and
reducing evening, doses of diuretics to reduce reduce heart failure hospitalisations for patients
nighttime waking or deal with incontinence who have severe, symptomatic heart failure and
issues. a widened QRS on ECG.

What to tell the examiner Further reading


McMurray JJ, Adamopoulos S, Anker SD, et al. ESC
Summarise the clinical case Guidelines for the diagnosis and treatment
Succinct summary of patient history including: of acute and chronic heart failure 2012: The
Understanding from patient of the aetiology Task Force for the Diagnosis and Treatment of
of their heart failure Acute and Chronic Heart Failure 2012 of the
Current symptoms and NYHA grading European Society of Cardiology. Developed
Current management in collaboration with the Heart Failure
Working diagnosis re: exacerbating factors Association (HFA) of the ESC. Eur Heart J 2012;
33:17871847.
Case 142: Raised lipids

Candidate information Rather than merely informing the patient that


your lipids seem to be raised and require further
Scenario attention, consider the following approaches:
You are the registrar in the cardiology clinic We have performed a screen of your lipid
reviewing a 56-year-old man, referred to the levels. Commonly we do this as part of a general
outpatient clinic due to raised lipids on blood health check or when we are concerned about
testing. His serum lipid profile is: a patients risk for heart disease or vascular
disease. In your case, I would like to know more
Total cholesterol 6.7 mmol/L about the reason for performing your tests as it
Triglycerides 3.4 mmol/L would appear that some of the levels are raised.
HDL cholesterol 0.8 mmol/L Were you expecting this? followed by What do
LDL cholesterol 4.4 mmol/L you understand to be the implications of raised
Please take a history from this man and lipid levels?
discuss the significance of the blood test results It may then be necessary to field questions
with him. from the patient, which will influence the
direction of the rest of the history. The strong
candidate will try to obtain a clear background
Actor information for the test results. In narrowing your diagnosis
On recent review at your local GP practice, you in a structured manner, attempt to distinguish
were told that your blood tests for cholesterol between primary and secondary causes of
were abnormal. You are surprised, given that dyslipidaemia. Thereafter, while dyslipidemia
you consider yourself to have a healthy lifestyle. itself usually causes no symptoms, probe
You are otherwise well. for resultant symptomatic vascular disease,
principally coronary artery disease and
peripheral arterial disease.
Prior to entering the room Patients with primary disorders may present
Conscious of the fact that hyperlipidaemia with:
in itself does not cause symptoms, the strong
candidate will consider the propensity for Eyelid xanthelasmas
associated conditions such as peripheral Prominent corneal arcus at a young age
vascular and coronary artery disease. Tendon (commonly achilles, knee and
elbow) or cutaneous xanthomas
Pancreatitis acute episodes may occur
On entering the room begin with in patients with significantly elevated
open questioning triglyceride levels
Whenever the explanation of results is required Common primary (familial) causes,
it is important to establish the reasoning behind most usefully categorised based on whether
ordering them, what prompted the investigation cholesterol, triglyceride or both are prinicipally
and how they relate to a patients ongoing health raised:
issues and management. The results require
context. It is useful to open the dialogue with
Familial hypercholesterolaemia
a review of the patients understanding and
Familial hypertriglyceraemia
expectations from the consultation. The nature
Familial combined hyperlipidaemia (raised
cholesterol and triglycerides)
of the results should guide you in this respect.
I understand you have attended to receive Importantly, a low HDL confers susceptibility
the results of your blood tests. Before we discuss to premature atherosclerosis and should also be
them, can you tell me what you understand considered.
about lipids and what you were expecting these Secondary, acquired, causes include:
tests to show?
The nervous, anxious or demanding patient Sedentary lifestyle with poor diet excessive
may not allow such an approach, insisting intake of saturated and trans fats. (Most
on delivery of the results before entering into common cause of dylipidaemia)
a dialogue. In such circumstances, it will be Diabetes mellitus
necessary to package or frame the results. Alcohol excess
272 Chapter 6 History taking (station 2)

Chronic kidney disease I think this has happened because I have


Hypothyroidism eaten too much fast food, can the damage be
Cholestatic liver disease such as primary reversed?
biliary cirrhosis I don't understand, I don't smoke and eat
Medication including: thiazides diuretics, healthily, how can this be the case?
beta-blockers, retinoids, highly active My friend takes a pill for his cholesterol and
antiretroviral treatment, and glucocorticoids. eats whatever he wants, can I just do that?
The patient may be asymptomatic,
Does this mean I will have a heart attack?
or complain of symptoms related to the
complications of hyperlipidaemia, such as What to tell the patient
angina or intermittent claudication. The patient should be informed of: the working
Thus, dyslipidemia is suspected in patients diagnosis; the current management plan; and
with characteristic physical findings or the implications on the patients cardiovascular
complications resulting from atherosclerotic health.
disease, and primary lipid disorders are suspected
when patients have physical signs of dyslipidemia,
suffer the onset of premature atherosclerotic What to tell the examiner
disease, or have a strong family history of Summarise the clinical case
hyperlipidaemia or atherosclerotic disease.
The discussion with the examiner should
include investigations, management and
Open questioning suggests a statements to the examiner that the candidate
diagnosis of raised lipids understands the role of lipid therapies in
modifying cardiovascular risk.
With this diagnosis questioning
should focus on the following Investigations
Establish the potential for treating the other
Where raised lipids are of concern, focus
modifiable risk factors and the secondary causes
closed questioning on risk stratification using
of hyperlipidaemia as discussed above:
the QRISK2 risk assessment tool to assess
cardiovascular disease risk for the primary Routine blood tests including full blood
prevention. Using the results of the lipid count, renal function, electrolytes and liver
screen (total cholesterol and HDL) combined function
with knowledge of other cardiovascular risk Ensure the lipid profile was a fasting sample,
factors (smoking history, blood pressure, explain and repeat if necessary
diabetes, family history of cardiovascular Check fasting glucose and an HbA1c
disease) and past medical history (including Check thyroid function
AF, chronic kidney disease, blood pressure Calculate body mass index
treatment and rheumatoid arthritis), it is
possible to estimate a 10-year risk of having Management
a heart attack or stroke. This can also be
Initial management will involve dietary and
presented to the patient as their heart
exercise advice to achieve weight loss and
age relative to their actual age, and their
improvement in lipid profile together with
relative risk of having a cardiovascular event
alcohol moderation and smoking cessation. The
compared to an average individual of the
challenge will be to encourage the patient to
same age, gender and ethnicity.
understand their risk status and work together
Do not use a risk assessment tool to assess
to improve their profile. If these measures are
cardiovascular disease risk in patients with type
ineffective, lipid-lowering drugs should be
1 diabetes, known cardiovascular or significant
considered.
chronic kidney disease, or inherited lipid
Encourage a diet low in fat:
disorders. These people are already at increased
cardiovascular risk. Total fat intake less than 30% total energy
intake
Explore concerns, ideas, and Saturated fat less than 7% total energy intake
Less than 300 mg/day cholesterol
expectations Replace saturated fats, where possible, with
Upon being afforded the opportunity, the monounsaturated and polyunsaturated
patient may offer concerns such as the following: fats
Case 143 Hypertension 273

Combined with 5 portions of fruit and or contraindications, a lower dose or lower


vegetables per day and 2 portions of fish per intensity preparation such as pravastatin may
week. It will be worth referring to a dietician be chosen. Fibrates and nicotinic acid should
to achieve an optimal 'cardioprotective' diet not routinely be considered for primary or
secondary prevention, and specialist opinions
Physical activity is recommended as should be sought in those with especially
30 minutes of physical activity a day (or high cardiovascular risk (such as inherited
cumulative 10 minute sessions throughout the dyslipidaemia), and in secondary prevention
day), of at least moderate intensity (brisk walk, patients with intolerances to multiple statins.
using stairs, cycling), at least 5 days a week. Advise patients to read the drug information
If the patient does not currently suffer leaflet carefully, paying particular attention
from ischaemic heart disease, peripheral to possible interactions (such as grapefruit
vascular disease and has no history of stroke juice, for example). If there is any history of
then primary prevention treatment with a previous generalised myalgia, irrespective
statin is only recommended where the risk of previous statin use, check creatine kinase
of developing such disease is high, (QRISK2 levels. Patients should seek medical attention
10year risk of greater than or equal to 10% if they develop muscle pains on statins, and
of developing cardiovascular disease) or if creatine kinase levels measured (with statin use
lifestyle modification has been ineffective. If stopped and future management re-evaluated).
however, the patient has a history of any form Asymptomatic patients should not have creatine
of atherosclerotic disease, they are inherently kinase levels routinely measured.
deemed high risk and the stratification process Liver transaminases should be measured at
is not necessary when deciding on whether baseline, 3 months and 12 months. Statin
to commence lipid lowering treatment (they therapy should be continued unless these are
are classed as requiring secondary prevention over 3 times the upper limit of normal.
treatment, with first line recommendation being Lipid levels should be monitored 3 months
atorvastatin 80 mg). after starting or changing therapies, aiming for
Treatment for the primary prevention of at least 40% reduction in nonHDL cholesterol,
cardiovascular disease should be initiated and at least annually thereafter.
in people who have a 10% or greater 10year
risk of developing cardiovascular disease,
and in all adults with type 1 diabetes who fit Further reading
one of the following criteria: are older than National Institute for Health and Care Excellence
40 years, have had diabetes for more than 10 (NICE). Lipid modification: cardiovascular risk
years, have established nephropathy, or have assessment and the modification of blood lipids
other cardiovascular risk factors. First line for the primary and secondary prevention of
therapy in such individuals is atorvastatin 20 cardiovascular disease. Clinical guideline 181.
mg. If there are potential drug interactions London; NICE, July 2014.

Case 143: Hypertension


on no medication, and has thus far advised
Candidate information the patient to stop smoking and improve his
Scenario diet.Please take a history from this man and
You are the registrar in the general medical discuss with him your management plan.
clinic. You are asked to review an anxious
30-year-old man in the general medical clinic, Actor information
referred by his GP in view of two previous BP You have been seeing your GP recently because
measurements of 158/90 and 165/88 mmHg. of high blood pressure, which was initially
The GP has been unsure as to whether to start noted at a well-man clinic. You are keen to
treatment in a patient this age, he is currently
274 Chapter 6 History taking (station 2)

know whether you need to take any tablets for recent exertion or caffeine or drug ingestion.
your blood pressure. You also wish to know the And establish the timing between readings.
reason why you have high blood pressure. Does the patient ever remember being told he
had a normal blood pressure?
Begin prompting when necessary,
Prior to entering the room and enquire as to the general features of
Hypertension is usually asymptomatic, uncontrolled hypertension or end organ
with symptoms heralding either a severe, damage. In so doing, attempt to gauge the
uncontrolled manifestation, or as the result severity of the problem. Ask specifically about
of long standing disease the symptoms of headaches, visual disturbance, epistaxis,
end-organ damage. Essential hypertension palpitations and any urinary abnormalities
is common and offers little in the way of (haematuria or frothy urine).
history; however, secondary hypertension may It will be useful to explain to the patient
involve complex constellations of symptoms why you are asking these questions I need
relating to renal or endocrine disorders. While to understand how your blood pressure is
hypertension is a common finding with age, in affecting you. Sometimes it can cause headaches
the young it should alert suspicion and prompt or problems with your sight, other times it
thorough investigation. The strong candidate can increase the work of your heart possibly
will be prepared to think laterally in this regard, causing pain, breathlessness or palpitations,
and make use of a review of symptoms to guide and sometimes it can cause problems with
further lines of enquiry when thinking about the your kidneys and affect your urine. Have you
secondary causes of hypertension. experienced anything along those lines?
Narrow the differential it may be necessary to
On entering the room begin with screen the patients past medical and family history
at an early stage in the consultation; however, this
open questioning can be useful to the structure of the consultation
It is sensible to start with a general statement in How is the rest of your health and How is your
this case, perhaps reiterating the GPs concerns, familys health? Are you aware of any conditions
to get a feeling for the patients understanding and that run in the family? can form relatively open
appreciation of the reason for his attendance. questioning in themselves. Furthermore, the
I understand you have been referred to the answers to such questioning may better direct the
clinic by your GP because he is concerned about closed questioning that follows.
your blood pressure. Can you tell me more about Be alert for a family history of atherosclerosis,
that? dyslipidaemia and vascular disease. Enquire
Alternatively, adopt the How can I help about renal disorders such as polycystic
you today? approach and lead in to the GPs kidneys and also consider diagnoses such as
concerns having established the patients phaeochromocytoma.
understanding of why he has attended.
In such a station it can be difficult to avoid
the inevitable rush towards closed questioning,
Open questioning suggests a
but taking clues from the scenario eluding to diagnosis of hypertension
an anxious patient it is important to encourage
a dialogue and avoid dominating the situation. With this diagnosis, closed
It is possible that the emphasis of the station questioning should focus on
will centre around sensitive subjects such as a
family history of similar symptoms with death of the following
a relative, worrying features like headaches that A review of symptoms may be more effective
the patient has previously omitted to mention to than asking questions about each of the known
the GP, or areas such as drug use. Alternatively, causes of secondary hypertension, but it is
the nervous disposition may be intended as important to screen for symptoms related to the
a reference to white coat hypertension and following conditions:
a barrage of questions may serve to merely
exacerbate the patients nerves at the outset. Cardiovascular
Where it hasnt been detailed in the
information or GP referral, elicit the In older patients, and those with risk factors,
circumstances in which the blood pressure consider ischaemic heart disease. Chest pain,
readings were taken and first noted to be high. palpitations, reduced exercise tolerance with
Ensure the patient was calm, rested with no breathlessness, ankle swelling and lethargy.
Case 143 Hypertension 275

Orthopnoea and paroxysmal nocturnal hypertension in pregnancy is a topic in its


dyspnoea own right, but would prompt consideration of
Intermittent claudication, rest pain, and poor pre-eclampsia. If the scenario involves failure
circulation with cold extremities of initial management, be sure to investigate
Coarctation of the aorta may be worth compliance, particularly related to side effects,
considering, and while this will have a in addition to considering medication that may
paucity of symptoms, early claudication interact with common anti-hypertensives.
may be a feature or suggested by surgery for A comprehensive social history will clarify
congenital heart defects and quantify caffeine (including carbonated
energy drinks), alcohol intake and smoking pack
Renal years. Get a feel for how his diet has changed
as mentioned in the information, particularly
Lethargy, oedema, pruritis and nausea,
in relation to salt intake. Attempt to estimate
while indiscriminate, may suggest renal
the amount of exercise he is undertaking as a
disease and uraemia. Changes in urine
guide to the value of lifestyle intervention in
and urination, as mentioned above, may
considering further management options. Ask
suggest renal parenchymal disease such as
specifically about illicit drug use, particularly
glomerulonephritis, infiltrative disease, previous
ecstasy, cocaine and amphetamines. Screen for
recurrent pyelonephritis with scarring, or
source and levels of stress and anxiety.
obstructive uropathy. Renovascular disease is
common and may be idiopathic or relate to the
cardiovascular aetiologies discussed above. Explore concerns, ideas, and
expectations
Endocrine
Ask the patient if there is anything he thinks may
Phaeochromocytoma labile blood pressure be causing his high blood pressure? Are there
with intermittent episodes of headache, underlying concerns with a family history of
palpitations, flushing, sweating and nausea related conditions such as aneurysms or stroke?
Hyperthyroidism anxiety, palpitations,
sweating, tremor, diarrhoea, weight loss, and
potentially eye signs dependent upon the cause What to tell the patient
Cushing's disease or syndrome (hypertension Raised blood pressure can be a problem even
secondary to sodium retention) Cushingoid if you dont have any obvious symptoms, so
facies may be evident on general inspection, managing it is all targeted at lowering your
enquire as to steroid use, weight gain, striae, long term risk of complications. In particular,
easy bruising and proximal myopathy the effects of raised blood pressure on the
Conn's syndrome lethargy, generalised cardiovascular system, to reduce the risk of heart
weakness and muscle cramps secondary to attack or stroke, and the effects on small blood
hypokalaemia vessels in the eyes and kidneys. However, firstly
Acromegaly headaches, visual disturbance, we should confirm the diagnosis. I would like to
changes in appearance and notable changes repeat the measurement today and depending
in shoe size upon the result consider monitoring your blood
pressure over a 24-hour period using a portable
White-coat hypertension machine to confirm whether it is truly raised and
Relates to raised measurements in a health warrants treatment.
care setting compared to normal levels during Depending on the scenario, it may be
ambulatory measurement. This is a diagnosis necessary to discuss, in depth, the underlying
of exclusion however, and abnormal, erratic, causes and the need for further investigation.
readings should always prompt consideration While hypertension is a common finding with
of the possibility of phaeochromocytoma or age, in the young it should alert suspicion and
unacknowledged drug use. prompt thorough investigation. Explain the
Despite being told that the patient is on no difference between essential, secondary and
medication, enquire as to over-the-counter white-coat hypertension.
items such as cold remedies containing I would also like to arrange some simple
pseudoephedrine, which may inadvertently raise investigations which will include looking at
measurements. Where the history pertains to a the back of your eyes, tracing your heart with a
woman, remember that the oral contraceptive 12-lead electrocardiogram, and testing a sample
can cause hypertension and will be an important of your urine to look for the unwanted effects of
feature of subsequent management. Similarly, chronically raised blood pressure.
276 Chapter 6 History taking (station 2)

I would like you to persevere with your Stage 1 hypertension clinic readings greater
achievements thus far in changing your diet than 140/90 mmHg and average (ABPM
and stopping smoking. These are important or HBPM) greater than or equal to 135/85
general health considerations from which you mmHg
will benefit greatly. The most common reversible Stage 2 hypertension - clinic readings greater
causes of raised blood pressure are smoking and than 160/100 mmHg and average ABPM
alcohol, and with regard to the latter I would or HBPM greater than or equal to 150/95
like you to moderate your intake before our mmHg.
next review if possible. Your GP has asked me to
Management options will depend upon the
consider whether we need to start medication,
classification of hypertension as above and
and certainly I agree that this may be necessary
further risk stratification. All patients with stage
to avoid serious complications in the future
2 hypertension should be started on medication.
such as heart disease and stroke. But first it is
Offer antihypertensive treatment to those
important that we confirm the diagnosis, and
patients with stage 1 disease if aged less than 80
rule out any other treatable causes. I suggest we
years and one or more of the following:
organise the various tests we have discussed and
review the situation in a couple of weeks time. end-organ disease
Does that sound agreeable? established cardiovascular disease or 10 year
cardiovascular risk of >20%
What to tell the examiner renal disease
diabetes
Summarise the clinical case
Secondary hypertension may be more likely:
Report to the examiner the main findings of the younger patients (<40 years)
patients history. hypokalaemia and hypernatraemia
suggestive of adrenal disease
Investigations elevated serum creatinine and reduce eGFR
A diagnosis of hypertension is made using three suggestive of renovascular disease
raised blood pressure measurements, taken in a sudden onset, labile, or worsening blood
suitable setting with the arm at rest at the level of pressure
the heart. Avoidance of preceding exertion and
Specific tests to consider (in appropriate
stimulation with substances such as nicotine or
selected cases):
caffeine is important. Use an appropriately sized
cuff and, for accuracy, ensure manual blood Blood tests urea, creatinine and
pressure measurements in cases of an irregular electrolytes, plasma glucose and serum
pulse. lipids. Thyroid function tests. 9 am cortisol,
Certain clinic readings should prompt dexamethasone suppression testing, and
immediate action, including: plasma aldosterone:renin activity ratio where
appropriate
220/120 mmHg or higher, or 180/110 mmHg
ECG
with signs of papilloedema and/or retinal
haemorrhage treat as malignant hypertension
Imaging with renal USS including Doppler
assessment of renal artery prompting
with same day admission to hospital
magnetic resonance angiography if stenosis
180/110 mmHg start anti-hypertensive
suspected.
treatment immediately and monitor
response closely thereafter
Echocardiogram particularly if ECG suggests
hypertrophy
There is a trend towards the use of 24 hour 24-hour urine collection for catecholamines
ambulatory blood pressure monitoring (ABPM)
or home blood pressure monitoring (HBPM) to Management
improve the accuracy of diagnosis where clinic Lifestyle advice should be discussed diet,
readings have been persistently above 140/90 smoking, alcohol and exercise changes should
mmHg. When using clinic or home ambulatory be outlined to the patient. For patients without
blood pressure monitoring devices, ensure that indications for immediate drug therapy this
they are validated by the British Hypertension will include 6 months of implementation of
Society (list of validated devices available on lifestyle changes to observe their effect, before
their website). considering the need for pharmacological
Formal diagnosis is considered as stage 1 or intervention if blood pressure remains raised
stage 2 hypertension based on the interpretation and no secondary cause is identified.
of blood pressure results:
Case 144 Breathlessness and wheeze new diagnosis of asthma 277

Choice of medication is dependent upon Specialist referral should be made (to renal
age, as defined by NICE guidelines on use or endocrinology) where investigations suggest
of ACE inhibitor, diuretics, calcium channel secondary cause requiring further assessment
blockers and beta-blockers. The aim is to and/or management.
achieve systolic pressure <140 mmHg and
diastolic <90 mmHg. In diabetes, tighter control
is desirable. In those aged over 80 years with
Further reading
treated hypertension, a target of <150/90 has National Institute for Health and Care Excellence
been proposed. (NICE). CG127, Hypertension: clinical
management of primary hypertension in
adults.London; NICE, 2011.

Case 144: Breathlessness and wheeze


new diagnosis of asthma

Candidate information Pneumothorax: this typically presents with


sudden onset dyspnoea or pleuritic pain. In
Scenario asthma or COPD patients it may present as
You are the SHO in the respiratory clinic. A sudden deterioration in the patients clinical
22-year-old man has presented several times to condition
Accident and Emergency over the last year with Pulmonary oedema: symptoms and signs
acute episodes of breathlessness associated with include wheeze, anxiety, pink, frothy sputum,
cough and wheeze. He is in generally good health sweating, tachycardia, and peripheral
but has complained to his GP of aches and pains shutdown. It may be preceded by myocardial
in his hands, linked to his job in construction, for infarction, positive cardiac history or risk
which he takes over the counter painkillers. Please factors for cardiac disease
assess the patient and offer a management plan. Pulmonary embolism: typical features
include pleuritic chest pain, haemoptysis,
and respiratory distress. Risk factors include
Actor information a family history or previous thrombosis,
You have been having intermittent breathing immobility including long haul travel/
problems over the past year. Your doctor prolonged bed rest, surgery, stroke,
mentioned this might be partially related to the malignancy, thrombophilia, pregnancy or
medications you have been taking for the pains postpartum, and the oral contraceptive pill
in your hands. Aspiration: aspiration of vomit, state of
decreased consciousness, e.g. during
epileptic attack, recent stroke
Prior to entering the room Anxiety: hyperventilation caused by stress or
In any scenario dealing with shortness of breath, emotion
cough, or wheeze, be alert to the context of the Anaphylaxis: upper airway swelling
symptoms.These symptoms are common to a threatening patency, rash, haemodynamic
wide range of conditions. Where there is doubt, instability possible in the atopic patient
proceed by considering the speed of onset to
effectively narrow the differential and prompt Causes of acute onset breathlessness include:
further targeted lines of questioning. Attempt to Asthma: episodic breathlessness, triggered
classify according to sudden, acute, or chronic by allergens or exercise, associated wheeze,
onset: diurnal variation, demonstrating reversibility
Causes of sudden onset breathlessness with improvement on inhalation of b-agonist
include:
278 Chapter 6 History taking (station 2)

Infection: productive cough, pleurisy, cough that is often nocturnal, and chest
mucupurulent or green sputum, associated tightness with or without wheeze. Symptoms
fever, rigors, general malaise are usually episodic with diurnal variation, or
Lung tumours: associated cough, consistent with exposure to an environmental
haemoptysis, weight loss, general malaise or occupational trigger. Bearing these features
and anorexia, smoking history in mind, start by open questioning looking to
Pleural effusion: pleuritic chest pain identify common themes between recurring
Cor pulmonale: fatigue, syncope. Usually episodes.
history of chronic lung disease, pulmonary Gauge the severity of the symptoms and
vasculature disorders and neuromuscular resultant impairment:
disease
As a compensatory mechanism for metabolic
Frequency of symptoms
acidosis
Timing of symptoms
Impact upon normal activity
With regard to chronic breathlessness,
consider cardiac versus respiratory. Also, be
mindful as to other chronic conditions that
Open questioning suggests
may result in breathlessness secondary to a diagnosis of asthma
generalised fatigue or anaemia.
With this diagnosis closed
Respiratory causes of breathlessness questioning should focus on the
Pulmonary vasculature: following
Pulmonary hypertension: chest pain, fatigue, Continue closed questioning and seek to
syncope establish a clear profile of potential triggers:
infections, pets, dust, cold weather, damp air,
Interstitium: stress/emotion, and drugs. Clarify working
COPD: usually older patients, history of conditions and any resolution of symptoms
smoking, productive sputum, dyspnoea, when away from the work place. Establish the
wheeze, recurrent chest infections, chronic impact of exercise on symptoms and quantify
bronchitis or bronchiectasis. Acute exercise tolerance. Evaluate evidence of sleep
exacerbations may present with infective disturbance with nocturnal symptoms. Specific
symptoms or pulmonary oedema due to features that would be in favour of a clinical
resultant diastolic dysfunction diagnosis of asthma include:
Fibrosing alveolitis: dry cough, exertional History of atopy: commonly eczema or
dyspnoea, malaise, weight loss, arthralgia hayfever
Bronchiectasis: persistent productive cough, Family history of atopy commonly found in
purulent sputum close family members
Cystic fibrosis: recurrent infections on a History of childhood asthma predisposes to, or
background of poor growth represents recurrence of, disease in adulthood
Chest wall: Drug history: symptoms worse after aspirin,
NSAIDs or beta-blockers
Neuromuscular myasthenia gravis/motor
neuron disease With regard to occupational asthma, ask:
Does the patient have symptoms during the
On entering the room begin with week but not weekends or periods of holiday?
open questioning Does the patient experience irritation of the
eyes or nose in the work place?
Wheeze can be difficult to establish from Do colleagues complain of similar symptoms?
history alone and often the term is used by
patients in a non-specific manner. Cough Look to exclude other known causes of
may in fact be the only presenting feature in dyspnoea from the past medical history
asthma. Consider the diagnosis in younger and elicit risk factors for other aetiologies.
patients who appear otherwise well. When Importantly, exclude a cardiac history,
considering a new diagnosis, the key is to as evidenced by concomitant chest pain,
elicit the main features of asthma, and identify palpitations or dizziness. Be seen to consider the
triggers. The cardinal features to expect are: importance of an infective component and the
shortness of breath or difficulty breathing, contribution of smoking. A significant pack year
Case 144 Breathlessness and wheeze new diagnosis of asthma 279

smoking history should prompt consideration Timely clinic review


of a chronic obstructive pulmonary disease, Trial of inhalers reliever to counteract
although co-existing disease is possible. symptoms caused by constriction. Consider
Thereafter, employ a review of symptoms as discussing the principals of inhaler technique
a means of evaluating recognised conditions Ensuring patient safety in the interim period
associated with asthma including acid reflux, paramount to educate as to red flags
polyarteritis nodosa (PAN), ChurgStrauss, symptoms that would necessitate urgent
and allergic bronchopulmonary aspergillosis medical review
(ABPA). It will not necessarily be important
Specific to this case, the patient will have
in this scenario to pursue such themes but
told you that he has been taking NSAIDs for
the strong candidate will at the very least
joint pains in the hands. Tell the patient you will
demonstrate an awareness of these conditions.
change him onto a different class of painkillers,
Acid reflux: retrosternal burning pain, worse aiming to eliminate the respiratory side effects
after meal and on lying flat, metallic taste in while maintaining effective analgesia. Tell him
the mouth that, if his hand symptoms become particularly
PAN: fevers, abdominal pain, general troublesome you will arrange for him to be seen
malaise and weight loss, arthralgia. Suspect by a rheumatologist.
in multisystem symptoms including renal,
vascular, cardiac, CNS, GI and skin
ChurgStrauss: asthma, eosinophilia and
What to tell the examiner
systemic vasculitis Summarise the clinical case
ABPA: wheeze, cough, sputum, dyspnoea, This patient describes symptoms consistent
recurrent pneumonia with occupational asthma. It is possible that
the use of analgaesia has contributed to
Explore concerns, ideas, and the episodes he describes, and it would be
important to distinguish the relative importance
expectations of this and the likely environmental allergen in
Themes to prepare for include: the work place.
Patients that raise concerns are their ability to
continue to work in current employment in Investigations
the face of occupational asthma balancing The likelihood or certainty of a formal diagnosis
the need to earn an income and support a should influence the initial course of action
family against personal health for instance gauged on the symptoms described and/
Consider difficulties with smoking cessation or available test results such as PEFR when
Ive tried to quit smoking before, I just cant symptomatic or formal spirometry.
do it If high probability: Proceed to treatment
Preconceptions surrounding the implications trial and continue to maintenance treatment if
of the diagnosis, both relating to severity and response satisfactory. If response is poor, check
limitations of daily life inhaler technique and compliance. Consider a
Stigma associated with carrying or using an course of steroids if the patient is symptomatic
inhaler in younger patients, particularly of at the time of clinic review.
school age If low probability: Explore symptoms further
and consider investigations required to establish
alternative diagnosis. Always be prepared to
What to tell the patient re-consider the diagnosis.
Communicate a working diagnosis of asthma. Intermediate probability: Further
Explain the main features in lay terms, focusing investigations and treatment trial before
on the concept of triggers causing hypersensitive confirmation of diagnosis. Objective evidence
airways to constrict or tighten with resultant provided by spirometry: if FEV1/FVC <0.7
wheeze and difficulty in breathing. Achieving consistent with an obstructive picture consider
a shared understanding will be beneficial in trial of treatment, particularly where reversibility
implementing management centred upon: can be demonstrated. If FEV1/FVC >0.7
PEFR chart to attempt to confirm the investigate further.
diagnosis Low FEV1 or PEFR: A normal spirometry
Identifying triggers and attempting to reading in asymptomatic patients is not
minimise contact/exposure exclusive of the diagnosis.
280 Chapter 6 History taking (station 2)

Management Step 5 Daily steroid tablet: Maintain high


dose inhaled steroid up to 2000 g/day.
Management considerations likely to be
Specialist input
discussed will include the avoidance of known
triggers, identifying others, encouraging Arrange regular review especially as
smoking cessation and weight reduction treatment is stepped down. Consider severity,
in obese patients with evidence of GORD. side effects, duration on current dose, patient
Thereafter, medical treatment will be as preferences in changing treatment. Maintain on
outlined by the British Thoracic Society lowest possible dose of inhaled steroid.
guidelines: Complete control as assessed by:
Step 1 Inhaled short-acting b2-agonist No daytime or nocturnal symptoms
Step 2 Add inhaled steroid 200800 g/day No rescue medication
Step 3 Inhaled long-acting b-agonist. If No exacerbations
inadequate response increase inhaled No limitations on activity or exercise
steroid to 800 g/day. Budesonide/ Normal lung function (FEV1/PEF >80% of
formoterol in a single inhaler as rescue predicted)
medication instead of a short-acting
b-agonist is an option
Step 4 Increase inhaled steroid up to Further reading
2000 g/day or add leukotriene receptor British Thoracic Society (BTS) and Scottish
antagonist, SR theophylline, b2-agonist Intercollegiate Guidelines Network (SIGN).
tablet British guideline on the management of asthma.
London and Edinburgh; BTS and SIGN, 2012.

Case 145: Shortness of breath and wheeze


established diagnosis of asthma

Candidate information Prior to entering the room


Scenario Be clear as to the accepted incremental and
stepwise increase in management according to
As the respiratory SHO in clinic you are asked BTS guidelines. It may be worth scribbling these
to see a new referral. A 19-year-old referred down to avoid confusion. Poor inhaler technique
by his GP due to poor asthma control in the should be considered before assuming the need
community. Please take a history from the for a change in medication. Also be seen to
patient, offer an explanation and propose a consider and rule out poor compliance.
management plan, before discussing the case
with the examiner.
On entering the room begin with
Actor information open questioning
You have had asthma since childhood. You have Use open questions to find out how long they
found you are increasingly dependent upon have had asthma for, who made the diagnosis,
your inhaler with minimal relief and significant their exact symptoms. Was their asthma ever
impact upon your daily living. More recently under complete control? Has anything changed
you have required treatment via Accident and recently? Does the patient have any idea why their
Emergency for severe attacks. asthma may not be under good control now?
Case 145 Shortness of breath and wheeze established diagnosis of asthma 281

Aim to establish the following: What to tell the patient


Degree of control by asking about daytime or From what you are telling me today I dont think
nocturnal symptoms your asthma is under best possible control. If
Use and frequency of rescue medication you could make one thing better for your asthma
Number of exacerbations (including hospital what would that be?
admissions and use of antibiotics) Where there is suspected concomitant
Limitations on exercise or other activity and pathology, or an alternative diagnosis, propose
context of exacerbations to organise further tests to ensure there
is nothing underlying the exacerbation of
Are there symptoms that make an alternative
symptoms.
diagnosis more likely? Sudden exacerbation
I think it is important we revisit the
of symptoms may indicate underlying
smoking situation as I really think stopping will
pneumothorax, while the presence of
significantly help your symptoms and improve
constitutional symptoms may signify underlying
the quality of your life. There are ways in which
ChurgStrauss especially in the context of
we can help out achieve this if you have decided
eosonophilia.
to try.
Review peak flow diary (if the patient keeps
In the first instance, I suggest you keep a
one). Completely controlled asthma should have
peak flow diary. You would be writing down
FEV1/PEF ratio >80% of predicted or best effort.
your peak flow scores regularly; this will help
Establish compliance and inhaler technique
you see when your asthma is improving or not.
as well as exact current asthma regime and
This way you can tell yourself what is happening
whether they have been on a stronger regime.
in your lungs, record how well you have been
and how effective are the medicines you are
Open questioning suggests a taking. It can be a major tool for helping you
controlling your asthma yourself and prevent
diagnosis of poorly controlled major exacerbations. Finally if you would like to,
asthma I can arrange for you to see the asthma specialist
nurse to review your inhaler technique and give
With this diagnosis closed you some helpful tips.
questioning should focus on Offer a personalised self-management plan
with simple verbal and written instructions
the following and advice on what to do in an emergency.
Review the patients past medical history, Provide asthma medicine card. Finish by saying:
medications, allergies and social setup, I suggest we also implement some changes
especially smoking and occupation. Take a in your medication today and review your
careful drug history to ensure they are not taking symptoms in 3 months time.
any medication that worsens asthma, such as
NSAIDs, beta-blockers or aspirin. Check for What to tell the examiner
side effects of medications such as tachycardia,
tremor or anxiety from b-agonists. Summarise the clinical case
Occupational history: This young man with a long history of asthma
Establish if symptoms are worse while at since childhood is no longer well controlled on a
work and improve when not working such as combination of inhaled steroids and long-acting
during the weekend b2-agonist. I suspect his deterioration might be a
Establish a job description and contact with combination of poor adherence to treatment and
materials poor inhaler technique. There are no symptoms
Concomitant rhinoconjunctivitis is often seen suggestive of other underlying disease.
High-risk occupations include food processing
or baking, sprays, animal work/farming, Investigations
healthcare, metal or wood work, fumes If an alternative diagnosis is possible, investigate
accordingly.
Explore concerns, ideas, and Peak expiratory flow measurement or
expectations spirometry
Prick testing may identify potential
How is the condition affecting the patients daily exacerbating allergens if the history is
life? suggestive
282 Chapter 6 History taking (station 2)

Management severity of asthma, side effects of treatment,


time on current dose and if beneficial effect is
Patient education regarding asthma and control,
achieved. Aim to maintain on lower possible
importance of keeping a PEF diary and how to
dose of inhaled steroid. Aim to review every
alter their medication in response to changes
3 months with an aim to decrease the doses of
in symptoms or PEF. General lifestyle advice
steroids by 2550% each time. If response is not
including smoking cessation, weight loss, and
as expected consider alternative diagnosis.
avoidance of known allergens. Check inhaler
Additional themes worth considering:
technique and consider alternative delivery
device if necessary. Asthma in pregnancy. Educate about
Medication changes: importance of good asthma control during
pregnancy. Long acting b2-agonist, inhaled
Use the step-approach (under new asthma
steroids and oral theophylline are safe in
diagnosis)
pregnancy. Leukotriene antagonists may be
For exercise-exacerbated asthma, if patient continued in women who have significantly
is otherwise controlled on inhaled steroids the benefited prior to pregnancy. Oral steroids
following can be added: should be reserved for severe asthma.
Treatment of acute asthma should be the
Leukotriene receptor antagonist same as for the non-pregnant women
Long-acting b-agonist
Oral b-agonist
Theophylline Further reading
Inhaled short-acting b-agonist immediately British Thoracic Society (BTS) and Scottish
before exercise Intercollegiate Guidelines Network (SIGN).
Arrange regular review to step down British guideline on the management of asthma.
treatment when and if appropriate. Consider London and Edinburgh; BTS and SIGN, 2012.

Case 146: Wheeze established chronic


obstructive pulmonary disease

Candidate information Prior to entering the room


Scenario Given that this man has a known diagnosis
of COPD, it will be important to quantify
A 72-year-old man, who was diagnosed with his exercise tolerance and frequency of
COPD 4 years ago by his GP, has been having exacerbations. The candidate will need to
frequent exacerbations. As the respiratory SHO establish disease severity and consider adjusting
in the outpatient clinic, please take a history the management plan accordingly.
from the patient, offer him your impression of
the case, and discuss your findings with the
examiner. On entering the room begin with
open questioning
Actor information Begin with open questions, which will give a
You saw your GP recently because you are chance to the patient to address their main
concerned that your breathing problems are symptoms and concerns regarding their COPD.
getting worse. You seem to be getting more Enquire about the disease course and
frequent chest infections. Additionally, you have duration. Is it predominantly chronic bronchitis
noticed you cannot walk as far as you used to be or emphysema? Chronic bronchitis is defined
able to before getting breathless. clinically as productive cough on most days for
Case 146 Wheeze established chronic obstructive pulmonary disease 283

3 months of 2 successive years. Emphysema is a Alpha-1 antitrypsin deficiency: suspect


histological diagnosis with enlarged air spaces in patients young patients <40 with
distal to terminal bronchioles and destruction of emphysema. Consider in patients with
alveolar walls. chronic liver disease, gallstones and
Find about their current symptoms including dyspnoea
cough, regular sputum production, wheeze and
Continue closed questioning find out if
limitations on activity and daily living.
there is past medical history of cardiac disease,
Ask about symptoms of weight loss, effort
establish current COPD regime, check on
tolerance, waking at night, ankle swelling,
vaccination history especially flu jab and
fatigue, chest pain and haemoptysis.
pneumococcus.
Assess the degree of breathlessness using the
Medical Research Council dyspnoea scale as
follows: Explore concerns, ideas, and
Grade 1 Troubled only on strenuous exercise expectations
Grade 2 Dyspnoea on exertion or walking Take a careful social history focusing on smoking
uphill and current psychosocial issues as these can be
Grade 3 Walks slowly on level ground or has prevalent in COPD patients.
to stop when walking at own pace Discuss functional aspects including strength
Grade 4 Stops for breath after 100 m or after a and exercise tolerance. Screen for commonly
few minutes on level ground encountered situations such as social isolation
Grade 5 Too breathless to leave house or and increased periods of sedentary activity, often
when dressing/undressing stemming from a fear of breathlessness resulting
in avoidance of any strenuous activity. Disease
Open questioning suggests a progression can be correlated to muscle wasting,
particularly of the proximal lower limbs and this
diagnosis of COPD is reflected in decreased strength or inability to
stand from a seated position unaided.
With this diagnosis closed
questioning should focus on What to tell the patient
the following It seems you are experiencing regular
Deterioration of symptoms in COPD can be due exacerbations which means your COPD is not
to acute and/or chronic causes. very well controlled at the moment. What do you
Acute causes include: think might be causing this?
I would propose we run some tests to check
Non-infective exacerbation: rapid and
your lung function and change your inhaler
sustained worsening of symptoms beyond
treatment accordingly.
day-to-day variations without infective
We also need to teach you how to manage
symptoms or signs
exacerbations early to avoid having a severe
Infective exacerbation: the commonest
exacerbation. I can give you a course of
presentation, fever, rigors, green sputum
antibiotic and steroid tablets to keep at home.
Pneumothorax: from ruptures bullae, will
I would like you to start the oral steroids if
present as sudden deterioration, cough,
you experience increasing breathlessness
sudden onset dyspnoea or pleuritic chest pain.
and it is interfering with your daily activities,
More common in patients with bullous disease
and starting the antibiotic when your sputum
Cor pulmonale: dyspnoea, fatigue and
becomes purulent. However, you should
syncope usually in patients with long-
contact your doctor if your symptoms fail to
standing COPD
improve.
Chronic causes include: Take into account patients needs and
preferences in achieving an informed
Cor pulmonale decision about treatment. Smoking cessation
Non-compliance with treatment is important but not always possible. Often,
Natural disease course involving pulmonary rehabilitation is the most
Also consider: effective way of achieving patient education,
improving compliance and encouraging
Bronchiectasis: frequent infections, smoking cessation in a motivated environment
persistent cough and copious purulent with like-minded individuals. Pulmonary
sputum with occasional haemoptysis rehabilitation also enables targeted strength
284 Chapter 6 History taking (station 2)

training to improve outcomes and developing consider a long-acting b-agonist (LABA) on


the ability to cope with symptoms including a regular basis or a long-acting muscarinic
breathlessness. antagonist (LAMA) on a PRN basis.
For patients with stable COPD who remain
breathless or have frequent exacerbations and
What to tell the examiner persistent breathlessness:
Summarise the clinical case
This patient with long-standing COPD who If FEV1 is <50% of predicted
is still a heavy smoker is still breathless with LABA and inhaled corticosteroid in
frequent exacerbations despite taking a combination inhaler or LABA and LAMA
long-acting b-agonist. This has resulted in if corticosteroid contraindicated or not
frequent hospitalisations. There are no symptoms tolerated or
in the history suggestive of progression to cor LAMA QDS (but stop LABA)
pulmonale or a lung malignancy but I would like If FEV1 is >50% of predicted
to examine the patient in full before proceeding LAMA QDS without LABA
to stepping up his treatment and offering some
other management advice. Persistent exacerbation and/or
For suspected cor pulmonale: Examine breathlessness in patients taking LABA and
patient for tachycardia, raised JVP, right inhaled steroid irrespective of FEV1:
ventricular heave, loud P2, pansystolic murmur, LAMA, LABA and inhaled corticosteroid
hepatomegaly and oedema.
Teach inhaler technique and/or provide
Investigations spacer. Nebulisers are an option for patients
breathlessness despite maximal therapy with
Assess the severity of COPD with inhalers.
post-bronchodilator spirometry in line with Oral corticosteroids are not routine for
NICE clinical guidelines chronic management. They are usually effective
Mild : FEV1 >80% of predicted for exacerbations as short courses of treatment.
Moderate : FEV1 5079% Some patients with advanced COPD may need
Severe : FEV1 3049% predicted maintenance with oral steroids if they cannot be
Very severe : FEV1 <30% successfully weaned off after an exacerbation.
Theophylline can be used in combination
Management with b-agonists and muscarinic antagonists after
trials of short acting and long acting treatment
General measures include: has failed. Note that it interacts with macrolides
Smoking cessation advice: offer nicotine and fluoroquinilones and you need to warn the
replacement therapy together with patient if they develop an infective exacerbation
a behavioural support programme. and require treatment with antibiotics.
Medications include varenicline for patients The aim of treatment is to minimise
who have expressed a desire to quit smoking the impact of exacerbations by giving self
Dietary advice and weight loss management advice on responding to
Mucolytics symptoms of exacerbation, starting appropriate
Offer vaccination to flu and pneumococcus treatment, using NIV, hospital at home, etc.
Offer pulmonary rehabilitation +/ On review check for improvement in:
theophylline: pulmonary rehabilitation symptoms, daily living, capacity, speed of
is suitable in patients including symptom relief
recent hospitalisation, functionally
disabled (MRC grade 3 and above) but Other management
contraindicated in unstable cardiac disease
Multidisciplinary team involvement considerations
including physiotherapy for chest exercise, Long-term oxygen therapy should be considered
dietetic advice to reduce BMI, occupational in patients with severe COPD (FEV1 3049%
therapy for help with activities of daily living, predicted). They need to fulfill the following
social services, palliative care team for end- criteria with measurement of arterial blood gas
stage COPD on 2 occasions 3 weeks apart when well:
With regard drug treatments, if the patient Pao2 <7.3 kPa for stable disease
is breathless and/or has exercise limitation, Pao2 >7.3 <8 kPa when stable and
Case 147 Haemoptysis in the returning traveller 285

Secondary polycythaemia Patients with hypercapnia or acidosis on


Peripheral oedema LTOT may require long term NIV.
Nocturnal hypoxaemia Surgical options do exist for appropriately
Pulmonary hypertension selected patients such as lung volume reduction
surgery or rarely lung transplantation.
Ambulatory oxygen therapy is a suitable option
Candidates should however, ensure they have a
for patients on LTOT who want to use oxygen
firm grasp on the medical management of COPD
outside the home and have exercise desaturation
before beginning discussions on the much less
and Pao2 <7.3 and improve on oxygen.
commonly used surgical options.

Case 147: Haemoptysis in the


returning traveller

Candidate information Take a careful travel history including all


areas visited and any stop-overs
Scenario Find out if they were travelling alone and
You are the medical registrar on call. You have what accommodation was used
been asked to see a 30-year-old man, who has The use (or otherwise) of preventative
recently returned from a prolonged stay in rural measures such as pre-travel immunisations
sub-Saharan Africa, has presented with frank and malaria prophylaxis should help you
haemoptysis. Please take a history from the narrow your differential
patient, offer a management plan based on a Symptoms of fever, lassitude, night sweats,
working diagnosis, and discuss your findings weight loss, anorexia, cough, sputum,
with the examiner. haemoptysis are highly suggestive of TB

Actor information Open questioning suggests a


You have recently returned from a year-long diagnosis of tuberculosis
trip to Zambia, where you were working for a
healthcare NGO. You have been troubled with a With this diagnosis closed
persistent cough for some time now. Recently, questioning should focus on
you have noticed that you have started to cough
up what looks to you like blood. the following
Explore other important causes of haemoptysis,
such as:
Prior to entering the room
A travel history with relevant destinations can be Pneumonia: fever, purulent sputum,
useful if considering a differential and tailoring productive cough, headaches, myalgia
the approach to history taking. A sensible Bronchial cancer: more chronic presentation,
approach would be to consider diseases endemic weight loss, cough, sputum with flecks of blood,
to the area visited, however, avoid jumping to too general malaise, anorexia, smoking history
many assumptions and risk overlooking basic Pulmonary oedema: dyspnoea, fatigue,
information in your quest for an exotic diagnosis. orthopnoea, PND, chest pain, nocturnal
cough, pink frothy sputum, previous cardiac
history or risk factors
On entering the room begin with Pulmonary embolism: suspect in recent
open questioning long-haul travel, usually sudden onset,
pleuritic chest pain dizziness or syncope and
Let the patient talk to you about their risk factors for PE (see other sections)
symptoms
286 Chapter 6 History taking (station 2)

Continue your closed questioning eliciting before waiting for the results of the tests. The
key features in the rest of the history. In the treatment duration is long, usually 6 months and
past medical history, ask about a history of it is very important you comply with treatment
previous TB, check for immunocompromise as it can cause serious consequences. Also you
such as in HIV, malignancy, diabetes. Explore should not stop the treatment before it is fully
risk factors for HIV. Take a careful drug history, complete even if you are feeling well, as you
including intravenous drug use or others. may still be infectious. I will provide further
Other medications such as phenytoin, warfarin information about the side effects but it is
and oestrogen containing pills that may important you have blood tests to check your
interact with treatment. Smoking and alcohol liver tests every month. It is also important to
history. prevent it from spreading to other people so we
A careful systems review to identify other are obliged to screen any close contacts you may
organ involvement: have had and offer them treatment too before
they become ill. I will bring you in touch with a
Neurological (meningeal TB): fever, named nurse specialist whom you can contact if
headache, nausea, vomiting, neck stiffness there are any problems at any stage.
and photophobia
Genitourinary TB: frequency, dysuria,
haematuria, loin pain What to tell the examiner
Bone pain: bony lower back pain
Summarise the clinical case
Skin: painful lower leg rash (erythema
nodosum) This young man with fever, haemoptysis and
night sweats who has returned from an endemic
area and has not previously been vaccinated
Explore concerns, ideas, and with BCG is at high risk of respiratory TB. He
expectations does not give a history suggestive of other organ
It is likely that the patient returning from areas involvement. Although his risk factors for HIV
with high incidences of tuberculosis and other are low I would ensure he has a test for HIV. I
diseases will have considered the possibility of would proceed to some investigations and start
such conditions. Ask the patient what concerns management immediately. I would also inform
they have about their health. the communicable diseases centre as TB is a
notifiable disease.

What to tell the patient Investigations


From the symptoms you are describing, my To achieve a diagnosis of active pulmonary TB:
main worry would be you might have caught TB
whilst you were abroad. Have you heard of TB or Chest X-ray
tuberculosis before? At least 3 sputum samples for TB microscopy
TB is a bacterial infection that anyone can and culture and prolonged incubation
catch and it is usually transmitted when infected on LwensteinJensen medium, before
people cough or sneeze. Most people become ill starting treatment or within 7 days of starting
within a few weeks or months of breathing the treatment
bacteria and have active TB. Bronchoscopy and bronchoalveolar lavage if
Some people never become ill because their sputum unavailable
immune system gets rid of the bacteria. In other In the presence of pleural effusion consider
people the bacteria hide in the body at low level pelural tap (low glucose, ph <7.3, high LDH
and the person does not get ill immediately but and lymphocytosis)
carries the infection and can become ill at a later Additional tests include:
stage. This is called latent TB.
Active TB usually affects the lungs but Pre treatment test colour vision as
can affect other body parts. Although you do ethambutol may cause reversible ocular
not describe any other symptoms it is worth toxicity
checking. To confirm the diagnosis you need to All patients with TB should have an HIV test
have some tests today, including a chest X-ray as well as routine blood tests
and sputum cultures. It is also important to check
for certain viruses, such as the HIV virus, which Management
sometimes pre-dispose or co-exist with TB. Remember communicable disease control
There is a cure for TB in the form of notification and contact tracing and screening.
antibiotic treatment, which we should start
Case 148 Persistent/change in cough investigation of cancer 287

The standard recommended regimen for material or cultures. If these remain positive
respiratory TB is: after 4 months of treatment, drug resistance
should be suspected.
First 2 months: Rifampicin, isoniazid, There is no routine follow-up after treatment
pyrazinamide and ethambutol completion. Follow-up for MDR TB for
Next 4 months: Rifampicin, isoniazid 12 months after completion.
Be aware of deviations from the standard
management in special circumstances such as
Screening for TB
meningeal TB and three times a week directly
observed therapy in patients at high risk for Diagnosing latent TB Screen all household
non-adherence such as street or shelter dwelling contacts and close non-household contacts with
homeless people, history of poor adherence or Mantoux testing and or interferon gamma test and
drug-resistant TB. chest X-ray. Mantoux negative results in household
Avoid admission to hospital unless clinical or contacts who are previously unvaccinated should
socio-economic need. If admission is necessary have interferon gamma testing 6 weeks after
provide a negative pressure room or vented Mantoux. Offer BCG vaccination to contacts
to the outside of the building. Patients can be with negative Mantoux tests, no previous BCG
discharged after 2 weeks of treatment if well and vaccination and age <35 years.
dont have MDR TB. Suspect latent in TB in patients with normal X-ray
Assign a key worker to facilitate education, or with Mantoux results who are:
involvement and adherence. Measures may <35 years old
include health education counselling, home HIV positive
visits, patient diary, pill counts and random Health workers
urine tests, social security benefits, housing and Mantoux positive >6 mm without previous
social services. BCG
Consider factors that increase risk for drug Mantoux positive >15 mm, interferon gamma
resistance, such as: positive and previous BCG
Treat for suspected latent TB in HIV positive
Previous TB
patients who have come to close contact with
Previous TB treatment failure
smear positive TB patients. Latent TB treatment:
Contact with known case of drug-resistant TB
6 months isoniazid or 3 months isoniazid with
Birth in a foreign county with high incidence
rifampicin.
of TB
HIV
Resident in London Further reading
Age 2544 years National Institute for Health and Care Excellence
Male (NICE) CG 117, Tuberculosis: Clinical diagnosis and
If resistance is suspected perform rifampicin management of tuberculosis, and measures for
resistance molecular tests on smear positive its prevention and control. London; NICE, 2011.

Case 148: Persistent cough or change in


cough investigation of cancer

Candidate information Emergency is concerned because she has seen


the patient cough up some blood stained mucus.
Scenario Additionally, she reports that the patient looks
You are the medical registrar on call. A 78-year- very emaciated. Please take a history from the
old man has been referred by his GP for a patient, tell them what you think is wrong and
persistent cough. The nurse in Accident and discuss your findings with the examiner.
288 Chapter 6 History taking (station 2)

Actor information TB-frank haemoptysis, cachexia, general


malaise, weight loss, fever, cough, night sweats
You are a life-long smoker. You have been to see
your GP a number of times for a cough. On the
COPD-mucoid/purulent sputum
last occasion, you told the GP that you have lost
Pulmonary oedema-pink/frothy sputum,
breathlessness (worse in recumbent
two stone in weight over the past 3 months.
position), leg oedema
Causes of a non-productive cough include:
Prior to entering the room
Be careful before making assumptions with Asthma-episodic, often seasonal wheezing
this scenario. Use it to form a preliminary and shortness of breath (see asthma section),
differential but do not rush to exclude other nocturnal deterioration
causes. The scenario might be presented in such GORD-retrosternal burning pain,
a way favouring one diagnosis but this may be precipitated by posture change, worse after
misleading; an older, emaciated man coughing heavy meals
and put in isolation because of a presumed Iatrogenic, e.g. ACE inhibitors
diagnosis of TB may actually be someone
suffering with lung cancer. Open questioning suggests a
The differential of persistent cough can
be wide. In the absence of significant co- diagnosis of lung carcinoma
morbidity, an acute cough is normally benign
and self-limiting. Apart from respiratory tract With this diagnosis closed
infection and inhalation of foreign body that questioning should focus on
are likely to be present with acute cough,
most other causes are probably secondary
the following
to chronic causes. Consider productive and Continue closed questioning on past medical
non-productive causes. Significant sputum and social history. Be sure to take a thorough
production indicates primary lung pathology. smoking history early in the encounter but avoid
Haemoptysis and weight loss will serve as a punitive tone or judgmental approach.
differentiating factors. When cough is present Heavy smokers may have a previous diagnosis
with haemoptysis be sure to consider lung of COPD or chronic bronchitis. Ask about other
cancer, pulmonary embolus or TB. previous respiratory, cardiac disease (also
predisposes to cough) or history of lung cancer.
Asbestos exposure significantly increases risk of
On entering the room begin with neoplastic disease, thus screen for occupational
open questioning history.
Consider extrapulmonary manifestations of
Attempt to explore the differential of cough,
bronchial carcinoma. Metastatic manifestations
focusing on onset and duration, frequency, and
may include:
quantity and appearance of sputum.
Screen for the characteristics of the cough. Pancoast tumour may cause pain in the
Is there diurnal variation and does it wake them shoulder and inner arm and Horners
up at night? Are there any triggers or aggravants? Horners: unilateral papillary constriction,
Where productive enquire as to the amount and slight ptosis and loss of sweating when
character of sputum (serous, mucoid, purulent). involving T1 root compression
Has there been haemoptysis? Is there associated Left recurrent laryngeal involvement may
chest pain and breathlessness? present with voice hoarseness and bovine cough
Screen for systemic health. Has there been Bone pain and neurological symptoms such
weight loss, general malaise and lethargy? as seizures or focal neurology may point to
metastatic disease
Narrow your differential Primaries in other sites that may spread
Causes of a productive cough include: to the lung renal, prostate, breast, bone,
gastrointestinal tract, cervical and ovarian
Lung cancer-weight loss and haemoptysis
(blood stained), breathlessness, chest pain Non-metastatic manifestations may be
from bony metastasis, long smoking history endocrine, neurological, vascular, skeletal and/
Pulmonary embolism-sudden onset or cutaneous in nature:
breathlessness, pleuritic chest pain, recent Ectopic secretion of ACTH, ADH, PTH-like
long-haul travel, leg swelling or DVT peptide
Case 148 Persistent/change in cough investigation of cancer 289

Myopathy, neuropathies, myasthenic on a background of symptoms suggestive of


syndrome, cerebellar degeneration chronic bronchitis. He is otherwise fit and
Thrombophlebitis migrans, anaemia healthy and has a good performance score. I
Hypertrophic pulmonary osteoarthropathy would first like to exclude an underlying lung
(clubbing, painful wrisits and ankles) malignancy.
Dermatomyositis, acanthosis nigricans
Investigations
Explore concerns, ideas, and Examination may give clues as to the
presence of significant masses or clinical signs
expectations related to associated pathology, commonly
Summarising the case may focus attention on unilateral pleural effusions, or organomegaly/
the likely diagnosis depending on the emphasis lymphadenopathy consistent with metastatic
placed on smoking, or other risk factors, and a disease.
potential link with current symptoms. This may Imaging with chest X-ray looking for a hilar
serve as a warning shot, preparing the patient or peripheral mass, collapse or consolidation of
for the discussion that will follow. the lung or pleural effusion (note that tumours
So you have been smoking for most your need to be at least 12 cm to be visible on a plain
life and have always had the occasional cough radiograph).
with sputum but recently you have developed An abnormal or normal X-ray alone would
a persistent cough for the last 8 weeks and it not establish/refute the diagnosis. I would
is gradually getting worse. You are concerned proceed to a staging CT scan to assess spread of
because recently you have started to cough up the tumour, including liver and adrenal glands.
blood stained sputum and your wife thinks you Abnormal CXR or CT should prompt urgent
have lost a lot of weight. referral to chest MDT. Recent NICE guidelines
What do you think may be causing your recommend offering PET-CT, endobronchial
symptoms? ultrasound-guided TBNA or endoscopic
Have you considered the possibility of cancer? ultrasound-guided FNA early in patients
with intermediate probability of mediastinal
What to tell the patient malignancy on CT.
Tissue diagnosis should be made where
I think it is important to run some tests to look possible. For peripheral tumours CT or US-
closely at your lungs. A simple chest X-ray can be guided needle biopsy. For central tumours,
very informative but if it were normal it would fibreoptic bronchoscopy enables sampling
be sensible to obtain a more detailed picture of nodes. Consider bronchoscopy to obtain
with a CT. This will look at your lungs in thin biopsy or washings. Alternatively CT-guided fine
slices and we will be able pick up a wide range of needle aspiration or core biopsy from peripheral
information which will help us better understand lesions if primary not accessible. Diagnostic
the cause of your symptoms. It will be important pleural tap for cytology for malignant cells and
to consider cancer as a potential diagnosis, and a biochemistry for exudates.
CT will be useful in establishing this. Depending on the history and examination
Depending on our findings we can discuss findings you might want to investigate for
treatment options. I do not want to rush ahead paraneoplastic syndromes such as ACTH,
and talk to you about this without knowing Cushings, PTHr-peptide.
with certainty what the underlying problem is
but I will make sure we have plenty of time at
your next appointment shortly after the scan to
Management
address all your questions. Management depends on staging and patient
I can organise the chest X-ray today if its ok suitability for major operation. Treatment and
with you and arrange for the CT scan in the next care should take into account patients needs
couple of days. Do you have any questions? and preferences. Clinical nurse specialist should
be involved in the early stages. The following
management points are based on NICE
What to tell the examiner guidelines (CG 121).
Summarise the clinical case Non-small cell lung cancer (NSCLC):
This 70-year old man with a 45-pack per year If medically fit may be a candidate
smoking history has presented with an 8-week for surgery, such as lobectomy or
history of cough, haemoptysis and weight loss segmentectomy if complete resection can be
290 Chapter 6 History taking (station 2)

achieved. Radical radiotherapy for patients Offer counselling and support groups for
with stage I, II or III NSCLC. All patients may both patient and family or carers. Consider
be suitable for multimodal treatment with general nursing issues. Home care or even
surgery, radiotherapy and chemotherapy. placement for patients with high needs.
Involve a thoracic oncologist and thoracic Review with results to discuss management
surgeon early in the decision-making process plan and address questions
and MDT. In general offer accurate and easy to
understand information to patients and
Small-cell lung cancer (SCLC):
cares. Explain tests and treatment options
For limited disease (T1-4, N0-3, M0) offer including survival benefits
cisplatin based chemotherapy or carbopoatin
Prognosis:
in the presence of abnormal renal function.
Consider concurrent radiotherapy if disease Depends on tumour type and staging
site can be encompassed in radiotherapy Bronchial carcinoma is the commonest
field and prophylactic cranial radiotherapy cancer in the west and leading cause of
Surgery might be an option for T1-2, N0, M0 cancer death in women. 90% are caused by
disease smoking. Only 5.5% are ever cured. It can be
For extensive SCLC (T1-4, N0-3, M1) offer divided into small cell and non-small cell.
platinum-based chemotherapy. Thoracic The latter is subclassified into squamous
radiotherapy after completion if response (40%), large cell (25%), adenocarcinoma
achieved (10%) and alveolar cell (12%)
Palliative treatment:
Adenocarcinoma is common in non-
smokers and is thought to be related to
Palliative radiotherapy may help for asbestos exposure. It occurs peripherally
symptoms control. Consider interventional and may spread locally or distant. Squamous
endobronchial treatment for obstruction cell usually spreads locally and metastasizes
(endobronchial or SVC). Other palliative late. Small cell (2030%) arises from
treatment would included malignant endocrine cells secreting polypeptide
pleural effusion drainage, interventions for hormones. Paraneoplastic syndromes are
breathlessness, opioids, dexamethasone for common (ACTH and Cushings). Small cell
symptomatic brain metastases tumours metastasize early but respond to
Offer multi-disciplinary support for chemotherapy
management of weight loss, loss of appetite,
difficulty swallowing, fatigue and depression.
Smoking cessation and nicotine replacement
Further reading
therapy is also important at any stage of National Institute for Health and Care Excellence
treatment (NICE). CG121, The diagnosis and treatment of
lung cancer. London; NICE, 2011.

Case 149: Change in bowel habit


malignancy

Candidate information constipation for the last 6 months, now suffering


from acute diarrhoea causing him significant
Scenario embarrassment and distress. Recent blood
You are the gastroenterology SHO. This 62-year- tests by the GP have shown an iron-deficient
old man has presented to the outpatient clinic anaemia. Please assess and offer advice on
with a change in bowel habit, complaining of further management.
Case 149 Change in bowel habit malignancy 291

Actor information loss, and general malaise represent red flag


symptoms. Screen for symptoms suggestive
You werent particularly concerned when you
of metastatic spread such as bone pain,
noticed constipation a few months ago, but now
breathlessness, or cough. Failure to pass wind/
you keep having episodes of diarrhoea. This has
flatus may represent a progression to bowel
caused you considerable social embarrassment
obstruction, which in the presence of pain,
and resulted in prolonged time off work.
bloating and vomiting would naturally require
Additionally, your GP has told you that you
expedient escalation of treatment.
are anaemic, which you assume explains your
In considering the differential, remain open
lethargy, but you are unsure of the significance
to the possibility of alternative causes of intra-
in relation to your other symptoms.
luminal obstruction or anal pathology, other
than colorectal carcinoma, such as:
Prior to entering the room Diverticular disease: recurrent episodes, left
Conceivably, scenarios revolving around an iliac fossa pain, often accompanied by low
altered bowel habit could focus upon a variety of grade fever
underlying aetiologies. The patient narrative and Stricturing on a background of inflammatory
context of the symptoms will be all important in bowel disease, particularly with previous
establishing the theme of the station and nature surgical intervention
of the discussion. The possibilities range from Fissure-in-ano or abscess: pain on
young patients, in whom symptoms may prompt defaecation and fresh rectal bleeding or pus
investigation of inflammatory bowel disease, to Haemorrhoids: straining at stool, pain
the older patient with red flag symptoms alerting on defaecation, blood on wiping, lump
to the possibility of a rectal mass, or where protruding from anus
constipation and diarrhoea represent a feature
of systemic disease or medication side effect. Be mindful of conditions associated
Employing a sensitive approach, to what are with constipation including endocrine and
commonly distressing and embarrassing topics neurological, such as:
for patients to broach, will be vital. Hypothyroidism: cold intolerance, hair/skin
changes, lethargy and proximal weakness,
On entering the room begin with with weight gain
Hypercalcaemia: nausea, vomiting,
open questioning abdominal pain, mental disturbance
Establish the nature of the symptoms, the speed Diabetes: polyuria, polydipsia, other
of onset and progression of the problem. Probe autonomic disturbance
as to what the patient actually understands by Parkinsons: resting tremor, rigidity,
the term constipation. Allow a full description of bradykinesia, difficulty with manual tasks
both new symptoms and what the patient would Spinal cord compression or demyelinating
consider to be a normal bowel habit for them. disease: urinary disturbance, sensory or motor
Be sure to ask about frequency of defaecation, lower leg neuropathy, sphincter dysfunction,
form of stool, the need for excessive straining perianal or perineal loss of sensation
and discomfort or pain on passing a motion, any
sensation of incomplete evacuation or anorectal
obstruction, and rectal bleeding. Recent or acute Open questioning suggests
constipation is more likely suggestive of organic a diagnosis of possible GI
disease, whereas chronic symptoms, of several
years, may point to functional problems.
malignancy
The challenge in this scenario will be to
discriminate between those cases requiring
With this diagnosis closed
further, often invasive, investigation, and questioning should focus on
those where reassurance will suffice. Be alert the following
to the likelihood of colorectal carcinoma with
Appreciate the need to clarify the patients past
a tendency to looser or more frequent stools,
medical history to specifically rule out disease
progressing to constipation, and the possibility
associations, such as those mentioned above.
thereafter of overflow diarrhoea. Associated
Thereafter, look to screen for alternative reasons
rectal bleeding without abdominal pain,
for the onset of altered bowel habit, particularly
symptomatic iron deficiency anaemia, weight
a careful drug history.
292 Chapter 6 History taking (station 2)

Enquire as to the intiation of any new practicalities of tests and investigations and the
prescriptions and screen for medication with time to results.
recognised associations with constipation and/
or gastrointestinal disturbance with blood loss
and anaemia, such as:
What to tell the examiner
Tricyclic anti-depressants
Summarise the clinical case
Anti-cholinergic medications This 62-year-old man complains of persistent
Anti-convulsants change in bowel habit with looser stools and
Calcium-channel blockers frequency up to 4 times a day over the last 8
Diuretics weeks. There is associated rectal bleeding, mixed
NSAIDs in with the stool and intermittent abdominal
Immodium pain. He has also lost 34 kilos over the last 3
Opioids months.
Iron supplementation
Investigations
Enquire regarding a family history of cancer
or other hereditary disease HNPCC, polyposis Digital rectal examination for palpable rectal
coli, PeutzJeghers. mass or identification of rectal bleeding
Considering the social history, smoking is Blood tests to look for iron deficiency
associated with increased risk of colorectal anaemia, tumour markers (CEA)
cancer. If no major co-morbidity colonoscopy would
be the gold standard for diagnosis with
Enquire about dietary change that might biopsies
have contributed to change in bowel habit Flexible sigmoidoscopy with barium enema
Be alert to behavioural factors: Poor bowel would be an alternative for patients with co-
habit/lifestyle-ignoring desire to defaecate morbidities unlikely to tolerate or have a safe
Anxiety and depression: anorexia nervosa, procedure
affective disorder CT colonoscopy is an alternative if there
are facilities, especially in elderly patients
Explore concerns, ideas, and or those with comorbidities or risk of
perforation during colonoscopy
expectations Contrast-enhanced staging CT of the chest,
The patient may raise a wide range of concerns, abdomen and pelvis if colorectal cancer
such as the following. Give the patient a chance diagnosed
to verbalise thoughts in relation to colorectal
cancer before you offer the idea in a completely Management
unprepared patient. Alternatively, the patient Management of colorectal cancer depends on
may not be particularly concerned regarding the patient expectations, co-morbidities and staging
underlying cause, but primarily keen to resolve and MDT decision. The following management
the symptoms, especially if, as in this case, hey points are based on NICE guidelines (CG 131).
are causing social embarrassment. In either Local colorectal tumours:
case, warning shots are an important principle.
Resectable primary rectal tumours: MRI
for further characterisation followed by
What to tell the patient resection. Short course pre-operative
I am concerned that your symptoms over the radiotherapy for moderate risk tumours and
last 6 months may be linked. I am conscious of pre-operative chemoradiotherapy for high
your anxiety in this regard but feel it important risk tumours
that we investigate further, as I am worried Unresectable colon or rectal tumours
about a potential diagnosis of cancer. I would or borderline resectable: pre-oeprative
like to perform an examination today in clinic chemoradiotherapy to allow for shrinkage
and arrange for a camera test to look at the and tumour response with high risk locally
bowels from the back passage. After the tests advanced tumours
are completed I will organise an appointment Laparoscopic or open surgery depending on
for you to come back and discuss the results the suitability for laparoscopic resection, the
and potential treatment options. Be prepared risks and benefits of both procedures, and
to answer questions relating to the details and the experience of surgeon
Case 150 Change in bowel habit established inflammatory bowel disease 293

Consider adjuvant chemotherapy after


Familial adenomatous polyposis: genetic
surgery for high-risk stage II/III colon cancer
testing for APC mutation at age 12
Advanced and metastatic colorectal tumours
years. Yearly flexible sigmoidoscopy. In
(metastatic or locally invasive and surgical
polyp identification full colonoscopy +/
excision with curative intent is unlikely):
prophylactic colectomy. Upper endoscopy
Hepatic metastases only hepatobiliary MDT
surveillance recommended too
for decision regarding possible surgery
Hereditary non-poluposis colorectal cancer:
Extra-hepatic metastases MRI for suspected
colonoscopic surveillance at 2025 years as
intracranial metastases and anatomic site-
most tumours are right sided. Genetic testing
specific MDT and PET-CT scan for decision
for 1st degree relatives
regarding radical surgery
PeutzJeghers: 2-yearly upper and lower
If both primary and metastatic tumours are
endoscopy, barium follow-through. Monitor
deemed respectable consideration of pre-
for gonadal, breast and cervical cancer
operative chemotherapy and possibility of
Factors in past medical history:
concomitant surgery for both tumours
Polyps: surveillance colonoscopy for
Whenever surgical options are considered, a adenomas >1 cm, villous or tubulovillous
discussion about stomas will be required. histology, multiple adenomas and dysplastic
polyps. 3 yearly if colon cleared of polyps
Screening for colorectal cancer Colorectal cancer: 36 months after surgery
For average risk patients surveillance colonoscopy for inspection of
<50 years of age no screening anastomotic site, at 1 year and 35 yearly
>50 years of age: thereafter
Annual faecal occult blood test Inflammatory bowel disease (see relevant
5-yearly flexible sigmoidoscopy case)
10-yearly colonoscopy Acromegaly: 3-yearly colonoscopy from age
For high risk patients: 40 years
Positive family history: if first or second degree
relative with CRC or adenomas in relative <60:
annual faecal occult blood tests and 5-yearly Further reading
flexible sigmoidoscopy, 10-yearly colonoscopy National Institute for Health and Care Excellence
starting at age 40 years. (NICE).CG131, Colorectal cancer: the diagnosis
Genetic syndromes: and management of colorectal cancer. London,
NICE, 2011.

Case 150: Change in bowel habit


established inflammatory bowel disease

Candidate information Actor information


Scenario Since you were last seen in the clinic, you have
had 4 episodes where your symptoms have
As the SHO in the gastroenterology clinic worsened, requiring steroid therapy. You wonder
you are asked to see a 35-year-old woman whether there are any other treatments available.
with a diagnosis of inflammatory bowel
disease, attending for her annual review.
Take a history from this patient and advise Prior to entering the room
on any changes in management that may be In a patient with established IBD the aim of the
necessary. history station would be to:
294 Chapter 6 History taking (station 2)

Assess disease severity Other manifestations (unrelated to colitis)


Identify any complications include:
Identify side-effects from treatment
Ankylosing spondylitis
Escalate management in chronic active
Sacroilitis
disease
PSC
Complications to have in mind would
Important points in the past medical history
include:
include:
Colorectal cancer
Previous surgical intervention
Cholangicarcinoma
Previous polyps
Primary sclerosing cholangitis
Falls and fractures in the context of nutritional
Ankylosing spondylitis/sacroilitis
deficiency and long-term steroid use
Strictures
Enquire about smoking in the social history
On entering the room begin with and any relation to flares. Be sure to take a family
history focusing on auto-immune disorders and
open questioning cancers.
Screen for current symptoms, which may include
bloody diarrhoea, pain, urgency or tenesmus.
Establish the duration of the disease and extent
Explore concerns, ideas, and
of colitis proctitis, left sided or more extensive expectations
(50% of patients have disease confined to colon/ Take time to explore what the individual patient
rectum, 30% beyond sigmoid, 20% pan-colitis). considers to be the main issue. For example,
Attempt to gauge the level of control achieved are they keen to explore other treatment
with existing treatment and clarify the measures options?
required to treat any flares since the last review.
What to tell the patient
Open questioning suggests a There is considerable scope for improvement
diagnosis of poorly controlled in the control of your disease. I propose we
inflammatory bowel disease make some changes to your current regime and
consider the use of an immunosuppressant.
With this diagnosis closed I would hope that this improves day-to-day
symptoms and reduces the number of flares.
questioning should focus on Additionally it would be sensible to arrange for
the following a repeat sigmoidoscopy, or if tolerable a full
screening colonscopy, to assess the current
Using closed questioning, establish disease
extent and severity of disease.
severity and the presence of systemic
disturbance to allow you to form an idea of how
well controlled their disease is and if treatment What to tell the examiner
escalation is advocated.
Highlight the importance of recent Summarise the clinical case
colonoscopy results if available in patients The patient has an existing diagnosis of
with long-standing disease that might require ulcerative colitis, identified at the age of
surveillance colonoscopies. Disease extending 18-year-old. She has a limited colitis affecting
past the rectosigmoid is associated with higher the left sided colon and is usually maintained
risk for malignancy. Risk relates to duration, on oral mesalazine. Over the last year, she has
severity, and extent of disease. had 4 flares, requiring oral steroids. Her father
Seek extra-intestinal manifestations of UC had colorectal cancer diagnosed at the age
related to disease activity: of 45 years which puts her at a higher risk of
colorectal cancer development in the future
Peripheral arthropathy although her colitis at present in limited to the
Erythema nodosum left side. The frequency of her flares suggests her
Episcleritis or anterior uveitis disease is not well-controlled and might require
Mouth ulcers second line treatment with a steroid sparing
Pyoderma agent.
Case 151 Difficulty in swallowing 295

Investigations Ciclosporin reserved as salvage therapy for


acute flares used for 36 months often as a
Blood tests include ESR/CRP, FBC to look
bridge to azathioprine
for anaemia of chronic disease or microcytic
anaemia from chronic blood loss, iron studies to Of proctitis:
look for iron deficiency, and LFTs particularly in
suspected PSC. Topical mesalazine with/without oral
When considering systemic agents: TPMT preparations
before starting AZA as reduced enzyme Add topical steroids if unresponsive
activity is associated with higher incidence of Formulation: suppositories for rectosigmoid,
myelosuppression and mercaptopurine may be foam liquid enemas for more proximal
indicated. FBC and LFTs before, and monthly disease
after the initiation of treatment. Add oral prednisolone if above fail
When suspecting a recent flare or active Suppositories for rectosigmoid
disease sigmoidoscopy should be sufficient. Non-stimulant osmotic laxatives if proximal
Colonoscopy for surveillance or more extensive faecal loading (PEG-based)
colitis, but avoid in active flares as patient will be Of mild active left-sided colitis or more
at higher risk of perforation (see below). extensive disease:

Management Oral mesalazine 2.44.8 g daily with/without


topical mesalazine (needs monitoring as risk
Maintenance therapy includes: (see Further of interstitial nephritis)
Reading for reference) Prednisolone 2040 mg reduced according to
Mesalazine 1.22.4 g/day orally disease response over 48 weeks
Topical mesalazine 1g daily for distal Confirm disease activity by sigmoidoscopy
disease Acute severe active colitis needs admission
For steroid-dependent disease, for intensive medical therapy, often intravenous
steroid-sparing agents should be considered: steroids, and close monitoring for escalation
of treatment with the possibility of surgical
Azathioprine 22.5 mg/kg/day. The intervention where poor response to medical
indications for azathioprine include: severe treatment or complications arise.
or frequent relapse; two or more steroid
courses within 12 months; relapse at
prednisolone doses <15 mg; relapse within Further reading
6 weeks of stopping steroids Mowat C, Cole A, Windsor A, et al. Guidelines for the
Mercaptopurine 0.751.5 mg/kg/day management of inflammatory bowel disease in
Methotrexate possible alternative adults. Gut 2011; 60:571607.

Case 151: Difficulty in swallowing

Candidate information Actor information


Scenario You have been suffering from heartburn for
many years but have managed it using over the
As the gastroenterology registrar you review counter antacids. Recently you have noticed
an urgent referral in clinic from a GP for a that you have had difficulty in swallowing. You
74-year-old man experiencing dysphagia. consulted your GP who has arranged for you to
Please assess and advise regarding further be seen today.
management.
296 Chapter 6 History taking (station 2)

Prior to entering the room Enlarged left atrium in mitral stenosis


In any dysphagia history you need to establish if
Para-oesophageal hernia intermittent
dysphagia, worse after a big meal, associated
the difficulty swallowing is associated with pain
hiccups
and whether it is limited to solids, liquids or both.
On entering the room begin with open
questioning Open questioning suggests
Explore the nature of dysphagia, how long it a diagnosis of possible
has been present for, is the problem intermittent
or constant, is the dysphagia worse for liquids malignancy
than solids and what other symptoms they are
experiencing. With this diagnosis closed
Narrow your differential considering the questioning should focus on
causes of dysphagia:
the following
In the wall If considering a diagnosis of oesophageal cancer
make sure you ask about symptoms suggestive
Gastrointestinal: of complications, such as:
Gastro-oesophageal reflux disease (GORD): Pulmonary, hepatic, brain and bone
intermittent, retrosternal burning pain, metastases are common with SCC
worse on lying, relieved on sitting forward, Adenocarcinoma spreads to regional and
worse after meals, especially rich in fat. distant lymph nodes and liver
Long-standing symptoms may predispose to Ask about breathlessness/haemoptysis, RUQ
Barretts or oesophageal cancer pain, confusion or headaches, and bone pain
Achalasia: intermittent difficulty, worse on systems review
for liquids than solids, associated food
regurgitation and heartburn Continue your closed questioning to
Oesophageal carcinoma: rapid onset and complete your history. Establish any past
deterioration of symptoms. Dysphagia to medical history of conditions predisposing to
solids first, then liquids. Haematemesis may oesophageal cancer such as achalasia, GORD/
be a feature. Associated anorexia, weight loss, Barretts and even previous malignancy
symptoms of anaemia. Occasionally voice especially head and neck SCC. Exclude drugs
changes in vocal cord paralysis from tumour that relax the lower oesophageal sphincter.
infiltration Check whether there is a family history of
PlummerVinson syndrome: dysphagia with achalasia or oesophageal cancer. Tobacco and
food sticking on upper part of oesophagus, alcohol history also important as they are both
middle aged female (web in upper risk factors for the development of oesophageal
oesophagus/post-cricoid web) carcinoma.
Barretts oesophagus or columnar lined
Infection: oesophagus is a precursor of development
Chagas disease: recent travel in endemic of oesophageal carcinoma. Its incidence
areas (e.g. Brazil, Venezuela, Argentina) has dramatically increased over the last
cardiac arrhythmias, gastrointestinal, years. Columnar lined epithelium may be
respiratory and urinary symptoms a complication of long-standing gastro-
oesophageal reflux disease. However, GORD
Autoimmune: may progress to adenocarcinoma without
Scleroderma developing into Barretts first making screening
important in cases of long-standing, severe
GORD.
Outside the wall
Pharyngeal pouch elderly patients,
regurgitation worse on lying down or Explore concerns, ideas, and
coughing, food sticking behind manubrium, expectations
neck swelling, halitosis, aspiration
Find out if they are suspecting what might be
pneumonia
the problem first and whether they mention the
Mediastinal tumours/bronchial ca-
possibility of a malignancy.
haemoptysis, weight loss
Case 152 Cirrhotic liver disease 297

What to tell the patient Investigations


I am worried that the symptoms you are In the first instance organise the following blood
describing are because of a blockage in the food tests: FBC may show anaemia of chronic disease
pipe. I would recommend an urgent endoscopy, or microcytic anaemia in which case iron studies
a test where a camera tube is passed from your may show a deficiency in the presence of chronic
mouth down into you stomach to establish what blood loss through a bleeding tumour. CRP is
might be causing the blockage. Depending upon often elevated in malignancy. Renal function
the findings we might be able to improve some may be deranged in dehydration or protracted
of the blockage to relieve your symptoms at the vomiting and albumin may be low because
time of the procedure. I will arrange a follow-up of malnutrition. Abnormal liver function may
appointment soon after the procedure to discuss suggest metastatic disease. Hypercalcaemia may
the results of biopsies if taken, and discuss be seen in SCC secondary to PTH related peptide.
treatment options. Other investigations would include:
Plain chest radiograph: might pick up large
What to tell the examiner tumours but also pulmonary spread
OGD: gold-standard for diagnosis and
Summarise the clinical case option for therapeutic dilatation. May show
This man with a long-standing history of tumour in the oesophagus, or the presence
GORD who has never had a screening OGD of pre-malignant Barretts oesophagus.
complains of rapid onset dysphagia with Biopsies are essential. Therapeutic dilatation
associated odynophagia and weight loss. I is a treatment option for patients with
am concerned there might be an underlying obstructing tumours or structuring disease
oesophageal malignancy and would like to CT for staging
refer him for urgent OGD and biopsy. In the
meantime I would examine him to look for Management
evidence of metastatic spread. If you are
Management of oesophageal cancer is via
confident enough you can expand upon the
an MDT approach. The exact management
likely aetiology in view of long-standing GORD
approach depends on staging. In general,
with a predisposition to adenocarcinoma
curative surgery may be an option in early stages
rather than SCC, but that biopsy will aid
with/without adjuvant chemoradiotherapy.
confirmation.
Therapeutic dilatation and stenting can be
offered for palliation.

Case 152: Cirrhotic liver disease

Candidate information Actor information


Scenario You saw your GP last month because your
family commented on a yellowed appearance
You are the gastroenterology registrar. A 60-year- of your eyes. As explored by your GP in the
old man with jaundice has been referred to the community, you drink approximately one bottle
outpatient clinic with suspected cirrhosis by the of red wine each evening after work. You drink
GP after routine blood tests showed abnormal more at the weekends. If the candidate asks you
LFTs and a cirrhotic liver on ultrasound. directly, you will admit that you are worried by
Please take a history from the patient and tell your drinking habits and are keen to seek help
him about any further investigations and/or to cut down.
treatments that are necessary.
298 Chapter 6 History taking (station 2)

Prior to entering the room Others including haemochromatosis, with


a history of diabetes, cardiomyopathy,
When you are dealing with such a scenario there
hypogonadism, skin pigmentation, and
are many things to consider. Abnormal LFTs,
arthritis, or a1-antitrypsin with cirrhosis and
jaundice and a cirrhotic liver probably give you
emphysema
the diagnosis of cirrhosis. Your job is to try to
establish the cause of liver disease, make an Next assess for history of complications of
educated guess as to how advanced the disease cirrhosis, such as:
is and if there is evidence of decompensation
or complications and of course provide a
Variceal bleeding: upper GI bleeding,
haematemesis, malaenia, epigastric pain
sound management plan that might prevent
progression to cirrhosis or prevent/reverse
Ascites
decompensation.
Hepatic encephalopathy
Hepatocellular carcinoma (especially in the
context of hepatits C)
On entering the room, begin Malnurishment
with open questioning Infection/SBP
Osteoporosis: check for low impact or
Let the patient guide you at first by asking spontaneous fractures
open questions to find out how long they have
been symptomatic (if at all), what prompted The presence of Jaundice, encephalopathy
investigation in the first place and about the or ascites indicates decompensated cirrhosis. If
general state of their health. the patient is describing such symptoms try to
establish the cause:
Open questioning suggests a Natural disease progression
diagnosis of chronic liver disease Recent alcohol binge/alcoholic hepatitis
Concomitant infection with hepatitis virus,
With this diagnosis closed
e.g. A
Infection/SBP: abdominal pain, distension,
questioning should focus on fever
the following Upper GI bleeding: malaena, haematemesis,
epigastric pain
Try to identify points in the history which
suggest the cause of their liver disease:
Dehydration: excessive vomiting,
malnutrition, decreased or concentrated
Exposure to hepatitis B or C (e.g. previous urinary output
blood transfusion, haemodialysis, maternal Constipation: may lead to encephalopathy
infection, needle exposure) or HIV Drugs: new or hepatotoxic
A history of other auto-immune conditions, Portal vein thrombosis: abdominal pain,
bowel pathology such as IBD fever, mesenteric infarction
Careful drug history, recreational drugs, BuddChiari: abdominal pain, ascites,
intravenous drug abuse, over the counter or hepatomegaly over several months
herbal. Drug history should ask specifically HCC: recent weight loss or abdominal pain
about use of NSAIDs, opiates, statins,
aminoglycosides that may cause worse
hepatic function or predispose to bleeding.
Explore concerns, ideas, and
Take a careful vaccination history especially expectations
for hepatitis B It is likely that any discussion surrounding the
Alcohol history, focusing on type of alcohol patients concerns will be primarily directed
and frequency of consumption, duration and towards alcohol use and dependency.
pattern of drinking
Recent travel history: job and occupational
exposure, especially healthcare workers, sex What to tell the patient
workers I suspect that all the problems we have
Sexual history for identification of risk factors discussed today are related to damage to liver
of sexually transmitted disease including caused by excessive alcohol over the years.
Hepatitis and HIV Alcoholic liver disease may be asymptomatic
Family history of auto-immune disease, for long periods. By the time it is identified,
genetic disease, diabetes usually on routine blood tests it means that the
Case 153 Previous renal transplant 299

liver is already damaged, which seems to have to exclude other causes, such as hepatitis C
happened in this case. The liver is a complex infection.
organ and among other things it helps filtering
toxins from the blood, regulates cholesterol Investigations
levels and helps fight infection. All of these
The investigative strategy in this patient would
functions are impaired when damage occurs
include the following:
and ultimately prolonged alcohol misuse
can seriously damage the liver irreversibly Full liver screen including: liver function
and even lead to death. Cirrhosis is the final tests, GGT, AST, hepatitis A, B and C, HIV,
stage of alcoholic liver disease. The damage is serum copper levels, ferritin, a1-antitrypsin
irreversible but stopping alcohol immediately FBC to look for anaemia, renal profile for
prevents any further damage and can lead to signs of dehydration/hyponatraemia/renal
gradual recovery of liver function. The treatment failure/hepatorenal syndrome, and clotting
options depend on you and whether you are screen which is likely to be derange due to
prepared to stop drinking. altered liver synthetic function
In the meantime I need to organise some Alpha-fetoprotein for suspected
tests to assess the degree of damage to your liver hepatocellular carcinoma
and rule out any other causes of liver disease. US liver and Doppler study to assess hepatic
portal veins
What to tell the examiner
Management
Summarise the clinical case When there is evidence of decompensation,
This 60-year-old man referred by his GP with admit the patient for further tests and treatment.
jaundice, abnormal LFTs and ultrasound Monitor closely for complications including
evidence of cirrhosis gives a history suggestive infection. Correct electrolyte disturbance and
of decompensated liver disease with jaundice provide nutritional support. Unless faced with
and ascites following a recent alcohol binge acute bleeding, consider routine OGD to look for
accompanied by a recent short history of varices and banding. Prophylactic propranolol
melaena and haematemesis indicating probable may be considered for prevention of variceal
variceal disease. Alcohol induced liver disease bleeding. Therapeutic drainage of ascites may be
is most likely implicated, however, I wish considered in symptomatic patients.

Case 153: Previous renal transplant

Candidate information tests have been deteriorating. Despite being


asymptomatic, you are concerned that your
Scenario transplanted kidney may be failing.
As the renal SHO, you have been asked to assess
a 61-year-old man who has been referred by Prior to entering the room
his GP with abnormal renal function tests and
decreasing urine output. He has previously been Decreasing urinary output and abnormal renal
a renal transplant recipient. Please take a history function indicate a failing graft. In general, most
and discuss your findings with the patient. A complications related to renal transplants will be
discussion with the examiner will follow. secondary to infection or graft rejection but you
should bear in mind all complications associated
with renal transplants, namely opportunistic
Actor information infection secondary to immunosuppression,
You have been sent to the renal clinic because hypertension, hyperlipidaemia, cardiovascular
your GP has noticed that your renal function disease (increased 10-fold compared to
300 Chapter 6 History taking (station 2)

the general population), diabetes mellitus Rejection


(pre-transplantion but also new onset secondary
Graft failure may be completely asymptomatic
to corticosteroids), tumours or disease
but may also present with decreasing urinary
recurrence. Recurrent renal disease accounts
output, hypertension and discomfort around
for fewer than 4% of graft failures. The degree
the transplant site. Needless to say you should
of immunosuppression is also directly linked
enquire about any symptoms consistent with
with the development of certain complications,
infection including fever, respiratory, urinary
as subtherapeutic levels will lead to higher
and bowel symptoms.
incidence of and rejection whereas toxic
levels may have a direct nephrotoxic effect. Hyperacute rejection usually occurs intra-
Immunosuppressants alone will lead to a higher operatively due to ABO or HLA class I
rate of opportunistic infections and malignant incompatibility.
transformation due to immune suppression. Acute rejection in the first 6 months from
In addition, graft rejection may be difficult to surgery occurs in 15% cases and often
clinically distinguish from infection, as the patients have recurrent episodes of rejection.
former may also present in a similar fashion or Presents as fever and graft pain
the two may co-exist. Late rejection after 6 months, usually a result
The most important aid in scenarios of immunosuppressive withdrawal after 6
involving renal transplantation is to establish months
early on the length of time after transplantation; Chronic rejection after 1 year usually
the type of complications is directly associated manifests as progressive loss of renal
to the length of time post transplant. function and proteinuria

On entering the room begin with Infection


open questioning Patients who have experienced previous
episodes of rejection may require high
Open questioning can be very informative as dose immunosuppression with a resultant
most patients with renal transplants and their susceptibility to infection and resultant
relatives will be very knowledgeable about their complications. Expect a history of fever, contact
condition. Do not underestimate the patient and with ill patients and/or mucocutaneous,
their ideas about the cause of their symptoms. respiratory, urinary, bowel symptoms.
Find out why and when they had their Consider opportunistic infections during the
transplant and if the source was cadaveric or 1st month post transplant and up to 6 months
from a living donor. Cadaveric transplants are when immunosuppression reaches its peak levels.
associated with more episodes of rejection,
infection and are associated with a lower Early: during the first month, usually related
survival rate (81% at 5 years compared to 91% in to peri-operative pathogens and localise
living donor). Was it a pre-emptive transplant in urinary tract (E. coli), line infections
and had they received renal replacement (Staphylococcus aureus or Streptococcus
therapy beforehand. Find out if there have viridans), wound (Streptococcus viridans,
been previous episodes of rejection or infection Staphylococcus aureus) and respiratory
and subsequent changes to medication. (Streptococcus pneumoniae)
The immunosuppressive regime should be Intermediate: during the first 6 months expect
established early on in the history as well as opportunistic infections. Fever, general
any recent medication changes and overall malaise, arhthralgia, myalgia. Most common
compliance with treatment. pathogens: CMV, PCP, Candida, Listeria,
Aspergillus. High fever, derraged LFTs and
leukopenia are often seen in CMV disease
Open questioning suggests a Late: after 6 months. The type of infection
diagnosis of deranged function depends on general health. If graft failure is
normal and on low dose immunosuppression
in a renal transplant expect infections similar to the general
population. In patients with poor functioning
With this diagnosis closed grafts, recurrent rejection episodes or high
questioning should focus on dose immunosuppression expect chronic
the following and opportunistic infections. Consider
infections with latent viruses such as CMV,
Complication of renal transplantation giving rise
VZV, EBV and hepatitis
to refection.
Case 153 Previous renal transplant 301

Cardiac/vascular glomerulonephritis are considered high risk for


transplant loss.
Cardiovascular complications are commoner
in pre-existing cardiac disease, hypertension,
steroids, diabetes, smoking and previous Explore concerns, ideas, and
episodes of rejection. Hypertension may be a
result of native kidney disease, graft dysfunction
expectations
or immunosuppressant. Most cases occur in the Many patients with renal transplantation have
first 12 weeks from surgery. a wide range of knowledge surrounding their
condition and may be medicalised. Therefore,
Renal artery thrombosis: usually presents afford them the time to voice any concerns as
in the immediate post-transplant period. they feel appropriate.
Sudden cessation of urine output
Renal vein thrombosis: early complication,
decreasing urinary output, haematuria, pain What to tell the patient
and swelling over graft site The obvious concern is one of potential
Renal artery stenosis: later. Uncontrolled transplant rejection. In order to better
blood pressure, peripheral oedema appreciate the current situation we will
perform repeat blood testing and arrange for
Urinary leak an ultrasound before reviewing the current
management and exploring potential options.
Common around 2 months: fever, abdominal
swelling and graft dysfunction.
What to tell the examiner
Malignancy Summarise the clinical case
Usually secondary to chronic
The case is one of a pre-emptive cadaveric
immunosuppression and viruses with malignant
kidney transplant 6-months ago presenting
potential such as EBV, HBV and HHV8. Kidney
with decreasing urine output in the context of
transplant patients are particularly at risk of
abnormal renal function indicative of a failing
non-melanoma skin cancer, kidney, non-
graft. The most common causes of graft failure
Hodgkins, and gynaecological cancer related
at 6-month include rejection and opportunistic
to HPV infection (cervix, vulvovaginal) but
infection. The history is not suggestive of an
in general higher incidence of most cancers
infectious cause although immunosuppressants
including lung, colon, bladder, prostate testis.
can mask symptoms. I would proceed with a full
work-up to rule out infection and other causes of
Drug nephrotocixity graft failure.
Although non-compliance is the commonest
cause of late acute rejection, ciclosporin and Investigations
tacrolimus are directly nephrotoxic and hence
Explain you would firstly examine the patient
tight level control is warranted.
fully for signs of sepsis, volume status and
Drug interactions are very important too
careful examination of transplanted kidney.
when considering levels especially the P450
You would record the temperature, blood
enzymes.
pressure and body weight as well as urinalysis.
Increased ciclosporin levels: calcium channel White cells are present in infection and
blockers such as diltiazem, antibiotics such obstruction, where as red cells may point to
as erythromycin glomerulonephritis.
Decreased levels: antibiotics such as Baseline blood tests should include
isoniazid, rifampicin (anti-TB prophylaxis is inflammatory markers and FBC to look for
very common in the transplant population), evidence of sepsis (leucocytosis, raised CRP),
anti-epileptics serum urea and creatinine which will be
Others: amphotericin, aciclovir, NSAIDs have elevated in the context of a failing graft (a 20%
a synergistic nephrotoxic effect (without rise in serum creatinine defines graft failure)
affecting levels) and very importantly immunosuppressive levels
(ciclosporin or tacrolimus usually) to rule out
Recurrent renal disease toxic or subtherapeutic levels.
Full septic screen including blood, urine
Focal segmental glomerulosclerosis, HUS,
cultures and stool cultures as well as chest
oxalosis, and membranoproliferative
302 Chapter 6 History taking (station 2)

X-ray. Ultrasound scan to look for obstruction or period of 25 years disease free should be
perinephric collection and colour-flow Doppler allowed before transplantation
of the renal transplant to look at vasculature and ABO blood group and HLA typing, CMV,
perfusion. Finally a graft biopsy may provide a EBV, VZV, hepatitis B, hepatitis C and HIV
definitive diagnosis. tests
Immunisation to HepB
Management Full cardiovascular work-up and screening
for cardiovascular disease. Patients with
Management depends on the underlying cause
the following should have cardiac stress
of transplant dysfunction. Treat reversible
tests: Age >49, diabetes, abnormal resting
causes such as infection, electrolyte disturbance
ECG and history of ischaemic cardiac
and volume abnormalities. Patients in
or cerebrovascular disease. Coronary
severe acute renal failure might even require
angiography or stress echocardiography
haemodialysis.
is recommended for further testing in
Optimise immunosuppression and
abnormal results
drug regime. In suspected drug-related
nephrotoxicity, if a trial of dose reduction does Types of graft: living related donor, live
not result in improved serum creatinine proceed unrelated donor, cadaveric grafts from
to biopsy. Consider corticosteroids in the brainstem dead donor.
context of rejection. Note that simultaneous kidney pancreas
Other considerations include: transplantation would be the treatment of
choice for patients with type 1 diabetes mellitus,
If blood transfusion is required, CMV
chronic renal failure including predictive date of
negative irradiated blood should be used.
requiring dialysis within 6 months or on dialysis
Intravenous hydrocortisone often used in
Pre-transplantation counselling including
patients on long-term steroids at risk of
potential risk of recurrent disease (accounts for
adrenal insufficiency
5% of allograft loss secondary to primary focal
Maintain a patient-centered focused
segmental glomerulosclerosis, IgA nephropathy,
approach, re-assuring the patient but at the
mesangiocapillary glomerulonephritis);
same time alert them to impending graft
smoking cessation advice as there is evidence
failure and the possibility of graft loss
that quitting smoking pre-transplant reduces the
Other general tips to consider: screening
relative risk of graft failure.
for malignancy at regular intervals, tight
diabetic, hypertensive, lipidaemia control for
Contraindications to renal transplantation:
Absolute contraindications include: active
optimal cardiovascular state
infection or systemic disease or metastatic
cancer, cancer, severe untreated heart
Additional themes worth disease, liver insufficiency and HIV (unless
considering CD4 count >200 for more than 6 months
and undetectable viral load on stable
Assessment prior to renal transplantation antiretroviral therapy for more than 3 months
Careful history for pre-exisisting and no major infectious or neoplastic
cardiovascular disease, CMV, HBV, TB and complications)
other chronic infections. EBV negative Relative contraindications include: HCV,
patients receiving EBV positive transplant HBV, morbid obesity, atherosclerosis, cardiac
have an increased risk of post-transplant disease, uncontrolled hypertension, smoking,
lymphoproliferative disorder (7-fold). In unresolved psychosocial issues
cases of previous malignancy a window
Case 154: Headache

Candidate information unsuccessful. In such cases it will be necessary


to demonstrate sufficient empathy to gain
Scenario trust and allow a fresh review of symptoms
You are the SHO in the neurology clinic. This to facilitate joint management decisions.
27-year-old female nurse has been experiencing Alternatively, it may be necessary to deal with
headaches of increasing frequency and severity. pre-conceived ideas as to the agenda for the
Please take a history from the patient. Inform consultation. Patients may have been referred
the patient of your findings and inform them of with the false promise of neuro-imaging or the
your management plan. prescription of stronger analgaesia. In such
situations it is important to reassure the patient,
consider appropriate testing or treatment if the
Actor information history warrants it, but in all likelihood, some
You started to have headaches a couple of negotiation will be required.
months ago. You have seen your GP four times Attempt to organise the symptoms and
with the same complaint. Additionally, you stratify using the framework of primary versus
presented to Accident and Emergency one secondary headaches. Making use of such
evening with a particularly severe headache. definitions will aid an organised approach to
You are concerned that you may have a brain a symptom with a wide and varied differential
tumour. You had been using over the counter diagnosis. It may be that in the first instance a
painkillers but despite taking the maximum candidate will exclude the features of common
doses the headaches have persisted and so in primary headaches before going on to ask closed
addition began using medication from hospital questions in respect of a secondary cause.
supplied on prescription by one of the doctors Primary disorders are those not associated
from the ward. with an underlying pathology and commonly
include migraine, tension-type, and cluster
headache.
Prior to entering the room Secondary disorders headache attributed to
Headache is one of the most common an underlying pathological condition. Includes
clinical presentations in general medicine any head pain of infectious, neoplastic, vascular
and frequently encountered in the PACES or drug-induced origin.
exam. Often the diagnosis is a clinical one and It is also useful to think about acute versus
investigation, which is often extensive and chronic in terms of an objective measure. While
invasive, should be reserved for cases where you are unlikely to encounter an acute headache
suspicion exists of a worrying underlying caused by a bleed or infection be precise when
pathology. It is the responsibility of the clinician attributing chronicity to a headache.
to discriminate between benign and sinister Narrow the differential and think about
features, reassuring where necessary. Often patterns of symptoms associated with headaches.
the need to implement trial management with
analgesia or lifestyle modification, and arrange Primary headaches
review as a safety net, is a prudent approach.
Migraine
Frequently unilateral and pulsating, with
On entering the room begin with an insidious onset lasting up to 72 hours.
open questioning Commonly associated with aura, nausea, and
Allow the patient to fully voice her concerns sensitivity to light and tinnitus or sensitivity to
and offer an account of her symptoms. Prompt sound. Worse with activity, preference to lie
the patient where necessary to establish onset, in the dark, resolution with sleep. Recurrent
duration, frequency and character. episodes aggravated by routine physical
Be prepared for the emotional patient exertion, with chronic history. Look for a family
where the symptoms have caused significant history.
distress, disruption to daily living and Tension-type headache
previous management from the GP has been Frequent or continuous, mild to moderate in
304 Chapter 6 History taking (station 2)

severity, and importantly bilateral. May be Mass effect neoplastic


described as band or vice-like, occipital or While the most common cause of early morning
frontal pain that spreads to the entire head that headaches is in fact migraine, classically this
is often worst at the end of the day. will raise alarm bells to the possibility of mass
Cluster headache effect. Seek to elicit evidence of diplopia,
Rapid onset of severe headaches, lasting from particularly on looking laterally (false localising
15 minutes to 3 hours. Commonly multiple sign). Pain aggravated by postural changes,
attacks on the same day. Prominent unilateral straining, or coughing. Altered behaviour where
orbitotemporal focus and may have trigeminal frontal lobe involvement. New onset seizures
involvement with lacrimation or flushing. possible. Vomiting or focal neurologic deficits
with significant mass effect. This may be what
Secondary headaches the patient is most worried about and may be
the elephant in the room. Be prepared to raise
Regarding secondary headaches consider the
the topic, even where the symptoms may not
diagnosis of secondary headache in patients
warrant it.
presenting with new onset headache or
Where there is a history of HIV or cancer the
headache that differs from their usual headache.
threshold for further investigating a headache
In order of importance in this setting:
is lower, thus ensure you ask the patient about
Analgesia headache any known diagnosis or consider screening for
Remains a diagnosis of exclusion but requires symptoms suggesting their involvement.
a clear history of significant analgesia use. This
should be excluded in all patients with chronic
headache, as defined above. Open questioning suggests a
Benign intracranial hypertension diagnosis of headache due to
This is a key diagnosis to consider where the persistent analgesia
scenario is with a young woman as there is
a predilection for young woman in the third With this diagnosis closed
decade. There is a migraine-like headache, but
it is associated with diplopia, pulsatile tinnitus,
questioning should focus on
and loss of peripheral vision. Be alert to risk the following
factors, classically obesity and medication such Be clear as to the exact analgaesia used.
as the oral contraceptive. There is some evidence Attempt to build up a timeline with the patient,
of a link to demyelinating disease and lupus and establishing at what point different medication
this may be a consideration. With a similar risk were trialed, in what strength and quantities. Not
factor profile, and in pregnancy, also consider only will this be crucial to any potential diagnosis
venous thrombosis. but it may prove insightful for the patient and aid
Infection comprehension of subsequent discussions.
Infection is worth excluding and may include
sinusitis or dental problems. Sinusitis is likely Explore concerns, ideas, and
with positional facial (on leaning forward like
a bowling ball) or tooth pain accompanied by expectations
a clear history of illness, fever, or rhinorrhea. What does the patient attribute the
Dental abscess occurs on a background of poor headaches to? Be prepared to discuss cancer
dentition, aggravated by mastication. Often it and think carefully as to the language used
causes pain that is sharp and shooting in nature, Pay particular attention to the social history
with pain referral to the ear or temple. and identify areas of stress
Acute angle-closure glaucoma Explore substance misuse. Does she believe
Unilateral: Halos around lights, decreased visual that the use of too many pain killers could
acuity, conjunctival injection, vomiting. cause her symptoms or represent a problem
in relation to substance misuse?
Giant cell arteritis
Screen for symptoms of depression, which
Unilateral throbbing pain: Exacerbated by touch may contribute to the presentation as a
often described as pain when combing hair. source of headache and negatively impact
Associated with visual disturbances and jaw upon the perception of symptoms and ability
claudication. to cope
Case 154 Headache 305

What to tell the patient Management


Be guided by the content in the history to Assess analgesia prescribing using the WHO
inform the patient as to what you think the pain ladder, advising the patient to avoid opiate-
underlying diagnosis is, what investigations may based analgesia. The treatment of analgesia
be necessary and any management options that headaches should be with abrupt withdrawal
may be used. in the case of simple analgesia or triptans, or
In this case, the patient has been using gradual withdrawal of opiates. Headaches
opiate-based analgesia and will need to stop are likely to initially worsen. Prophylactic
such medications given the likely diagnosis of medication may be of some benefit.
headache due to overuse of analgesia. The management of other types of headache
Arrange a shared plan with the patient either is discussed below:
to review results or to return (to this case to her
GP, but in other situations a repeat clinic review)
Acute management of migraine involves
simple analgesia and the use of tryptans.
to assess progress.
Subsequent management involves the
avoidance of precipitants and, if necessary,
What to tell the examiner the use of prophylactic medications.
Importantly women with migraine associated
Summarise the clinical case with aura, or over 35 years and without
The patient has a recent history of headaches, aura, should not use a combined oral
without red flag symptoms, which has continued contraceptive pill due to increased relative
in the setting of persistent opiate use and in the risk of stroke
absence of symptoms of another diagnosis. Benign intracranial hypertension should be
investigated with CT and lumbar puncture.
Investigations Management is weight loss and stopping of
Full neurological examination, fundoscopy and medication thought to be causing symptoms.
blood pressure check are indicated. Diuretics or steroids may be useful and
Neuroimaging is not indicated in patients surgical treatment can be considered, where
with a clear history of primary headache; there is concern about eyesight, with optic
without red flag features of a potential nerve sheath fenestration and shunting
secondary headache; and a normal neurological Giant cell arteritis investigated with ESR
examination. and CRP, biopsy, occasionally imaging with
In settings where imaging with CT or MRI angiography and treatment predominantly
is agreed, it is important to counsel the patient with steroids
with regard to what you are looking to exclude Headaches in pregnancy should be treated in
and warn them that such investigations may the first instance with paracetamol. MRI with
demonstrate incidental findings not related contrast venography should be considered
to their symptoms that may warrant further where concern relates to potential venous
investigation in of themselves. thrombosis
Case 155: Visual disturbance

Candidate information of establishing this in retrospect without the


luxury of examination when symptomatic.
Scenario Partial or total
As the registrar in the rapid-access TIA clinic, Transient or persistent establish the
you are asked to see a 73-year-old man who duration of the episode
has been referred by Accident and Emergency Sudden or gradual in onset
overnight having presented with an episode of Discriminating between monocular and
acute visual loss. Please take a history from the binocular visual loss can be invaluable in
patient. Inform the patient of your findings and considering the underlying cause. Monocular
inform them of your management plan. visual loss implies a disorder anterior to the
optic chiasm, likely affecting the structure of
Actor information the eye or the optic nerve. Transient binocular
Last night you noted deterioration in your vision visual loss suggests a more posterior process,
that occurred suddenly. You promptly presented involving the optic chiasm, tracts, or radiations,
to Accident and Emergency where you were or the visual cortex.
reviewed by a doctor who indicated you might Thereafter, consider partial versus total and
have suffered a mini-stroke. acute versus gradual visual loss.

Partial versus total visual loss


Prior to entering the room Causes of partial visual disturbance include:
The history of acute visual disturbance
may involve the discussion of a plethora of Stroke homonymous hemianopia
symptoms ranging from flashing lights and Pituitary tumour bitemporal hemianopia
floaters to blurring, disturbance of colour, or Retinal vascular occlusion horizontal
loss of vision. While the nature of the visual Glaucoma peripheral field loss
disturbance will doubtless provide sufficient Macular degeneration central field loss
grounds for discussion in of itself, be aware Migraine
that the underlying aetiology is likely to prompt Causes of total visual loss include:
discussion of larger themes in medical practice Acute:
such as diabetes, transent ischaemic attacks and Amaurosis fugax
stroke, or conditions predisposing to vascular Temporal arteritis
occlusions such as sickle cell disease. Thus, be Optic neuritis
sure to take cues from the station information Central retinal vein/artery occlusion
and the patient in tailoring your approach to the Papilloedema raised ICP
discussion. The strong candidate will succinctly
gather details of the visual disturbance before Gradual:
considering information relevant to the context, Cataract
such as risk stratification of cerbrovascular Glaucoma
disease, or diabetes control, for instance. Diabetic retinopathy
A typical history of painless monocular,
On entering the room, begin transient, total visual loss is consistent with
with open questioning amaurosis fugax and transient ischaemic
attack. The patient may describe the classical
Allow the patient to describe their symptoms, curtain descending across the visual field. The
initially without prompting, but be sure to symptoms should satisfy the criteria of TIA,
establish key features that will enable the lasting typically for a period of minutes, and
differential to be narrowed. In all cases of visual resolving completely within 24 hours. Where
loss, establish the following features: symptoms last more than 24 hours with no
Painful or painless residual disability, consider the diagnosis of a
Monocular or binocular visual loss non-disabling stroke.
although important, recognise the limitations
Case 155 Visual disturbance 307

examine the patient in full, looking for any


Open questioning suggests a evidence of neurologic deficit but also perform
transient ischaemic attack ophthalmoscopy to look for cholesterol emboli
and auscultate for carotid bruits.
With this diagnosis closed
questioning should focus on the Investigations
following Investigations are preferably performed within
1 week of symptoms. The ABCD2 score, if high,
Seek further evidence of carotid territory attacks may mandate inpatient investigations.
including hemiparaesis, hemisensory loss, or
dysphasia. If high risk (ABCD2 score >or crescendo
Clarify: TIA in whom vascular territory is uncertain)
diffusion-weighted MRI head
Whether aspirin was given at the time of the Carotid Doppler to look for atherosclerotic
symptoms plaques
Identify whether previous episodes possible. Blood pressure
Importantly, two or more TIAs in a week Risk stratification tests including fasting
constitute crescendo TIAs and correlates to a lipids and glucose
higher risk of stroke ECG and Holter monitor to look for atrial
Establish ABCD2 risk of stroke fibrillation and echocardiogram to rule out
cardiac thrombus
Explore concerns, ideas, and
expectations Management
Management aspects include lifestyle
Visual loss is a concerning symptom. The patient
modification:
may know the relevance of TIAs in relation to
stroke risk and may be concerned about the Wight loss
implications in relation to future disease. Stop smoking
Exercise
What to tell the patient
I suspect you have had a transient ischaemic Further reading
attack. The risk of stroke after a TIA in the first Johnston SC, Rothwell PM, Nguyen-Huynh MN, et al.
5 years is approximately 7% per annum. It is Validation and refinement of scores to predict
important to treat your modifiable risk factors, very early stroke risk after transient ischaemic
such as treating hypertension, stopping smoking attack. Lancet 2007; 369:283292.
and reducing your cholesterol, to prevent a Ferguson GG, Eliasziw M, Barr HW, et al. The North
stroke from happening in the future. I would like American Symptomatic Carotid Endarterectomy
to organise some investigation today in our clinic Trial: surgical results in 1415 patients. Stroke
to rule out common sources of problems that we 1999; 30:17511758.
may be able to treat and reduce your overall risk. European Carotid Surgery Trialists Collaborative
Group.Randomised trial of endarterectomy for
recently symptomatic carotid stenosis: final
What to tell the examiner results of the MRC European Carotid Surgery
Summarise the clinical case Trial (ECST). Lancet 1998; 351:13791387.
This patient describes symptoms consistent
with a transient ischaemic attack. I would
Case 156: Tremor

Candidate information tremor, exclude drugs, toxins or trauma as a


cause.
Scenario Body parts affected predominantly upper/
You are the registrar in the general medical lower limbs, but also consider the face
clinic. You are asked to see a 68-year-old retired Alleviating or exacerbating factors such as
university lecturer who is attending for a review caffeine and alcohol
following a recent hospital admission. An Narrow the list of differential diagnoses
accompanying letter from his General Practitioner based on the character of the tremor.
raises concerns over a tremor that has been Resting tremor, occurring with the limb
present for the last 18 months. Last month a fall supported from gravity:
resulted in a minor head injury necessitating a
short stay for observation, although the discharge Parkisnons disease low frequency, pill
summary made no reference to the tremor. rolling, unilateral classically of the hand
Please take a history from the patient. Inform the Drug-induced Parkinsonism
patient of your findings and inform them of your
Postural tremor, with the limb maintained in
management plan.
a position against gravity:

Actor information Essential slow, coarse or fine, symmetrical


tremor, usually involving the upper limbs but
Your GP has referred you to the medical clinic often also the head and voice
because of a tremor that has become apparent Dystonic
over the past year and a half. Your wife has also Drug induced
noted that you have been getting more forgetful Alcohol or drug withdrawal associated with
over a similar period. diaphoretic symptoms and hallucinations
Hyperthyroidism
Prior to entering the room Decompensated liver disease or
renal failure often in the context of
In assessing tremor as a presenting complaint,
encephalopathy
be mindful that the most common tremors
encountered in clinical practice are essential
Physiological fine, rapid tremor occurring
in healthy individuals commonly alleviated
tremor and those related to Parkinsonism.
or suppressed by alcohol
The strong candidate will take cues from the
scenario in considering the latter but not rush Intention tremor, during purposeful
to a presumptive diagnosis. The key to this voluntary movement:
station will be to evaluate the tremor leading
to a larger diagnosis and gauge the patients Cerebellar pathology alcohol/
insight and readiness for such news, employing demyelinating disease/CVA
warning shots where possible and arranging for
further investigation or review to ensure a safety Open questioning suggests a
net.
diagnosis of parkinsonism
On entering the room begin with With this diagnosis closed
open questioning questioning should focus on
For all tremors, consider categorising based the following
on those occurring at rest, related to posture, The Parkinons tremor classically manifests as an
or related to movement/intention. This will insidious onset, over a period of months, usually
itself discriminate between a wide range of after the age of 60 years, of a hand at rest. The
causes. Thereafter, consider a general review of legs, chin, lips and trunk may also be affected.
symptoms and evaluate: Typically described as pill-rolling, a coarse
Duration of symptoms and acuity of onset tremor at a frequency of 57 Hz, in a unilateral
insidious or abrupt. In any case of abrupt onset distribution. Symptoms may progress, with time,
Case 156 Tremor 309

to become bilateral and may disappear with worry and suspicion, or the pragmatic approach.
action. I dont mind the tremor; I think it is just my
Complete the diagnostic triad by enquiring nerves. My leg shook like that for years. My fall was
specifically about concomitant bradykinesia and just a silly lack of concentration crossing the street.
rigidity. Rigidity often precedes tremor in many Surely they have nothing to do with each other.
patients. Look for functional difficulties such as Theres nothing serious going on is there doctor?
changes in co-ordination relating to problems My father experienced something very
with every day tasks like buttoning shirtsleeves similar, as a family we never talked about it and
or tasks requiring a finer degree of manual I hoped it wouldnt happen to me but I fear the
dexterity. worst doctor.
Ask about balance impairment and changes Is this Parkinsons doctor? I have done plenty
in gate, posture, or recurrent falls. Commonly of reading around the subject but I am confused
falls occur when attempting to turn quickly or as to how you can confirm the diagnosis. What
with unexpected loss of sure footing with an do we need to do? I am keen to start treatment
inability to react due to loss of postural reflexes. early if it will make things better.
A collateral history will be invaluable,
particularly in relation to signs of dementia.
Family members or friends might have
What to tell the patient
commented on the patients facial expression Your symptoms are suggestive of Parkinsons
(hypomimia) and flat mood or signs of disease, a type of degenerative disease that
depression. Interestingly, sleep problems may affects the movement pathways in the brain
be an early sign of Parkinsons disease, even and typically gives rise to the kind of tremor you
before motor symptoms have begun. Some of describe. Its treatment can be difficult and the
the common sleep problems for Parkinsons disease tends to progress with time. I will give
patients include: you some more information to read about the
disease and its complications and bring you
Insomnia back to discuss all this at length.
Nightmares Where appropriate discuss falls prevention
REM sleep behavior disorder acting out and consider an early discussion about cognitive
dreams during sleep decline, personality changes, depression,
Restless leg syndrome lethargy and fatigue, financial planning,
Sleep apnoea insurance, and disability applications.
Review the patients medications and exclude
neuroleptics and anti-emetics that may cause What to tell the examiner
akineticrigid syndrome, mimicking the early
signs of bradykinesia in Parkinsons disease. Summarise the clinical case
Be sure to get a feel for the social history and This patient describes symptoms consistent
the patients functional capacity as previously with an akineticrigid syndrome, with unilateral
alluded to. Ask about the patients ability to carry hand tremor, rigidity and bradykinesia, all
out activities of daily living unaided establish suggestive of Parkinsons disease. Diagnosis is
the home set-up, support network and any predominantly clinical. Tremor without other
pre-existing level of care package. It is important characteristic Parkinsons symptoms suggests
that you are seen to assess, the functional and either early disease or the potential for an
psychosocial effects of the tremor. alternative diagnosis.
Spend time discussing alcohol intake and
its relation to symptoms. Also review smoking Investigations
history and pack years in view of the established
Differentiating between Parkinsons disease and
inverse association with Parkinsons.
Parkinsonism is largely clinical, assessing the
relative importance of drugs or family history
Explore concerns, ideas, and and balancing the context of examination
expectations findings.
Neuroimaging can be of limited use:
Allow the patient to express their ideas and
concerns as to the potential diagnosis. Prepare MRI brain to look for cerebrovascular disease,
for a variety of responses from the patient that multiple infarcts and presence of Lewy bodies
could conceivably develop themes of denial, (if Lewy body dementia is suspected)
310 Chapter 6 History taking (station 2)

SPECT scan in cases of uncertain COMT inhibitors (entacapone)-blocks L


Parkinsonism to differentiate between dopa metabolism
Parkinsons disease and atypical Dopamine receptor agonists (pramipexole
parkinsonisms such as multiple system and ropinirole)
atrophy and progressive supranuclear palsy Apomorphine pump
and other causes (unassociated with loss of
Patients may require anti-emetics for nausea,
dopaminergic neurons) such as essential
atypical neuroleptics and even cholinesterase
or dystonic tremor, medication induced
inhibitors for cognitive impairment or
Parkinsonism or tremor.
hallucinations.
Further discussion around Parkinson-plus
Management syndromes is possible. These include:
Levodopa is the gold standard for motor
symptoms used with a peripheral DOPA Multiple system atrophy extra pyramidal
decarboxylase inhibitor such as carbidopa. symptoms and autonomic failure
Long-term therapy may lead to reduced Progressive supra nuclear palsy extra
efficacy, on-off (fluctuations) and pyramidal symptoms, postural instability,
dyskinesias. falls from failure of downward gaze and
MAO-B inhibitors (selegiline) as dementia
monotherapy or adjuncts for early onset Lewy body dementia Parkinsonian features
motor fluctuations with dementia

Case 157: Established multiple sclerosis

Candidate information team, offering patients an opportunity to talk


openly about their condition, seek reassurance,
Scenario and gain support for often seemingly
A 40-year-old woman with known multiple insurmountable challenges. Aim to cover a
sclerosis attends the neurology clinic for review. review of symptoms, an assessment of disease
As the SHO covering the clinic you have been progression, evaluation of any response to
asked to take a history from the patient before treatment and challenges related to side effects.
reviewing the case with your consultant. The
patient has been attending the clinic for many
years and is keen to hear your opinion on her
On entering the room begin with
case. Summarise your understanding and offer a open questioning
management plan. It can be useful in such a scenario, where the
patient returns on a regular basis, to enquire as
Actor information to their health and circumstances since the last
review. This will encourage the discussion to
You have presented for your yearly review. develop around themes driven by the patient.
That said, all patients with chronic disease, and
Prior to entering the room particularly those with MS where the diagnosis
is typically made early in the third or fourth
Chronic disease, diagnosed early in life,
decade, develop close relationships with their
predisposes to a multitude of problems. Be
healthcare team. Thus, consider introducing
prepared to encounter issues relating to disease
yourself as a new member of the team, and
progression and treatment, often complicated
asking them to give you a summary of their
by anxiety and depression. Reviews in clinic
case.
represent an important link to the healthcare
Case 157 Established multiple sclerosis 311

Attempt to explore the details surrounding Enquire specifically about current symptoms
their diagnosis, including symptoms, tests and and resultant level of disability, commonly:
results as the patient understands them.
Establish the type of MS, and in doing
Visual problems such as eye movement
trouble, diplopia or oscillopsia
so, clearly demonstrate to the examiner an
understanding of the distinction between
Spasticity and/or tremor with mobility issues
relapsingremitting and progressive disease:
Pain and paroxysmal features
Bladder disturbance including urgency,
Complete or near-complete resolution of retention and recurrent UTIs
symptoms between attacks: relapsing Impotence
remitting MS Fatigue
Development of steady progression without Depression
resolution between exacerbations after an Memory and thinking
initial course of relapsingremitting disease:
Screen past medical history, specifically
secondary progressive MS
asking the patient if they ever experienced
Steady progression without resolution of
seizures or have a pre-existing diagnosis of
symptoms between exacerbations: primary
epilepsy. Clarify current medication and
progressive disease
treatments. Be sure to gather a focused social
Attempt to highlight the variation in history including assessment of the patients
symptoms between attacks, consistent with support network, package of care, and coping
indiscriminate demyelination throughout the strategies. Screen for signs of depression where
central nervous system. Patients are likely to appropriate, even if thought unlikely. Be alert to
have experienced a range of symptoms, albeit excessive alcohol use.
subtle in many cases, but commonly one or
more of:
Explore concerns, ideas, and
Visual disturbance associated with optic expectations
neuritis, often pain and partial loss of vision
of one eye Be prepared for varying themes, introduced by
Blurred vision or monocular blindness due to the patient, which may change the direction of
ocular palsy/inter-nuclear ophthalmoplegia the consultation. Three common examples are
Loss of colour vision discussed here: childbearing, urinary problems
Sensorimotor disturbance and treatment issues.
Assymetrical spastic paraparesis My partner and I are keen to start a family;
Paraesthesia MS commonly affects young women of child
Dysaesthesia of limbs bearing age necessitating clear information from
Thermal dysaesthesia the candidate surrounding pregnancy related
questions:
Enquire as to the effect of hot weather, hot MS should not prohibit patients from
baths or saunas on symptoms as related to becoming pregnant
Uhthoffs phenomenon. There is no evidence that MS is inherited
Symptoms may in fact improve during
Open questioning suggests a pregnancy in some patients however there is
thought to be a high risk of relapse postnatally
diagnosis of multiple sclerosis Current recommendations relating to disease
modifying drugs such as a-interferon suggest
With this diagnosis closed stopping prior to conception where possible.
questioning should focus on I cant seem to pass enough urine, constantly
the following feel like I want to go, and keep suffering from
There is likely to be a central theme that infections. A friend from my support group
will become the focus of the discussion in self-catheterises, but Im just not sure that I
this scenario. Moving from open to closed could cope with that. Involvement of the spinal
questioning will require tact given the sensitive column often results in bladder dysfunction,
nature of themes that will range from pregnancy ranging from mild urgency or hesitancy, to
or self catheterisation in young women to partial retention or incontinence, and frank
worsening symptoms with poor prognosis and incontinence in advanced disease. Treatment
disease progression in the older patient. of recurrent infections due to retention is not
312 Chapter 6 History taking (station 2)

uncommon and can have a significant bearing symptoms that were most troublesome to the
on quality of life, including sexual appetite patient.
and wellbeing. Self-catheterisation can offer
patients control and reduce infections when Investigations
used with appropriate antibiotic cover. Self-care
The diagnosis of multiple sclerosis is
can be facilitated through specialist nurses and
predominantly a clinical one. However imaging
community teams to offer guidance and support
and investigations can be used to support the
initially.
diagnosis, in line with the McDonald criteria.
Poor response to treatment: I feel like
A diagnosis can be made on the basis of 2 or
things are just getting worse, are there other
more relapses and 2 or more objective clinical
treatments that might help? Be sure to consider
lesions.
not only disease modifying drugs and the use
of steroids for acute flares, but also treatment of
MRI evidence of cerebral, classically
periventricular, or cord plaques. There is no
symptoms in of themselves and, importantly,
consensus on the role for serial imaging with
depression. This may necessitate a discussion
MRI to monitor the response to treatment or
of management with the patient other than
progression of disease
predominantly with the examiner see below.
Evoked potentials relate to electrical signals
generated in the central nervous system in
What to tell the patient response to sensory stimulation of peripheral
The discussion with the patient will very nerves. Visual evoked potentials are slowed
much depend upon the themes that form the Lumbar puncture with CSF analysis
focus of the history. Naturally, where there oligoclonal IgG bands
is any doubt as to the established diagnosis
then this should be reflected in the proposal of Management
further investigation, although this would be Be clear as to the rationale for treatment of
more likely in an appropriate scenario of new acute flares versus disease modifying drugs and
diagnosis. Be mindful that diagnosis treatment of specific symptoms.
relies on a thorough history to establish Acute relapses:
discrete neurological manifestations,
which, in the first instance, are often subtle. Steroids used to reduce severity and length
Thereafter, MRI brain and spinal cord to of acute relapse.
demonstrate plaques consistent with MS. Disease modifying drugs:
Where there is doubt, evoked potentials and/or
cerbrospinal fluid testing for oligoclonal bands Beta interferon decreases the frequency
is prudent. of relapses and delays disability but does
A hollistic approach to the ongoing not alter disease progression. No role in
management will be key. The strong candidate progressive disease
will try to offer a plan going forward that Glatiramer analogue of a component of
will incorporate specialist referrals where myelin basic protein thought to act as a decoy
necessary, community support from specialist for the immune system
nursing teams, adequate information re Mitoxantrone immunosuppressant used in
support groups and sources of additional progressive disease
information, and timely subsequent clinic Natalizumab monoclonal antibody,
review. Changes to, or initiation of, drug directed against adhesion molecule
treatment should not distract from the wealth a4-integrin, reducing leukocyte passage
of alternative discussion points. across the bloodbrain barrier
Symptom control:
What to tell the examiner Spasticity baclofen or dantrolene,
Summarise the clinical case botulinum toxin and physiotherapy
Succinct summary of the patients diagnosis,
Tremor clonazepam, gabapentin
profile of symptoms and frequency of relapses,
Fatigue amantadine, selegiline
current management and ongoing issues.
Bladder disturbance anti-cholinergics, e.g.
oxybutinin or tolterodine, intermittent self-
Thereafter, targeted discussion of salient
catheterisation or it may be that the switch
features from the history. Try to steer the
from self-catheterisation to long-term or
discussion towards topics of interest, such
supraprubic catheterisation is necessary
as novel treatments, or focus on specific
Case 158 Fever in the returning traveller 313

Impotence sildenafil Neuropsychological referral


Depression tricyclic or SSRI, consider
Multi-disciplinary team involvement is
counselling referral
important and may require input from a wide
Pain and paroxysmal features
range of healthcare professionals (neurologist,
carbamazepine, gabapentin
GP, specialist nurse, occupational therapy,
Visual problems ophthalmology referral
physiotherapy, counselling team).
(exclude acute flare)

Case 158: Fever in the returning traveller

Candidate information On entering the room begin with


Scenario open questioning
You are the medical registrar on call. This Explore the patients symptoms and establish
40-year-old man has presented to Accident the onset and duration of symptoms in relation
and Emergency with febrile illness for the to travel history. Prompt the history where
last 3 weeks and has been referred for further appropriate with a generalised review of
investigation. Please take a history from the symptoms.
patient. Inform the patient of your findings and Take a full travel history including short
management plan. stays and airport transfers considering the
possible infections associated with different
areas:
Actor information Tropics or sub-Saharan Africa (malaria)
You have recently returned from an extended Africa (rickettsia, ameobiasis, katayama,
trip to Zambia and have been feeling unwell visceral leishmaniasis)
since your arrival back in the UK. You did not South/Southeast Asia (enteric fever)
take malaria prophylaxis. Middle East (brucellosis)
Others (leptospirosis, trypanosomisasis and
viral hemorrhagic fever)
Prior to entering the room
Definitions for pyrexia of unknown origin Enquire about associated symptoms:
(PUO) vary, but it refers to a fever, persistent for Fever with rash (dengue, HIV, ricketsia,
a number of weeks with the cause not found schistosomiasis )
despite appropriate initial investigations. Fever with jaundice (leptospirosis, viral
Consider the differential diagnosis: hepatitis, yellow fever)
Malaria Fever, headache, myalgia, arthralgia and
Enteric fever (typhoid/paratyphoid) malaise (malaria)
Viral hepatitis Constipation/diarrhoea, dry cough (enteric
Ricketsia fever)
Dengue Rash, night sweats, weight loss, diarrhoea
(often seen in HIV)
The most common causes of PUO are:
Ask about accommodation, confirming
Malignancy whether it was rural or urban. Go on to
TB enquire about the activities undertaken, with
Endocarditis associated risk in mind, including sexual
Gallbladder disease encounters, drug use and exposure to foreign
HIV infection healthcare systems.
Auto-immune disorders
314 Chapter 6 History taking (station 2)

Open questioning suggests a breathing problems or liver problems. Do you


have any questions?
diagnosis of malaria
With this diagnosis closed What to tell the examiner
questioning should focus on Summarise the clinical case
the following This patient gives a history suggestive of malaria.
Establish pre-travel precautions, such as The main differential would be enteric fever
anti-malaria prophylaxis (note that correct (although he has not returned from an endemic
prophylaxis with full adherence does not area), HIV seroconversion and viral hepatitis.
exclude malaria), and the use, if any, of a I would proceed to a full examination, to look
mosquito net or insect repellents. for evidence rash, eschar, hepatosplenomegaly,
Exclude complications: lymphadenopathy or jaundice although
examination findings might be non-specific.
Confusion, seizures or a reduction in
Glasgow coma scale: cerebral malaria Investigations
Dark red or black urine: blackwater fever
Investigations would include:

Explore concerns, ideas, and Blood film and rapid diagnostic test for
malaria (regardless of whether or not malaria
expectations prophylaxis has been taken)
If the patient has traveled to an endemic area (or FBC lymphopenia common in viral
if they know people who have had the condition) infection and typhoid, eosinophilia often
they may be concerned about malaria. Allow seen in parasitic or fungal infection, and
them to vocalise their concerns and address thrombocytopenia in malaria, dengue, acute
these appropriately. HIV, typhoid
Serum save
HIV testing in all patients
What to tell the patient Hepatitis A,B,C
I suspect the symptoms you are describing Urinalysis: haemoglobinuria in malaria,
may be related to an infection you might proteinuria and haematuria in leptospirosis
have caught whilst abroad, namely malaria. Blood cultures
It is a tropical disease that is transmitted by Stool and urine cultures
mosquitoes through bites. You can become G6PD (especially before primaquine treatment)
sick very quickly but it is entirely treatable. We
can confirm the diagnosis with blood tests, Management
which will also direct the appropriate treatment.
The management of malaria depends on
However it is safer for you to be admitted to
whether the causative parasite is falciparum
hospital to undergo the investigations and
or non-falciparum. Chloroquine followed by
receive the correct treatment and we can
primaquine for non-falciparum. Quinine and
monitor you for any complications such as
doxycycline for uncomplicated falciparum.
Case 159: Sore, stiff hands

Candidate information Systemic features

Scenario Fever
You are the registrar in rheumatology clinic. This
Weight loss
40-year old woman has been referred by her GP
Tiredness
with a history of sore, stiff hands. Please take a
Breathlessness
history from the patient. Inform the patient of
your findings and your management plan. Open questioning suggests a
diagnosis of rheumatoid arthritis
Actor information
You have been referred to clinic with worsening
With this diagnosis closed
pain and stiffness in your hands. This is affecting questioning should focus on
your ability to carry out simple tasks such as the following
opening jars.
Important negatives include:

Prior to entering the room Psoriatic arthritis: rash


Reiters syndrome: urethritis, conjunctivitis
Consider the differential diagnosis of sore, stiff Reactive arthritis: post dysentery
hands: Ankylosing spondylitis: low back pain and
Arthritis stiffness, asymmetrical peripheral arthritis
Rheumatoid (hips, knees, shoulders)
Osteoarthritis Gout: mono-articular arthritis, usually big
Psoriatic toe or knee
SLE Establish current medication. Is there relief
Crystal Arthropathy Gout from NSAIDs suggestive of inflammatory
arthritis? Does the patient take any diuretics,
On entering the room begin with especially thiazides, which may precipitate gout?
Ask about extra-articular manifestations:
open questioning Eyes
I understand from your GP you have been
troubled with stiff hands. Could you tell me Keratoconjunctivitis sicca
about the problems that you are experiencing? Scleritis: painful red eye
Screen for a rheumatological history. Episcleritis: uncomfortable red eye
Establish the nature of pain, stiffness, swelling, Cardioplumonary
deformity, disability and systemic illness. Be
clear in relation to the onset, duration and Pericarditis
timing of symptoms, particularly early morning Pleuritic chest pain or shortness of breath
stiffness. Interstitial fibrosis
Unless already covered, enquire about: Neurological
Other joint involvement:
Carpal tunnel
Symmetrical PIP, MCP, wrist, knee, ankle, Peripheral neuropathy
MTPs commonly seen in RA
C-spine, thoracolumbar, first CMC, DIP,
patellofemoral commonest in OA Explore concerns, ideas, and
First MTP commonest in gout expectations
Associated joint symptoms Are they particularly concerned about the lack
of function? The patients job is of particular
Stiffness, especially morning stiffness
importance given the relative dependence on
Swelling
manual dexterity in both the work place and
Deformity
activities of daily living.
316 Chapter 6 History taking (station 2)

What to tell the patient Rheumatoid factor


I suspect you have a condition called rheumatoid
Anti-CCP antibodies
arthritis, an inflammatory disease that affects the
FBC-normocytic anaemia, raised WCC and
raised platelets with acute phase response)
joints. I need to examine you and organise some
tests to confirm the diagnosis. Depending on the
ESR may be elevated
tests and your symptoms we will discuss treatment
Plain radiographs for diagnosis of erosions.
Radiological changes in affected joints are
which often involves painkillers and often disease-
loss of bone density on either side of the
modifying drugs to suppress disease activity.
joint, soft tissue swelling, erosions, loss
of joint space, deformity (subluxation or
What to tell the examiner complete dislocation)
Synovial fluid aspiration: straw coloured with
Summarise the clinical case raised WCC (not done routinely)
The patient gives a history suggestive of
rheumatoid arthritis as evidenced by morning Management
stiffness lasting more than 30 minutes for more
Important points in the patients management
than 6 weeks, involving 3 joint areas, including
include:
both wrists.
Mention if there is evidence of extra-articular Symptomatic relief with adequate pain relief,
manifestations. reduction of stiffness and swelling
I would examine all the joints, looking for Early referral to the multi-disciplinary team
boggy synovial thickening and joint effusions as DMARDs within 12 weeks of disease onset
well as rheumatoid nodules. methotrexate remains first line
Biological medications such as infliximab or
Investigations etanercept
An appropriate investigative strategy would
include:

Case 160: Back pain

Candidate information Traumatic


Infective osteomyelitis, TB
Scenario Inflammatory ankylosing spondylitis,
You are the SHO in the rheumatology clinic. discitis
A 30-year-old man has been referred with Neoplastic
insidious onset lower back pain and stiffness. Metabolic osteoporosis, osteomalacia
Please take a history from the patient and Degenerative osteoarthritis
discuss a relevant management plan. Vascular aortic aneurysm/dissection

Actor information On entering the room begin with


You have noted lowed back pain and stiffness, open questioning
which has been worsening recently. You cannot Ask about the onset of the pain. Sudden onset
remember any accidents or trauma that could back pain can be trauma-related, due to disc
have caused the back pain. lesions or pathological fracture. Gradual pain
may suggest degenerative disease.
Prior to entering the room Consider exacerbating/relieving factors,
such as:
Consider the differential diagnosis of back pain,
which includes:
Case 160 Back pain 317

Aggravation on movement/relief by rest in The patient is young and male and has no
mechanical red-flag symptoms. The pain is worse in the
Worst after rest: inflammatory morning. The most likely diagnosis is ankylosing
Worse on cough/strain: intervertebral disc spondylitis.
prolapse
Morning stiffness: ankylosing spondylitis
Explore concerns, ideas, and
Consider the patients age as a predictor of expectations
likely aetiology:
This young man has potentially debilitating
1530: traumatic, ankylosing spondylitis symptoms, which have been chronic. Explore
(especially in males) functional limitations and impact with potential
3050: degenerative, prolapsed disc, disability.
malignancy
>50: degenerative, osteoporosis, myeloma
Older women: osteoporosis What to tell the patient
From what you tell me, I suspect your back
Screen for associated symptoms: pain is caused by a condition called ankylosing
Abdominal pain, dysuria, menorrhagia spondylitis, where there is inflammation in
Night sweats, fever in infective the spine. We will need to carry out some
Sensorimotor symptoms/bladder involvement investigations to confirm the diagnosis. Once
(degenerative/cord compression) the investigations are done we will see you again
Sciatica (degenerative) in clinic to advise on specific treatment options.
In the interim I would like to offer you suitable
The past medical history may offer important analgaesia and make a pre-emptive referral to
context for current symptoms: physiotherapy.
Malignancy
Menopause (osteoporosis) What to tell the examiner
Gastrectomy (osteomalacia)
Diabetes (infective) Summarise the clinical case
Immunosuppression (infective) I suspect the diagnosis is ankylosing
spondylitis, given the young age of the male
patient, sacroiliac joint involvement and
Open questioning suggests stiffness. I would complete my diagnosis after
a diagnosis of ankylosing examination to look for features of ankylosing
spondylitis spondylitis such as exaggerated thoracic
kyphosis, compensatory hyperextension of neck,
With this diagnosis closed loss of lumbar lordosis, fixed flexion of knees,
compensatory flexion of knees.
questioning should focus on
the following Investigations
Systems review should include questions about FBC: normochromic, normocytic anaemia
extra-articular manifestations/complications: ESR: may be elevated
Iritis
Rheumatoid factor: negative
Plantar fasciitis
Radiology: spinal radiographs may show:
sacroilitis, squaring of lumbar vertebrae,
Hip/knee involvement
bamboo spine. Further imaging may be
Crohns/UC
required
Rashes: psoriasiform dermatitis
Peri-/myocarditis
Management
Consider Red Flag symptoms, including: Principles of management include:
<20 years or >55 years Conservative and symptomatic
Weight loss Exercise, analgesia
Pyrexia DMARDs for peripheral, but not axial
Painful spine in all movements symptoms
Localised bony tenderness Biological therapy
Continuous non-mechanical pain
Case 161: Established diabetes

Candidate information of diabetes and side effects of current


medications.
Scenario
You are the endocrinology SHO. This 60-year- Diabetic control
old woman is attending the diabetes clinic for HbA1c levels and home fingerprick glucose
annual review of her type-2 diabetes. Please levels
take a history from the patient. Discuss any History of hypoglycaemia attacks: frequency,
concerns in relation to current disease control, severity and awareness
explaining any proposed changes to her Hyperosmolar hyperglycaemic state (HHS)
management. Weight and trend of weight loss or gain

Actor information Complications of diabetes


You were diagnosed with type 2 diabetes Vascular disease
10 years ago. You have come to clinic today
for your annual review. Despite an absence Exertional angina or breathlessness
of symptoms, your GP tells you that the blood Intermittent claudication
tests you have had done recently mean that you Cardiovascular disease may be clinically
need tablets to control the sugar levels. You silent
were started on medication 1 year ago and had Kidneys
another tablet added 2 months ago.
Known microalbuminuria/proteinuria
Prior to entering the room Haematuria or other urinary symptoms/
retention
In a patient with established diabetes, seek to:
Retinopathy
Optimise glycaemic control
Assess modifiable risk factors Visual disturbance
Screen for complications Last retinal screening
Any retinal laser phototherapy

On entering the room, begin Neuropathy


with open questioning Sensorimotor neuropathy: numbness,
Explore the diagnosis, clarifying the initial tingling and burning. Include issues relating
symptoms and subsequent management. to diabetic feet such as non-healing minor
Be clear on the progress of the disease and injuries, painless ulceration, deformity
reasons necessitating changes to medication requiring podiatry input
and control of current symptoms. Attempt to Mononeuritis multiplex: III, VI nerve
gauge compliance not only with medication involvement
but with modifiable risk factors relevant to tight Amyotrophy: painful, wasted quadriceps
glycaemic control including diet, alcohol intake, or pelvifemoral muscles. Ask for difficulty
and exercise. climbing stairs
Autonomic neuropathy: orthostatic
hypotension, urine retention, erectile
Open questioning suggests a dysfunction
diagnosis of poorly controlled Ask about the drug history including the
and/or complicated diabetes current diabetes regimen. Is the patient having
any side-effects from their medications? Are
With this diagnosis closed they on appropriate medications to lower
their cardiovascular risk, such as aspirin, ACE
questioning should focus on inhibitors and statin medications?
the following The social history should include an
Closed questioning should focus on symptoms assessment of the patients smoking history. It is
relating to diabetic control, complications also important to clarify their driving status.
Case 162 Heat intolerance and weight loss 319

Explore concerns, ideas, and seems to be developing peripheral neuropathy.


I would like to perform a full body examination,
expectations including a formal neurological examination.
This is the patients annual review. Have there
been any new developments in the past year that Investigations
the patient is concerned about? Are there any
Appropriate investigations include:
ongoing and/or unresolved issues from previous
clinical reviews? Bed side tests: blood pressure, urinary
dipstick for proteinuria/microalbuminuria
What to tell the patient Blood tests HbA1c, renal profile, liver
function (look for raised transaminases
Where concern relates to poor glycaemic control, which may suggest non-alcoholic fatty liver
explain I see you have had diabetes for quite a disease) and a fasting lipid profile
while now. I am concerned that your diabetes is Urine protein:creatinine ratio to assess for
not well-controlled as you seem to be developing proteinuria
some of the recognised complications. It is Fundoscopy
important to try to achieve consistent sugar
control to prevent these complications from Management
progressing further. Therefore, I would like to
proceed to examining you and then organise The management of patients with diabetes
some tests. My impression is you might need is provided through a multi-disciplinary
to go to the next step of treatment, with the approach.
consideration of insulin. In the interim, I cannot General health promotion in diabetes
stress the importance of helping yourself by includes promoting exercise, weight loss, a
being more physically active, losing weight, balanced diet, and good glycaemic control.
eating a healthy diet and stopping smoking. It Regarding glycaemic control, if the patient
is very important to tell you doctor or GP if you is already established on metformin and
start developing hypoglycaemic symptoms, these sulfonylurea, consideration of a third oral agent
might interfere with your ability to drive. (thiazolidinedione or dipeptidyl peptidase
4 inhibitor) is possible or injectable agents
(insulin or GLP-1 receptor agonists).
What to tell the examiner Ensure the patient is on appropriate
anti-hypertensive medications, such as ACE
Summarise the clinical case inhibitor or ARB.
This patient seems to have poorly controlled
diabetes on two oral anti-diabetic agents and

Case 162: Heat intolerance and weight loss

Candidate information yourself wearing very few clothes even when


others were wrapping up. Although you are happy
Scenario that you are saving on central heating you are
You are the SHO in the endocrinology clinic. concerned because your GP told you they think
This 45-year-old woman presents with heat you may have some problems with your thyroid.
intolerance and weight loss over the past
5 months. Please take a history from the patient. Prior to entering the room
Inform the patient of your findings and inform
them of your management plan. Consider the differential diagnosis of weight
loss, which includes, but is not restricted to, the
following:
Actor information
You have become concerned that you have lost
Malignancy, cardiac failure, respiratory
disease, malabsorption, renal or liver failure
weight recently. Through the winter you found
320 Chapter 6 History taking (station 2)

Hyperthyroidism, diabetes, Addisons disease Is there a family history of auto-immune


TB/HIV thyroiditis? Is there a history of medullary
Depression, anorexia thyroid cancer that might raise your suspicions
of MEN2?
Systems review:
On entering the room begin with
open questioning Cardiorespiratory: symptoms of heart failure
in thyrotoxic cardiomyopathy, angina, atrial
The patient gives you a 5-month history of fibrillation
weight loss of 2 stone. She has a good appetite Ophthalmology: visual acuity, colour vision
and a good dietary intake. She also describes loss
heat intolerance, sweating but no night sweats. Skin: pre-tibial myxoedema, onycholysis,
vitiligo
Open questioning suggests a
diagnosis of hyperthyroidism Explore concerns, ideas, and
expectations
With this diagnosis closed
Allow the patient time to inform you of any
questioning should focus on particular concerns they have. Is there one
the following particular symptom that is worrying them
Screen for systemic manifestations of more than others? For example, tremor, which
hyperthyroidism: may be considered as relatively minor, may
be considered by the patient to be the most
Tremor debilitating.
Palpitations, dyspnoea, angina
Diarrhoea
What to tell the patient
Hair loss
Menstrual irregularity Your symptoms are suggestive of an over-
Anxiety, insomnia active thyroid gland. There can be many causes,
Dysphagia, dyspnoea, hoarseness: invasion although the commonest is antibodies that you
of malignant lesion have made against your thyroid. I would like
to examine you next and order some tests to
Important negatives: help establish the diagnosis before I propose
Polydypsia and polyuria in diabetes treatment.
Anorexia, malaise, nausea and vomiting,
diarrhoea in Addisons (usually cold What to tell the examiner
intolerance)
Change in bowel habit, PR blood in GI Summarise the clinical case
malignancy I suspect this patient has hyperthyroidism.
Episodic chest tightness, pins an needles, I would proceed to examine her and look for
sweating, abdominal pain, vomiting, syncope evidence of hyperthyroidism such as goitre, fine
in phaeochromocytoma tremor, atrial fibrillation and associated features
such as ophthalmopathy.
Past medical history:
Cardiorespiratory disease, renal or liver Investigations
failure may all lead to weight loss Investigations would include:
Other auto-immune conditions: Addisons,
diabetes, pernicious anaemia, vitiligo: may Thyroid function tests: In hyperthyroidism
point to polyendocrine syndrome or one would expect an undetectable TSH with
autoimmunity elevated free T3 and T4
Drug history:
Antithyroid peroxidise and TSH receptor
antibodies
Lithium and amiodarone may precipitate Technetium-99 radioisotope scan to
thyroid disease differentiate Graves from nodular goitre in
Radiation exposure is a risk factor for benign appropriate cases
and malignant thyroid disease
Self-medication
Case 163 Diabetes insipidus 321

Management Thyroid suppression: Carbimazole (side


effects: agranulocytosis, headache, rash,
Treatment options in hyperthyroidism include:
alopecia) or propylthiouracil
Symptom control: propranolol Partial thyroidectomy
Radioiodine

Case 163: Diabetes insipidus

Candidate information || Hypokalaemia


|| Hypercalcaemia
Scenario || Tubulo-interstitial disease
As the endocrinology registrar you have been Hereditary (X-linked)
asked to review a 47-year-old man referred Excessive fluid intake
by his GP with with polyuria and polydipsia. Psychogenic
When assessed by his GP in the community, he Drug-induced from anticholingergics
also mentioned problems with his vision and Drugs, such as diuretics
you note that he has also been referred to the
neurology clinic for this complaint. Please take
a history from the patient. Inform the patient
On entering the room begin with
of your findings and discuss your management open questioning
plan with the examiner. The patient is likely to complain of excessive urine
production often accompanied by frequency of
Actor information micturition and excessive thirst, feeling dry no
matter how much fluid intake they have. This may
You have been experiencing worrying symptoms disrupt their daily routine and lead to irritability,
recently, including increased thirst and blurred feeling run down and generally unwell.
vision. You are concerned because your job as Is there a diurnal pattern to the excessive
an accountant requires extended periods of urination? While prostatic disease is relatively
concentration, however you have been suffering unlikely in a 47-year-old man, all things being
from a lack of sleep due to the need to pass equal it is important to screen for prostatic
urine during the night, and are consequently symptoms.
struggling during the day and falling behind
with clients and deadlines.
Open questioning suggests a
Prior to entering the room diagnosis of diabetes insipidus
Consider the differential diagnosis of polyuria With this diagnosis closed
and polydipsia:
questioning should focus on
Diabetes mellitus
the following
Diabetes insipidus
Cranial Check for visual symptoms in pituitary tumours
|| Idiopathic (ADH deficiency or evidence of anterior pituitary hormonal
production) imbalance such as symptoms of thyroid,
|| Posterior pituitary lesions endocrine, or menstrual problems.
(craniopharyngioma/pineal gland Past medical history:
tumours)
|| Cranial surgery
Head trauma, head surgery, brain tumours
|| Head trauma
Diabetes
Nephrogenic Chronic kidney disease, recent hospital
admission with sepsis and haemorrhage all
|| Chronic kidney disease
predispose to ATN
|| Drugs (lithium, demeclocycline)
322 Chapter 6 History taking (station 2)

Malignancy and hyperparathyroidism cause Investigations


chronic hypercalcaemia which can cause
Investigations would include:
polyuria (nephrogenic diabetes insipidus)
Cardiac history: may implicate fluid Blood tests:
balance management and preclude use of Renal function and electrolytes for renal
vasopressin in treatment failure, hypernatraemia or hypokalaemia
Drug history: Drugs to specifically ask about
Glucose
in the drug history include:
Paired urine and serum osmolality (also
seen the urine for urinary sodium and
Therapeutic diuretics specific gravity)
Opiates (inhibit ADH secretion) If the biochemical picture suggestive
Lithium/demeclocycline of diabetes insipidus (urine specific
Anti-cholingergics (stimulate thirst) gravity <1.005 with urine osmolality
Nephrotoxic drugs (may precipitate tubular <200 mOsm/kg, random plasma osmolality
necrosis which can result in severe polyuria) >287) then proceed to a water deprivation
test. In diabetes insipidus water deprivation
Take a full family history. Nephrogenic increases plasma osmolality and sodium
diabetes insipidus is rarely inherited. with no change in urine osmolality. In central
diabetes insipidus. Exogenous ADH increases
Explore concerns, ideas, and urine osmolality >50%. In nephrogenic
diabetes insipidus exogenous ADH has little
expectations or no effect
From the candidate instruction it is impossible MRI brain to look at the pituitary gland
to tell what the patients particular concerns (if
any) will be. The polyuria and polydipsia may Management
be of secondary importance to him that the
Management of central diabetes insipidus
visual symptoms. The impact upon his ability to
includes:
work is likely to be of significance and demand
discussion in relation to speed of treatment and Treat the underlying cause
possible options to improve symptoms. He may Medication:
also be concerned in relation to his ability to Hormonal: desmopressin or vasopressin
drive due to the deterioration in his vision. (avoid the latter in coronary artery disease
or give nifedipine)
What to tell the patient Nonhormonal agents: chlorpropamide,
carbamazepine, clofibrate, diuretics
Your symptoms are compatible with a condition (thiazides), and NSAIDs (indomethacin
called diabetes insipidus, which is very different though limited efficacy)
from diabetes mellitus, the common form of
diabetes. It results from a deregulation in the Management of nephrogenic diabetes
water level system. If it is left untreated it can insipidus includes:
lead to severe dehydration and shock. We need
to run some more tests to confirm the diagnosis
Conservative management: normal thirst
mechanism can self-regulate.Withdraw
and provide treatment according to the cause.
offending drugs. Optimise renal impairment
Medication: non-hormonal agents
What to tell the examiner In an emergency:
Summarise the clinical case Replace urinary losses with dextrose and
The patient gives a history of recent onset polyuria water or other hypo-osmolar fluid
and polydipsia with symptoms of dehydration on Avoid hyperglycaemia
a background of visual disturbance suggesting Avoid volume overload
central diabetes insipidus possibly from a pituitary Avoid rapid correction of hypernatraemia.
tumour such as a craniopharingioma. Reduce Na by 0.5 mmol/L every hour
Case 164: Weight gain

Candidate information the patient to take control of consultations, and


become more demanding. Often patients will
Scenario be very well informed and in possession of vast
You are the registrar in endocrinology clinic. A mounts of detailed knowledge pertaining to
65-year-old woman has been referred by her GP their disease. As such you must be clear on the
with concerns relating to weight gain. She is well scope of the history and attempt to establish a
known to the respiratory service with chronic shared understanding with the patient as to the
restrictive airways disease but recently missed limitations of time and prioritise the aspects of
her annual review, although she was seen in the the case that require more detailed discussion
cardiology clinic 1-month prior. Please obtain from the outset.
a history, an examination is not required, and
discuss your thoughts with the patient including On entering the room begin with
plans for further investigation or management.
open questioning
Invite the patient to give you an overview of
Actor information her recent symptoms. Where appropriate use
You were diagnosed at the age of 45 years with the patient information to open up details of
idiopathic pulmonary fibrosis. Treatment has the case while maintaining an open approach
proved problematic and you have required I understand that you are normally reviewed
long periods of high-dose steroids, which in the respiratory clinic, can you tell me more
have offered significant benefit and reduced about that. Place the symptoms in context of the
symptoms of breathlessness. Your most recent past medical history.
cardiology review was reassuring with no Seek to quantify the weight gain. Screen for
evidence of heart failure. The weight gain has additional symptoms and attempt to narrow the
been progressively worsening over 3 months differential. Common causes of weight gain, and
and is particularly noticeable around your associated symptoms, include:
tummy. In fact your legs are very thin. You
have also noted your skin to be of poor quality Endocrine
despite liberal use of moisturiser and you have
developed prominent stretch marks. While Cushings hirsutism, acne, stretch marks,
you are not so concerned with regard to your thin skin, depression, easy bruising,
appearance, worryingly your exercise tolerance impotence, muscle weakness
has been impacted. The GP first referred you Hypothyroidism tiredness and lethargy,
to a cardiologist fearing that it may be fluid weakness, cold intolerance, anorexia, dry
retention but the doctor who reviewed you skin, constipation, menorrhagia
in clinic seemed more concerned about your Polycystic ovarian syndrome hirsutism,
steroids. obesity, irregular menses, diabetes

Fluid retention
Prior to entering the room Cardiac failure exertional symptoms
Patients with chronic diseases often pose including chest pain and breathlessness,
particular challenges, for a number of reasons. peripheral oedema, orthopnoea, and
The history can often be extensive, demanding paroxysmal nocturnal dyspnoea
an ability to steer the conversation in order to Renal failure polyuria, lethargy, peripheral
obtain the most relevant information. Achieving oedema
this without seeming dismissive of information Liver disease ascites
upon which the patient places great emphasis,
but which in reality is of limited value, can be
difficult. The potential for complications and
Physiological
side effects increases complicating the case Increased intake with reduced exercise
further. The relative increase in exposure to Pregnancy amenorrhoea, nausea, urinary
medical professionals will also likely empower frequency, breast tenderness
324 Chapter 6 History taking (station 2)

Take a detailed drug history paying particular The GP mentioned the possibility of
attention to exogenous steroid. Be sure to side effects from steroids but was reluctant
remain flexible with regard to the differential to change my medication due to the nature
diagnosis and be seen to screen for other and severity of the fibrosis and current
medication with the potential for weight gain: breathlessness:
Lithium and amiodarone, may cause Does this mean I need to stop steroids, and if
hypothyroidism so how will that impact upon my fibrosis?
Calcium channel blockers may lead to If I stop the steroid, will the weight gain
peripheral oedema be reversible or can I expect symptoms to
Anabolic steroids, growth hormone continue to get worse?
Anti-psychotics such as olanzapine which
commonly increases appetite
What to tell the patient
From what you tell me, I suspect that the
Open questioning suggests weight gain you have experienced is due to a
a diagnosis of corticosteroid condition called Cushings syndrome, caused
by the steroid used to treat your fibrosis. I would
excess like to arrange for some tests to confirm the
diagnosis. It will be important to get the input
With this diagnosis closed of the respiratory team in making adjustments
questioning should focus on to your medication and consider alternative
the following management options.
Screen for the full constellation of signs and
symptoms commonly associated with Cushings What to tell the examiner
syndrome, including:
Cushingoid or moon facies Summarise the clinical case
Centripetal adiposity Demonstrate an understanding of the potential
Intra-scapular fat pad buffalo hump causes of Cushings syndrome, other than
Thin atrophic skin exogenous steroid. Be clear in relation to
Poor wound healing and easy bruising Cushings disease, causing Cushings syndrome
Abdominal striae as the result of excess pituitary production of
Weakness with evidence of proximal ACTH, usually secondary to a pituitary tumour.
myopathy Re-iterate that the patient did not complain
of symptoms of raised intracranial pressure of
Consider the potential for conditions
visual disturbance to offer clinical context.
commonly associated with prolonged exposure
to raised levels of corticosteroid include:
Investigations
Hypertension In suspected Cushings syndrome investigations
Insulin resistance or diabetes seek to confirm the diagnosis, rule out
Recurrent infections significant complications, and distinguish
Osteoporosis between the potential causes.
Mental disturbance including low mood and The diagnosis is largely clinical but it is
depression useful to confirm raised cortisol levels with
Despite the likelihood of an iatrogenic random testing and 24-hour urinary collection
cause, consider signs and symptoms that measuring free cortisol.
might suggest a pituitary adenoma including Urine dipstick can be useful in assessing
headaches and visual disturbance which is for glycosuria and excluding hypokalaemia on
classically a bitemporal hemianopia. Polyuria blood testing would be prudent.
and galactorrhoea may also be seen. Dexamethasone testing and ACTH
measurement:
Low dose dexamethasone suppression
Explore concerns, ideas, and testing demonstrates a failure to suppress
expectations cortisol levels
Be prepared to consider different themes Where the low dose dexamethasone test is
introduced by the patient. positive ACTH is measured. ACTH levels
Case 164 Weight gain 325

will be undetectable in primary adrenal Management


disorders, whereas ACTH will be high if
the cause is either Cushings disease or an
Management depends on the underlying
cause
ectopic ACTH producing tumour
Where the low dose dexamethasone test is
Metyrapone can be used to decrease plasma
cortisol levels
positive and the ACTH is raised, performing
a 48-hour dexamethasone test will confirm
Surgery can be considered for adrenal
adenomas, pituitary tumours, or tumours
Cushings disease as evidenced by a partial
with evidence of ectopic ACTH production
suppression of cortisol. Imaging will then be
required with CT or MRI brain for pituitary
disease
Chapter 7

Communication skills and


ethical scenarios
(station 4)
Case 165: Informed consent for an invasive
investigation or procedure chest drain
insertion

Candidate information fluid, resulting in both an improvement in your


symptoms and allowing for analysis to suggest
Scenario the likely cause of the fluid accumulation. By
You are the medical registrar on call. A 68-year- doing this, you have also explained the benefit of
old man has presented with a large unilateral the procedure, an important part of any consent
pleural effusion. Your task is to consent the form.
patient for chest drain insertion. You do not have Explain, in terms understandable to the
to take a detailed history. patient, what will happen during the procedure.
Firstly, we will mark a suitable entry point
Actor information using an ultrasound probe. Then, using sterile
technique and local anaesthetic, a needle will
You have been experiencing worsening be inserted through the skin and chest wall into
breathlessness for a few months now. It is the fluid-filled chest cavity. A tube is then placed
affecting how far you can walk. You presented into the chest, which will drain into a bottle by
to your GP today who has sent you up to the side of the bed. The tube will be stitched
hospital for urgent assessment. You have been securely into place and covered with a sterile
told you have fluid on the lung that needs dressing.
to be removed. The candidate will explain Explain the risks. GMC guidance states
the procedure to you and seek your consent. that the discussion of risks will usually involve
Underlying concerns relate to a previous discussion of:
smoking history, recent weight loss and change
in cough with blood in your sputum. Your father Side effects
passed away from lung cancer and you have Complications
refused to seek medical attention sooner due Failure of intervention to meet its desired aim
to apprehension surrounding the suspected
It is convenient to start any conversation
diagnosis.
about procedural risk with the statement Every
procedure has risks. Then go on to discuss
Approach to the case both the serious and common risks. You are
When seeking consent for an invasive not expected to know every risk for every
investigation or procedure in PACES, several medical procedure, but you should be familiar
principles apply: with common invasive investigations and
The procedure should be necessary. Obtain procedures that are considered to be common
a brief history, confirming current symptoms competencies of a medical registrar.
or previous investigation and results that are
consistent with a need for the investigation or Common competencies
procedure that you are consenting for.
The patient should have capacity to give Examples of procedures with which candidates
consent for the procedure. Assessing the should be comfortable consenting for include:
patients ability to understand and make a Abdominal paracentesis
decision based on the information provided in DC cardioversion
your discussion is important. Formal capacity Knee aspiration
assessment may be required where there is Chest drain insertion with ultrasound
doubt based on your interaction. guidance effusion versus pneumothorax
Explain the need for the investigation or Central line insertion
procedure. The reason why we wish to insert Lumbar puncture
a chest drain today is because the symptoms
and previous tests you have had suggest that Summarise what has been said and ask the
the breathlessness you are experiencing is due patient if there is anything they want further
to a collection of fluid in the chest. By carrying explained and if they have any other questions.
out this procedure we aim to drain off the If the patient is happy to consent, tell them that
Case 166 The patient refusing to consent 329

you will fill out the consent form and ask them
treatment options in a way they can understand,
to sign it.
and respect their right to make decisions about
Where the patient has capacity, and refuses
their care. You should see getting their consent as
to consent based on inadequate information
an important part of the process of discussion and
relating to either the indication for, or the
decision-making, rather than as something that
practical steps involved with, the procedure,
happens in isolation.
seek to provide further sources of information.
Additional themes worth consideration:
If practical, allowing a period of contemplation
The anxious patient where reassurance fails,
with access to material such as pamphlets,
overriding concern regards risk, and consent
internet sites, nursing or senior colleagues is
is not obtained
appropriate. Signposting to support groups may
The confused patient where capacity to
also be worthwhile.
consent is in doubt requiring a formal
Be prepared for a discussion of the wider
capacity assessment
condition, such as:
The skeptical patient who has experienced
Causes of the effusion is this cancer complications in the past
doctor? I want general anaesthetic; I dont want to be
Recurrent effusions failure of treatment of awake for any of it. Explaining safe sedation
the underlying disease and anaesthesia
Duration of drain and need for inpatient stay
Further reading
GMC guidance General Medical Council (GMC). Consent: patients
You must work in partnership with your patients. and doctors making decisions together. London;
You should discuss with them their condition and GMC, 2008.

Case 166: The patient refusing to consent

Candidate information that your GP is concerned about cancer, but


are unwilling to undergo endoscopy, as you
Scenario are concerned that the discomfort will be
You are the gastroenterology SHO. You have intolerable. Despite any reassurances given to
been asked to consent a 67-year-old woman for you, you will refuse endoscopy, but are willing
upper and lower GI endoscopies to investigate to consider alternative investigations.
iron-deficiency anaemia in the context of
significant weight loss. Please obtain this Approach to the case
patients consent.
Introduce yourself to the patient and build a
rapport with the patient. What does the patient
Actor information understand to be the purpose of todays visit?
You are a 67-year-old woman who has been Explain to her what having an upper and lower
investigated by her GP for weight loss. You GI endoscopy entails and explain the risks and
have lost approximately 7 kg in weight in the benefits. Her consent should be sought: In this
last 6 months (dropping two dress sizes). Your scenario the patient will refuse consent.
GP sent you for blood tests and told you that Explore the patients reasons for withholding
you were anaemic. They also tested some of consent, bearing in mind that patients with
your stool and told you there was blood in it. capacity do not actually need to give a reason to
You have had a change in bowel habit recently, withhold consent.
tending towards constipation. You are aware
330 Chapter 7 Communication skills and ethical scenarios (station 4)

Assess the patients capacity by explaining potential investigation modalities including


to her the indications for the investigations capsule endoscopy and CT colonography.
and the potential implications of late diagnosis The patient should be invited to have a
of a GI cancer. It should be ensured that she conversation with the consultant, avoiding
understands, retains, and weighs up the the impression of punitive consequences to
information given to her. Ensure that she can her refusal, stressing the need to identify an
communicate back her refusal and reasons for alternate plan of investigation. Reassure her
doing so. that she will not be forced to undergo any
If the patient has capacity (and we must investigations or treatments with which she is
assume she does unless we have evidence to uncomfortable, and where possible alternative
the contrary) she cannot be forced to have the approaches will be explored. The fundamental
endoscopy. premise of the case is the underlying suspicion
The strong candidate will go on to seek to of cancer and the need to establish a diagnosis
find alternative ways to investigate the patients to facilitate treatment.
anaemia. Discuss with the patient other

Case 167: Consenting for a clinical trial

Candidate information Approach to the case


Scenario Introduce yourself to the patient and inform
them of the purpose of the consultation. As
This 60-year-old woman has recently suffered the Cardiology registrar I have been asked to
an ST-elevation myocardial infarction. As the meet with you today to discuss the possibility
cardiology registrar, you have been asked to of you participating in a clinical trial for a new
consent her for a double-blind, randomised medication to treat high cholesterol in patients
control trial for a new anti-lipid drug. The who have recently had a heart attack. I would
primary end point is reduced mortality over like to tell you a bit about the drug and the trial
5 years. The protocol states that should she to see if you would be interested in taking part. If
consent, she would receive either the trial you have any questions or concerns at any stage,
drug or placebo once daily for the 5 years please feel to interrupt me.
of the study. The drug is to be added to her Make an early statement that reassures the
current medication. Side effects in phase patient that there is no obligation for her to
2 trials included headaches and occasional participate. Let me say from the outset, that
gastrointestinal upset. Your task is to discuss participation in this study is entirely voluntary
participation in the trial with the patient and and if, at any stage, you decide that you do not
seek her consent for involvement in the trial. want to take part that will be absolutely fine. Let
me also reassure you that if you decide not to
Actor information consent to participation in the trial it will have
You have recently been hospitalised for a heart absolutely no effect on your care and you will
attack. You have been asked to meet with the still receive the best medical treatment we can
doctor to discuss taking a new medication provide.
as part of a trial. You are not sure what this Provide the patient with some background
will involve and would like some further information regarding the medication and the
information. Assuming all your concerns are condition it is designed to prevent/treat. There
addressed, you are happy to participate in the is a constant desire to develop new drugs to treat
trial. and prevent heart attacks more effectively and
Case 167 Consenting for a clinical trial 331

therefore help people to live longer. We know someone taking it may have a side-effect that
that having higher cholesterol puts people at has not been noted before.
higher risk of heart attack and there are several Re-iterate to the patient that, if they decide
drugs that help to lower cholesterol. These have not to consent to involvement in the trial, such a
been shown to help in patients who have had decision will have no effect whatsoever on their
heart attacks by reducing the risk of further heart current and future care. It is vitally important
attacks and death. that patients understand this, otherwise
Inform the patient of the aim(s) of the trial they may feel they are being pressurised into
(primary end-point). We know that the drug participation.
reduces cholesterol so now we want to see if it Summarise to the patient the information
has a beneficial effect on life-expectancy. You that has been covered. They should go on to
would take tablets every day for 5 years, coming ask them if they have any further questions or
to see us in the clinic every few months for check concerns.
ups. Ask the patient if they consent to being
Explain the structure of the trial. Should you involved in the trial. Whatever their decision,
agree to participate, you will be given tablets to the candidate should repeat their decision
take every day. These will either be the trial drug back to them and tell the patient that this will
or a placebo (a tablet which has no medication be documented in their notes. This acts as
in it whatsoever). The study is known as a a safety net so all parties can be sure of the
double-blind, randomised control trial. The decisions that have been made. Thank you for
double blind bit means that neither you nor I consenting to participate in the clinical trial. I
will know whether you are taking the actual drug will document your decision to give consent in
or the placebo. The randomised bit means that your notes. If required or requested, a period of
the decision whether you get the actual drug or contemplation should be offered.
the placebo is entirely random, thus preventing Conclude by asking the patient to sign the
any bias coming into selection. trial consent form and offer the patient written
Explain the potential benefits. The potential information on the trial.
benefit to you is that you have a 50:50 chance Additional themes worth consideration:
of taking a drug which we believe will have a
positive effect on your cholesterol and therefore The patient in whom issues arise surrounding
on your life-expectancy. suitability. For instance concerns
Clearly lay out any potential draw-backs and surrounding compliance with current
include a discussion of side-effects. There is medication and as such potential for non-
also a 50:50 chance that you will be taking an adherence with trial
extra tablet every day which contains no drug. Patient refusal on the grounds of concerns in
We know from previous research on this drug relation to side effects from medication
that people taking it can experience headaches, The patient propositioning to ensure
joint pains, or abnormalities in liver function. As placement in the treatment arm of the trial
with any drug, there is also the possibility that and related ethical dilemmas
Case 168: Assessing capacity

Candidate information two criteria for having capacity an ability to


understand the information they are given and
Scenario the ability to weigh it up.
You are the on-call SHO for the medical wards.
You have been asked by the nurse in charge
Criteria for capacity
of the Care of the Elderly ward to review an
82-year-old man, who is refusing antibiotics to For a person to have capacity to make decisions
treat a lower leg cellulitis. Please speak to the regarding their own care, the following four
patient, assess his capacity to refuse treatment, criteria must be met: The patient must be able to
and seek to resolve the situation. understand the information they are given, retain
that information, weigh up the information and
Actor information communicate their decision back to the clinician.
You have been admitted for cellulitis of Always bear in mind that paragraph 64 of the GMC
your right leg. You are refusing to accept guidance on capacity (2008) states that: You must
your intravenous antibiotics, which were work on the presumption that every adult patient
started 5 days ago. Your reasoning being that has the capacity to make decisions about their
the intravenous access through which the care...
antibiotics are being given has required frequent
replacement (only disclose this if asked directly It is important to understand why the patient
why you are refusing antibiotics). You are not is refusing treatment. Ask the patient directly
confused. why, despite understanding the importance
of the antibiotics, he is still refusing to accept
them. He will reply that the recurrent need to
Approaching the case insert new cannulae is causing him considerable
Although this case is primarily about assessing distress. It is important to elicit this piece of
capacity, there are also several other important information as it now affords the candidate
elements that will be assessed, including several options for resolution of the scenario, as
communication skills and consideration of discussed later in the case.
causes of potential confusion. The key principles Do not forget to screen for reversible causes
in approaching scenarios of this kind are for confusion. It will be prudent to request the
discussed below within an appropriate method patients notes, observation charts and most
of how to approach the case. recent blood results.
Introduce yourself to the patient and Check that the patient has retained the
explain your role and the reason for your information that was previously given to him.
interaction. I have been asked by the nurse Forgive me for asking again, but may I check
in charge to have a chat with you about the that you are clear as to the reasons why we are
antibiotics you have been prescribed. Would prescribing you intravenous antibiotics, can
that be acceptable? you repeat my explanation back to me? He will
Summarise the situation, as you understand give a suitably accurate account. With this, the
it. Ive been told that you do not want your patient meets the third criterion for possessing
intravenous antibiotics. Is that correct? The capacity the ability to retain information.
patient will respond in the affirmative. Ask the patient if, despite you explaining the
An assessment of the patients capacity is key. need for intravenous antibiotics, he still refuses
The reason why we would like you to take the to receive them. By his response, he meets the
antibiotics is to treat the infection you have in final criterion for having capacity the ability to
your leg. Without the antibiotics, the infection communicate ones decision.
will may not get better and indeed is likely to get After having decided that the patient does
much worse. Do you understand such reasons indeed have capacity to refuse treatment,
for concern? The patient will respond that he your task then changes to how to resolve the
does understand. He realises the importance situation. There can be no hard and fast rules
of the antibiotics. At this stage the candidate with this, but suitable tactics could include:
can now say that the patient has met the first
Case 169 The patient who lacks capacity 333

Reassurance with regard to further cannula Alternatively, offer an assessment of his


insertion. Discussion with regard to an current progress including an examination,
appropriate choice of anatomical site in review of observation chart and blood tests,
addition to options for securing the access and consider the potential for conversion to
or protecting it with bandaging. Involvement oral antibiotics
of the nursing staff to demonstrate a
Stress to the patient that the final decision
concerted effort within the team to promote
in relation to his treatment lies with him. The
longevity of the access and reduce the need
patient will state that he is happy to receive oral
for replacement would be sensible. Offer
antibiotics.
your services as a senior member within the
Additional themes worth consideration:
team with relative skill and experience as
compared with junior doctors, nursing staff The confused patient acute confused state,
or phlebotomist, should this be an issue with delirium, secondary to infection/sepsis
the patient The confused patient co-morbidity of
advanced dementia

Case 169: The patient who lacks capacity

Candidate information dissatisfaction with the current package of care


received, and the consequence of mobility
Scenario issues on the ability of your husband to return
One of the patients on your ward has been home. You have been considering the option
admitted with a non-resolving diabetic ulcer on of a nursing home and this deterioration in his
his heel, which has failed to respond to several health has highlighted your inability to cope.
courses of intravenous antibiotics. The advice
of the vascular MDT meeting has been that an Approach to the case
amputation is the most appropriate intervention
in this case. As well as other co-morbidities, the Important principles to remember when dealing
patient has a history of vascular dementia. It has with a patient who lacks capacity are:
been judged by your consultant that the patient Decisions should be made that serve to act in
lacks capacity to make decisions regarding his the patient's best interests
own care. Please discuss the plan of action with Treatment initiated in the patient's best
the patients next of kin. interests must be the least restrictive
It is important to keep lines of
Actor information communication open with the patient's next
You are the wife of a 60-year-old man who has of kin. This acts both to reduce the likelihood
had diabetes for many years. He has had a of conflict and also to give an idea of what
problem with an ulcer on his foot for the past 18 the patient's wishes may have been, had they
months and has been in hospital four times for possessed capacity
prolonged courses of intravenous antibiotics. Ensure that if any legal documentation exists,
He has been diagnosed with vascular dementia in the form of an advance directive or relating
and requires considerable care and supervision. to power of attorney, these are given due
The doctor will inform you that they feel the consideration
best course of action is for an amputation of the To commence the consultation, introduce
affected foot. Initially you are against this plan yourself to the patients relative. They should
due to concerns about whether your husband confirm their name and their relationship to the
would want it. Underlying concerns relate to patient. Clarify their status as the next of kin.
334 Chapter 7 Communication skills and ethical scenarios (station 4)

Ask the patients relative what they station is to seek the relative's consent. As the
understand the current situation to be. This patient has been judged to be incapacitated,
is an effective tactic in all communication any interventions that are carried out on his
situations as it gives you an idea of what level behalf are done so in what is deemed to be
of understanding the patient/relative has of the his best interests, as decided by the medical
scenario and allows the candidate to judge how team. The candidate is trying to foster
much information is required and at what level agreement with his relatives and defuse any
to pitch it at. conflict, but is not seeking their consent
Establish a joint understanding of the Is amputation the least restrictive
seriousness of the situation and the current intervention? The candidate information is
thinking in relations to management options. deliberately ambiguous. Based solely on the
Explain that, due to the persistence of the information provided, one could argue that
infection in the heel ulcer, demonstrated by options, which are least restrictive, include
both recurrent admissions and poor response further courses of intravenous antibiotics, or
to treatment, there is significant concern that it to perform limited tissue debridement. This
will be impossible to eradicate the infection with is obviously a complex case that provides
antibiotics alone. There is additional concern plenty of scope for discussion between the
in relation to the potential for complications, candidate and the examiner
primarily sepsis, and that such an eventuality
would likely carry a poor prognosis, even death.
Go on to explain that a meeting of a range Mental Capacity Act 2005
of specialists involved in the patients care have (England and Wales)
come to a joint decision that the best treatment The following points are adapted from the GMC
option would be to perform an amputation of guidance (2008) on the Mental Capacity Act. This
the foot. The benefits of this would be to remove states that actions taken under the act must abide
all of the infected and necrotic tissue as a means by several principles:
of eradicating the infection. It would be hoped Unless it has been established otherwise, the
that the stump and scar would subsequently patient should be assumed to have capacity
heal. At this stage pause and ask if the patient's All practicable steps should be taken to aid a
relative has any questions. patient to become able to make a decision on
Explain that all the team realise that their own care
amputation is a major step and that it would not Just because a person makes unwise
be considered if it was not felt to be a necessary decisions does not make them unable to
step to manage a difficult and serious situation. make decisions on their own care
Seek to identify and address the relatives views Any action or decision made under the remit
now that the aspects of the case above have been of the Mental Capacity Act 2005 must be
discussed with them. done in the patient's best interests
Below are some additional points for The action taken must be considered to be
consideration, which may form part of a the least restrictive on the patient's rights
discussion with the examiner: and freedom of action.
Ethics committee: In most PACES scenarios,
any conflict will be resolved in the course
of the discussion. In the event of persistent
disagreement, suggestion to the examiner Further reading
of referral to the hospitals clinical ethics General Medical Council (GMC). Consent: patients
committee is a sensible plan and doctors making decisions together. London;
Be clear that the relative is unable to GMC, 2008.
refuse the treatment suggested. It would Department of Constitutional Affairs. Mental
be a misunderstanding of the candidate Capacity Act 2005. Code of practice. London;
instructions to think that the point of the The Stationary Office, 2007.
Case 170: Breaking bad news to a relative

Candidate information overloading them with information. Gauge what


they want and need to know. Often it is best
Scenario to impart small amounts of key information
You are the cardiology registrar. You have been repeatedly to ensure mutual understanding as
asked to meet the wife of a 70-year-old man who to the severity of the situation. Where necessary
was admitted with an extensive anterior STEMI. employ the use of warning shots to prepare a
His background includes active metastatic relative for bad news often indicated by non-
prostate cancer, which has so far been refractory verbal cues.
to treatment, and vascular dementia with a Summarise the situation and attempt to
background MMSE of 19/30. He has had all paint the picture of what is an irretrievable
appropriate treatments but is in severe cardiac situation. It should be stated very clearly that
failure and is not expected to survive his current it is the opinion of the medical team that the
hospital stay. Please talk with his wife and patient will not survive this current admission.
inform her of the prognosis. Pause. It is essential that the patients relative
is given time to absorb the information. Often,
Actor information an uncomfortable but necessary silence will
ensue, demanding the confidence not to talk or
Your husband was admitted after having an continue the discussion too hastily.
episode of chest pain at home. He has dementia Do not be afraid to encourage the relative to
and has been unwell for an extended period discuss how they are feeling and express their
of time with prostate cancer, for which he has emotions.
been told the treatment is not working. You are Encourage questions relating to what
concerned about your husbands chances of happens next. Gauge the ability of the relative
surviving his heart attack but are prepared for in any given scenario to continue with difficult
the worst. Your children both live overseas and discussions, which may include:
they are unable to travel home for another few
days but are hopeful that they will be able to say The decision to resuscitate
goodbye and are looking for such an assurance Withdrawing care
from the doctor. Brainstem death testing
Organ harvesting
Approach to the case Requesting a postmortem

Breaking bad news often arises in the context of Look to give some form of indication as to
a discussion with the next of kin. In the majority prognosis and the expected speed of deterioration
of cases this will involve a patient in whom the but avoid committing to specifics. The relative
prognosis is poor or indeed is dying. Often the may be keen to resuscitate or escalate care until
initial consideration is one of confidentiality her children have arrived. The discussion will
in that the discussion with the relative may require an emphasis on the patients best interests
occur where the patient is incapacitated and focus on developing hope tempered with
and as such cannot consent to disclosure of realism. Her husband may survive long enough
information. The basic premise is that it is in for her children to see him but that if he does not,
the patients best interest to discuss the case then this is something they should prepare for and
openly with appropriate members of his family. an emphasis on quality of life is prudent.
An appreciation of confidentiality should be Summarise, check understanding and offer
made clear however, to satisfy the examiners to see her again with her children or other
expectations a statement along the lines of relatives if they would find it useful. Offer your
Your husband is very ill, and while ordinarily I condolences where appropriate.
would look for his consent to discuss the details Additional themes worth considering:
of the case with you, I think it would be in his best The relative that demands more be done to
interests for you to understand the situation he is treat the patient. Difficulties establishing a
in to make decisions and support him at this time. ceiling of care and managing expectations
Establish the relatives current understanding Chronic conditions where death is not
of the patients current condition. Be prepared imminent but there is likely steady decline
to fill gaps or clarify statements that are either without the potential for improvement or
incorrect or lack appropriate detail but avoid reversibility
Case 171: The decision to resuscitate

Candidate information Incorporating her terminology may engage her


in discussion and help build a rapport.
Scenario It should be made clear that, given the
You are the general medical SHO working in patients weight of co-morbidities and his
the team responsible for the care of an 80-year- current poor condition, the consultant in charge
old man with a background of Alzheimers of his care and the rest of the medical team are
dementia, chronic kidney disease and ischaemic all in agreement that, in the event of the patients
heart disease. He is dependent on care for all heart stopping it would not be in her fathers
activities of daily living and lives in a nursing best interests to try and revive him.
home. He suffered an ischaemic stroke 7 days Be clear that despite the best possible
ago and subsequently developed a severe medical care, her father is not expected to
hospital-acquired pneumonia. He is not survive and that the end could come soon.
expected to survive this admission and over However, although a DNAR has been put in
the past 24 hours has experienced a significant place, treatment is still ongoing.
deterioration. Your consultant has asked you to
talk to the next of kin about his decision, taken
Official guidance for resuscitation
during the morning ward round, to sign a Do
Not Attempt Resuscitation form. decision making
In 2007, a joint statement was released by the British
Actor information Medical Association, the Resuscitation Council (UK),
You are the patients daughter and next of kin. and the Royal College of Nursing. Entitled Decisions
You are aware of the stroke but until today you Relating to Cardiopulmonary Resuscitation it
were unaware of the severity of the pneumonia addresses various issues surrounding resuscitation.
and are concerned that he appears much worse The document notes that Healthcare professionals
than when you last visited. The most important have an important role in helping patients to
aspect of his care from your perspective is that participate in making appropriate plans for future
he does not suffer, having had a difficult and care in a sensitive but realistic manner, making clear
traumatic experience of prolonged end of life whether or not attempted CPR could be successful.
care leading up to the death of your mother. Helping patients to reach a clear decision about
The doctor will discuss with you the current their wishes ... should be regarded as a marker of
prognosis and explain the decision in relation good practice ... The joint statement notes that
to resuscitation, with which you agree, but decisions made regarding resuscitation must
you are concerned about a lack of palliative comply with the Human Rights Act 1998. When
care involvement in the case. Based on your discussing the communication of DNAR decisions to
experience with your mother in the same patients, the joint statement remarks that clinicians
hospital, your knowledge of the system appears should Offer patients as much information as they
to be greater than that of the doctor you are want, provide information in a manner and format
speaking to and this is a source of frustration which patients can understand, answer questions
and worry if not openly acknowledged. honestly and explain the aims of treatment.

With tact and compassion, you must


Approach to the case explain that in this case, given the extensive
Introduce yourself and confirm that the woman co-morbidities and limited functional reserve,
to whom you are speaking is the patients next CPR is not considered to be in the best
of kin. interests of the patient and would therefore be
Explore her current understanding of his inappropriate. It would be prudent to further
underlying state of health, recent events and discuss ceilings of care and clarify while
current condition. treatment is ongoing, any further requirement
Thereafter, having gauged the daughters level such as respiratory support would only be
of insight, succinctly summarise the case for considered, at most, in the context of the ward
her. Attempt to mirror some of her statements. environment, and not with an admission to
Case 172 Withdrawing (life-dependent) care 337

HDU or ITU. This is likely to prompt discussion for agreement, you are not seeking the next of
in this case of end-of-life care. kins consent.
Attempt to incorporate an explanation
that CPR is classified under law as a medical
The involvement of relatives/friends in intervention, and as such, the final decision
resuscitation decisions regarding its use lies with the medical team. Even
When considering the discussion of DNAR in scenarios where disputes arise, it is important
decisions with relatives or those close to the to be seen by the examiner to be putting yourself
patients, note that the joint statement says that on the side of the patient and working with the
this is not only good practice but is also likely next of kin to reach agreement. Remember, the
to be a requirement of the Human Rights Act solution is not to argue with the next of kin, it is to
(Articles 8 and 10) [and] the Mental Capacity educate them on why CPR is inappropriate in the
Act 2005 (England and Wales). Section 9.2 of given situation.
the Joint Statement deals with Adults who If any disagreement persists, offer to arrange
lack capacity, have neither an attorney nor an
a meeting with the consultant in charge of the
advance decision but do have family or friends.
patients care and be seen to involve the palliative
The statement notes that the final decision rests
care team and senior nursing staff on the ward.
with the senior clinician in charge of the case (the
Often the matron on the ward is a useful link to
patients consultant). It goes on to say that the
services available and in coordinating care.
views of those close to the patient should
Close the scenario by briefly summarising
be sought ... to determine any previously
what has been discussed to ensure shared
expressed wishes.
understanding.

In some cases, the patients relative may Further reading


try to insist that the patient receives CPR in
Decisions relating to cardiopulmonary resuscitation.
the event of a cardiac arrest. This changes the
A joint statement from the British Medical
focus of the consultation. In such cases, you
Association, the Resuscitation Council (UK) and
must be clear that while you are seeking to
the Royal College of Nursing. J Med Ethics. 2001;
explain the rational for a DNAR, and hoping
27:310316.

Case 172: Withdrawing


(life-dependent) care
Candidate information to respond to intravenous antibiotics. Currently
she is in pulmonary oedema with hypoxia and
Scenario hypotension. She is anuric with evidence of
As the on-call registrar, you received handover acute kidney injury. Her inflammatory markers
prior to the weekend that a 92-year-old are worsening and she has become confused
patient on the care of the elderly ward had and drowsy. You feel that continuing treatment
been deteriorating over a period of weeks. The is not in her best interests given the irretrievable
consultant responsible for management set the situation. The patients co-morbidities include
ceiling of care as ward-based management. On dementia, chronic airways disease and
review there is evidence of multi-organ failure as heart failure. You have been asked to discuss
a result of advanced chest sepsis that has failed withdrawing care with her son.
338 Chapter 7 Communication skills and ethical scenarios (station 4)

Actor information and the poor response, continuing current


treatment is inappropriate. Ask the son how he
Your mother has been in hospital for the last
feels about what has been said. The purpose of
3 weeks with a chest infection that never really
such a discussion is not to seek consent but to
got better. The nurses called you this afternoon
establish from him, and the wider family, what
to tell you that her condition has worsened
the patients wishes might have been.
and that you should come to the hospital. Your
If the son asks why his mother has not been
understanding is that she is close to death. You
moved to intensive care, take the time to explain
wish to ensure that she does not suffer in any
that given her frail condition and concomitant
way however you are anxious about withdrawing
health problems, it is unlikely that a stay in
care. Your mother has always been a fighter
intensive care would benefit her. The concept
and would be reluctant to give up albeit her
of a lack of functional reserve is a useful one,
quality of life has been poor since her dementia
explaining that the patient is unable to mount
and heart failure worsened.
a sufficient response, and that this is the reason
for a lack of recovery as opposed to a failure of
Approach to the case treatment per se. As such it would be expected
Introduce yourself and confirm the identity of that intensive care might prolong life, but not
the patients relative and their relationship to the aid recovery and ultimately not be in the best
patient including status as next of kin. Explore interests of the patient.
the level of understanding of his mothers current In the event of conflict, the appropriate
illness and progress. This will give an indication as course of action would be to contact the
to whether he has insight into the seriousness of consultant on-call for the weekend and arrange
the situation. Summarise the case as appropriate. for the relative to meet them to discuss the
It should be explained to the patients son case. The principle to focus upon when the
that given the number of problems affecting his patient lacks capacity is the medical teams
mother and the number of organs that arent responsibility to act in what they perceive to be
working properly, the medical team feel that the patients best interests, taking into account
this is an illness from which the patient will not any advanced directives and the relatives view
recover. Time should be afforded to absorb such on what they understand the patient would have
information, check for understanding, and invite wanted.
questions to aid clarification. The case should conclude by discussing
Proceed to tell the son that the medical team the role of palliative medicine and end-of-life
feels that, given the current clinical condition care with the aim of making the patient as
comfortable as possible.

Case 173: Brainstem death testing

Candidate information Actor information


Scenario You are the mother of a 30-year-old who
sustained severe head injuries after being hit by
You are the intensive care SHO. A 30-year-old a car last week. He has been in the intensive care
man is currently a patient in your unit after unit on life support ever since. From the outset
being hit by a motor vehicle a week ago. He things did not look good and you are aware
has suffered severe diffuse traumatic brain of the grave prognosis. Over the past 24 hours
injury and is on a ventilator. He is not expected the consultant has informed you that they do
to survive and the team is preparing to carry not think your son will recover. They have told
out brainstem testing. You have been asked to you they will be doing some tests on his brain
discuss with the patients next of kin, his mother, (brainstem testing), which are expected to show
the details of brainstem testing. his dependence upon machines. You have had
Case 173 Brainstem death testing 339

limited access to the consultant in charge of Assess response to apnoea testing. Apnoea
your sons care and have found a large amount is confirmed when there are no spontaneous
of the information relayed by the SHO thus far to respiratory movements when the patient is
be technically complicated and in language that disconnected from the ventilator and the
you struggle to understand. You are seeking a PaCO2 is allowed to reach 6.65 kPa measured
clear explanation of your sons clinical state and by arterial blood gas testing
what brainstem testing actually means.
Brain stem death: time of death
Approach to the case One potentially confusing aspect of brainstem
Relatives will often struggle to come to terms death, and therefore one which will require
with the loss of a loved one where the removal careful explanation is the time of death once
of care is required, instead clinging to the brainstem testing is completed. Although
possibility of recovery. Individual cases will vary the patient is technically still alive until the
with content and emphasis, but the general key conclusion of the second repeat testing, upon
for success in this scenario is to use knowledge reaching a diagnosis of brainstem death, the
of the principles of brainstem death, combined time of death is considered as the time of
with effective communication skills, to explain conclusion of the first test.
what can be a complicated concept.
Definition of death
What is brainstem death?
There is no legal definition of death in the UK. A
Brainstem death is a clinical state where there is Code of Practice for the Diagnosis and Confirmation
total loss of brainstem function, resulting in an of Death recommends that death should be
apnoeic coma that, without artificial ventilation, defined as the irreversible loss of the capacity for
would result in death. consciousness, combined with irreversible loss of
the capacity to breathe. The document argues that
How do we test for brainstem death? since brainstem death produces the clinical state
Given that the diagnosis of brainstem death is set out above, brainstem death equates to the
a life-ending episode, it is not surprising that death of the patient.
there are specific rules regarding testing for it. The following are conditions under which
These are laid out clearly in A Code of Practice the diagnosis of brainstem death should be
for the Diagnosis and Confirmation of Death considered:
(Academy of Medical Royal Colleges, 2008). The patients condition is due to irreversible
A general algorithm for diagnosing brainstem brain damage for which the cause is known
death includes: The patient is deeply unconscious
Not influenced by drugs (medications or
Identify coma otherwise)
Exclude possible reversible factors Not due to primary hypothermia
(hypothermia, disturbances in metabolism, Not due to reversible endocrine, metabolic or
drugs) circulatory factors
Testing is done by two clinicians, both of Mechanical ventilation is necessary due to
who are 5 or more years post-full registration inadequate/absent spontaneous ventilation
with the General Medical Council and Be aware that where brainstem function remains
one of whom is a consultant. The length of intact, despite loss of cortical function, a diagnosis
time between the two episodes of testing is of persistent vegetative state is made.
regarded as a matter of clinical judgement
Pupillary light reflex, corneal reflex (take care
not to damage the cornea as it may be used
in transplant), and vestibulo-ocular reflexes Further reading
are absent Academy of Medical Royal Colleges. A Code of
Absent motor response within the cranial Practice for the Diagnosis and confirmation
nerve distribution [due to] stimulation of of death. London: Academy of Medical Royal
any somatic area in combination with No Colleges, 2008
limb response to supraorbital pressure and
an absent gag reflex are compatible with
brainstem death
Case 174: Organ donation and harvesting

Candidate information for whom organ donation is a non-starter, but


for those who will consider giving consent, will
Scenario act as a gentle introduction to the subject. Where
You are the intensive care SHO. A 30-year-old there is reluctance to engage in discussion, the
man is currently a patient in your unit after challenge will be to impress upon the relative
being hit by a motor vehicle a week ago. He has the time sensitive nature of the matter.
suffered severe diffuse traumatic brain injury The relative should be asked whether, to the
and is on a ventilator. He is not expected to best of their knowledge, the patient had ever
survive and the team is preparing to carry out expressed any views on organ donation and if so,
brainstem testing. The patient does not carry what were they?
a donor card and is not on the organ donor Explain the process of organ donation. If the
register. He does not have an advance directive. relatives were to give consent, after death was
You have been asked to talk with the patients confirmed with brainstem testing, they would be
next of kin, his mother, with a view to obtaining given a short time to spend with him, and then
consent for organ harvesting. You do not need to he would be taken to the operating theatre for
discuss the details of brainstem testing. harvesting/recovery of organs. Organs, ranging
from his heart, lungs, liver, kidney and corneas
Actor information could be used. His body would then be returned
to the family for burial.
You are the mother of a 30-year-old who Questions the relative may ask include:
sustained severe head injuries after being hit by
a car last week. He has been in the intensive care Will the body be disfigured? There will be
unit on life support ever since. From the outset scars, but they will not be visible with the
things did not look good and you are aware body presented in the standard way by the
of the grave prognosis. Over the past 24 hours undertakers (with a shroud)
the consultant has informed you that they do Will it delay the funeral? No. The process of
not think your son will recover. They have told harvesting should not ordinarily delay any
you they will be doing some tests on his brain funeral arrangements
(brainstem testing), which are expected to show Will all his organs be used? Organs will be
his dependence upon machines. The doctor harvested and decisions on their suitability
will ask you for consent for organ harvesting. for transplant will be taken on an individual
Your son had never expressed a view on organ basis
donation but you think he would have been in He was a smoker; does that mean he can't
favour of it. Before you give consent you have donate his organs? Not necessarily. Of
some concerns. As long as the doctor answers course, it may be that organ damage from
these concerns appropriately, you will give smoking may preclude transplantation of
consent. some organs, most obviously the lungs in
a heavy smoker. It is unlikely, however that
a 30-year-old would have such significant
Approach to the case smoking damage that would prevent lung
Ask the patients mother what has been transplantation
discussed with her in the last 24 hours to Will we get to know about who gets the
confirm that she understands that brainstem organs? Relatives are given a limited amount
testing is about to happen and is likely to of information about the recipient: age (by
confirm brainstem death. decade), gender, and the outcome of the
A discussion on organ harvesting should be transplant
instigated with a statement such as: I know it
is a difficult topic to talk about, but whenever a Additional themes worth consideration:
patient is considered as unwell as your son, we Possession of a donor card or member of
are obliged to talk to their relatives about the the organ donation register yet the relative
possibility of organ donation. Do you feel in a is refusing consent. The current position,
position to talk about this now? A statement which many would argue is unsatisfactory,
such as this will quickly identify those relatives is that even if the patient is in possession
Case 175 Requesting a hospital postmortem 341

of a donor card or has signed the organ the patient in whom death has occurred has
donation register or has expressed wishes an advance directive indicating their wish
to donate their organs, if the next of kin for organ donation. Advance directives are
refuses consent, organ donation often does legally binding
not occur. Candidates should note, however,
that the Human Tissue Act 2004 contains
provisions that make it lawful to take organs
Further reading
for transplantation where the deceased NHS Blood and Transplant (NHSBT) and the British
consented before his death. However it does Transplant Society (BTS). Guidelines for consent
note that good practice dictates that relatives for solid organ transplantation in adults.
are consulted Cheshire; NHSBT and BTS, 2011.
Next of kin refusing to accept death and
unwilling to discuss organ donation where

Case 175: Requesting a hospital


postmortem

Candidate information down her bed. When the doctor raises the
possibility of a postmortem you are reluctant.
Scenario You dont want your dead mother cut into. Ask
You are the general medical SHO. A 68-year- the doctor about what exactly the postmortem
old woman was admitted on Friday evening entails. You are also concerned about the
with a history of significant weight loss and funeral arrangements, as you are unsure how
confusion. The impression on the post- a postmortem will affect these. Providing the
admission ward round was of likely malignancy doctor addresses your concerns appropriately,
with unknown primary. On the ward round a you will give consent for a postmortem.
plan was made for various investigations to take
place after the weekend. On Sunday evening, Approach to the case
48 hours after admission, the patient suffered
a cardiac arrest and died. In the context of a Introduce yourself and begin by offering your
sudden deterioration without a firm diagnosis, condolences.
the consultant in charge has advised that a Ask the patients relative what they
postmortem would be advisable to identify the understand to be the recent course of events.
cause of death. You have been asked to speak to If necessary, they should be informed of any
the patients son to obtain consent. details they may not know/understand. Explain
that due to uncertainty regarding the exact
cause of death of their mother, the consultant
Actor information in charge of her care feels that a post-mortem
Your mother had been complaining of weight would provide invaluable information as to the
loss for approximately 3 months. You took her cause of death.
to see her GP last Friday as she had become Discuss with them any questions they may
increasingly confused. The GP sent her up to have about postmortems. Once the discussion
the hospital where you were told she required is complete, the relative should be asked a
admission for further investigation. On Sunday very clear question whether they consent to a
evening you received a telephone call from the postmortem.
nurse in charge of the ward to let you know Areas for discussion relevant to postmortems:
that she had passed away suddenly. Nobody
has been able to tell you exactly what she has Why do you need to do a postmortem? It
died of, but you overheard the nursing staff wont make any difference to the fact my
mentioning cancer when they were turning mother is dead, will it? It is important to
342 Chapter 7 Communication skills and ethical scenarios (station 4)

answer this question frankly and accept the I have read about scandals where hospitals
relative's point that it will not change the fact have kept peoples organs. How can I be
that his mother had died. It may however, sure this wont happen to my mum? If this
help give closure if they know exactly question is raised, it is important to accept
why she died. Much more a secondary that, yes, there have been problems with this
consideration is that a postmortem may in the past, but after extensive investigation,
shed light on a particular condition and strict rules and regulations have been put
thus potentially provide information to the in place to ensure it does not happen again.
medical profession to help other patients in Unless the relative is specifically asked for
the future permission, no organs will be retained.
Will the scars disfigure her? A useful way of After examination, they will be returned to
answering this question is to tell the relative the body. It may be that a request is made
that scars from the postmortem will not be ahead of the postmortem for an organ to
visible with the body dressed in a funeral be retained. This may for various reasons,
shroud. You can then ask the relative if either because it is thought to be relevant to
they want to know further details about the the cause of death, of for relevant research.
incisions (many will not). If they do, tell them Written consent would be sought before the
that there is an incision made in the front event. There is a half-way house in which
to allow access to the chest and abdominal consent may be sought to keep the organ for
organs. To examine the brain, a scalp incision a limited period, after which it would either
is made at the back, again, not visible with be returned to the patient's next of kin or
the body on its back disposed of by the hospital
What about the funeral arrangements? A
postmortem typical takes place within 2 or Mandatory postmortems
3 days, after which the body is released to the
Consent for a postmortem is not required
funeral director. Thus, there should not be a
when a coroner orders one to be carried out.
significant delay to the funeral (bear in mind
Examples of such scenarios are when the death
that different religions have specific beliefs
is sudden or unexpected; if there are suspicious
about funerals regarding timing so be careful
circumstances; or if their doctor had not seen the
not to be blas about significant delays)
deceased in the 14 days preceding their death.

Case 176: Driving with disease

Candidate information has been improving but you have experienced


two hypoglycaemic episodes, with symptoms
Scenario of profuse sweating and feeling faint, without
A 48-year-old man with type 2 diabetes has warning, attributed to difficulty managing your
recently had changes to his medication. Please insulin requirements. Despite being informed
discuss the implications of the addition of of your obligation to inform the Driver and
insulin to his current driving status. Vehicle Licensing Agency (DVLA) of a change in
circumstances you have not done so and have
Actor information continued to commute to work 40 miles from
your home, including stretches of motorway.
You have had type 2 diabetes for 3 years and
despite weight loss, diet modification and
treatment with tablets your control has remained Approach to the case
poor. The doctor who reviewed you clinic When faced with a communication station case
4 months ago was concerned about the effects based on the topic of driving regulations, several
on your eyes and kidneys and started you on key principles apply:
insulin twice a day. Since then your blood sugar
Case 176 Driving with disease 343

Knowledge of driving regulations. The DVLA dependent diabetic patients, only where
website provides information on a wide there is use of oral medication with known
range of conditions via an AZ list risk of hypoglycaemia, or where episodes of
The three main conditions to know about hypoglycaemia have occurred.
are diabetes, epilepsy, and coronary artery In all cases of diabetes, the DVLA must be
disease. A fundamental understanding of the informed if the patient does not have awareness
main factors of each condition prohibiting of hypoglycaemic episodes.
driving is important. However, the full
guidance is often complicated and detailed Group 2 diabetes
thus it is always sensible to caveat any
To qualify for a group 2 license,
discussion with a patient by stating that you
insulin-dependent patients must demonstrate:
will give them clear written advice
The way in which information is projected is Full awareness of hypoglycaemic episodes
important. You should remember to remove and understanding of risks involved in
yourself from the driving regulations. You are relation to driving
not telling that patient that they cannot drive; No hypoglycaemic episodes over the
rather that you are duty bound to inform previous 12 months (where they were
them that the DVLA is telling them that they dependent upon others for their wellbeing)
cannot drive this is not just semantics Willingness to perform blood glucose
Do not let the patient leave the station monitoring 2 hours prior to driving and
thinking you have made a judgment on every 2 hours thereafter. Regular blood
their driving. The message is not that people glucose monitoring should be performed
with any given condition are bad drivers. using a glucometer with a memory function
Instead, the message is that there is an allowing review of a 3-month period during
inherent danger, totally independent of the assessment with an independent specialist
patient's driving ability, due to the nature of on an annual basis
the condition or possible complications of
Where criteria are met, a 1-year license may
treatment, that give rise to the potential for
be issued.
serious harm to themselves and/or others
The same rules apply to non-insulin
Be sure to clarify the patients occupation and
dependent diabetic patients only where
relate this to either dependency upon travel
there is use of oral medication with known
or a group 2 license (see below)
risk of hypoglycaemia, or where episodes of
Take the time to explore the patient's
hypoglycaemia have occurred. That said, all
concerns and questions
diabetics must inform the DVLA and undergo
Be clear that the rules for individual medical regular medical review. Non-insulin dependent
conditions vary depending on whether the patients may be issued a license for 1, 2, or
patient holds a group 1 (car or motorbike) or 3-year periods.
group 2 (heavy goods vehicle) license.
Group 1 epilepsy
Group 1 diabetes Isolated seizure, which refers to a single seizure
To qualify for a group 1 license, insulin- or multiple seizures within a 24-hour period,
dependent patients must demonstrate: having never suffered a seizure previously,
requires a 6-month period off driving.
Awareness of hypoglycaemic episodes
A formal diagnosis of epilepsy under group
Less than 1 hypoglycaemic episode over
1 licensing is considered by the DVLA as
the previous 12 months (where they were
two seizures within a 5-year period. To qualify
dependent upon others for their wellbeing)
for a license the patient must be seizure free
Willingness to perform blood glucose
with or without medication for a period of 1 year.
monitoring 2 hours prior to driving and every
Exceptions to this include:
2 hours thereafter
Normal standards for visual acuity, without Pattern over 1 year, exclusively of sleep
diplopia or visual field defects seizures full license granted even if sleep
seizures continue
Where criteria are met, a 1, 2, or 3-year
license may be issued.
Likewise, seizures where consciousness and
ability to control a vehicle are not impaired
The same rules apply to non-insulin
demonstrated for 1 year
344 Chapter 7 Communication skills and ethical scenarios (station 4)

Having been seizure free for a period of 1 year to negotiate with the patient by informing them
on medication, and where agreed with a that many employers will allow the patient
medical practitioner to come off medication, to retrain or move to a different part of the
breakthrough seizures may occur. If a workforce. In such a situation where the patient
breakthrough seizure occurs, a seizure free informs the candidate that they will continue to
period of 6 months is required on the same drive despite being advised of the DVLA rules, the
medication to regain license. strategy should include the following:
Confirm that the patient understands the
Group 2 epilepsy information they have been given
A formal diagnosis of epilepsy under group
Re-advise them of the legal requirements
2 licensing is considered by the DVLA as 2 If they still refuse to stop driving,
seizures within a 10-year period. To qualify for a demonstrate to the examiner, that you are aware
license patients must be seizure free for 10 years of all the appropriate steps that should be taken,
without medication. including:
Following an isolated seizure a patient is
required to adhere to 5 years off driving from the Seek the help of a senior doctor (registrar or,
date of the seizure. preferably, consultant)
Where a seizure is thought to be provoked Assuming no senior is available, the patient
by an event, which is unlikely to occur again, should be informed that in a situation such
such as head injury, the DVLA may consider as this, both the DVLA and the patient's GP
this an exceptional circumstance. Importantly, will be informed. In addition to this, the
seizures in the context of alcohol, drug misuse, patient will receive a letter documenting the
sleep deprivation or medication changes are not medical advice and what actions the medical
usually considered to be provoked. Each case is team have taken to inform the DVLA
considered on an individual basis. This applies It should be confirmed with the patient that
to group 1 licenses also. they understand what they have been told
Other themes worth consideration:
Group 1 acute coronary syndrome
Other conditions that may arise include
and coronary intervention transient ischaemic attacks, stroke, and
Following coronary angioplasty a period of 1 arrhythmias
week off driving must be adhered to. A return to Patients working as taxi drivers can be
driving is then possible provided there are no particularly challenging. Often, local
plans for further intervention within a 4-week authorities require group 2 licenses,
period, the LV ejection fraction is >40%, and muddying the waters somewhat
there are no other disqualifying conditions. Vehicles that do not require a license forklift
A period of 4 weeks off driving is required trucks, farm vehicles and sit-on lawn mowers.
following CABG. The Health and Safety Executive advises that
the standards for operating these vehicles
Group 2 acute coronary syndrome should be similar to that of comparative road
and coronary intervention vehicles. Employers are required to satisfy
health and safety regulations
Following coronary angioplasty a period of 6 Electric wheelchairs and mobility scooters
weeks off driving must be adhered to. A return are not considered vehicles, but if they are
to driving is based on an ability to satisfy used on the road then an application must be
functional/exercise testing and where no other made to the DVLA
disqualifying conditions exist. The impact upon insurance premiums will
also be impacted and patients should be
Breaking confidentiality and made aware of this
informing the DVLA
One aspect that may come up in the examination Further reading
is the scenario where the patient refuses to stop
Driver and Vehicle Licensing Agency. At a glance
driving. Reasons given for this may include
guide to the current medical standards of fitness
livelihood or other need to drive (hence
to drive. For medical practitioners. Swansea:
importance of addressing this). It may be possible
Driver and Vehicle Licensing Agency, 2014
Case 177: Genetic counselling

Candidate information Summarise your understanding of the


patients knowledge and outline the areas that
Scenario you wish to discuss in more detail, including:
A 26-year-old man has been referred by his GP. Clinical features of Huntingtons disease and
The patients father has recently been diagnosed age of onset of symptoms
with Huntingtons disease and as such the GP is Basic genetics and mode of inheritance with
requesting genetic counselling. As the neurology consideration of the implications on the
registrar you are asked to review the case, patient and his family
provide appropriate information and discuss the Options with regard to genetic testing and
potential implications. how such a service is accessed

Actor information
You are a 26-year-old married man with two
Clinical features
young children. You have a poor relationship Onset is usually during middle age, between
with your father who has been suffering with 30 and 50 years of age.
severe depression complicated by psychosis Symptoms are usually insidious and relate
for many years. His physical health started to dementia or psychiatric disturbance,
to deteriorate, to the best of your knowledge highlighting the importance of the fathers
this year, but you are unaware of the nature of mental health history in the vignette, and
the symptoms. The diagnosis of Huntingtons movement disorders.
disease is a recent one and you know nothing Abnormal movements include chorea,
about the condition other than it is inherited. myoclonic jerks, ataxia and gait disturbance.
You were reluctant to attend because you dont Be clear as to the untreatable nature of the
understand the need given that you have no disease with a poor prognosis. Life expectancy
symptoms and are in excellent health. You are is approximately 1015 years from onset
worried about the potential for developing of symptoms with considerable associated
similar mental health problems as your father morbidity.
but will only discuss this if asked directly and
with some persistence. Genetics and mode of inheritance
Huntingtons has autosomal dominant
Approach to the case inheritance.
Explain in simple terminology that the
Introduce yourself and clarify your role. Explore disease is related to inheritance of abnormal
the patients understanding and expectations genetic code excessive CAG repeats of the
of the referral from his GP. Attempt to manage Huntingtin gene (HTT). HTT is passed from
expectations from the outset, in that you are not affected parent to child and that, given the
in a position to offer formal genetic counselling mixing of genetic material between father and
nor test the patient, but rather offer information mother, he has a 50% chance of inheriting the
surrounding the condition to allow the patient to disease.
make a decision on whether he wishes to pursue
formal testing. If he has inherited HTT with excessive CAG
Allow the patient to talk openly with repeats, then he will develop Huntington's,
regard to his fathers disease, exploring his linked to high penetrance
initial presentation and current symptoms. If he has not inherited his fathers HTT, then
Attempt to gauge the patients current level of he will definitely not develop Huntington's
understanding of Huntingtons disease, pitching
Usually asymptomatic until after middle age
the subsequent interaction at a level that will
so the patient may already have children, as in
be easily understood. Remain flexible with your
this case.
approach, clearly demonstrating compassion
If he does not have the abnormal genetic
and empathy in relation to both the fathers
code, there will be no need to test his children. If
current mental and physical health and the
he does, then they would require testing, as they
impact upon the patients relationship with him.
would have the same 50% chance of inheritance.
346 Chapter 7 Communication skills and ethical scenarios (station 4)

Be sure to pause and ensure adequate of testing, this should be made clear to the
understanding of the details discussed. When patient prior
explaining inheritance of disease to patients Privacy and confidentiality should be
who have no prior knowledge of genetics, a considered, particularly in relation to
simple family tree diagram will help to visually disclosure to third parties, which may
describe the pattern of inheritance. include family members, who may also
The patients understanding of the content require testing, employers, and insurance
of the conversation should be checked. If he companies. Consider the Huntingtons
seems to be struggling with the information, patient in whom they refuse to disclose a
take the time to go back over the basics. If he is positive test result to their children
judged to have a good understanding and he The risks of testing also need to be made
has no specific questions, then the principle clear, albeit applicable predominantly to
of anticipation should be covered. This is the prenatal testing and risk to pregnancy
phenomenon by which Huntingtons tends to
Hypothetical discussion of a positive test
display more severe symptoms earlier in the life
may ensue. If the patient were to test positive,
of successive generations. It is un-necessary to
themes for further discussion would likely
go into the details of expansion of trinucleotide
focus upon the implication of the result and
repeats with the patient; however, this may form
ensuring that the patient understands the
the basis of a discussion with the examiner.
disease. Thereafter, it is possible that issues
of confidentiality may arise it is prudent to
Genetic testing consider situations such as a reluctance to
Formal genetic counselling and testing can be inform family members and the refusal of wider
accessed with a referral to specialist services, family testing. While confidentiality must be
however the examiner may require evidence of maintained, encouraging an open attitude with
an understanding of the principles involved: full disclosure to those closest to the patient will
be important, albeit a challenge demanding
Consent for testing requires that the patient
great skill and tact. Indeed, this highlights the
understands the nature of the disease being
importance of pre-testing discussion with family
tested for, underpinning the importance of
members, with joint decision-making, and is
the conversation outlined above
a vital point to stress with the examiner, as a
The indications for testing must be
means to avoid such a situation.
understood. The implication of a positive
Where the patient has not yet started a
result must be fully appreciated by the
family, a discussion in relation to assisted
patient and linked to predictive versus
conception with in vitro fertilisation and
diagnostic testing. Importantly with
pre-implantation testing, or pre-natal testing,
Huntingtons disease, a positive result
may arise.
indicates a sufficiently long CAG repeat
which predicts likelihood of onset of disease
provided the patient lives long enough. Genetic testing and insurance
As such a positive result is considered
predictive, while symptoms onset would The British Government and the Association of
be diagnostic. For other conditions genetic British Insurers set out in the document
testing may in fact be used for risk profiling Concordat and Moratorium on Genetics and
(e.g. BRCA testing in breast cancer or familial Insurance in 2001 that anyone who has had
adenomatous polyposis), identifying carrier a predictive test to assess their susceptibility
status (e.g. cystic fibrosis), or prenatal testing to genetic conditions can take out significant
(e.g. trisomy 21) insurance cover without disclosing the results. This
Clearly identify the benefit of testing. This is agreement has been extended to 2017 with the
often linked to the mode of inheritance or to next review due in 2014.
the ability to initiate treatment and improve
outcomes. In Huntigtons disease, where no At the end of the consultation, revise the
treatment is available the benefit of testing basic information with the patient to be sure
centres upon informed decision making in that he is going away with the right factual
relation to family planning understanding. Refer to support groups for the
The accuracy of testing should be disease. Offer a further clinic appointment, and
understood. Where there is likely to be a the opportunity to return with family members if
degree of uncertainty, due to limitations desirable.
Case 178 Percutaneous endoscopic gastrostomy feeding 347

Additional themes worth consideration: Further reading


Genetic cases may be used by examiners in Association of British Insurers. Concordat and
the context of breaking bad news to patients. Moratorium on Genetics and Insurance.
Consider situations requiring the explanation London: UK Government & Association of
of investigation results, confirming a British Insurers, 2011
diagnosis such as cystic fibrosis or multiple
sclerosis, and related discussion

Case 178: Percutaneous endoscopic


gastrostomy feeding

Candidate information able to tolerate fluids or oral foods. He should be


informed in simple language that the standard
Scenario management is to insert a PEG tube to ensure he
A 60-year-old man with head and neck meets his nutritional requirements during this
cancer attends the gastroenterology clinic period.
for pre-assessment. He is due to undergo At this point, the patient may or may not
chemoradiotherapy and as a result will be have any idea about what a PEG is. As with all
unable to tolerate food and drink for an explanations to patients in MRCP PACES, this
extended period of time. Please counsel him must be explained in a manner that:
about the need for PEG feeding. Contains no medical jargon: A PEG is a tube
that is inserted through the skin into the
Actor information stomach. The medical abbreviation stands for
You have recently been diagnosed with a head percutaneous endoscopic gastrostomy
and neck cancer. You have been told that you Is alert to the patient's sensitivities
will need a feeding tube put into the stomach Addresses what you understand to be the
while you are having your treatment. You are patient's concerns
due to meet the doctor today to discuss the
Explain to the patient, in basic terms, the
details. While apprehensive about the procedure
technique of PEG insertion, involving upper GI
you are hoping to understand the practicalities
endoscopy, and then an incision in the skin to
of placement and want to be clear on the
insert the tube into the stomach. Discuss the need
possible risks and associated complications.
for light sedation, often with midazolam, or simply
lidocaine throat spray. Most patients tolerate it
Approach to the case well although it can be an uncomfortable and
Introduce yourself and explain the purpose distressing experience in a minority. As with all
of the consultation. The patient should be procedures, inform the patient of the common
asked about what has happened thus far, what and serious complications, but at the same time
he understands of his diagnosis, proposed explain that the balance of risk and benefit lies
treatment and need for PEG feeding. An open strongly towards benefit. Complications can be
approach will allow you to judge his level of divided into those related to the PEG insertion
knowledge and help build up a rapport but and those related to feeding via a PEG
beware spending a disproportionate amount of Complications related to PEG insertion
time on history taking. Attempt to focus swiftly include:
on to the topic of PEG feeding. Peritonitis is a rare but serious complication.
It should be explained to the patient that due It can occur due to leakage of the stomach
to the intensive treatment given to head and contents into the peritoneum via the
neck oncology patients, it is unlikely he will be iatrogenic defect in the stomach wall.
348 Chapter 7 Communication skills and ethical scenarios (station 4)

This is a life-threatening complication their concerns. The patient should be offered


requiring urgent surgical intervention some information sheets to take home with
Wound site infection. There is evidence to them and signposted to relevant websites or
show that the risk of wound site infection is online resources.
significantly reduced by use of prophylactic Additional themes worth consideration:
antibiotics at the time of PEG insertion
Alternative situations in which PEG
Complications related to feeding via the PEG feeding is commonly encountered such
include: as stroke rehabilitation, where it has been
demonstrated to be highly beneficial and
Irritation of the surrounding skin (due to
improve prognosis, and dementia, where its
leakage of gastric contents)
value remains controversial
Blocked PEG tube
Situations where families either insist on PEG
The PEG tube can fall out, requiring
feeding, where it is not in the patients best
replacement
interests, or alternatively where they resist,
The tube and its buffers can be internalised,
and are not in agreement despite potential
drawn into the skin due to raised tension
benefit. Be prepared to compare PEG
Allow the patient time to ask any questions feeding with alternatives such as short term
they have. If they have any specific questions nasogastric tub feeding, or total parenteral
that exceed your knowledge base, the patient nutrition. Such cases may have an increased
should be told that the question will be focus towards capacity, consent and the
discussed with either a member of the oncology incapacitated patient
team (for cancer-related questions) or the
gastroenterology consultant or PEG nurse (for
PEG insertion-related questions) and the answer
Further reading
relayed back to the patient. Avoid ambiguity if Nicholson FB, Korman MG, Richardson MA.
uncertain and do not feel embarrassed to admit Percutaneous endoscopic gastrostomy: a review
to not knowing an answer. of indications, complications and outcome.
As the consultation nears its close, ask the J Gastroenterol Hepatol 2000; 15:2125
patient if they think they are happy to go ahead Jain NK, Larson DE, Schroeder KW, et al. Antibiotic
with the PEG. The majority of patients will say prophylaxis for percutaneous endoscopic
yes. In this case a date can be arranged. If they gastrostomy. A prospective, randomized, double-
say no, they can be offered a discussion with blind clinical trial. Ann Intern Med 1987; 107:
one of the gastroenterologists to seek to assuage 824828.

Case 179: HIV testing recent


unprotected sex

Candidate information experiencing difficulties. 48 hours prior you


were on a night out with your girlfriends and
Scenario consumed an excessive amount of alcohol.
A 29-year-old woman is attending the GUM You had sexual intercourse with a man that
clinic, concerned about the risk associated you had met for the first time in a club. Due to
with recent unprotected sexual intercourse, intoxication you cannot recall the full details of
requesting medication. Assess the case and the encounter and are unclear as to the use of
advise the patient on an appropriate course of protection. Concerns relate primarily to the risk
action. of HIV transmission.

Actor information Approach to the case


You are in a long-term relationship with your Encouraging an open discussion relies upon
boyfriend however recently you have been making the patient comfortable and ensuring
Case 179 HIV testing recent unprotected sex 349

a non-judgmental approach to what are often In the highest risk groups, men who have
embarrassing and difficult topics to articulate. A sex with men, engaging in unprotected anal
matter of fact attitude can be useful but risks the intercourse the risk of transmission, where
impression of being cold and uncaring. Often, HIV status is unknown, is less than 1 in 1000,
reassurance is all that is required. increasing to approximately 1 in 100 if known
HIV positive. Your risk is considerably less than
Sexual health history taking this. As a heterosexual engaging in unprotected
vaginal intercourse, the risk is likely between 1
A brief history is a useful place to start. Avoid
in 30,000 and 1 in 200,000. Avoid focusing on
overcomplicating such an approach, but the
absolute numbers, conscious that they will vary
basic content of a sexual health history may offer
with time and between populations, the point
the platform on which to develop the relevant
here is that the relative risk may in fact be quite
themes of the scenario.
low.
Allow the patient to explain her reason for
However, as alluded to, risk may be increased
attending and express her concerns. Attempt to
by certain factors. Most importantly, where
understand:
possible it needs to be established as to whether
Date of last sexual contact and the number of the sexual contact is known to be HIV positive.
partners in the last 3 months Thereafter, consideration is given to the nature
Establish the gender of the partners more of the sexual intercourse in relation to acts
relevant with men who have sex with men performed, ejaculation, and the possibility of
(MSM) but also women who may have trauma and exposure to blood.
bisexual partners In reality, such information is usually
Nature of intercourse by anatomical site obtained in clinic and referenced to a pro-
vaginal, anal and oral forma to risk stratify patients. For the purposes
Use of protection of MRCP PACES, be mindful that the most
Suspicion of previous or current infection by important features pertaining to high-risk
symptoms or confirmed diagnosis encounters include:
Existing diagnosis of blood borne virus and
Known HIV positive partner (in which
history of previous HIV/hepatitis testing
case establishing the viral load would be
paramount)
Understanding HIV and AIDS Unprotected intercourse
Where the patient raises concerns or worries in Receptive, greater than insertive, anal
relation to transmission of HIV establish what intercourse
they understand of the disease. Clarify and Trauma
summarise the salient points as necessary.
Attempt to gauge the validity of concerns It should be made clear that testing can
in relation to HIV exposure. Establish whether and should be offered regardless of perceived
there is evidence to suggest that the sexual risk. Testing may be routine blood tests or
contact has a confirmed diagnosis of HIV. potentially more expedient point of care
For instance have they been contacted with testing. A repeat test is always required 12
information to that effect? Indeed, has the weeks later (newer antigen testing may be
partner disclosed recent blood testing, including feasible at 4 weeks).
viral load and details of current treatment? Where exposure is considered highrisk,
While such a situation may seem implausible, testing should be performed and post
it is the first step in moving the conversation exposure prophylaxis (PEP) offered provided
towards a consideration of relative risk. presentation is within 72 hours. Counselling
should include:
Risk of transmission related to An understanding that regardless of whether
exposure, need for testing, and the baseline test is negative continued
compliance with a 4-week course of antiviral
consideration of post exposure medication is required
prophylaxis The need to have a second HIV test 12 weeks
The concept of risk needs to be introduced. post completion of PEP
While statistics are rarely welcomed in such a The side-effects of the drugs and the support
scenario, often with anxious patients fearing the available in the clinic and in the community
worst, they can be used to offer context. It is not to help adherence
recommended to rote learn figures, but useful to Promotion of safer sex, particularly over the
employ something along the lines of: initial 4-month period
350 Chapter 7 Communication skills and ethical scenarios (station 4)

Issues around disclosure and confidentiality for commonly encountered sexually transmitted
(see below) infections including hepatitis.
Psychological support as necessary, often Conclude by checking understanding with
relating to high-risk behaviour complicated the patient. Reiterate the salient points of the
by substance misuse or the impact of discussion, particularly the need for compliance
potential infection with medication if prescribed and the need for a
repeat test at 12 weeks.
In the first instance a starter pack is often
The method of communicating results to the
made available, containing a combination of
patient should be agreed, either with a planned
anti-viral medication in addition to medication
return visit to the clinic or by text message. It is
to aid tolerance such as anti-emetic and anti-
good practice for the clinic to deliver the results
diarrhoeal medication, which will last the
directly and never via a third party.
patient up to 5 days. Review thereafter in the
Additional themes worth consideration
GUM clinic allows for review and potential
include:
alteration of the regime based on further
information, which may have been obtained in The patient who requests PEP but does
the interim. not wish to be tested for HIV. HIV testing is
mandatory for all patients receiving PEP
Principles of confidentiality and Drug resistance in HIV positive individuals
may influence the choice of PEP in patients
information sharing in sexual health who are concerned about exposure and risk
Routinely, the attendance at, and results from, of transmission from that person. This should
sexual health clinics are not shared with other not delay start PEP where indicated but may
medical health professionals, including a result in modification of the regime at a later
patients GP. Exceptions to this include where date when details of resistance available
the patient was in fact referred in by their GP The pregnant patient. Pregnancy is not a
or where the patient consents to sharing of the contraindication for PEP but expert advice
information, often to allow continuity of care, should be sought
particularly where conditions are chronic and Individuals with repeated high-risk exposure.
may impact upon other health issues for which Repeated exposure should be considered
the GP is responsible. as cumulative risk when making a decision
There are certain instances where to prescribe PEP. When exposed repeatedly
confidentiality cannot be maintained and during a period of PEP, the medication does
sharing of information is required, including: not need to be extended beyond the existing
In response to a court order 28-day course
Notifiable diseases (Health Protection The individual in whom PEP has been started
Agency) and situations where there is but noncompliance has proven troublesome
deemed considerable risk to others A patient demanding PEP when it is not
Where there are concerns about safeguarding indicated. There is often a public perception
vulnerable adults or children of a right to medication. This scenario
challenges the candidate to explain the
The intention to share information should risk versus benefit principle of antiviral
be discussed with the patient first, unless medication, often best centred upon the
prohibited by a court order. potential adverse side effects of medication

Other infections associated with Further reading


high risk behaviour British Association for Sexual Health and HIV. http://
Where concern exists with regard to HIV www.bashh.org/.
infection, screening should also be performed
Case 180: HIV testing needlestick injury

Candidate information Check the exact time of the incident. Where


appropriate, treatment should be instigated
Scenario within 12 hours of exposure for the most
You are the on-call medical registrar. Your SHO effective reduction of risk. Furthermore, it
has been putting in a central venous catheter on should be ensured that practical measures
a patient who presented with respiratory failure. have been taken, including encouraging the
She reports that she received a needlestick wound to bleed but not sucking or squeezing
injury. She is concerned because the patient with excessive pressure, irrigation and washing
has a history of intravenous drug abuse. The with warm water and soap and first aid where
local policy suggests that the SHO should be risk necessary.
assessed in Accident and Emergency however Clarify the details of the potential exposure.
the site manager has asked if you would While this scenario indicates a needlestick
review her due to pressures in the emergency injury, be aware of the potential for exposure
department. resulting from a splash of fluid involving mucus
membranes or broken/damaged skin.
Actor information Specifically related to needlestick injuries,
establish features associated with increased risk:
You are the on call medical SHO. Earlier,
you were asked to insert a central line into a Hollow bore needle (higher risk as compared
40-year-old man who presented with respiratory to solid bore needle). When discussing
failure and is on the high-dependency unit needlestick injuries during procedures in
(HDU). After you had inserted the central which more than one sharp implement is
venous catheter, you were clearing up the involved, be precise in finding out which
sharps to dispose of them into the sharps bin. sharp caused the injury. For example, in a
You sustained a needlestick injury on the central line insertion there is the hollow-bore
hollow-bore needle which had been used to introducer needle, the scalpel, and the solid
obtain central access. You had sterile gloves on, bore suture needle
but the needle punctured these and the skin. Deep injuries and those that draw blood
You washed the wound for 10 minutes under are both associated with increased risk of
running water. You are concerned because the transmission of blood-borne viruses
patient in question has a history of intravenous
drug abuse. You are worried about the risk of
blood-borne virus transmission. Features associated with high risk
Mechanism:
Approach to the case Deep injury
Hollow bore needles
Presume that you have not previously met the Donor known to be blood-borne virus
SHO and introduce yourself. I understand positive
that you have sustained a needlestick injury. Body fluid: Predominantly concern relates to blood
What I suggest we do is start with talking about exposure however body fluid in which virus may
exactly what happened, and then we can go on be present and also represent high-risk includes:
to discuss whether or not you need to consider Amniotic fluid
post-exposure prophylaxis. Finally, we can Vaginal secretions
talk about further testing. Is that ok? It may be Semen
sensible to gauge the doctors understanding of Breast milk
the steps required following potential exposure CSF
to blood-borne viruses or make a statement Peritoneal, pleural, and pericardial fluid
such as I am going to work through the protocol Synovial
for this assuming you have no prior knowledge, Bodily fluids including urine, faeces, sputum and
for that I apologise in advance but in my vomit are generally considered low risk unless
experience it is the safest way of ensuring we contaminated by blood.
dont miss anything important.
352 Chapter 7 Communication skills and ethical scenarios (station 4)

Identify the patient, referred to as the and 24 weeks. Occupational health will also be
donor. The SHO, or recipient, may have limited responsible for obtaining the donor results and
information on the donors medical background liaise with recipient with the outcome. Where
or current case but attempt to establish: positive, they will refer for specialist care and
advice on fitness to work. Furthermore, they
Suspected or confirmed history of
will also report confirmed BBV exposures to the
blood-borne virus (HIV, HepB, HepC)
health protection agency.
High-risk features, including ethnic origin
The GUM clinic will normally facilitate
correlating to areas of high prevalence,
ongoing provision of PEP for a 4-week period
intravenous drug use, sex worker status, and
and monitor for toxicity with weekly blood
(increasingly less relevant) haemophilia with
testing.
history of transfusion pre 1985
What are they currently in hospital for? In
General considerations:
this case, the donor has respiratory failure
In this case, you should send the doctor home
in the context of a history of intravenous
and tell the examiner you would discuss with
drug use. It would be helpful to know
the site manager to arrange cover for the rest of
what the working diagnosis is and see
the shift.
the plain chest radiograph to consider
The hospitals infection control team is
pneumocystis pneumonia suggestive of
responsible for monitoring implementation
immunocompromised state
of sharps policy. The doctor should be offered
Do they have capacity to consent to testing formal training on preventing sharps injuries.
for blood-borne viruses? Regardless of the actual scenario, be aware that
High-risk situations demanding the Department of Health guidance notes that
consideration of PEP: many exposures result from a failure to follow
recommended procedures, including the safe
If the donor has confirmed HIV, the doctor handling and disposal of needles and syringes.
should be advised to commence post- Part of the discussion with the examiner may
exposure prophylaxis, as soon as possible involve addressing how to consent the donor
from the time of the sharps injury. A PEP for HIV testing. Ultimately the patient is under
consent form must be signed by the recipient no obligation to consent. Testing for the sake of
In cases where there is a suspicion of HIV the recipients best interests is not necessarily
but no confirmatory evidence, discussion in the donors best interest. Caution should be
should take place between you and the on- used to avoid placing pressure or coercing the
call infectious disease doctor to discuss the donor into testing for blood-borne viruses. The
risks of the needlestick and potential need concept of informed consent is paramount. Pre-
for empirical PEP therapy until test results for and post-test counselling should be in place.
HIV are available Where the patient is incapacitated and unable
High risk of hepatitis B may require treatment to consent, the physician responsible for their
with immunoglobulin care should be consulted and evidence that it
The doctor who has sustained the needlestick is in the patients best interest sought. Blood
injury will need to have blood tests for blood- testing should be performed either by you, as
borne viruses (HIV, hepatitis B and C). They the registrar on call, or by the doctor responsible
will require repeat testing at time intervals up for the care of the patient provided it is not the
to 6 months to ensure seroconversion has not recipient.
occurred
The doctor who sustained the needlestick
injury should fill in a clinical incident form Further reading
Department of Health. Guidance for clinical health
Thereafter, referral should be made to
care workers: protection against infection with
occupational health; where necessary they
blood-borne viruses. London; Department of
will facilitate immunisation such as hepatitis
Health, 1998.
B booster and undertake repeat testing at 6, 12
Case 181: Dealing with complaints
medical errors

Candidate information a situation avoid commenting on colleagues


actions but rather seek to resolve the matter.
Scenario You must neither try to cover up actions made
As the medical registrar covering the acute ward by colleagues nor convict them in the court of
a patient has requested a meeting with you to public opinion.
discuss concerns in relation to the care that he Explain how you wish to resolve the situation
has received. and explain the actions you intend to take
to ensure that similar does not occur. In this
Actor information scenario, consider:
You are a 60-year-old man who has been in Practical measures to ensure completion of
hospital for 2 days with a chest infection. Your a course of appropriate antibiotics. Change
condition has improved and you are due to the prescription and ensure no delay to
be sent home to complete a course of oral discharge by involving the sister in charge of
antibiotics. Despite telling the doctor who saw the ward or alerting pharmacy
you in casualty that you are allergic to penicillin Completion of an incident form
(when you had penicillin before your throat Feedback to the colleague and their
closed over) you have been given amoxicillin supervising consultant, with training or
as your oral treatment to go home with. You teaching where necessary
are extremely upset as you think your life may Furthermore, a review of the ward process
have been put in danger. You wish to make a for such things as the ordering of discharge
complaint and have asked to see the doctor. medication that should involve check points
for allergy status including nursing staff and
the dispensing pharmacist
Approach to the case
Introduce yourself and clarify your role. Explain Where an apology is deemed insufficient
that you understand the patient has expressed direct the patient to the Patient Advice and
a wish to make a complaint and that you would Liaison Service to support further options
like to deal with any concerns he may have. Give open to the patient in line with the complaints
the patient the opportunity to explain the nature procedure.
of their complaint, listening attentively and
resisting the temptation to interrupt, become Complaints procedures
defensive, or challenge the information outright.
Summarise the issues raised and repeat Where a patient is not satisfied with a verbal
them back to the patient to ensure a shared apology, it is important to have an appreciation of
understanding of the areas for discussion. Make the complaints procedure.
efforts to understand the situation from the All patients have the right to complain about any
patients perspective, acknowledge distress if aspect of their care. Furthermore, individuals may
caused and empathise where possible. complain on behalf of a patient where consent to
Apologise for any distress caused. If do so has been obtained. Complaints should be
appropriate apologise for the consequences received as soon as possible after the event, or
of an action but you are often not in a position from the time when they were aware of a reason to
to apologise for the act itself, unless you were complain, generally within a 12-month window.
personally responsible. This may seem like In the first instance patients should be
avoiding the issue, but you should recognise encouraged to complain at a local level
your limitations in this situation and be clear on to the provider of the service from which
the concept of apologising on behalf of the team they received their care and to whom the
not the individual. complaint refers.
Discussing obvious errors made by Alternatively, complaints can be made to
colleagues is an extremely difficult topic. Clich the commissioner of the service Clinical
tells us that we all make mistakes so it is Commissioning Group or NHS England.
tempting to defend colleagues and attempt to Every service should make available, on
justify their actions. That, however, would not be request, a copy of their complaints procedure.
the right and proper action. If faced with such
354 Chapter 7 Communication skills and ethical scenarios (station 4)

Often there will be a named individual who Definition of clinical negligence and
deals with all complaints.
the Bolam test
It is good practice to ensure a dated response
within 3 working days of receipt of a written The doctor must be proven to owe a duty of
complaint. This should include an outline the care to a patient
actions initiated, offer a meeting to discuss/ A breech of that duty of care must be
negotiate a plan to deal with the issues raised, and demonstrated, often demanding application
outline a timeline for a formal response (no fixed of the Bolam test to reveal liability.
time scale). Comparing the treatment or management
Where the outcome or response is unsatisfactory, implemented to that which would be
patients have a right to appeal to the considered reasonable or applicable by the
Parliamentary and Health Service Ombudsman. wider medical community
In exceptional circumstances a judicial review may The breech of duty of care should be shown
be required. to have caused harm, referred to as causation.
Support for complaints: That is to say, harm that has resulted which
Patient Advice and Liaison Service would not otherwise have occurred
Complaints Advocacy Service
Citizens Advice Bureau Common scenarios for complaints relate to:
Complaints cannot be directed to the Care Quality
Commission however the commission can be
The patient in whom a test result has
been misinterpreted with either distress
informed of concerns with services.
resulting from misdiagnosis or inappropriate
Patients can make complaints about doctors
implementation of management/treatment
directly to the General Medical Council. The
options
GMC will review the complaint and if deemed
appropriate proceed with an investigation, or
The lost test result with the need for repeated
testings and delay to diagnosis or treatment
alternatively it may be referred back for local
resolution. In such instances, doctors should
Access to treatment issues. Often complaints
relate to novel treatments for conditions with
contact their defence union or British Medical
poor prognosis, where NHS availability is
Association for advice and consider the need for
limited or ruled out by high cost and poor
legal representation.
evidence of benefit. Answering this sort of
complaint requires a detailed understanding
Additional themes worth consideration:
of the relevant NICE guideline(s) and of the
Patients demanding financial compensation way NHS care is funded
from a complaint. Be clear that financial Delays to definitive management, with the
compensation cannot be obtained from need to apply principles of waiting lists
the standard complaints procedure in the and supply versus demand on the basis of
NHS. Legal action is required to secure the prioritisation
possibility of financial compensation, where
clinical negligence can be demonstrated
Case 182: The trainee in difficulty

Candidate information Slowness and inefficient use of time. Often


paradoxically, the doctor who appears
Scenario hyper-vigilant in arriving early and leaving
As the medical registrar you have been working late from work may in fact be attempting to
with the new intake of junior doctors for over compensate for problems coping
3 months. Concerns have been raised by the Difficulty prioritising work, and an inability
nursing staff about the performance and to compromise or accept constructive
behaviour of one of the junior doctors with criticism
whom you have also experienced problems. Teamwork dynamics. Members of the team,
There is a general feeling that the doctor is including nurses and other doctors, seek to
having difficulty coping with the demands of bypass the need for involving the individual
the job and you have been asked to discuss the in question. This is a coping strategy that fails
situation with them. to deal with the root cause of the problem
Temper or behavioural outbursts on the ward
Actor information often with a lack of insight and defensive
responses to criticism with counter-
You are a 24-year-old foundation year doctor challenges relating to the performance of
having graduated 6 months previously and others
moved to the city to start your training.
Throughout medical school you avoided
spending time on the wards during clinical
Why is it important?
attachments, instead, prioritising time in the GMC guidance states: The safety of patients
library. The exposure to clinical medicine has must come first at all times. If you have concerns
been a shock and you have struggled to adapt that a colleague may not be fit to practise, you
but have felt unable to discuss the problem with must take appropriate steps without delay, so
your peers who seem more confident and able. that the concerns are investigated and patients
Your reaction to the situation has been to isolate protected where necessary. This means you
yourself and minimise contact with your team must give an honest explanation of your
members. Increasingly irritable, low in mood, concerns to an appropriate person.
de-motivated and lacking in confidence you are
frustrated by the downward spiral in which you Understanding the process of
find yourself and discussing the issues comes as raising concerns
a welcome relief.
Distinguishing between the trainee with
difficulties versus the difficult trainee is an
Approach to the case important concept. The former represents
Be clear on the reasoning and evidence for any a failure to make satisfactory progress with
concerns raised particularly if others have raised training, for whatever reason, with resultant
them. Avoid conjecture and be wary of fuelling difficulties relating to their standard of clinical
gossip or rumours. It is important to focus upon practice or ability to perform their role. The
observable behaviours rather than personal difficult trainee poses a problem as a result
characteristics or traits. of attitudes or behaviours impacting upon
professionalism and conduct. Both have
Recognising the trainee in difficulty a potential impact upon patient safety but
practical solutions to the problems are likely to
Difficulties may relate to: be markedly different.
Lack of reliability. Persistent lateness, Aim to deal directly with the individual.
frequent sick leave, issues with not replying Sharing your concerns with an early conversation
to bleeps, and disappearing during may rapidly identify the causes of any difficulty.
clinical duties such as extended breaks or Attempt to discuss the situation in a non-
unaccounted periods between ward and judgmental way, encouraging an open and honest
clinic work discussion. Where the individual is reluctant
356 Chapter 7 Communication skills and ethical scenarios (station 4)

to discuss the perceived problems, attempt to information with their consultant who is
explain the responsibilities you have in dealing often the clinical supervisor, or alternatively,
with the matter now that you have been made educational supervisor. If met with a refusal to
aware of it. This may offer a degree of separation, such a suggestion, a tactful explanation of your
in that you are helping to discuss the concerns of duty to do so on their behalf should follow. By
others, not necessarily your own. Avoid a punitive escalating to seniors, the challenge of deciding
or threatening tone but explain the potential for the need for wider involvement including
escalation if no progress can be made. occupational health, human resources, the
Common causes of difficulties include: deanery, and the GMC is somewhat deferred.
That said you should understand your
Educational problems, balancing work
responsibility to protecting the best interests of
pressures with revision and exams
patients and the potential need to personally
Career progression and anxiety in relation to
contact the GMC should you believe that there
lack of support in making career choices
are issues relating to fitness to practice.
Perceived lack of team support, difficult team
Where possible be seen to link the use
dynamics, or understaffing
of the trainees e-portfolio and work-based
Personal or family health issues including assessments, both as a way of identifying
substance misuse or dependency trainees in difficulty, but also as a means of
Personal relationship difficulties setting goals and monitoring improvement.
Domestic responsibilities or pressures. Where assessments have been used honestly
Inability to strike a life-work balance and openly they may be used to identify areas
of weakness, such as communication skills,
Interventions and solutions practical procedures, clinical reasoning, or team
Formulating a plan of action, which is in the best working. Multi-source feedback can be utilised
interests of patients. to offer context to any complaints or concerns.
Understanding the reasons for difficulty In addition to identifying areas of support,
should better allow you to identify avenues for and need for specialist input or management,
help. Where legitimate problems are identified, trainees may also be able to consider flexible
doctors should be encouraged to share the training or an extension their training period.

Anda mungkin juga menyukai